Пропорции математика. Пропорция – это равенство двух отношений. Соотношение.
Альфашкола
Статьи
Пропорция в математике
Пропорция – это равенство, утверждающее, что два отношения равны. Пропорциональный — значит находящийся в определенном отношении к какой-либо величине. Четыре величины \(4, 2, 8 \) и \(4\) находятся в отношении, если \(\frac{4}{2}=\frac{8}{4}\). Произведение крайних членов пропорции равно произведению средних.
Пропорция всегда включает равные коэффициенты. Когда соотношение остается постоянным, это соотношение называется пропорциональным.
Если \(\frac{A}{B} = \frac{C}{D}\), то
\(AB=CD\)
\(AD=BC\)
Пропорция состоит из двух равных отношений. Однако если \(\frac{A}{B}\) не равно \(\frac{C}{D}\), то \(A, B, C, D \) не называются пропорцией.
Три величины считаются пропорциональными, если отношение первого ко второму равно соотношению второго и третьего.
\(A, B , C\) находятся в постоянной пропорции, если \(\frac{A}{B} =\frac{C}{D}\)
Если \(A, B ,C \) находятся в постоянном отношении, то \(B\) называется средней в пропорции.
В косвенной пропорции как одно значение увеличивается, так и другое значение уменьшается.
Задача 1. За \(5\) дней и \(12\) человек построили забор. Сколько дней это займет у \(6\) людей?
Решение.
\(12\) человек → \(5\) дней
\(6\) человек → \(x\) дней
\(\frac{12}{6} = \frac{x}{5}\)
умножаем крест на крест члены пропорции и сокращаем на \(6\):
\(12*5=6x\)
\(60=6x\)
\(x=10\)
Ответ: \(6\) людей будут работать \(10\) дней, чтобы закончить работу.
Задача 2. Найдите значение \(x\), если \(\frac{2}{5}=\frac{x}{15}\)
Решение:
\(2*15=5x\)
\(30 =5x\)
Делим на 5 обе части равенства: \(\frac{30}{5}=x\), откуда находим
Задача 3. Что должно быть добавлено к каждому из четырех чисел 10, 18, 22, 38, чтобы сделать их пропорцией?
Решение:\(\)
\((10+x)(18+x)=(22+x)(38+x)\)
\(380+48x+2x=396+40x+2x\)
\(8x=16\)
\(x=2\)
Задача 4. Найти четвертый член пропорции \(6,10\) и \(12\)
Решение:
\(\frac{6}{10}=\frac{12}{x}\)
6×х = 120
x = 120/6
x = 20
Больше уроков и заданий по математике вместе с преподавателями нашей онлайн-школы «Альфа». Запишитесь на пробное занятие уже сейчас!
Запишитесь на бесплатное тестирование знаний!
Нажимая кнопку «Записаться» принимаю условия Пользовательского соглашения и Политики конфиденциальности
Наши преподаватели
Ирина Демьяновна Хоухлянцева
Репетитор по математике
Стаж (лет)
Образование:
Могилевский государственный педагогический институт им. А. Кулешова
Проведенных занятий:
Форма обучения:
Дистанционно (Скайп)
Сабина Витальевна Рабцевич
Репетитор по математике
Стаж (лет)
Образование:
Белорусский государственный университет
Проведенных занятий:
Форма обучения:
Дистанционно (Скайп)
Екатерина Сергеевна Яковлева
Репетитор по математике
Стаж (лет)
Образование:
Донецкий Национальный Университет
Проведенных занятий:
Форма обучения:
Дистанционно (Скайп)
Предметы
Математика
Физика
Химия
Русский язык
Английский язык
Обществознание
История России
Биология
География
Информатика
Похожие статьи
Сложение и вычитание отрицательных чисел
Многочлены
РУДН: факультет ветеринарии
Арифметическая прогрессия
Наводим красоту: топ-10 простых и красивых причесок для девушек (разного возраста)
Топ 20 бесполезных покупок к школе
Что такое педикулез и как с ним бороться
Чем заняться в новогоднюю ночь всей семье?
Нажимая кнопку «Записаться» принимаю условия Пользовательского соглашения и Политики конфиденциальности
§ Пропорции 6 класс.
Тема пропорции
Похоже, вы используете блокировщик рекламы. Наш сайт существует и развивается
только за счет дохода от рекламы.
Пожалуйста, добавьте нас в исключения блокировщика.
Скрыть меню
На главную страницу
Войти при помощи
Темы уроков
Начальная школа
Геометрия: начальная школа
Действия в столбик
Деление с остатком
Законы арифметики
Периметр
Порядок действий
Разряды и классы. Разрядные слагаемые
Счет в пределах 10 и 20
Математика 5 класс
Взаимно обратные числа и дроби
Десятичные дроби
Натуральные числа
Нахождение НОД и НОК
Обыкновенные дроби
Округление чисел
Перевод обыкновенной дроби в десятичную
Площадь
Проценты
Свойства сложения, вычитания, умножения и деления
Среднее арифметическое
Упрощение выражений
Уравнения 5 класс
Числовые и буквенные выражения
Математика 6 класс
Масштаб
Модуль числа
Окружность. Площадь круга
Отношение чисел
Отрицательные и положительные числа
Периодическая дробь
Признаки делимости
Пропорции
Рациональные числа
Система координат
Целые числа
Алгебра 7 класс
Алгебраические дроби
Как применять формулы сокращённого умножения
Многочлены
Одночлены
Системы уравнений
Степени
Уравнения
Формулы сокращённого умножения
Функция в математике
Геометрия 7 класс
Точка, прямая и отрезок
Что такое аксиома и теорема
Алгебра 8 класс
Квадратичная функция. Парабола
Квадратные неравенства
Квадратные уравнения
Квадратный корень
Неравенства
Системы неравенств
Стандартный вид числа
Теорема Виета
Алгебра 9 класс
Возрастание и убывание функции
Нули функции
Область определения функции
Отрицательная степень
Среднее геометрическое
Алгебра 10 класс
Иррациональные числа
Алгебра 11 класс
Факториал
Для утвердительного ответа достаточно лишь одного слова — «да». Все прочие слова придуманы, чтобы сказать «нет».Дон-Аминадо
на главную
Введите тему
Русский язык
Поддержать сайт
Пропорции
Задачи на пропорции
Запомните!
Пропорция — это равенство двух отношений.
Рассмотрим два равных отношения:
Соединив их знаком равенства, мы получим пропорцию.
В пропорции различают крайние и средние члены.
8 и
5 называют крайними членами.
4 и 10 — средние члены.
Если пропорция составлена верно, то есть отношения, составляющие эту пропорцию действительно равны,
то для пропорции верно следующее:
Запомните!
Произведение крайних членов пропорции равно произведению средних.
Правило выше и называется основным свойством пропорции.
Чтобы правильно применять правило, мы предлагаем вам запомнить правило (креста) «X».
Рассмотрим его на примере пропорции.
Убедимся, что пропорция составлена верно.
Теперь запишем пропорцию и нарисуем карандашом поверх знака равенства крест.
Нарисовав крест, гораздо легче составить нужное произведение (выполнить основное свойство пропорции).
Прямая пропорция — это математическое сравнение двух чисел, при котором отношение двух чисел равно постоянному значению. Определение пропорции гласит, что когда два отношения эквивалентны, они пропорциональны. Символ, используемый для обозначения пропорций, — «∝». Давайте узнаем больше о прямой пропорции в этой статье.
1.
Определение прямой пропорциональности
2.
Формула прямой пропорции
3.
График прямой зависимости
4.
Прямая пропорция против обратной пропорциональности
5.
Часто задаваемые вопросы о прямой пропорциональности
Определение прямой пропорциональности
Определение прямой пропорциональности гласит: «Когда отношения между двумя величинами таковы, что если мы увеличим одну, то увеличится и другая, а если мы уменьшим одну, то уменьшится и другая величина, то говорят, что две величины равны прямо пропорционально». Например, если есть две величины x и y, где x = количество конфет, а y = общая сумма потраченных денег. Если мы покупаем больше конфет, нам придется платить больше денег, а если мы покупаем меньше конфет, то мы будем платить меньше денег. Итак, здесь мы можем сказать, что x и y прямо пропорциональны друг другу. Он представлен как x ∝ y. Прямая пропорция также известна как прямая вариация.
Ниже приведены некоторые реальные примеры прямой пропорциональности:
Количество продуктов питания прямо пропорционально общей сумме потраченных денег.
Выполненная работа прямо пропорциональна количеству рабочих.
Скорость прямо пропорциональна расстоянию за фиксированное время.
Формула прямой пропорции
Формула прямой пропорции говорит, что если величина y прямо пропорциональна величине x, то мы можем сказать, что y = kx для константы k. y = kx также является общей формой уравнения прямой пропорциональности.
где
k — коэффициент пропорциональности.
y увеличивается с увеличением x.
y уменьшается по мере уменьшения x.
График прямой зависимости
График прямой зависимости представляет собой прямую линию с восходящим наклоном. Посмотрите на изображение, приведенное ниже. На оси x отмечены две точки, а на оси y две точки, где (x) 1 < (x) 2 и (y) 2 < (у) 2 . Если мы увеличим значение x с (x) 1 до (x) 2 , мы увидим, что значение y также увеличится с (y) 1 до (y) 2 . Таким образом, линия y=kx графически представляет прямую пропорциональность.
Прямая пропорция против обратной пропорциональности
Существует два типа пропорциональности, которые можно установить на основе отношения между двумя заданными величинами. Это прямо пропорциональные и обратно пропорциональные. Две величины прямо пропорциональны друг другу, если увеличение или уменьшение одной ведет к увеличению или уменьшению другой. С другой стороны, две величины называются обратно пропорциональными, если увеличение одной величины приводит к уменьшению другой, и наоборот. График прямой пропорциональности представляет собой прямую линию, а график обратной пропорциональности — кривую. Посмотрите на изображение, приведенное ниже, чтобы понять разницу между прямой пропорцией и обратной пропорцией.
Темы, относящиеся к прямой пропорциональности
Проверьте эти интересные статьи, связанные с концепцией прямой пропорциональности.
Доля
Константа пропорциональности
Обратно пропорциональная
Процентная доля
Примеры прямой пропорциональности
Пример 1: Предположим, что y напрямую зависит от x, и y = 36, когда x = 6. Используя формулу прямой пропорции, найдите значение y, когда x = 80?
Решение:
Используя формулу прямой пропорции, у = кх Подставьте данные значения x и y и найдите k. 36 = к × 6 к = 36/6 = 6 Уравнение прямой пропорциональности: y = 6x Теперь подставьте x = 80 и найдите y. y = 6 × 80 = 480
Ответ: значение y равно 480.
Пример 2: Если стоимость 8 фунтов яблок составляет 10 долларов, какова будет стоимость 32 фунтов яблок?
Решение:
Учитывая, что Вес яблок = 8 фунтов Стоимость яблок весом 8 фунтов = 10 долларов США. Рассмотрим вес по параметру x и стоимость по параметру y. Чтобы найти стоимость яблок весом 32 фунта, воспользуемся формулой прямой пропорциональности. у=кх 10 = k × 8 (при подстановке значений) к = 5/4 Теперь, положив значение k = 5/4 при x = 32, мы имеем .
Стоимость яблок весом 32 фунта = 5/4 × 32 у =5×8 y = 40
Ответ: Стоимость яблок весом 32 фунта составляет 40 долларов.
Пример 3: Генри получает 300 долларов за 50 часов работы. Сколько часов он проработал, если получил 258 долларов?
Решение: Пусть сумма, полученная Генри, рассматривается как y, а количество отработанных им часов как x. Подставляем данные значения x и y в формулу прямой пропорции, получаем 300 = к × 50
⇒ k=300/50 к = 6 Уравнение: y = 6x. Теперь подставьте y = 258 и найдите x. 258 = 6 × х
⇒ х = 258/6 = 43 часа Следовательно, если Генри получил 258 долларов, он проработал 43 часа.
перейти к слайдуперейти к слайдуперейти к слайду
Есть вопросы по основным математическим понятиям?
Станьте чемпионом по решению проблем, используя логику, а не правила. Узнайте, что стоит за математикой, с нашими сертифицированными экспертами
Записаться на бесплатный пробный урок
Практические вопросы по прямой пропорции
перейти к слайдуперейти к слайду
Часто задаваемые вопросы о прямой пропорциональности
Что такое прямая пропорция в математике?
Две величины называются прямо пропорциональными, если увеличение одной ведет к увеличению другой величины, и наоборот. Например, если a ∝ b, это означает, что если «a» увеличивается, «b» также увеличивается, а если «a» уменьшается, «b» также уменьшается.
Что обозначает символ ∝ в формуле прямой пропорциональности?
В формуле прямой пропорции символ пропорциональности ∝ обозначает соотношение между двумя величинами. Это выражается как y ∝ x и может быть записано в уравнении как y = kx для константы k.
Что такое прямая пропорция и обратная пропорция?
Прямая пропорция, как следует из названия, указывает на то, что увеличение одной величины также увеличивает значение другой величины, а уменьшение одной величины также уменьшает значение другой величины. В то время как обратная пропорция показывает обратную зависимость между двумя данными величинами. Это означает, что увеличение одного приведет к уменьшению значения другого количества и наоборот.
Как представить формулу прямой пропорциональности?
Формула прямой пропорциональности описывает соотношение между двумя величинами и может быть понята с помощью следующих шагов:
Определите две величины, которые различаются в данной задаче.
Определите вариант как прямой вариант.
Формула прямой пропорции: y ∝ kx.
Что такое уравнение прямой пропорциональности?
Уравнение прямой пропорциональности: y = kx, где x и y — заданные величины, а k — любая постоянная величина. Некоторыми примерами уравнений прямой пропорциональности являются y = 3x, m = 10n, 10p = q и т. д.
Как решить задачи на прямую пропорцию?
Чтобы решить задачи на прямую пропорцию, выполните следующие шаги:
Определите две величины, которые различаются в данной задаче.
Убедитесь, что изменение прямо пропорционально.
Составьте уравнение относительно y = kx и найдите значение k на основе заданных значений x и y.
Найдите неизвестное значение, подставив значения x и известную переменную.
Как показать связь между двумя величинами с помощью формулы прямой пропорциональности?
Прямо пропорциональную зависимость между двумя величинами можно определить, используя следующие ключевые моменты.
Определите две величины, данные в задаче.
Если x/y постоянны, то величины имеют прямо пропорциональную зависимость.
Когда две величины связаны друг с другом обратно пропорционально, т. е. когда увеличение одной величины приводит к уменьшению другой и наоборот, говорят, что они находятся в обратной пропорции . В обратной пропорции произведение данных двух величин равно постоянной величине. Давайте узнаем об этом подробнее в этой статье.
1.
Что такое обратная пропорция?
2.
Формула обратной пропорции
3.
График обратной пропорциональности
4.
Прямая и обратная пропорция
5.
Часто задаваемые вопросы об обратной пропорции
Что такое обратная пропорция?
Определение обратной пропорциональности гласит: «Говорят, две величины находятся в обратной пропорциональности, если увеличение одной ведет к уменьшению другой величины, а уменьшение одной ведет к увеличению другой величины». Другими словами, если произведение обеих величин, независимо от изменения их значений, равно постоянной величине, то говорят, что они находятся в обратной зависимости. Например, возьмем количество рабочих и количество дней, необходимых им для выполнения данного объема работы, как x и y соответственно.
Количество рабочих (x)
Требуемое количество дней (г)
16
3
12
4
8
6
4
12
Количество рабочих и количество дней находятся в обратно пропорциональной зависимости? Давай выясним.
Внимательно соблюдайте значения, указанные в таблице. Вы обнаружите, что для каждой строки произведение x и y одинаково. Это означает, что если есть 16 рабочих, они выполнят работу за 3 дня. Итак, здесь x × y = 16 × 3 = 48. Теперь, уменьшая количество рабочих, очевидно, что меньшее количество рабочих будет выполнять ту же работу за большее время. Но мы видим произведение x и y здесь, это 12 × 4 = 48. Опять же, для 8 рабочих за 6 дней произведение равно 48. И то же самое для 4 рабочих за 12 дней. Итак, произведение двух величин в обратной пропорции всегда равно.
Формула обратной пропорции
Формула обратной пропорции помогает установить связь между двумя обратно пропорциональными величинами. Пусть x и y — две величины, и предположим, что x уменьшается, когда y увеличивается, и наоборот. Пример: Скорость обратно пропорциональна времени. С увеличением скорости время, затрачиваемое нами на преодоление того же расстояния, уменьшается. Приняв скорость за y, а время за x, мы можем сказать, что y обратно пропорционально x и математически записывается в виде формулы обратной пропорциональности.
Формула обратной пропорциональности записывается как
y = k/x
, где
k — константа пропорциональности.
y увеличивается по мере уменьшения x.
y уменьшается с увеличением x.
Здесь символ ∝ обозначает пропорциональное соотношение между двумя величинами.
График обратной пропорциональности
График обратной пропорциональности обычно представляет собой кривую, которая изгибается к началу координат, образуя форму гиперболы. Если есть любые две случайные точки на оси x и оси y на графике обратной пропорции (x) 1 , (x) 2 , (y) 1 и (y) 2 , такие, что (x) 1 < (x) 2 и (y) 1 < (8) y) 2 , график будет выглядеть следующим образом:
Это означает, что если мы увеличим значение x от \(x_{1}\) до \(x_{2}\), значение y уменьшается от \(y_{2}\) до \(y_{1}\).
Прямая и обратная пропорция
Существует два основных вида пропорциональности — прямая пропорция и обратная пропорция. Говорят, что две переменные x и y прямо пропорциональны, когда y ∝ x (или x ∝ y). Отсюда следует, что y = kx для константы k. В то время как две переменные x и y называются обратно пропорциональными, если y ∝ 1/x (или x ∝ 1/y). Отсюда следует, что y = k/x для константы k. В прямом вариационном отношении отношение двух переменных одинаково при любых значениях, а в обратной пропорциональности произведение двух переменных равно при любых значениях. Посмотрите на изображение, приведенное ниже, чтобы наглядно понять разницу между прямой и обратной пропорцией.
Похожие темы
Проверьте эти интересные статьи, связанные с концепцией обратной пропорции.
Доля
Константа пропорциональности
Прямая пропорция
Процентная доля
Часто задаваемые вопросы об обратной пропорции
Что такое косвенная пропорция?
Косвенная или обратная пропорция представляет собой отношение между двумя величинами, при котором увеличение одной ведет к уменьшению другой, и наоборот. Это прямо противоположно прямой пропорциональности.
Что обозначает символ ∝ в формуле обратной пропорции?
В формуле обратной пропорции символ пропорциональности ∝ обозначает соотношение между двумя величинами. Он выражается как x ∝ 1/y. Отсюда следует, что x = k/y, где k — константа пропорциональности.
Как найти K в обратной пропорции?
K в обратной пропорции представляет собой константу пропорциональности, которая одинакова независимо от значений данных переменных. Чтобы найти k в обратной пропорции, найдите произведение x и y. Формула y = k/x, что дает нам k=xy.
Как представить формулу обратной пропорциональности?
Формула обратной пропорциональности изображает отношение между двумя величинами, которые можно понять по приведенной ниже формуле:
Определите две величины, которые различаются в данной задаче.
Определите, что существует обратная вариация. х ∝ 1/год
Примените формулу обратной пропорции x = k/y.
В чем разница между прямой и обратной пропорцией?
Разница между прямой и обратной пропорцией заключается в том, что прямая пропорция показывает прямую зависимость между двумя величинами, при которой увеличение одной ведет к увеличению другой величины и наоборот.
Парабола известна математикам уже очень давно, а название этой функции дал древнегреческий математик Аполлоний Пергский в III в. до н.э., изучавший свойства сечений конуса.
Также изучением параболы занимались Архимед и Папп Александрийский.
В дальнейшем разные учёные показали, что многие явления можно описать параболой, так, например, была открыта траектория движения снаряда.
Основные определения и строение параболы
Вершина параболы — это точка, находящаяся на минимальном расстоянии от директрисы параболы $d$.
Фокус $F$ параболы — это точка, через которую проходит ось симметрии параболы, перпендикулярная прямой, находящаяся на расстоянии $d$. Фокус расположен на расстоянии $\frac{p}{2}$ от вершины.
Координаты фокуса классической параболы можно определить из её уравнения.
Фокус и вершина являются основными точками, характеризующими параболу.
Параметр $p$ параболы иначе называется фокальным параметром и является расстоянием между фокусом и директрисой. 2$. Если он отрицательный, то парабола перевёрнутая по отношению к обычной и её ветви смотрят вниз, а если положительный – то её ветви смотрят вверх.
Также модуль коэффициента $a$ влияет на степень пологости (ширину) параболы, чем меньше модуль $a$, тем парабола более широкая (пологая), и чем больше модуль $a$, тем она более узкая (крутая).
Далее необходимо посмотреть на коэффициент $c$. Коэффициент $c$ обозначает смещение по оси $OY$ относительно пересечения осей координат.
Это легко проверить, если приравнять $x$ к нулю в имеющемся уравнении.
Если коэффициент $c$ — положительный, то парабола смещена вверх относительно точки $(0;0)$, а если отрицательный – то вниз.
В случае если $c=0$ — парабола проходит через точку начала координат.
Теперь можно найти вершину параболы, её координаты вычисляются по формуле:
$x = — \frac{b}{2a}$ (1).
Чтобы найти $y$, нужно подставить полученный по формуле $x$ в уравнение. {2}+y_{0}}
Парабола определяется как набор точек, расстояние до фиксированной точки (фокуса) и фиксированной прямой линии (директрисы) равны. Но, вероятно, его легче запомнить как U-образную изогнутую линию, созданную при построении квадратичного уравнения.
Многие объекты реального мира движутся по параболе. Когда вы бросаете баскетбольный мяч, траектория мяча образует параболу.
Пример
Парабола, которая направлена вверх, имеет форму буквы «U», а парабола, направленная вниз, имеет форму перевернутой буквы U.
Открывается вверх
Открывается вниз
Парабола не обязательно должна быть вертикальной, но горизонтальные параболы не являются функциями (они не проходят тест вертикальной линии). Некоторые функции, которые вы видите, будут горизонтальными параболами с ограниченными областями (). Вы узнаете больше об этом, когда будете изучать коники.
Пример
х = -у 2
х = у 2
Вершина параболы — это точка, в которой парабола меняет направление и где график наиболее изогнут. На графиках квадратичных чисел он находится в самом верху или в самом низу квадратичного уравнения. Вершина — это точка параболы на оси симметрии.
Для квадратичных уравнений стандартной формы ax 2 + bx + c ось симметрии можно найти с помощью уравнения x = . Чтобы найти координату y вершины, найдите ось симметрии и подставьте это значение x в исходное уравнение.
Пример
f(x) = x 2 + 6x + 11
a = 1; б = 6; c = 11
x = = -3
f(-3) = (-3) 2 + 6(-3) + 11
f(-3) = 9 — 18 + 11 = 2
Вершина в (-3,2)
Расчет фокуса и директрисы
Ниже приведен пример расчета фокуса и директрисы, который поможет лучше понять математическое определение параболы, приведенное выше:
Пример
Фокус — это точка, расположенная на той же прямой, что и ось симметрии, а директриса — это прямая, перпендикулярная оси симметрии. Для парабол фокус всегда находится внутри параболы, а директриса никогда не касается параболы. Поскольку вершина находится на одинаковом расстоянии от фокуса и директрисы, директриса находится прямо напротив фокуса.
Для параболы в форме вершины y = a(x — h) 2 + k фокус расположен в точке (h, k + ), а направляющая расположена в точке y = k — .
Пример
y = (x — 3) 2 + 2
h = 3 ; к = 2; a =
Фокус: (3,2 + ) = (3,4)
Директриса: y = 2 — = 0
Для горизонтальных парабол вершина x = a(y — k) 2 + h , где (h,k) — вершина. Фокусы парабол в этой форме имеют фокус, расположенный в точке (h + , k), и направляющую в точке x = h — . Ось симметрии расположена в точке y = k.
Вершинная форма параболы
Вершинная форма параболы – это другая форма квадратичной функции f(x) = ax 2 + bx + c. Вершинная форма параболы:
f(x) = a(x — h) 2 + k
А в вершинной форме параболы соответствует а в стандартной форме. Если a положительно, парабола направлена вверх. Если а отрицательно, парабола направлена вниз.
В вершинной форме (h,k) описывает вершину параболы, а парабола имеет линию симметрии x = h.
Форма вершины очень похожа на общее выражение для преобразования функций. Форма вершины значительно упрощает построение параболы, потому что она упрощает построение вершины.
Пример
f(x) = -(x — 1) 2 + 4
Из этого уравнения уже можно сказать, что вершина параболы находится в точке (1,4), а ось симметрии находится в точке x = 1. Теперь все, что нужно сделать, это подставить точки вокруг вершины, а затем построить график.
Вы можете использовать завершение квадрата для преобразования квадратного числа в стандартной форме в вершинную форму. Вы также можете преобразовать форму вершины, зная, что вершина лежит на оси симметрии.
Значение x вершины равно x = — , поэтому значение y равно y = f (-). Подставьте эти значения для h и k в уравнение формы вершины. Затем a одинаково в обеих формах, так что просто скопируйте это в уравнение вершинной формы.
Пример
f(x) = x 2 — x — 20
a = 1; б = -1; c = -20
x = h = — = — =
f() = () 2 — — 20 = — — 20 = -20
f(x) = (x — ) 2 — 20
Чтобы преобразовать параболу в вершинной форме в стандартную форму, разверните уравнение и упростите его.
Пример
f(x) = 5(x + 3) 2 — 6
f(x) = 5(x 2 + 6x + 9) — 6
f(x) = 5x 2 + 30x + 45 — 6
f(x) = 5x 2 + 30x + 39
12.3: Парабола — Математика LibreTexts
Последнее обновление
Сохранить как PDF
Идентификатор страницы
3289
OpenStax
OpenStax
Цели обучения
Графические параболы с вершинами в начале координат.
Запишите уравнения парабол в стандартной форме.
Параболы графа с вершинами не в начале координат.
Решение прикладных задач на параболах.
А вы знали, что олимпийский огонь зажигают за несколько месяцев до начала игр? Церемониальный метод зажжения пламени такой же, как и в древние времена. Церемония проходит в храме Геры в Олимпии, Греция, и уходит своими корнями в греческую мифологию, отдавая дань уважения Прометею, который украл огонь у Зевса, чтобы раздать его всем людям. Одна из одиннадцати действующих жриц помещает факел в фокус параболического зеркала (рис. \(\PageIndex{1}\)), которое фокусирует солнечные лучи и зажигает пламя.
Рисунок \(\PageIndex{1}\): Олимпийский факел завершает свое кругосветное путешествие, когда от него зажигают олимпийский котел во время церемонии открытия. (Источник: Кен Хэкман, ВВС США)
Параболические зеркала (или отражатели) способны улавливать энергию и фокусировать ее в одной точке. О преимуществах этого свойства свидетельствует обширный список параболических объектов, которые мы используем каждый день: спутниковые антенны, подвесные мосты, телескопы, микрофоны, прожекторы и автомобильные фары, и это лишь некоторые из них. Параболические отражатели также используются в устройствах альтернативной энергии, таких как солнечные плиты и водонагреватели, потому что они недороги в производстве и не требуют особого обслуживания. В этом разделе мы рассмотрим параболу и ее использование, в том числе недорогие и энергоэффективные солнечные конструкции.
Графическое изображение парабол с вершинами в начале координат
Ранее мы видели, что эллипс образуется, когда плоскость пересекает прямой круговой конус. Если плоскость параллельна ребру конуса, образуется неограниченная кривая. Эта кривая представляет собой параболу (рис. \(\PageIndex{2}\)).
Рисунок \(\PageIndex{2}\): Парабола
Подобно эллипсу и гиперболе, парабола также может быть определена набором точек на координатной плоскости. Парабола — это множество всех точек \((x,y)\) на плоскости, которые находятся на одинаковом расстоянии от фиксированной линии, называемой директриса , а фиксированная точка ( фокус ) не на директрисе.
Ранее мы узнали о вершине параболы и оси симметрии. Теперь мы расширим обсуждение, включив в него другие ключевые свойства параболы (рис. \(\PageIndex{3}\)). Обратите внимание, что ось симметрии проходит через фокус и вершину и перпендикулярна директрисе. Вершина — это середина между директрисой и фокусом. Отрезок, проходящий через фокус и параллельный директрисе, называется 9-м.0243 широкая прямая кишка . Концы широкой прямой кишки лежат на кривой. По определению расстояние d от фокуса до любой точки \(P\) на параболе равно расстоянию от \(P\) до директрисы.
Рисунок \(\PageIndex{3}\): Основные характеристики параболы
Для работы с параболами в координатной плоскости мы рассмотрим два случая: с вершиной в начале координат и с вершиной в точке . точку, отличную от исходной. Начнем с первого.
Рисунок \(\PageIndex{4}\) 92=4py\) когда ось y является осью симметрии. Эти стандартные формы приведены ниже вместе с их общими графиками и ключевыми характеристиками.
СТАНДАРТНЫЕ ФОРМЫ ПАРАБОЛ С ВЕРШИНОЙ \((0,0)\)
Таблица \(\PageIndex{1}\) и рисунок \(\PageIndex{5}\) суммируют стандартные характеристики парабол с вершиной в Происхождение.
Таблица \(\PageIndex{1}\)
Ось симметрии
2=4py\)
\((0, р)\)
\(у=-р\)
\((\pm 2p, p)\)
Рисунок \(\PageIndex{5}\): (a) Когда \(p>0\) и осью симметрии является ось x, парабола открывается вправо. (b) Когда \(p<0\) и осью симметрии является ось x, парабола открывается влево. (c) Когда \(p<0\) и осью симметрии является ось y, парабола раскрывается. (d) Когда \(p<0\) и осью симметрии является ось y, парабола открывается вниз.
Ключевыми характеристиками параболы являются ее вершина, ось симметрии, фокус, директриса и широкая прямая кишка (рис. \(\PageIndex{5}\)). Получив стандартное уравнение для параболы с центром в начале координат, мы можем легко определить ключевые особенности для построения графика параболы. Прямая называется касательной к кривой, если она пересекает кривую ровно в одной точке. Если мы нарисуем линии, касающиеся параболы в конечных точках прямой кишки , эти линии пересекутся на оси симметрии, как показано на рисунке \(\PageIndex{6}\). 92=4px\), тогда
осью симметрии является ось \(x\), \(y=0\)
набор \(4p\) равный коэффициенту \(x\) в данном уравнении, которое нужно решить для \(p\). Если \(p>0\), парабола открывается вправо. Если \(p<0\), парабола открывается влево.
используйте \(p\) для нахождения координат фокуса, \((p,0)\)
используйте \(p\) для нахождения уравнения направляющей, \(x=−p\)
используйте \(p\) для нахождения концов широкой прямой кишки, \((p,\pm 2p)\). В качестве альтернативы подставьте \(x=p\) в исходное уравнение. 92=4py\), тогда
осью симметрии является ось \(y\), \(x=0\)
набор \(4p\) равный коэффициенту \(y\) в данном уравнении, которое нужно решить для \(p\). Если \(p>0\), парабола раскрывается. Если \(p<0\), парабола раскрывается вниз.
используйте \(p\) для нахождения координат фокуса, \((0,p)\)
используйте \(p\) для нахождения уравнения направляющей, \(y=−p\)
используйте \(p\) для нахождения концов широкой прямой кишки, \((\pm 2p,p)\) 92=4px\). Таким образом, осью симметрии является ось x . Отсюда следует, что:
\(24=4p\), значит \(p=6\). Так как \(p>0\), парабола выходит вправо
координаты фокуса \((p,0)=(6,0)\)
уравнение направляющей \(x=−p=−6\)
конечные точки широкой прямой кишки имеют одинаковые координаты x в фокусе. Чтобы найти конечные точки, подставьте \(x=6\) в исходное уравнение: \((6,\pm 12)\)
92=4py\). Таким образом, осью симметрии является ось \(у\). Отсюда следует, что:
\(−6=4p\), поэтому \(p=−\dfrac{3}{2}\). Так как \(p<0\), парабола направлена вниз.
координаты фокуса \((0,p)=(0,−\dfrac{3}{2})\)
уравнение направляющей \(y=−p=\dfrac{3}{2}\)
концы широкой прямой кишки можно найти, подставив \(y=\dfrac{3}{2}\) в исходное уравнение, \((\pm 3,−\dfrac{3}{2})\)
Далее наносим фокус, директрису и 92=8у\). Определите и обозначьте фокус, направляющую и конечные точки прямой кишки latus .
Ответить
Фокус: \((0,2)\)
Директриса: \(y=−2\)
Конечные точки широкой прямой кишки: \((\pm 4,2)\). 2=4p(y−k)\) для парабол, у которых ось симметрии параллельна оси \( у\)-ось. Эти стандартные формы приведены ниже вместе с их общими графиками и ключевыми характеристиками.
СТАНДАРТНЫЕ ФОРМЫ ПАРАБОЛ С ВЕРШИНОЙ \((H, K)\)
Таблица \(\PageIndex{2}\) и рисунок \(\PageIndex{11}\) суммируют стандартные характеристики парабол с вершиной в точка \((h,k)\).
использовать данное уравнение для определения \(h\) и \(k\) для вершины, \((h,k)\)
использовать значение \(k\) для определения оси симметрии, \(y=k\)
набор \(4p\) равный коэффициенту \((x−h)\) в данном уравнении для решения \(p\). Если \(p>0\), парабола выходит вправо. Если \(p<0\), парабола открывается влево.
используйте \(h\), \(k\) и \(p\) , чтобы найти координаты фокуса, \((h+p, k)\) 92=4p(y−k)\), тогда:
использовать данное уравнение для определения \(h\) и \(k\) для вершины, \((h,k)\)
использовать значение \(h\) для определения оси симметрии, \(x=h\)
набор \(4p\) равный коэффициенту \((y−k)\) в данном уравнении для решения \(p\). Если \(p>0\), парабола раскрывается. Если \(p<0\), парабола раскрывается вниз.
используйте \(h\), \(k\) и \(p\) , чтобы найти координаты фокуса, \((h, k+p)\)
используйте \(k\) и \(p\) для нахождения уравнения направляющей, \(y=k−p\)
используйте \(h\), \(k\) и \(p\) , чтобы найти концы широкой прямой кишки, \((h\pm 2p, k+p)\)
Постройте вершину, ось симметрии, фокус, директрису и прямую кишку, а затем нарисуйте плавную кривую, чтобы сформировать параболу.
Пример \(\PageIndex{4}\): построение параболы с вершиной \((h, k)\) и осью симметрии, параллельной оси \(x\) 92=4p(x−h)\). Таким образом, ось симметрии параллельна оси \(х\). Отсюда следует, что:
вершина равна \((h,k)=(−3,1)\)
ось симметрии \(y=k=1\)
\(−16=4p\), поэтому \(p=−4\). Так как \(p<0\), парабола открывается влево.
концы широкой прямой кишки равны \((h+p,k\pm 2p)=(−3+(−4),1\pm 2(−4))\), или \((−7,− 7)\) и \((−7,92=4(х-8)\). Определите и обозначьте вершину, ось симметрии, фокус, направляющую и конечные точки прямой кишки latus .
Ответить
Вершина: \((8,−1)\)
Ось симметрии: \(y=−1\)
Фокус: \((9,−1)\)
Директриса: \(x=7\)
Концы широкой прямой кишки : \((9,−3)\) и \((9,1)\).
Рисунок \(\PageIndex{13}\)
92&= 4⋅7⋅(y+8) \end{align*}\]
Отсюда следует, что:
вершина \((h,k)=(4,−8)\)
ось симметрии \(x=h=4\)
так как \(p=7\), \(p>0\) и таким образом парабола раскрывается
уравнение направляющей имеет вид \(y=k−p=−8−7=−15\)
концы широкой прямой кишки равны \((h\pm 2p,k+p)=(4\pm 2(7),−8+7)\) или \((−10,−1)\) и \((18,−1)\) 92=−20(y−3)\). Определите и обозначьте вершину, ось симметрии, фокус, направляющую и конечные точки прямой кишки latus .
Ответить
Вершина: \((−2,3)\)
Ось симметрии: \(x=−2\)
Фокус: \((−2,−2)\)
Директриса: \(y=8\)
Конечные точки широкой прямой кишки : \((−12,−2)\) и \((8,−2)\).
Рисунок \(\PageIndex{15}\)
Решение прикладных задач с использованием парабол
Как мы упоминали в начале раздела, параболы используются для проектирования многих объектов, которые мы используем каждый день, таких как телескопы, подвесные мосты, микрофоны и радиолокационное оборудование. Параболические зеркала, такие как то, которое использовалось для зажигания олимпийского огня, обладают уникальными отражающими свойствами. Когда лучи света, параллельные оси симметрии параболы, направляются на любую поверхность зеркала, свет отражается прямо в фокус (рис. \(\PageIndex{16}\)). Вот почему олимпийский факел зажигается, когда его держат в фокусе параболического зеркала.
Параболические зеркала способны фокусировать солнечную энергию в одной точке, повышая температуру на сотни градусов за считанные секунды. Таким образом, параболические зеркала используются во многих недорогих, энергоэффективных солнечных продуктах, таких как солнечные плиты, солнечные обогреватели и даже дорожные разжигатели огня.
Пример \(\PageIndex{6}\): решение прикладных задач, связанных с параболами
На рисунке \(\PageIndex{17}\) показано поперечное сечение конструкции переносного солнечного пожарного пускателя. Солнечные лучи отражаются от параболического зеркала к объекту, прикрепленному к воспламенителю. 2=4py\), где \(p>0\). Воспламенитель, который является фокусом, находится на \(1,7\) дюймах выше вершины тарелки. Таким образом, мы имеем \(p=1,7\). 92&=6.8y\qquad \text{Подставить } 2.25 \text{ вместо } x\\ y&\ приблизительно 0.74\qquad \text{Решить для } y \end{align*}\]
Блюдо примерно \(0.74 \) дюймов глубиной.
Упражнение \(\PageIndex{6}\)
Солнечные плиты размером с балкон были разработаны для семей, живущих в Индии. Верх блюда имеет диаметр \(1600\) мм. Солнечные лучи отражаются от параболического зеркала в сторону «варки», расположенной \(320\) мм от основания.
Найдите уравнение, моделирующее поперечное сечение солнечной плиты. Предположим, что вершина параболического зеркала является началом координатной плоскости и что парабола выходит вправо (т. е. имеет 92=4р(у-к)\)
Ключевые понятия
Парабола — это множество всех точек \((x,y)\) на плоскости, находящихся на одинаковом расстоянии от фиксированной прямой, называемой направляющей, и фиксированной точки (директрисы). фокус) не на директрисе.
Для построения графика параболы можно использовать стандартную форму параболы с вершиной \((0,0)\) и осью x в качестве оси симметрии. Если \(p>0\), парабола открывается вправо. Если \(p<0\), парабола открывается влево. См. пример \(\PageIndex{1}\).
Стандартная форма параболы с вершиной \((0,0)\) и осью y в качестве оси симметрии может быть использована для построения графика параболы. Если \(p>0\), парабола раскрывается. Если \(p<0\), парабола раскрывается вниз. См. пример \(\PageIndex{2}\).
Зная фокус и направляющую параболы, мы можем записать ее уравнение в стандартной форме. См. пример \(\PageIndex{3}\).
Для построения параболы можно использовать стандартную форму параболы с вершиной \((h,k)\) и осью симметрии, параллельной оси \(x\). Если \(p>0\), парабола открывается вправо. Если \(p<0\), парабола открывается влево. См. пример \(\PageIndex{4}\).
Для построения параболы можно использовать стандартную форму параболы с вершиной \((h,k)\) и осью симметрии, параллельной оси \(y\) .
Поиск материала «Задачник по высшей математике, Шипачев В.С., 2003» для чтения, скачивания и покупки
Ниже показаны результаты поиска поисковой системы Яндекс. В результатах могут быть показаны как эта книга, так и похожие на нее по названию или автору.
Search results:
B.C.ШипачевЗАДАЧНИКПОВЫСШЕЙMATEMATИКЕ Издание…
B.C.Шипачев ЗАДАЧНИК ПО ВЫСШЕЙ MATEMATИКЕ Издание третье, стереотипное.
yagu.s-vfu.ru
Задачникповысшейматематике — Шипачев В.С.
Пособие написано в соответствии с программой по высшей математике для вузов. Содержит задачи и примеры по следующим важнейшим разделам: теория пределов, аналитическая геометрия на плоскости и в пространстве, дифференциальное и интегральное исчисления функций одной и нескольких переменных, высшая алгебра, ряды и дифференциальные уравнения. Приведены основные теоретические сведения, решения типовых примеров и задач, задачи и упражнения для самостоятельной работы с ответами, решениями и указаниями.
В учебнике излагается материал по важным разделам высшей математики, таким, как сведения из теории множеств и теории вещественного числа, теория пределов последовательностей и функций, основы дифференциального и интегрального исчисления функций одной и нескольких переменных, элементы аналитической геометрии на плоскости и в пространстве, некоторые вопросы линейной и векторной алгебры, теории рядов и теории обыкновенных дифференциальных уравнений.
11klasov.net
Задачникповысшейматематике. Шипачев В.С.
Пособие написано в соответствии с программой по высшей математике для вузов. Содержит задачи и примеры по следующим важнейшим разделам: теория пределов, аналитическая геометрия на плоскости и в пространстве, дифференциальное и интегральное исчисления функций одной и нескольких переменных, высшая алгебра, ряды и дифференциальные уравнения. Приведены основные теоретические сведения, решения типовых примеров и задач, задачи и упражнения для самостоятельной работы с ответами, решениями и указаниями.
Шипачев В. С., 2003, Файл, скачать. Пособие написано в соответствии с программой по высшей математике для вузов. Содержит задачи и примеры по следующим важнейшим раздела…
mathprofi.com
Шипачев В.С. Задачникповысшейматематике
Задачник содержит 6 тысяч задач по всем основным разделам высшей математики. Наряду с учебными и совершенно стандартными здесь содержится немало весьма тонких и непростых задач. Опытный преподаватель сумеет найти в книге задания как д…
Методические указания к выполнению контрольных заданий по высшей математике Ульяновск: УлГТУ, 1998, — 32 с. Методические указания написаны в соответствии с программой по высшей математике для студентов ускоренной формы обучения.
www.studmed.ru
B.C.Шипачев ВЫСШ. MATEMATHK ИЗДАНИЕ ЧЕТВЕРТОЕ…
Настоящий учебник написан автором на основе многолетнего опыта чтения лекций и ведения практических занятий по высшей математике на нематематических факультетах в Московском госу- дарственном университете и отвечает всем необходимым требова- ниям, предъявляемым к современному математическому образова- нию. В учебнике излагаются основы высшей математики, поэтому он может быть использован как в университетах, так. высших технических учебных заведениях, а также в гимназиях, лицеях и кол- леджах
lib.maupfib.kg
В.С Шипачев. Задачникповысшейматематике, издание 3, 2003г
Книга написана согласно программе по дисциплине Высшая Математика для технических университетов. Здесь представлены задачи и упражнения по: — теории пределов; — аналитической геометрии; — дифференциалам и интегралам, — высшей алгебре. — рядам и дифференциальным уравнениям. Также в книге можно ознакомиться с теорией, решением стандартных примеров, задачами и упражнениями для самостоятельных занятий, доступны ответы, решения и указания.
www.student-you.ru
Задачникповысшейматематике — Шипачев В. С. — Скачать…
Задачник по высшей математике. Математика. Автор: Шипачев В. С. Год: 2003.
События книги. Формат: DJVU (9866 Kb). СКАЧАТЬ. Currently 0/5. Рейтинг: 0/5 (Всего голосов: 0). Аннотация. Задачник по высшей математике.
www.rulit.me
Высшаяматематика. Шипачев В.С.
Высшая математика. Шипачев В.С. В учебнике излагаются элементы теории множеств и вещественных чисел, числовые последовательности и теория пределов, аналитическая геометрия на плоскости и в пространстве, основы дифференциального и интегрального исчислений функций одной и нескольких переменных, элементы высшей алгебры, теория рядов и обыкновенные дифференциальные уравнения. Теоретический материал иллюстрируется большим количеством примеров.
11klasov.net
Задачникповысшейматематике. Шипачев В.С.
Пособие написано в соответствии с программой по высшей математике для вузов. Содержит задачи и примеры по следующим важнейшим разделам: теория пределов, аналитическая геометрия на плоскости и в пространстве, дифференциальное и интегральное исчисления функций одной и нескольких переменных, высшая алгебра, ряды и дифференциальные уравнения. Приведены основные теоретические сведения, решения типовых примеров и задач, задачи и упражнения для самостоятельной работы с ответами, решениями и указаниями.
Автор: Шипачев В.С. Название: Задачник по высшей математике Формат: PDF Размер: 9,86 Мб Язык: Русский. Скачать по прямой ссылке. Пособие написано в соответствии с программой по высшей математике для вузов. Содержит задачи и примеры по следующим важнейшим разделам: теория пределов, аналитическая геометрия на плоскости и в пространстве, дифференциальное и интегральное исчисления функций одной и нескольких переменных, высшая алгебра, ряды и дифференциальные уравнения.
www.psyoffice.ru
122729: Шипачев В.С. Задачникповысшейматематике
Нажмите на ссылку Шипачев_Задачник.pdf, чтобы просмотреть файл. Пропустить Версия для слабовидящих.
Письменный Д.Т. Конспект лекций по высшей математике. Высшая математика в упражнениях и задачах. часть 1.
yagu.s-vfu.ru
Шипачев В.С. Задачникповысшейматематике
Пособие написано в соответствии с программой по высшей математике для вузов. Содержит задачи и примеры по следующим важнейшим разделам: теория пределов, аналитическая геометрия на плоскости и в пространстве, дифференциальное и интегральное исчисления функций одной и нескольких переменных, высшая алгебра, ряды и дифференциальные уравнения. Приведены основные теоретические сведения, решения типовых примеров и задач, задачи и упражнения для самостоятельной работы с ответами, решениями и указаниями.
язык пособия: чисто русский. описание на украинском языке: завантаження. . . Название: Задачникповысшейматематике Язык: только русский. Автор: Ш. Вы здесь: Главная сайта ГДЗ Сборник задач Пособие ШипачевЗадачникповысшейматематике бесплатно.
Пособие написано в соответствии с программой по высшей математике для вузов. Содержит задачи и примеры по следующим важнейшим разделам: теория пределов, аналитическая геометрия на плоскости и в пространстве, дифференциальное и интегральное исчисления функций одной и нескольких переменных, высшая алгебра, ряды и дифференциальные уравнения. Приведены основные теоретические сведения, решения типовых примеров и задач, задачи и упражнения для самостоятельной работы с ответами, решениями и указаниями.
Теория + множество примеров. Изложены элементы теории множеств и вещественных чисел, числовые последовательности и теория пределов, аналитическая геометрия на плоскости и в пространстве, основы дифференциального и интегрального исчислений функций одной и нескольких переменных, элементы высшей алгебры, теория рядов и обыкновенные дифференциальные уравнения. Теоретический материал иллюстрируется большим количеством примеров. Для студентов высших учебных заведений.
Пособие написано в соответствии с программой по высшей математике для вузов. Содержит задачи и примеры по следующим важнейшим разделам: теория пределов, аналитическая геометрия на плоскости и в пространстве, дифференциальное и интегральное исчисления функций одной и нескольких переменных, высшая алгебра, ряды и дифференциальные уравнения. Приведены основные теоретические сведения, решения типовых примеров и задач, задачи и упражнения для самостоятельной работы с ответами, решениями и указаниями.
cdnpdf.com
Шипачев В.С. Задачникповысшей математке
Пособие написано в соответствии с программой по высшей математике для вузов. Содержит задачи и примеры по следующим важнейшим разделам: теория пределов, аналитическая геометрия на плоскости и в пространстве, дифференциальное и интегральное исчисления функций одной и нескольких переменных, высшая алгебра, ряды и дифференциальные уравнения. Приведены основные теоретические сведения, решения типовых примеров и задач, задачи и упражнения для самостоя…
Пособие написано в соответствии с программой по высшей математике для вузов. Содержит задачи и примеры по следующим важнейшим разделам: теория пределов, аналитическая геометрия на плоскости и в пространстве, дифференциальное и интегральное исчисления функций одной и нескольких переменных, высшая алгебра, ряды и дифференциальные уравнения. Приведены основные теоретические сведения, решения типовых примеров и задач, задачи и упражнения для самостоятельной работы с ответами, решениями и указаниями.
school-textbook.com
Основы высшейматематики — Шипачев В.С.
Основы высшей математики — Шипачев В. С. В пособии изложен общий курс математики для студентов вузов. Основная особенность книги — сочетание необходимого теоретического материала с широким использованием методов решения основных типов задач по всем разделам курса. Пособие отличается высоким уровнем строгости и методической продуманностью изложения, точностью формулировок основных понятий и теорем, краткостью и доступностью доказательств.
11klasov.net
Задачникповысшейматематике | Шипачев В.С. | скачать книгу
Шипачев В.С. Скачать книгу бесплатно (djvu, 9.63 Mb) | Читать «Задачник по высшей математике».
bookscat.org
Шипачев В.С. Высшаяматематика
Книга содержит упражнения по курсу «Высшая математика» для студентов I и II курсов естественных факультетов государственных университетов, где на преподавание математики отводится до 200 учебных часов. Некоторые задачи, включенные в сборник заимствованы из различных распространенных сборников задач по высшей математике, в частности, из «Сборнике задач и упражнений по математическому анализу» Б. П. Демидовича и «Сборника задач по высшей математике…
www.studmed.ru
Высшаяматематика. | ВКонтакте
*** Файлы DJVU открываются с помощью программы WinDjView. Скачать ее можно тут: http
Высшая математика в упражнениях и задачах. (с ответами) Часть 1 + Часть 2.
Шипачев В.С. Высшая математика. Учебник. Издание четвертое, стереотипное 1998, 479 с.
Шипачев В.С. Задачник по высшей математике. Издание третье, стереотипное 2003 г.
vk.com
B.C.ШипачевВЫСШАЯ MATEMATT B.C. Шипаче Matemati
B.C.Шипачев. ВЫСШАЯ MATEMATT.
ББК 22.11. Рецензент: д-р пед. наук, проф. А.Г. Мордкович. Шипачев, В.С. . Высшая математиқа: Учеб. для вузов/В.С.
www.nntu.ru
Шипачев В.С. Задачникповысшейматематике
Лунгу К.Н., Письменный Д.Т., Федин С.Н., Шевченко Ю.А. Сборник задач по высшей математике. 1 курс (Документ).
Эвнин А.Ю. Задачник по дискретной математике (Документ). Ляшко И.И., Боярчук А.К., Гай Я.Г., Головач Г.П. Справочное пособие по высшей математике.
nashaucheba.ru
«Задачникповысшейматематике» — читать…
«Задачник по высшей математике» (Шипачев Виктор Семенович), НИЦ ИНФРА-М. В электронно-библиотечной системе Znanium.com.
Пособие написано в соответствии с программой по высшей математике для вузов. Содержит задачи и примеры по следующим важнейшим разделам: теория пределов, аналитическая геометрия на плоскости и в пространстве, дифференциальное и интегральное исчисления функций одной и нескольких переменных, высшая алгебра, ряды и дифференциальные уравнения.
Znanium.com
Высшаяматематика 8-е изд., пер. и доп. Учебник и практикум…
Цель данного учебного пособия показать в простом изложении как четкость и конкретность, так и доступность для широкого круга читателей основных понятий и теорем высшей математики. В книге имеется большое количество подробно решенных типовых примеров и задач, поясняющих теоретический материал и способствующих более глубокому его пониманию. Полная версия книги. Высшая математика 8 е изд., пер. и доп. Учебник и практикум.
Пособие написано в соответствии с программой по высшей математике для вузов. Содержит задачи и примеры по следующим важнейшим разделам: теория пределов, аналитическая геометрия на плоскости и в пространстве, дифференциальное и интегральное исчисления функций одной и нескольких переменных, высшая алгебра, ряды и дифференциальные уравнения.
*С методикой построения графиков функций можно познакомиться в кн.: Шипачев В. С. Основы высшей математики.
ru.b-ok.cc
Читать ШипачевВысшаяматематика онлайн
Название: Шипачев Высшая математика. Язык: только русский. Автор: Шипачев В. С. Размер: 2. 9 Страниц: 479 Год: 2005 Читать Шипачев Высшая математика о.
ВС математика. ГИА: ЕГЭ/ОГЭ. как скачать.
gdz-online.ws
Шипачев В.С. Задачникповысшейматематике
Лунгу К.Н., Письменный Д.Т., Федин С.Н., Шевченко Ю.А. Сборник задач по высшей математике. 1 курс (Документ).
Эвнин А.Ю. Задачник по дискретной математике (Документ). Ляшко И.И., Боярчук А.К., Гай Я.Г., Головач Г.П. Справочное пособие по высшей математике.
nashaucheba.ru
Книга Задачникповысшейматематике (Шипачев. ..)
Читать онлайн книгу Задачник по высшей математике автора Шипачев В.С.
bookree.org
На данной странице Вы можете найти лучшие результаты поиска для чтения, скачивания и покупки на интернет сайтах материалов, документов, бумажных и электронных книг и файлов похожих на материал «Задачник по высшей математике, Шипачев В.С., 2003»
Для формирования результатов поиска документов использован сервис Яндекс.XML.
Нашлось 9 млн ответов. Показаны первые 32 результата(ов).
Дата генерации страницы:
404 Cтраница не найдена
Размер:
AAA
Изображения
Вкл.
Выкл.
Обычная версия сайта
К сожалению запрашиваемая страница не найдена.
Но вы можете воспользоваться поиском или картой сайта ниже
Университет
Майкопский государственный технологический университет – один из ведущих вузов юга России.
История университета
Анонсы
Объявления
Медиа
Представителям СМИ
Газета «Технолог»
О нас пишут
Ректорат
Структура
Филиал
Политехнический колледж
Медицинский институт
Лечебный факультет
Педиатрический факультет
Фармацевтический факультет
Стоматологический факультет
Факультет послевузовского профессионального образования
Факультеты
Кафедры
Ученый совет
Дополнительное профессиональное образование
Бережливый вуз – МГТУ
Новости
Объявления
Лист проблем
Лист предложений (Кайдзен)
Реализуемые проекты
Архив проектов
Фабрика процессов
Рабочая группа «Бережливый вуз-МГТУ»
Вакансии
Профсоюз
Противодействие терроризму и экстремизму
Противодействие коррупции
WorldSkills в МГТУ
Научная библиотека МГТУ
Реквизиты и контакты
Работа МГТУ в условиях предотвращения COVID-19
Документы, регламентирующие образовательную деятельность
Система менеджмента качества университета
Опрос в целях выявления мнения граждан о качестве условий оказания образовательных услуг
Абитуриентам
Подача документов онлайн
Абитуриенту 2023
Экран приёма 2022
Иностранным абитуриентам
Международная деятельность
Общие сведения
Кафедры
Новости
Центр международного образования
Академическая мобильность и международное сотрудничество
Академическая мобильность и фонды
Индивидуальная мобильность студентов и аспирантов
Как стать участником программ академической мобильности
Дни открытых дверей в МГТУ
Подготовительные курсы
Подготовительное отделение
Курсы для выпускников СПО
Курсы подготовки к сдаче ОГЭ и ЕГЭ
Онлайн-курсы для подготовки к экзаменам
Подготовка школьников к участию в олимпиадах
Малая технологическая академия
Профильный класс
Индивидуальный проект
Кружковое движение юных технологов
Олимпиады, конкурсы, фестивали
Архив
Веб-консультации для абитуриентов
Олимпиады для школьников
Отборочный этап
Заключительный этап
Итоги олимпиад
Профориентационная работа
Стоимость обучения
Студентам
Студенческая жизнь
Стипендии
Организация НИРС в МГТУ
Студенческое научное общество
Студенческие научные мероприятия
Конкурсы
Команда Enactus МГТУ
Академическая мобильность и международное сотрудничество
Образовательные программы
Подготовка кадров высшей квалификации
Аспирантура
Ординатура
Расписание занятий
Расписание звонков
Онлайн-сервисы
Социальная поддержка студентов
Общежития
Трудоустройство обучающихся и выпускников
Информация о Центре
Цели и задачи центра
Контактная информация
Положение о центре
Договоры о сотрудничестве с организациями, предприятиями
Партнеры
Работодателям
Размещение вакансий
Ярмарки Вакансий
Студентам и выпускникам
Вакансии
Стажировки
Карьерные мероприятия
Карьерные сайты
Сегодня Современный Государственный Университет — это один из самых крупных многопрофильных вузов Поволжья, обеспечивающий формирование интеллектуального потенциала и способствующий социально-экономическому развитию региона.
HeadHunter
Работа в России
Факультетус
Карьерные возможности для лиц с инвалидностью и ОВЗ
Трудоустройство иностранных студентов
Обеспеченность ПО
Инклюзивное образование
Условия обучения лиц с ограниченными возможностями
Доступная среда
Ассоциация выпускников МГТУ
Перевод из другого вуза
Вакантные места для перевода
Наука и инновации
Научная инфраструктура
Проректор по научной работе и инновационному развитию
Научно-технический совет
Управление научной деятельностью
Управление аспирантуры и докторантуры
Точка кипения МГТУ
О Точке кипения МГТУ
Руководитель и сотрудники
Документы
Контакты
Центр коллективного пользования
Центр народной дипломатии и межкультурных коммуникаций
Студенческое научное общество
Новости
Научные издания
Научный журнал «Новые технологии»
Научный журнал «Вестник МГТУ»
Научный журнал «Актуальные вопросы науки и образования»
Публикационная активность
Конкурсы, гранты
Научные направления и результаты научно-исследовательской деятельности
Основные научные направления университета
Отчет о научно-исследовательской деятельности в университете
Результативность научных исследований и разработок МГТУ
Финансируемые научно-исследовательские работы
Объекты интеллектуальной собственности МГТУ
Результативность научной деятельности организаций, подведомственных Минобрнауки России (Анкеты по референтным группам)
Студенческое научное общество
Инновационная инфраструктура
Федеральная инновационная площадка
Проблемные научно-исследовательские лаборатории
Научно-исследовательская лаборатория «Совершенствование системы управления региональной экономикой»
Научно-исследовательская лаборатория проблем развития региональной экономики
Научно-исследовательская лаборатория организации и технологии защиты информации
Научно-исследовательская лаборатория функциональной диагностики (НИЛФД) лечебного факультета медицинского института ФГБОУ ВПО «МГТУ»
Научно-исследовательская лаборатория «Инновационных проектов и нанотехнологий»
Научно-техническая и опытно-экспериментальная база
Центр коллективного пользования
Конференции
Международная научно-практическая конференция «Актуальные вопросы науки и образования»
VI Международная научно-практическая онлайн-конференция
Международная деятельность
Иностранным студентам
Международные партнеры
Академические обмены, иностранные преподаватели
Академическая мобильность и фонды
Индивидуальная мобильность студентов и аспирантов
Как стать участником программ академической мобильности
Объявления
Факультет международного образования
Информация о факультете
Международная деятельность
Кафедры
Кафедра русского языка как иностранного
Кафедра иностранных языков
Центр Международного образования
Академическая мобильность
Контактная информация
Контактная информация факультета международного образования
Сведения об образовательной организации
мягкий вопрос — Как читать книгу по математике?
спросил
Изменено
1 год, 1 месяц назад
Просмотрено
62к раз
$\begingroup$
Как получилось, что вы читаете книгу по математике?
Ведете ли вы тетрадь определений? А теоремы?
Вы делаете все упражнения? Сосредоточиться на доказательствах или игнорировать их?
Я читал Мункреса, Артина, Халмоса и т. д., но обычно немного теряюсь где-то в середине. Кроме того, о том, как быстро вы должны читать это? Нужен любой совет, я только что достиг высшего уровня дивизиона.
мягкий вопрос
$\endgroup$
12
$\begingroup$
Этот метод хорошо сработал для меня (но то, что хорошо работает для одного человека, не обязательно будет работать для всех). Беру в несколько заходов:
Чтение 0: Не читайте книгу, не читайте статью в Википедии и не спрашивайте друга, о чем идет речь. Узнайте о важных вопросах, заданных в предмете, и основах теорем, которые на них отвечают. Часто наиболее важными идеями являются те, которые можно изложить лаконично, поэтому вы должны быть в состоянии запомнить их, когда будете читать книгу.
Прочтите 1: Пусть ваш взгляд перескакивает с определения на лемму и теорему, не читая промежуточных доказательств, если только что-то не привлекает ваше внимание или не беспокоит. Если в книге есть упражнения, посмотрите, сможете ли вы выполнить первое из каждой главы или раздела по ходу дела.
Чтение 2: Прочитайте книгу, но на этот раз прочтите корректуру. Но не волнуйтесь, если вы не получите все детали. Если какой-то логический переход не имеет полного смысла, не стесняйтесь игнорировать его по своему усмотрению, если вы понимаете общий ход рассуждений.
Прочтите 3: Прочтите сквозь призму скептика. Проработайте все корректуры гребешком с мелкими зубьями и задайте себе каждый вопрос, который придет вам в голову. Вы никогда не должны спрашивать себя, «почему» вы доказываете то, что вы доказываете в данный момент, но у вас есть шанс уточнить детали.
Этот подход хорошо подходит для многих учебников по математике, которые, кажется, написаны для чтения людьми, которые уже понимают предмет. Большинство «классических» учебников названы таковыми потому, что они всеобъемлющи или хорошо организованы, а не потому, что они хорошо излагают сложные абстрактные идеи для непосвященных.
(Шаги 1-3 основаны на трехэтапном эвристическом методе написания доказательств: убедить себя, убедить друга, убедить скептика)
$\endgroup$
8
$\begingroup$
От Сахарона Шелаха, «Теория классификации и количество неизоморфных моделей»; цитируется в Just and Weese, «Discovering Modern Set Theory I»:
Теперь мы объясним, как читать книгу. Правильный способ — положить
на вашем столе днем, под подушкой ночью, посвящая
себя к чтению и решению упражнений до тех пор, пока не усвоите их
сердце. К сожалению, я подозреваю, что читатель ищет совета по
как не читать, т.е. что пропускать, а еще лучше, как только читать
некоторые отдельные моменты.
Извините… Я просто люблю эту цитату.
$\endgroup$
1
$\begingroup$
Случайно зашел на этот вопрос-обсуждение только сегодня.
Тема нескольких ответов и комментариев, что много чтений в разных стилях лучше, я бы второй, по крайней мере, до определенного момента.
Я бы не согласился со всеми советами отказываться двигаться вперед без «предварительного овладения всеми деталями»… конечно, почти для всех учебников и даже многих монографий более высокого уровня. Причина в том, что учебники в настоящее время, похоже, имеют стиль, в котором подробно описываются все возможные детали во имя «строгости», а также довольно невербально. То есть относительное значение разных деталей/лемм/чего бы то ни было вовсе не очерчено. Поскольку по крайней мере 90 процентов деталей вовсе не «опасны» и даже не слишком удивительны или поучительны, это приводит к полной неэффективности. Учебники в 10 раз длиннее, чем нужно, а важные моменты теряются в 10-кратно большей мешанине суетливых деталей. Ужасный.
Единственный серьезный подход к тому, чтобы не утонуть в ложной строгости и суетливых деталях, состоит в том, чтобы сделать хотя бы один просмотр материала, чтобы увидеть основные моменты, сюжетные арки более высокого уровня. Это придает согласованность деталям более низкого уровня. «Задним числом» своего рода.
В частности, «учения» — чрезвычайно изменчивый вопрос. Современные учебники «должны» включать множество упражнений, чтобы угодить издателям и оправдать другие ожидания. Таким образом, человек имеет скудное представление о природе данного! Кроме того, во многих текстах можно наблюдать раскол между «теоретическим» характером главы и «решающим проблемы» характером упражнений, с недостатком прототипов в самой главе для поддержания своего рода ошибочной «чистоты».
Итак: различать относительную значимость деталей и видеть общую сюжетную арку — вот самые важные вещи, которые нужно развивать. Некоторое знакомство с деталями более низкого уровня, безусловно, полезно, но предполагаемое «предельное» значение деталей низкого уровня в основном является артефактом того, как математика преподается в школе.
$\endgroup$
2
$\begingroup$
Позвольте мне поделиться с вами первым абзацем предисловия к моему учебнику по математике:
Книга по математике требует другого типа чтения, чем роман или рассказ. Каждое предложение в учебнике по математике полно информации и логически связано с соседними предложениями. Вы должны внимательно прочитать предложения и подумать над их значением. Читая, помните, что математика строится сама на себе. Обязательно читайте с карандашом и бумагой: делайте расчеты, рисуйте эскизы и делайте заметки.
$\endgroup$
5
$\begingroup$
Это действительно зависит от книги. Будут определенные книги, которые вам не понравятся или с которыми вы не сможете справиться. Другие книги, которые вы пролистаете и сразу получите удовольствие.
У меня много книг, но я склонен считать, что большинство книг по математике написаны скорее лаконично, чем интересно (под этим я подразумеваю, что читатели должны сами находить интересные фрагменты, а не автор, передающий свой интерес). Я не поклонник этого стиля, но вы должны заметить, что вы почти всегда можете найти дополнительные материалы, чтобы заполнить эти пробелы.
Иногда для получения надежных знаний в определенной области требуется сочетание ресурсов.
$\endgroup$
1
$\begingroup$
Нельзя слишком сильно подчеркнуть, что длинный математический аргумент можно полностью понять
при первом чтении только тогда, когда оно действительно очень элементарно по отношению к математическому пониманию читателя.
знания. Если кому-то нужна только суть, он может прочитать такой материал только один раз; а иначе он
должен ожидать, чтобы прочитать его по крайней мере еще раз. Серьезное чтение математики лучше всего делать сидящим болтом
прямо на жестком стуле за письменным столом. Карандаш и бумага почти незаменимы; ибо всегда есть
цифры должны быть набросаны, а шаги аргумента должны быть проверены расчетом.
Л. Дж. Сэвидж
$\endgroup$
0
$\begingroup$
Как сказал мой профессор: » Смотри, пока не поймешь »
Мой подход заключается в том, чтобы сначала просмотреть каждую главу, чтобы знать, что я собираюсь делать. Я считаю, что это психологически полезно, и это поддерживает мою мотивацию и желание получить следующий «блок знаний» — усталость плоха и контрпродуктивна. Затем я читаю каждый раздел медленно и дважды, чтобы получить уверенность в атакующих упражнениях. Я стараюсь уделить серьезное внимание каждой задаче, но я не трачу дни на набор задач (максимум 1-2 дня, в зависимости от книги).
Хороший способ сохранить важное положительное отношение к обучению — помнить: » Этот материал может показаться трудным, но если я буду читать его медленно, то я все пойму. »
Когда я закончу книгу, то я быстро прочитаю ее снова, чтобы освежить память. Тогда я должен быть в порядке с материалом.
$\endgroup$
0
$\begingroup$
Когда вы читаете книгу,
(Прекрасная книга!)
Прочтите первую часть и посмотрите, как выглядит макет.
Если некоторые разделы являются факультативными
Тогда не читайте их, будьте избирательны
Со своими книгами книгами книгами книгами книгами книгами
Будьте избирательны, и вы будете плыть дальше по своим книгам.
Когда вы приходите на занятия
В тетрадь
Держите блокнот и карандаш рядом с тетрадью.
Решайте каждую интересную задачу,
Все легкие и некоторые сложные
В ваших книгах книги книги книги книги книги
Не забывайте упражнения в вашей книге.
$\endgroup$
$\begingroup$
Я просто добавлю, что чтение текста по математике может быть очень пугающим поначалу (особенно для математики на уровне степени). Вы не можете ожидать, что поймете все, что читаете с первого раза. Это также будет зависеть от вашего уровня математических знаний и сложности текста. Но прежде чем читать, вам нужно установить, что вы собираетесь получить от чтения. Вы читаете, чтобы сдать экзамен или для самореализации?
Следовательно, я склонен сначала читать, чтобы получить общее представление о предмете, а затем мои последующие чтения будут заключаться в том, чтобы понять более мелкие детали. Хорошо понимать доказательства, и делать заметки тоже может быть полезно. Некоторые доказательства могут быть слишком сложными, чтобы их можно было полностью понять за одно чтение. Вы по-настоящему понимаете математику только в том случае, если можете решать задачи в соответствующих областях — чтение математики может быть ложно обнадеживающим, если вы не кладете ручку на бумагу.
Наткнулся на несколько полезных советов по чтению математики, которыми поделюсь здесь (https://www.people.vcu.edu/~dcranston/49).0/handouts/math-read.html):
Попытайтесь понять общую картину: «Чтение по математике — это вовсе не линейный опыт… Понимание текста требует перекрестных ссылок, сканирования, пауз и повторного просмотра»
Не будьте пассивным читателем: «Трехстрочное доказательство тонкой теоремы — квинтэссенция многолетней деятельности. Чтение математики… предполагает возвращение к мышлению, которое вошло в написание»
Не читай слишком быстро
Сделайте идею своей собственной: проследите идею до ее истоков и заново откройте ее для себя
Познай себя: убедись, что ты целевая аудитория книги
Я бы порекомендовал книгу Лары Алкок, которая показалась мне действительно полезной: https://www.amazon.co.uk/Study-Mathematics-Degree-Lara-Alcock/dp/0199661324. Вся глава 7 посвящена чтению математики.
Она также написала о том, «Как думать об анализе», что было еще полезнее: https://www.amazon.co.uk/Think-About-Analysis-Lara-Alcock/dp/0198723539/ref=pd_lpo_sbs_14_t_2?_encoding =UTF8&psc=1&refRID=0WK87FS6FG62H50Z29РД
$\endgroup$
$\begingroup$
Это пост семилетней давности, но я делюсь здесь своим опытом изучения математики в студенческие годы.
Во-первых, я хотел бы немного объяснить, что обычно находится внутри учебника по математике. Каждая глава или разделы обычно состоят из двух частей: содержания и упражнений. Цель содержания состоит в том, чтобы позволить вам раскрыть свои знания, в то время как упражнения позволяют вам совершенствоваться до тех пор, пока вы действительно хорошо не поймете материал. Так что, прежде всего, не расстраивайтесь, если вы не можете решить некоторые упражнения, так как вы выполнили упражнение на своем уровне! Если вы легко решаете все упражнения или они настолько сложны, что не можете решить ни одно из них, эта книга не для вас. Для меня подходящей для вас книгой будет та, в которой вы сможете решить не менее 60% вопросов большую часть времени. Кроме того, не пытайтесь решить каждое отдельное упражнение из книги, так как это приведет к уменьшению отдачи. Есть много забавных предметов, на которые стоит обратить внимание. Этот абзац должен дать вам правильное отношение к упражнениям по математике. Сейчас я поделюсь своим способом изучения книги по математике.
Первый уровень — Кратко прочитать главы. Если вы следуете курсу, класс лекций будет на этом уровне. Если вы занимаетесь самообразованием, прочитайте главу, в которой вы сможете понять хотя бы половину содержания, включая доказательства. Если на этом этапе вам требуется так много времени, чтобы понять (даже первую главу), подумайте о том, чтобы перейти на более легкую книгу.
Второй уровень — Подробно прочитать главы. Запишите концепцию или часть доказательств, которые вы не можете понять. Перепишите все формулировки теорем и определения на лист бумаги. Это очень полезно, когда вы пытаетесь выполнять упражнения.
Третий уровень — Выполните упражнения в каждой главе. После борьбы не стесняйтесь искать ответы на вопросы, которые вы не можете решить. Убедитесь, что понимаете, как они решают эту проблему.
Четвертый уровень — После прохождения половины книги. Пришло время ревизии! Попробуйте еще раз доказать все в тексте. Повторите упражнение, которое вы сделали. Если вы не можете доказать некоторые из них, перечитайте еще раз и постарайтесь запомнить ключевые части доказательств.
Все готовые решения ИДЗ Рябушко к 1,2,3 и 4 части к сборнику по высшей математике под редакцией Рябушко А.П. Чтобы сдать высшую математику на хорошую оценку, нужно как минимум понимать этот предмет. Но, к сожалению, не каждому дан математический склад ума. И не каждый преподаватель может доходчиво объяснить этот предмет. А о преподнесении информации в учебниках по дисциплине «Высшая математика» лучше вообще промолчать. Пишут для академиков, а не для студентов. Зато требования в ВУЗах высокие: чтобы получить 4 и 5 за экзамен, нужно сдать преподавателю правильное решение ИДЗ Рябушко А.П., например.
Полные и подробные решения индивидуальных заданий по высшей математике Рябушко с пояснениями.
В интернете многие сайты предлагают скачать все варианты решений этого сборника (часть 1,2,3 или 4), но, как правило, после скачивания оказывается, что нужный вариант отсутствует, либо написан от руки так, что сам автор не разберет. Некоторые бюро и конторы предлагают индивидуальное решение вашего варианта. Тут приходится попрощаться с деньгами и ждать 3-4 дня. А если нужно срочно, то и денег приходится отдать в 2 раза больше.
Наверное, самый оптимальный вариант – это купить уже готовое решение.
Во-первых, так можно сэкономить немало времени, особенно, когда сдача решения ИДЗ уже «на носу»,
Во-вторых, обычно такие варианты написаны в Word, с пояснениями: не придется сидеть и думать, что за цифра тут написана,
И наконец, стоит это намного дешевле, чем, если бы кто-то решал с нуля.
Кому это нужно?
Студенты-очники, у которых есть склонность к математике, не станут искать готовые решения. Они, как правило, прекрасно справляются сами. Но есть люди с гуманитарным складом ума, для которых сборник индивидуальных заданий по высшей математике – это настоящая китайская грамота. И есть заочники, у которых в голове не сборник задач, не формулы и не учебники, — а работа и семья. И минимум свободного времени. Для них гдз к сборнику ИДЗ Рябушко А.П. – просто спасение.
Решения и ответы ко всем вариантам ИДЗ для любой части доступны онлайн. 1,2,3 и 4 том учебника по высшей математике.
Мы используем cookie. Это позволяет нам анализировать взаимодействие посетителей с сайтом и делать его лучше. Продолжая пользоваться сайтом, вы соглашаетесь с использованием файлов cookie.
ГДЗ к сборнику ИДЗ Рябушко А.П. 1 часть
Готовые домашние задания к сборнику ИДЗ Рябушко А.П. 1 части индивидуальных заданий по высшей математике. ГДЗ для 1 части Рябушко А.П. содержит 14 ИДЗ. Выберите необходимый номер из списка.
ИДЗ №:
1.1,
1.2,
2.1,
2.2,
3.1,
3.2,
4.1,
4.2,
5.1,
5.2,
6.1,
6.2,
6.3,
6.4
1. Определители. Матрицы. Системы линейных алгебраических уравнении
1.1. Определители и их свойства. Вычисление определителей
1.4. Методы решения систем линейных алгебраических уравнеуравнений
1.5. Индивидуальные домашние задания к главе 1
1.6. Дополнительные задачи к главе 1
2. Векторная алгебра.
2.1. Векторы. Линейные операции над векторами. Проекция вектора на ось. Координаты вектора
2.2. Деление отрезка в данном отношении. Скалярное произпроизведение векторов и его приложения
2.3. Векторное и смешанное произведения векторов и их прилоприложения
2.4. Индивидуальные домашние задания к главе 2
2.5. Дополнительные задачи к главе 2
3. Плоскости и прямые
3.1. Плоскость
3.2. Прямая в пространстве. Прямая и плоскость
3.3. Прямая на плоскости
3.4. Индивидуальные домашние задания к главе 3
3.5. Дополнительные задачи к главе 3
4. Линии и поверхности
4.1. Линии второго порядка
4.2. Поверхности второго порядка
4.3. Линии, заданные уравнениями в полярных координатах и параметрическими уравнениями
4.4. Индивидуальные домашние задания к главе 4
4.5. Дополнительные задачи к главе 4
5. Функции. Пределы. Непрерывность функций
5.1. Числовые множества. Определение и способы задания функции
5.2. Пределы последовательностей и функций. Раскрытие простейших неопределенностей
5.3. Замечательные пределы
5.4. Сравнение бесконечно малых функций. Непрерывность функций
5.5. Индивидуальные домашние задания к главе 5
5.6. Дополнительные задачи к главе 5
6. Дифференциальное исчисление функции одной переменной и его приложения
6.1. Производная, ее геометрический и физический смысл. Правила и формулы дифференцирования
6.2. Логарифмическое дифференцирование
6.3. Производные высших порядков
6.4. Дифференциалы первого и высших порядков и их приложения
6.5. Теоремы о среднем. Правило Лопиталя — Бернулли
6.6. Исследование поведения функций и их графиков
6.7. Схема полного исследования функции и построение ее графика
6. 8. Практические задачи на экстремум
6.9. Дифференциал длины дуги и кривизна плоской линии
6.10. Индивидуальные домашние задания к главе 6
6.11. Дополнительные задачи к главе 6
Приложения
Рекомендуемая литература
Мы используем cookie. Это позволяет нам анализировать взаимодействие посетителей с сайтом и делать его лучше. Продолжая пользоваться сайтом, вы соглашаетесь с использованием файлов cookie.
Экономьте деньги на приложениях для Android и iPhone
Выйти
Популярные купоны
Последние обзоры приложений
НОВОСТИ: AppGrooves запускает первую услугу по экономии денег для пользователей приложений
Приложения могут воплотить ваши мечты о видеоблогерах в реальность
Винный погреб 101: как собрать и сохранить свою коллекцию
Как победить растительную пищу
Новый и улучшенный инструмент сравнения AppGrooves
Обновите свой гардероб — из дома и со смартфона
Как нахлыст может расслабить разум
Как приложения повышают опыт кулинарного мастер-класса
Вот способы, которые помогут вам создать счастливую перспективу
Лучшие способы позаботиться о стареющих родителях
Посмотреть все
Наборы приложений по жизненным целям
Забыли пароль?
Введите адрес электронной почты, связанный с вашей учетной записью, и мы отправим вам ссылку для сброса пароля
Скидка до 90% на
Приложения и игры
Зарегистрируйте бесплатную учетную запись, чтобы получить
специальные купоны
Зарегистрируйтесь, чтобы сохранить
Лучший
Контент
Создайте бесплатную учетную запись для полного
Опыт AppGrooves
Зарегистрируйтесь, чтобы поделиться своими
Рекомендация
Создайте бесплатную учетную запись для полного
Опыт AppGrooves
Зарегистрируйтесь, чтобы поделиться своими
Обзор
Создайте бесплатную учетную запись для полного
Опыт AppGrooves
Присоединяйтесь к Grooviest
Сообщество
Создайте бесплатную учетную запись для полного
AppGrooves опыт
Зарегистрируйтесь, чтобы сохранить это
App
Создайте бесплатную учетную запись, чтобы получать новые
купоны для ваших любимых приложений каждую неделю.
Зарегистрироваться
Войти
или продолжить с
Мы никогда никогда не будем передавать или продавать ваши данные третьим лицам. Вы можете отказаться в любое время.
Условия
Сервис | Конфиденциальность
Политика
Заполните свой профиль
Завершите настройку своей учетной записи для полного использования AppGrooves
Мы никогда никогда не будем передавать или продавать ваши данные третьим лицам. Вы можете отказаться в любое время.
Условия
Сервис | Конфиденциальность
Политика
Успехов!
Проверьте свой почтовый ящик на наличие специального приветственного подарка!
Настройте свой опыт
Меня интересует…
Выбрать все
Убрать выделение со всего
Вы можете обновить свои настройки позже
ОТПРАВИТЬ МНЕ КУПОНЫ НА…
iOS Андроид
Пожалуйста, выберите ОС
Проверьте свою электронную почту, чтобы получить свои первые предложения AppGrooves.
Спешите! Истекает через:
24
Часы
00
Минуты
00
Секунды
Перейти на вашу электронную почту
Все еще ищете другие игры?
Подпишитесь, чтобы получать обзоры, советы и похожие рекомендации по играм!
Все еще ищете самые надежные приложения?
Подпишитесь, чтобы получать обзоры, рекомендации и многое другое с помощью AppGrooves!
Зарегистрироваться
Войти
или продолжить с
Мы никогда никогда не будем передавать или продавать ваши данные третьим лицам. Вы можете отказаться в любое время.
Условия
Сервис | Конфиденциальность
Политика
Загрузка…
Зарегистрируйтесь, чтобы опубликовать свой отзыв!
И получайте рекомендации похожих приложений, которые стоят вашего времени.
Зарегистрироваться
Войти
или продолжить с
Мы никогда никогда не будем передавать или продавать ваши данные третьим лицам. Вы можете отказаться в любое время.
Условия
Сервис | Конфиденциальность
Полис
Проверьте свою электронную почту и сэкономьте на популярных приложениях уже сегодня!
Спешите! Истекает через:
24
Часы
00
Минуты
00
Секунды
Перейти на вашу электронную почту
{Название трека}
{formattedPrice}
Загрузка…
Бесплатные купоны
заданий, решений и пояснений. Егэ по математике Решить задачи по
Среднее общее образование
Линия УМК Г.К. Муравина. Алгебра и начала математического анализа (10-11) (углубленный)
Линия УМК Мерзляк. Алгебра и начала анализа (10-11) (У)
Математика
Разбираем задачи и решаем примеры с учителем
Экзаменационная работа профильного уровня длится 3 часа 55 минут (235 минут).
Минимальный порог — 27 баллов.
Экзаменационная работа состоит из двух частей, различающихся по содержанию, сложности и количеству заданий.
Отличительной особенностью каждой части работы является форма заданий:
часть 1 содержит 8 заданий (задания 1-8) с кратким ответом в виде целого числа или конечной десятичной дроби;
часть 2 содержит 4 задания (задания 9-12) с кратким ответом в виде целой или конечной десятичной дроби и 7 заданий (задания 13-19) с развернутым ответом (полная запись решения с обоснованием выполняемые действия).
Панова Светлана Анатольевна , учитель математики высшей категории школы, стаж работы 20 лет:
«Для получения школьного аттестата выпускник должен сдать два обязательных экзамена в форме ЕГЭ, один из которых – математика. В соответствии с Концепцией развития математического образования в Российской Федерации ЕГЭ по математике делится на два уровня: базовый и специализированный. Сегодня мы рассмотрим варианты профильного уровня.
Задание №1 — проверяет умение участников ЕГЭ применять навыки, полученные в ходе 5-9оценки по элементарной математике в практической деятельности. Участник должен иметь вычислительные навыки, уметь работать с рациональными числами, уметь округлять десятичные дроби, уметь переводить одни единицы измерения в другие.
Пример 1 В квартире, где проживает Петр, установлен счетчик (счетчик) холодной воды. Первого мая счетчик показывал расход 172 куб. м воды, а на первое июня — 177 куб. м. Какую сумму должен заплатить Петр за холодную воду за май, если цена 1 куб. м холодной воды 34 рубля 17 копеек? Дайте ответ в рублях.
Решение:
1) Найти количество израсходованной воды в месяц:
177 — 172 = 5 (куб. м)
2) Найти, сколько денег будет выплачено за израсходованную воду:
34,17 5 = 170,85 (руб.)
Ответ: 170,85.
Задание №2 — это одно из самых простых заданий ЕГЭ. С ней успешно справляется большинство выпускников, что свидетельствует о владении определением понятия функции. Тип задания № 2 по кодификатору требований — задание на использование полученных знаний и умений в практической деятельности и повседневной жизни. Задание № 2 состоит в описании с помощью функций различных реальных отношений между величинами и интерпретации их графиков. Задание №2 проверяет умение извлекать информацию, представленную в таблицах, схемах, графиках. Выпускники должны уметь определять значение функции по значению аргумента при различных способах задания функции и описывать поведение и свойства функции по ее графику. Также необходимо уметь находить наибольшее или наименьшее значение по графику функции и строить графики исследуемых функций. Допущенные ошибки носят случайный характер при чтении условий задачи, чтении схемы.
#ADVERTISING_INSERT#
Пример 2 На рисунке показано изменение курсовой стоимости одной акции горнодобывающей компании в первой половине апреля 2017 года. 7 апреля бизнесмен приобрел 1000 акций этой компании. 10 апреля он продал три четверти купленных акций, а 13 апреля продал все оставшиеся. Сколько бизнесмен потерял в результате этих операций?
Решение:
2) 1000 3/4 = 750 (акций) — составляют 3/4 всех купленных акций.
6) 247500 + 77500 = 325000 (рублей) — получил бизнесмен после продажи 1000 акций.
7) 340 000 — 325 000 = 15 000 (рублей) — бизнесмен потерял в результате всех операций.
В этом разделе мы готовимся к ЕГЭ по математике как базовому, профильному уровню — представляем разборы задач, тесты, описание ЕГЭ и полезные рекомендации. Используя наш ресурс, вы как минимум поймете, как решать задачи и сможете успешно сдать ЕГЭ по математике в 2019 году.. Начинать!
ЕГЭ по математике является обязательным экзаменом для любого ученика 11 класса, поэтому информация, представленная в этом разделе, актуальна для всех. ЕГЭ по математике делится на два вида — базовый и профильный. В этом разделе я привожу анализ каждого типа задач с подробным объяснением двух вариантов. Задания ЕГЭ строго тематические, поэтому к каждому номеру можно дать точные рекомендации и дать теорию, необходимую именно для решения данного типа заданий. Ниже вы найдете ссылки на задания, перейдя по которым вы сможете изучить теорию и разобрать примеры. Примеры постоянно обновляются и обновляются.
Структура базового уровня ЕГЭ по математике
Экзамен по математике базового уровня состоит из одной части , в том числе 20 заданий с кратким ответом. Все задания направлены на проверку развития базовых умений и практических навыков применения математических знаний в повседневных ситуациях.
Ответ на каждое из заданий 1–20: целое число , конечное десятичное число или последовательность цифр .
Задание с кратким ответом считается выполненным, если правильный ответ зафиксирован в бланке ответов №1 по форме, предусмотренной инструкцией по выполнению задания.
Скоро сказка рассказана, да не скоро дело сделано
К сожалению, новые обучающие материалы не появляются на сайте с той скоростью, которой хотелось бы многим посетителям. Неудивительно, ведь на создание качественного урока уходит много времени. Но что делать? Вот вам тема или отдельное задание, которое вы не нашли на моем ресурсе…. А разобраться нужно сегодня!
Поэтому по мере развития проекта возникла необходимость создания банка готовых решений по различным разделам высшей математики. И эта страничка точно должна вас порадовать, ведь банк совершенно БЕСПЛАТНО ! Прямо сейчас у вас есть возможность бесплатно скачать файловые архивы с решенными задачами по разным башенным темам.
Источник квеста?
Все задачи выполняются лично мной, что гарантирует высокое качество решения.
Типовой архив содержит, как правило, 200-300 заданий с готовыми решениями, и я постарался собрать самые разнообразные материалы, включая редкие задания и примеры повышенной сложности. Солянка состоит из тестов, индивидуальных заданий, выполненных в свое время мной на заказ. Кроме того, порадую студентов дневного отделения — в архиве типовые расчеты из сборника Кузнецова, ИПД из сборника Рябушко, а также теория вероятностей из сборника Чудесенко. По причинам, указанным в обзоре Полезные математические сайты Я раздаю их бесплатно. Да, расчетов, конечно, не очень много, но некоторым ученикам очень везет!
Только, товарищи, не будем жаловаться и обижаться. Да, качество высокое, все примеры и задания проверены преподавателями, но в решениях могут быть опечатки, неточности и даже ошибки. Задания представлены «как есть». Не имеет смысла оповещать меня о найденных косяках, так как, во-первых, их технически сложно исправить, а во-вторых, просто нет времени (одна перепаковка и перезалив архива чего стоит). Исключением являются pdf-файлы, расположенные непосредственно на сервере, пожалуйста, напишите мне, если обнаружите в них СЕРЬЕЗНУЮ оплошность.
Возможно, у некоторых возникнут недопонимания и даже возникнут неправильные представления. Но все же практически все посетители сайта имеют ненулевой уровень математической подготовки, поэтому типовые решения принесут гораздо больше пользы, чем вреда; идеально, если вы уже хорошо разбираетесь в той или иной теме и хотите найти дополнительные примеры.
Каждый сборник задач сопровождается краткими комментариями о содержимом архива. Более того, банк время от времени пополняется новыми материалами.
Другими словами, присоединяйтесь к нам!
Готовые задачи с решениями по теме Линейная алгебра и аналитическая геометрия
В архиве около трехсот задач по линейной алгебре и аналитической геометрии, в том числе несколько расчетов Кузнецова и несколько ИПД Рябушко. Решены почти все типичные задачи алгебры: решение систем уравнений, вычисление определителей, изучение систем на совместность, задачи на определение ранга матрицы, матричных уравнений, задачи на собственные значения/собственные векторы. Аналитическая геометрия представлена как задачами на плоскости, так и в трехмерном пространстве.
Примеры решения лимита
В архиве пара сотен сплит-лимитов, включая лимиты последовательности. Рассмотрены примеры решения пределов с помощью замечательных эквивалентностей. Сборник содержит 3 расчета Кузцева и 3 варианта ИДЗ Рябушко. Не скажу какие именно варианты — наслаждайтесь лотерейной игрой и развлечениями! Если вы читали существующие уроки по теме, то понять многие ограничения сборника не составит труда.
Разграничение функций одной переменной
В архиве представлены в основном типовые расчеты из сборника Кузнецова (10 вариантов), есть 3 варианта от Рябушко. Примеры позволят вам значительно улучшить вашу технику дифференциации, так как производные из коллекции Кузнецова довольно тяжелые. Более того, некоторые производные приходится находить в несколько этапов! Помимо производных есть типовые задачи: нахождение производных произвольного, «n-го» порядка, примеры по формуле Лагранжа; задачи на построение касательной и нормали и несколько задач на нахождение производной по заданию производной.
Функция Исследования
Пожалуй, самая трудоемкая и кропотливая подборка. Решение содержит 69 полностью исследованных функций одной переменной, включая полиномиальные функции, дробно-рациональные функции, функции с показателями степени, логарифмы и другие более редкие примеры. Вполне вероятно, что вы найдете свой пример или очень похожую функцию. Копилка выступает в роли приложения темы «Функции и графики» и предназначена для самопроверки/обучения.
Готовые решения неопределенных интегралов
В основном неопределенные интегралы, но есть и определенные интегралы. Коллекция включает более двухсот интегралов, в том числе очень сложных. Рассмотрено много примеров, которые не влезли в мои уроки! В архиве 6 типовых расчетов Кузнецова и 2 И.Д.С. Рябушко. Расчеты Кузнецова содержат задачи на нахождение объемов тел, длин дуг и площадей поверхностей, причем не только в декартовой, но и в полярной системе координат, а также для параметрически заданных функций.
Задачи для числового и функционального ряда
Еще один тяжелый кирпич. Конечно, есть примеры исследования числовых рядов на сходимость, причем есть и творческие ряды, требующие опыта и нестандартного подхода. Моя гордость — не было ряда, который я не смог бы взломать. Рассмотрены задачи нахождения суммы ряда, различные задачи на приближенные вычисления с использованием ряда. Функциональные ряды представлены общей проблемой нахождения области сходимости (опять же с творческими случаями). К вашим услугам популярная задача расширения частного решения ТЭ в серию. Примеров решений рядов Фурье около десятка. И даже выявление неопределенностей внутри с помощью расширения ряда. В архиве 13 видов Кузнецова и пару вариантов Рябушко. Приятного аппетита!
Готовые решения дифференциальных уравнений
Добрая сотня диффузоров. В задачах используются различные методы решения дифференциальных уравнений первого порядка, в том числе и те, которые я не рассматривал на уроках. Имеются диффузы повышенной сложности — ДЭ второго порядка и выше, примеры с использованием метода вариации произвольных констант. Системы дифференциальных уравнений прилагаются. В архиве 8 стандартных расчетов Кузнецова и 2 варианта Рябушко. Кроме того, я упаковал сборник экзаменационных работ, в них помимо дифференциальных уравнений есть задачи на ряды и кратные интегралы.
Примеры решения кратного и криволинейного интегралов
Двойной интеграл, изменить порядок обхода площади, вычислить площадь с помощью двойного интеграла. Тройной интеграл, вычисление объема тела с помощью тройного интеграла. Примеры решений не только в декартовой, но и в полярной, цилиндрической системах координат. Основные применения кратных интегралов: нахождение центра тяжести плоской пластины, центра масс тела и др. задачи. Криволинейные интегралы по кривой, по контуру. Хит-задача на нахождение работы силы по контуру и по формуле Грина.
Ты правда думаешь, что это сложно?!
Примеры решения задач по теории поля
Помимо прочего, в сборник включены наиболее популярные задачи с решениями следующих типов: нахождение производной по направлению и градиента, вычисление потока векторного поля через часть плоскости, вычисление течения векторного поля через замкнутую поверхность по формуле Остроградского-Гаусса, вычисление циркуляции векторного поля по контуру. В архиве несколько расчетов из коллекции Кузнецова.
Примеры решения задач по комбинаторике и теории вероятностей
В следующих pdf-ках вы можете найти дополнительные задания к урокам по комбинаторике и теория вероятностей . При этом у меня не было цели собрать сотни примеров, наоборот, я старался отсеивать явные дубликаты. Кроме того, в большинстве файлов задания упорядочены по возрастанию сложности:
Задания экономического содержания
Многие знают, как сложно «раскачаться» после долгого зимнего отпуска, и однажды после праздников я нашла отличный способ вернуться в рабочий ритм – создавать сборники готовых задач с ярко выраженного экономического содержания, которые законсервированы в моей коллекции и никому не приносят пользы. Это некоторые задачи экономико-математического моделирования, это задачи финансовой математики (проценты, рента и др.) , и, наконец, это задачи, более связанные с экономикой. Итак, исправим хмурую январскую погоду следующими полезными материалами:
Задачи по финансовой математике
На самом деле это слишком громко — в следующем файле вы найдете актуальные и практически полезные задачи на проценты и расчет выплат по вкладам/кредитам. Я отретушировал несколько найденных решений и сделал их максимально понятными! Более того, по ходу дела я сам кое-что придумал =)
И, конечно же, калькулятор для задач! Да хотя бы для того, чтобы моментально найти процент процентов от любого числа:
Задача межотраслевого баланса (модель Леонтьева)
Одна из самых известных экономико-математических моделей, описывающая многие отрасли экономики, часть продукции которых взаимно расходуется в результате производства, а другая часть конечный продукт. Возникли проблемы с матрицами прямых/общих затрат, кропотливыми инверсиями и дробным умножением матриц? Следующая программа моментально проведет расчеты, которые вы просто устанете делать на обычном микрокалькуляторе:
Эта программа создана в MS Excel и доступна активным пользователям библиотеки . Или за умеренную плату.
Демо-версию можно посмотреть. Расчеты проводились для наиболее распространенного образовательного случая — три отрасли; Более того, решение можно распечатать и вручить!
И что особенно приятно, вот пример практического использования матричных алгебр .
Типовые задачи для экономических показателей
Следующий скромный пдф, конечно, не охватывает всего многообразия экономических показателей, но я досконально проанализировал и систематизировал, пожалуй, все разновидности самой популярной задачи на индексы цен/физический объем продаж/оборот + задача аналогична стоимостным и суммарным стоимостным показателям . Даже ребенок поймет:
… а если он еще и использует с этой программой , то никогда не ошибайтесь!
Задачи экономической статистики
Как и в предыдущем файле, вся высшая математика здесь ограничивается простейшими арифметическими действиями =) … И было бы смешно, если бы не было так грустно (Экономика не мой профильный предмет, но, тем не менее, ваш скромный прислуге однажды пришлось замучить несколько десятков задач по социально-экономической и производственной статистике, которые были «довесками» к контрольной работе по математической статистике . В связи с этим супер качество не гарантирую, хотя с другой стороны халтуры тоже нет :
И наконец, на радость студентам дневного отделения, открываю еще две раздачи:
Типовые расчеты по теории вероятностей из сборника Чудесенко бесплатно!
Первые 22 задачи решены, какие-то больше, какие-то меньше. Первый вариант для демонстрации выложен прямо на сайте: Остальные варианты (2, 5, 7, 8, 9, 10, 11, 12, 15, 20, 21, 24, 25) в силу их легкого веса, Перешел с файлообменника на только что созданный сайт библиотеки. Также архив может быть полезен тем, что в сборнике Чудесенко собраны достаточно сложные задачи по теории вероятностей, и, возможно, вы найдете то, что искали.
ИДЗ из задачника Рябушко — бесплатно!
Солидный довесок ко всем архивам + задачи, которые еще не были предложены на этой странице, в частности примеры с функциями нескольких переменных . Конечно далеко не все 4 тома закрыты, но в некоторых разделах будет добрая половина вариантов, а то и больше! Решенные индивидуальные задания можно использовать как «по прямому назначению», так и в качестве дополнительной самоподготовки/обучения.
Похоже, что здесь ничего нет…Может, попробуете воспользоваться поиском?
Искать:
Свежие записи
Куда сходить с детьми в Каспийске
Викторина про животных для детей 7-10 лет, с ответами
Куда сходить с ребенком в Ессентуках
Второстепенные члены предложения: таблица
Куда сходить с ребенком в Мытищах
Архивы
Архивы
Выберите месяц Ноябрь 2022 Октябрь 2022 Сентябрь 2022 Август 2022 Июль 2022 Июнь 2022 Май 2022 Апрель 2022 Март 2022 Февраль 2022 Январь 2022 Декабрь 2021 Ноябрь 2021 Октябрь 2021 Сентябрь 2021 Август 2021 Июль 2021 Июнь 2021 Май 2021 Апрель 2021 Март 2021 Февраль 2021 Январь 2021 Декабрь 2020 Ноябрь 2020 Октябрь 2020 Сентябрь 2020 Август 2020 Июль 2020 Июнь 2020 Май 2020 Апрель 2020 Март 2020 Февраль 2020 Январь 2020 Декабрь 2019 Ноябрь 2019 Октябрь 2019 Сентябрь 2019 Август 2019 Июль 2019 Июнь 2019 Май 2019 Апрель 2019 Март 2019 Февраль 2019 Январь 2019 Декабрь 2018 Ноябрь 2018 Октябрь 2018 Сентябрь 2018 Август 2018 Июль 2018 Июнь 2018 Май 2018 Апрель 2018 Март 2018 Февраль 2018 Январь 2018 Декабрь 2017 Ноябрь 2017 Октябрь 2017 Сентябрь 2017 Август 2017 Июль 2017 Июнь 2017 Май 2017 Апрель 2017 Март 2017 Февраль 2017 Январь 2017 Декабрь 2016 Ноябрь 2016 Октябрь 2016 Сентябрь 2016 Август 2016 Июль 2016 Июнь 2016 Май 2016 Апрель 2016 Март 2016 Февраль 2016 Январь 2016 Декабрь 2015 Ноябрь 2015 Октябрь 2015 Сентябрь 2015 Август 2015 Июль 2015 Июнь 2015
Рубрики
РубрикиВыберите рубрикуАНАЛИЗЫАФИША СОБЫТИЙБЕЗОПАСНОСТЬ РЕБЕНКАБЕРЕМЕННОСТЬ И РОДЫБОЛЕЗНИ У ДЕТЕЙВОПРОСЫ-ОТВЕТЫДЕТСКАЯ АПТЕКАДЕТСКАЯ КОМНАТАДЕТСКАЯ ЛОГОПЕДИЯДЕТСКАЯ ПСИХОЛОГИЯДЕТСКАЯ СТОМАТОЛОГИЯДОМ И РЕБЕНОКЗДОРОВЬЕ РЕБЕНКАМЕТОДИКИНАЧАЛЬНАЯ ШКОЛАНОВОРОЖДЕННЫЙОБУЧЕНИЕ И ОБРАЗОВАНИЕОСНОВНАЯ ШКОЛАПАТРИОТИЧЕСКОЕПИТАНИЕ РЕБЕНКАПОЛОВОЕПРИВИВКИПРОФОРИЕНТАЦИЯПСИХОЛОГИЯ И РАЗВИТИЕПУТЕШЕСТВИЯ И ОТДЫХРАЗВИТИЕ И ОБУЧЕНИЕРЕЛИГИОЗНОЕРЕЦЕПТЫСАМОСТОЯТЕЛЬНАЯ РАБОТАСЕМЕЙНОЕСТАРШИЕ КЛАССЫТВОРЧЕСТВОТОВАРЫ ДЛЯ ДЕТЕЙФИЗИЧЕСКОЕ ВОСПИТАНИЕФИНАНСОВОЕЧЕМ ЗАНЯТЬ РЕБЕНКАЭТИЧЕСКОЕ
Метки
1 класс
2 класс
3 класс
4 класс
5 класс
6 класс
7 класс
8 класс
9 класс
10 класс
11 класс
Викторины
География
ДОУ
Дошкольный возраст (3-7 лет)
Игры
История
Литература
Математика
Младенчество (до 1 года)
Младший школьный возраст (7-12 лет)
Подростковый возраст (12-15 лет)
Ранний возраст (1-3 года)
Русский язык
СЛАЙДЕР
Страницы
Вопросы-ответы
Часто задаваемые вопросы: список
Дом и ребенок
Детская комната
Товары для детей
Досуг
Афиша событий
Путешествия и отдых
Чем занять ребенка
Творчество
Здоровье
Безопасность ребенка
Беременность и роды
Болезни у детей
Здоровье ребенка
Анализы
Детская аптека
Детская стоматология
Новорожденный
Прививки
Питание ребенка
Рецепты
О персональных данных пользователей
Обратная связь
Обучение и образование
Начальная школа
Самостоятельная работа
Основная школа
Старшие классы
Профориентация
От редакции
Пользовательское соглашение
Психология и развитие
Воспитание ребенка
Патриотическое
Половое
Религиозное
Семейное
Финансовое
Этическое
Детская психология
Методики
Развитие и обучение
Детская логопедия
Физическое воспитание
Рубрики
Страница не найдена — Детский возраст
Похоже, что здесь ничего нет. ..Может, попробуете воспользоваться поиском?
Искать:
Свежие записи
Куда сходить с детьми в Каспийске
Викторина про животных для детей 7-10 лет, с ответами
Куда сходить с ребенком в Ессентуках
Второстепенные члены предложения: таблица
Куда сходить с ребенком в Мытищах
Архивы
Архивы
Выберите месяц Ноябрь 2022 Октябрь 2022 Сентябрь 2022 Август 2022 Июль 2022 Июнь 2022 Май 2022 Апрель 2022 Март 2022 Февраль 2022 Январь 2022 Декабрь 2021 Ноябрь 2021 Октябрь 2021 Сентябрь 2021 Август 2021 Июль 2021 Июнь 2021 Май 2021 Апрель 2021 Март 2021 Февраль 2021 Январь 2021 Декабрь 2020 Ноябрь 2020 Октябрь 2020 Сентябрь 2020 Август 2020 Июль 2020 Июнь 2020 Май 2020 Апрель 2020 Март 2020 Февраль 2020 Январь 2020 Декабрь 2019 Ноябрь 2019 Октябрь 2019 Сентябрь 2019 Август 2019 Июль 2019 Июнь 2019 Май 2019 Апрель 2019 Март 2019 Февраль 2019 Январь 2019 Декабрь 2018 Ноябрь 2018 Октябрь 2018 Сентябрь 2018 Август 2018 Июль 2018 Июнь 2018 Май 2018 Апрель 2018 Март 2018 Февраль 2018 Январь 2018 Декабрь 2017 Ноябрь 2017 Октябрь 2017 Сентябрь 2017 Август 2017 Июль 2017 Июнь 2017 Май 2017 Апрель 2017 Март 2017 Февраль 2017 Январь 2017 Декабрь 2016 Ноябрь 2016 Октябрь 2016 Сентябрь 2016 Август 2016 Июль 2016 Июнь 2016 Май 2016 Апрель 2016 Март 2016 Февраль 2016 Январь 2016 Декабрь 2015 Ноябрь 2015 Октябрь 2015 Сентябрь 2015 Август 2015 Июль 2015 Июнь 2015
Рубрики
РубрикиВыберите рубрикуАНАЛИЗЫАФИША СОБЫТИЙБЕЗОПАСНОСТЬ РЕБЕНКАБЕРЕМЕННОСТЬ И РОДЫБОЛЕЗНИ У ДЕТЕЙВОПРОСЫ-ОТВЕТЫДЕТСКАЯ АПТЕКАДЕТСКАЯ КОМНАТАДЕТСКАЯ ЛОГОПЕДИЯДЕТСКАЯ ПСИХОЛОГИЯДЕТСКАЯ СТОМАТОЛОГИЯДОМ И РЕБЕНОКЗДОРОВЬЕ РЕБЕНКАМЕТОДИКИНАЧАЛЬНАЯ ШКОЛАНОВОРОЖДЕННЫЙОБУЧЕНИЕ И ОБРАЗОВАНИЕОСНОВНАЯ ШКОЛАПАТРИОТИЧЕСКОЕПИТАНИЕ РЕБЕНКАПОЛОВОЕПРИВИВКИПРОФОРИЕНТАЦИЯПСИХОЛОГИЯ И РАЗВИТИЕПУТЕШЕСТВИЯ И ОТДЫХРАЗВИТИЕ И ОБУЧЕНИЕРЕЛИГИОЗНОЕРЕЦЕПТЫСАМОСТОЯТЕЛЬНАЯ РАБОТАСЕМЕЙНОЕСТАРШИЕ КЛАССЫТВОРЧЕСТВОТОВАРЫ ДЛЯ ДЕТЕЙФИЗИЧЕСКОЕ ВОСПИТАНИЕФИНАНСОВОЕЧЕМ ЗАНЯТЬ РЕБЕНКАЭТИЧЕСКОЕ
Метки
1 класс
2 класс
3 класс
4 класс
5 класс
6 класс
7 класс
8 класс
9 класс
10 класс
11 класс
Викторины
География
ДОУ
Дошкольный возраст (3-7 лет)
Игры
История
Литература
Математика
Младенчество (до 1 года)
Младший школьный возраст (7-12 лет)
Подростковый возраст (12-15 лет)
Ранний возраст (1-3 года)
Русский язык
СЛАЙДЕР
Страницы
Вопросы-ответы
Часто задаваемые вопросы: список
Дом и ребенок
Детская комната
Товары для детей
Досуг
Афиша событий
Путешествия и отдых
Чем занять ребенка
Творчество
Здоровье
Безопасность ребенка
Беременность и роды
Болезни у детей
Здоровье ребенка
Анализы
Детская аптека
Детская стоматология
Новорожденный
Прививки
Питание ребенка
Рецепты
О персональных данных пользователей
Обратная связь
Обучение и образование
Начальная школа
Самостоятельная работа
Основная школа
Старшие классы
Профориентация
От редакции
Пользовательское соглашение
Психология и развитие
Воспитание ребенка
Патриотическое
Половое
Религиозное
Семейное
Финансовое
Этическое
Детская психология
Методики
Развитие и обучение
Детская логопедия
Физическое воспитание
Рубрики
Рабочие листы по математике для 5-го класса в формате pdf, экзаменационные листы по математике для 5-го класса
Десятичные модели
Практика десятичных моделей с помощью этого бесплатного печатного листа по математике.
Распечатать здесь >>>
Иллюстрированное десятичное упражнение
Попрактикуйтесь в иллюстрированном десятичном упражнении с помощью этого бесплатного печатного листа по математике.
Распечатать здесь >>>
Умножение десятичных дробей
Практика умножения десятичных дробей с помощью этого бесплатного математического листа, который можно распечатать.
Распечатать здесь >>>
Округлить десятичные дроби
Практика округления десятичных дробей с помощью этого бесплатного печатного листа по математике.
Распечатать здесь >>>
Вычитание с десятичными дробями
Практика вычитания с десятичными дробями с помощью этого бесплатного печатного листа по математике.
Распечатать здесь >>>
Деление в длинное деление
Практика деления в длинное в этом бесплатном математическом листе для печати.
Распечатать здесь >>>
Разделить 2 на однозначные числа
Потренируйтесь делить 2 на однозначные числа с помощью этого бесплатного печатного листа по математике.
Распечатать здесь >>>
Разделить 3 на однозначные числа
Потренируйтесь делить 3 на однозначные числа с помощью этого бесплатного печатного листа по математике.
Распечатать здесь >>>
Упражнение с делением по кругу
Практикуйте упражнение по упражнению с делением по кругу с помощью этого бесплатного распечатываемого листа по математике.
Распечатайте здесь >>>
Упражнение на соответствие делений
Попрактикуйтесь в упражнении на соответствие делений с помощью этого бесплатного печатного листа по математике.
Распечатать здесь >>>
Деление целых чисел на дроби
Практикуйте деление целых чисел на дроби с помощью этого бесплатного печатного листа по математике.
Распечатать здесь >>>
Упражнение с таблицей делений
Потренируйтесь выполнять упражнение с таблицей делений с помощью этого бесплатного печатного листа по математике.
Распечатать здесь >>>
Деление в длину с остатком
Практикуйте деление в длину с остатком с помощью этого бесплатного печатного листа по математике.
Распечатать здесь >>>
Деление в длину
Потренируйтесь в делении в длину с помощью этого бесплатного печатного листа по математике.
Распечатать здесь >>>
Сложение дробей
Практика сложения дробей с помощью этого бесплатного печатного листа по математике.
Распечатать здесь >>>
Сложение смешанных дробей
Попрактикуйтесь в сложении смешанных дробей с помощью этого бесплатного печатного листа по математике.
Распечатайте здесь >>>
Преобразование дробей в десятичные
Потренируйтесь преобразовывать дроби в десятичные с помощью этого бесплатного математического листа, который можно распечатать.
Распечатать здесь >>>
Деление дробей
Потренируйтесь в делении дробей с помощью этого бесплатного математического листа, который можно распечатать.
Распечатать здесь >>>
Деление смешанных дробей
Практика деления смешанных дробей с помощью этого бесплатного математического листа, который можно распечатать.
Распечатать здесь >>>
Эквивалентные дроби
Практикуйте эквивалентные дроби с помощью этого бесплатного печатного листа по математике.
Распечатать здесь >>>
Дробь чисел
Практикуйте дроби чисел с помощью этого бесплатного печатного листа по математике.
Распечатать здесь >>>
Умножение смешанных дробей
Потренируйтесь в умножении смешанных дробей с помощью этого бесплатного математического листа, который можно распечатать.
Распечатать здесь >>>
Упростить дроби
Практикуйтесь в упрощении дробей с помощью этого бесплатного математического листа, который можно распечатать.
Распечатать здесь >>>
Вычитание дробей
Практика вычитания дробей с помощью этого бесплатного печатного листа по математике.
Распечатать здесь >>>
Вычитание смешанных дробей
Попрактикуйтесь в вычитании смешанных дробей с помощью этого бесплатного печатного листа по математике.
Распечатать здесь >>>
Площадь кругов 2
Попрактикуйтесь в области кругов 2 с помощью этого бесплатного печатного листа по математике.
Распечатать здесь >>>
Площадь кругов
Практика площади кругов с помощью этого бесплатного печатного листа по математике.
Распечатать здесь >>>
Окружность круга
Попрактикуйтесь в измерении окружности с помощью этого бесплатного математического листа, который можно распечатать.
Распечатайте здесь >>>
Определение сегментов геометрии
Попрактикуйтесь в определении сегментов геометрии с помощью этого бесплатного математического листа, который можно распечатать.
Распечатать здесь >>>
Измерение углов транспортиром
Попрактикуйтесь в измерении углов транспортиром с помощью этого бесплатного печатного листа по математике.
Распечатать здесь >>>
Периметр и площадь сложных фигур
Практикуйте периметр и площадь сложных фигур с помощью этого бесплатного математического листа для печати.
Распечатать здесь >>>
Периметр и площадь L-образных фигур
Попрактикуйтесь в периметре и площади L-образных фигур с помощью этого бесплатного математического листа для печати.
Распечатать здесь >>>
Периметр и площадь прямоугольников
Попрактикуйтесь в периметре и площади прямоугольников с помощью этого бесплатного печатного листа по математике.
Распечатать здесь >>>
Многоугольники Стороны Вершины
Попрактикуйтесь в построении сторон многоугольников с помощью этого бесплатного математического листа, который можно распечатать.
Распечатать здесь >>>
Теорема Пифагора
Практикуйте теорему Пифагора с помощью этого бесплатного печатного листа по математике.
Распечатайте здесь >>>
Фигуры
Попрактикуйтесь в фигурах с помощью этого бесплатного математического листа, который можно распечатать.
Распечатать здесь >>>
Твердые и плоские фигуры
Практикуйте объемные и плоские фигуры с помощью этого бесплатного математического листа, который можно распечатать.
Распечатать здесь >>>
Симметричные фигуры
Практикуйте симметричные фигуры с помощью этого бесплатного печатного листа по математике.
Распечатать здесь >>>
Симметричные объекты из реальной жизни
Практикуйте симметричные объекты из реальной жизни с помощью этого бесплатного печатного листа по математике.
Распечатать здесь >>>
Симметрия с фигурами
Практикуйте симметрию с фигурами с помощью этого бесплатного математического листа, который можно распечатать.
Распечатайте здесь >>>
Практический лист по гистограммам и линейным графикам
Практический лист по гистограммам и линейным графикам с помощью этого бесплатного печатного листа по математике.
Распечатайте здесь >>>
Координаты
Практика координат с помощью этого бесплатного печатного листа по математике.
Распечатать здесь >>>
Свобода настройки графика
Практикуйте свободу настройки графика с помощью этого бесплатного печатного листа по математике.
Распечатайте здесь >>>
Графики и данные
Практикуйте графики и данные с помощью этого бесплатного печатного листа по математике.
Распечатать здесь >>>
Графики, расположенные по X Y
Практические графики, расположенные по X и Y с помощью этого бесплатного печатного листа по математике.
Распечатать здесь >>>
Линейные графики
Попрактикуйтесь в построении линейных графиков с помощью этого бесплатного математического листа, который можно распечатать.
Распечатать здесь >>>
Построение графиков
Потренируйтесь строить графики с помощью этого бесплатного математического листа, который можно распечатать.
Распечатать здесь >>>
Таблицы
Практические таблицы с этой бесплатной печатной таблицей по математике.
Распечатать здесь >>>
Диаграммы Венна 2
Попрактикуйтесь в диаграммах Венна 2 с помощью этого бесплатного математического листа, который можно распечатать.
Распечатайте здесь >>>
Диаграммы Венна
Попрактикуйтесь в построении диаграмм Венна с помощью этого бесплатного печатного листа по математике.
Распечатайте здесь >>>
X Y Упражнение с координатами
Упражнение с координатами x y с этим бесплатным распечатываемым математическим заданием.
Распечатать здесь >>>
Преобразование весов мм см фут ярд
Попрактикуйтесь в переводе весов мм см фут ярд с помощью этого бесплатного математического листа, который можно распечатать.
Распечатать здесь >>>
Измерение углов
Практика измерения углов с помощью этого бесплатного математического листа, который можно распечатать.
Распечатать здесь >>>
Измерение прямоугольников и вычисление площади
Практикуйтесь в измерении прямоугольников и вычислении площади с помощью этого бесплатного математического листа, который можно распечатать.
Распечатать здесь >>>
Измерения термометра
Попрактикуйтесь в измерениях термометра с помощью этого бесплатного распечатываемого листа по математике.
Распечатать здесь >>>
Операции смешивания
Попрактикуйтесь в операциях смешивания с помощью этого бесплатного печатного листа по математике.
Распечатать здесь >>>
Умножение на однозначные числа
Потренируйтесь умножать на однозначные числа с помощью этого бесплатного печатного листа по математике.
Распечатать здесь >>>
Умножение на двузначные числа
Практика умножения на двузначные числа с помощью этого бесплатного печатного листа по математике.
Распечатайте здесь >>>
Сопоставление умножения
Потренируйтесь сопоставлять умножение с помощью этого бесплатного печатного листа по математике.
Распечатать здесь >>>
Умножение дробей
Практика умножения дробей с помощью этого бесплатного математического листа, который можно распечатать.
Распечатать здесь >>>
Упражнение с таблицей умножения
Попрактикуйтесь в упражнении с таблицей умножения с помощью этого бесплатного печатного листа по математике.
Распечатать здесь >>>
Умножение с денежными значениями
Попрактикуйтесь в умножении с денежными значениями с помощью этого бесплатного печатного листа по математике.
Распечатайте здесь >>>
Шаблоны счета
Потренируйтесь в счете шаблонов с помощью этого бесплатного печатного листа по математике.
Распечатайте здесь >>>
Коэффициенты
Попрактикуйтесь в коэффициентах с помощью этого бесплатного распечатываемого листа по математике.
Распечатать здесь >>>
Наибольший общий делитель
Практикуйте наибольший общий делитель с помощью этого бесплатного математического листа, который можно распечатать.
Распечатайте здесь >>>
Наименьшее общее кратное
Практикуйте наименьшее общее кратное с помощью этого бесплатного печатного листа по математике.
Распечатать здесь >>>
Простые и составные числа
Практикуйте простые и составные числа с помощью этого бесплатного печатного листа по математике.
Распечатать здесь >>>
Римские цифры
Практикуйте римские цифры с помощью этого бесплатного печатного листа по математике.
Распечатать здесь >>>
Округлить числа до миллионов
Потренируйтесь округлять числа до миллионов с помощью этого бесплатного печатного листа по математике.
Распечатайте здесь >>>
Округление чисел
Попрактикуйтесь в округлении чисел с помощью этого бесплатного печатного листа по математике.
Распечатать здесь >>>
Научное обозначение 1
Потренируйтесь в научном обозначении 1 с помощью этого бесплатного печатного листа по математике.
Распечатать здесь >>>
Научное обозначение 2
Потренируйтесь в научном представлении 2 с помощью этого бесплатного печатного листа по математике.
Распечатать здесь >>>
Комплекс узоров
Практикуйте комплекс узоров с помощью этого бесплатного печатного листа по математике.
Распечатать здесь >>>
Patterns Easy
Практикуйте узоры легко с помощью этого бесплатного печатного листа по математике.
Распечатать здесь >>>
Вычитание до ста тысяч
Потренируйтесь вычитать до ста тысяч с помощью этого бесплатного печатного листа по математике.
Распечатайте здесь >>>
Вычитание до миллионов
Практика вычитания до миллионов с помощью этого бесплатного печатного листа по математике.
Распечатать здесь >>>
Упражнение на вычитание круга
Практикуйте упражнение на вычитание круга с помощью этого бесплатного печатного листа по математике.
Распечатать здесь >>>
Упражнение с таблицей вычитания
Попрактикуйтесь в упражнении с таблицей вычитания с помощью этого бесплатного печатного листа по математике.
Распечатайте здесь >>>
Определение времени до полудня
Практикуйтесь в определении времени до полудня с помощью этого бесплатного печатного листа по математике.
Распечатать здесь >>>
Часы с римскими цифрами для определения времени
Практикуйтесь в определении времени по часам с римскими цифрами с помощью этого бесплатного печатного листа по математике.
Распечатать здесь >>>
Время после определенного времени
Практика времени после определенного времени с помощью этого бесплатного печатного листа по математике.
Распечатать здесь >>>
Рабочие листы по математике для 5-го класса — Поскольку у нас есть базовые навыки, которым ребенок учится в 5-м классе, мы взяли на себя смелость разбить их и отсортировать по группам в соответствии с их ответвлениями. Это позволяет нам разработать более простой подход для студентов, таким образом, мы разработали рабочие листы для каждой категории и ее подкатегории, если это необходимо. Так как в этом году учащиеся будут изучать дроби, поднимая десятичные операции на новый уровень; им понадобится способ практиковать и совершенствовать свои концепции. Именно поэтому мы разработали наши рабочие листы с учетом национальной учебной программы, чтобы убедиться, что мы помогаем детям практиковать и развивать правильные методы. Думайте об этих задачах как о коде, который будет взломан, как только ученик решит все задачи. Помня об этой концепции, мы разработали наши рабочие листы таким образом, чтобы они служили интересным инструментом; они будут восприняты как интересный рабочий лист. Эти рабочие листы помогут учащемуся сосредоточиться на поставленной задаче и убедиться, что он охватил аспект и понял концепцию. Наши рабочие листы помогут им усовершенствовать навыки и методы, которые они изучили, и отшлифовать их дальше. Это простой, но эффективный метод, когда ребенок во что-то вникает и получает удовольствие от процесса; учатся быстрее и возвращаются, чтобы учиться с большей охотой. Как только они узнают, что они хороши в чем-то, они будут стремиться стать лучше. Мы разработали эти рабочие листы для детей в возрасте 9 лет.до 10, а в соответствии с национальной учебной программой учащиеся должны были освоить основные действия с десятичной дробью, а также начать знакомиться с дробями и процентами. Поэтому мы разработали наши рабочие листы, чтобы помочь учащимся в различных аспектах и на разных уровнях обучения.
В 5-м классе у вашего ребенка будет наблюдаться стабильное развитие логических, математических и критических способностей мышления. Объем учебной программы этого года будет отражать такой рост. Он закрепит понятия, которые ребенок усвоит в начальной школе, и заложит основу для его математического образования в средней школе. Если ваш ребенок следует регулярному режиму изучения и применения своих математических знаний, он будет на пути к достижению компетенций в математике 5-го класса . Как родители, так и учителя могут подтвердить, что обучение детей может быть довольно тяжелым испытанием, особенно когда речь идет о подготовке планов уроков и материалов. Признавая эту трудность, мы предоставляем достаточную коллекцию листов по математике для 5-го класса для ваших учебных нужд. Учебная программа обычно начинается с рассмотрения простейших элементарных математических тем, включая разрядные значения (которые могут достигать миллиона разрядных значений) и многозначные задачи на четыре арифметических действия. Усвоение таблицы умножения к 5-му классу создаст надежную основу для того, чтобы ваш ребенок с легкостью справлялся с арифметическими задачами до конца своего учебного пути. Таблица умножения также поможет быстрее узнать о множителях и множителях различных чисел. Ваш ребенок обнаружит отношения между числами благодаря их наибольшим общим делителям и наименьшим общим кратным. В этом году мы углубимся в дроби и десятичные дроби, а также в связь между ними. Дети будут практиковаться в том, как умножать дроби на целые числа или другие дроби, а также преобразовывать неправильные и смешанные дроби друг в друга. На этом этапе дети могут мысленно вычислять многозначные операции, содержащие дроби. Геометрия на этом уровне заключается в том, чтобы больше узнать о свойствах 2D- и 3D-форм, а также об их преобразованиях, обычно состоящих из вращений, отражений, расширений, а также перемещений. Кроме того, задачи на объем, площадь и периметр останутся главной особенностью геометрии 5-го класса. Задачи, содержащие несколько арифметических действий и групп, станут более распространенными в 5-м классе. Чтобы вашего ребенка не смущали такие сложные проблемы, он будет знакомиться с порядком действий. В зависимости от вашего региона это может быть PEMDAS, GEMDAS, BEDMAS или другие варианты. Несмотря на изменение названия, все эти аббревиатуры относятся к одному и тому же стандартному порядку выполнения нескольких операций. 906:55 В течение года уроки вероятности и анализа данных снабдят вашего ребенка способностью рассчитывать и сравнивать шансы и определять вероятность определенных результатов. Они узнают, как представлять указанные данные в виде графиков и текста. Уроки логики, рассуждений и подготовки доказательств в конце года помогут вашему ребенку развить алгебраическое мышление для средней школы. Вы всегда можете загрузить и распечатать наши рабочие листы по математике для 5-го класса в соответствии с потребностями вашего ребенка в обучении. Используйте их в качестве шаблонов для следующей серии викторин или украсьте их более сложными вопросами перед экзаменами. Вы можете раздавать их в качестве заданий или сидячих мест или даже проводить групповые занятия по решению некоторых из наших особенно сложных рабочих листов.
Рабочие листы по математике для 5-го класса: БЕСПЛАТНО и для печати
Если вы ищете полную коллекцию БЕСПЛАТНЫХ печатных рабочих листов по математике для 5-го класса, которые помогут вашим ученикам повторить основные математические темы, то эта страница как раз для вас.
Здесь вы найдете большую коллекцию бесплатных математических упражнений и рабочих листов, которые помогут вашим пятиклассникам подготовиться и попрактиковаться.
Нажмите на каждую тему и загрузите рабочий лист по математике для учащихся 5-х классов.
Существует также БЕСПЛАТНО
Надеюсь, вам понравится!
ВАЖНО: УСЛОВИЯ АВТОРСКОГО ПРАВА: Эти листы предназначены для личного использования. Рабочие листы нельзя загружать в Интернет в любой форме, включая классные/личные веб-сайты или сетевые диски. Вы можете скачать рабочие листы и распечатать их столько, сколько вам нужно. У вас есть разрешение на распространение печатных копий среди ваших учеников, учителей, наставников и друзей.
У вас НЕТ разрешения на отправку этих листов кому бы то ни было (по электронной почте, текстовым сообщениям или другими способами). Они ДОЛЖНЫ загрузить рабочие листы самостоятельно. Вы можете отправить адрес этой страницы своим ученикам, репетиторам, друзьям и т. д.
Рабочие листы по математике для 6 класса
Рабочие листы по математике для 7 класса
Рабочие листы по математике для 8 класса
Абсолютная лучшая книга
для ACE 5 -го класса математика Тест
5 -й класс.
Нечетные или четные
Сложение и вычитание
Сложение двузначных чисел
Вычитание двузначных чисел
Сложение трехзначных чисел
Adding Hundreds
Adding 4–Digit Numbers
Subtracting 4–Digit Numbers
Multiplication and Division
Multiplication
Division
Long Division by One Digit
Division with Remainders
Mixed operations
Округление и оценка
Оценка сумм
Оценка разностей
Оценка произведений
Пропущенные числа
Теория номеров
Факторинговые номера
Прайс -факторизация
Наибольший общий фактор
Наименее распространенный множество
Правила дивизии
и графики
Граф.
Линейные графики
Гистограммы
Шаблоны и последовательности
Повторяющийся шаблон
Шаблоны роста
Patterns: Numbers
Money
Add Money Amounts
Subtract Money Amounts
Money: Word Problems
Measurement
Convert Measurement Units
Metric Units
Distance Measurement
Weight Measurement
Время
Чтение часов
Цифровые часы
Измерение времени
Лучшая книга, которая поможет вам добиться успеха на математическом тесте 5 -го класса
Геометрический
Идентификации углов: острые, правые, тупые и прямые углы
Обработки
Triangles
Кв. Периметр и площадь квадратов
Периметр и площадь прямоугольников
Найдите площадь или длину пропущенной стороны прямоугольника
Площадь и периметр
Объем кубов
Three-Dimensional Figures
Identify Three–Dimensional Figures
Count Vertices, Edges, and Faces
Identify Faces of Three–Dimensional Figures
Symmetry and Lines
Line Segments
Identify Lines of Symmetry
Подсчет линий симметрии
Параллельные, перпендикулярные и пересекающиеся прямые
Дроби
Сложение дробей с одинаковыми знаменателями
Вычитание дробей с одинаковыми знаменателями
Сложение и вычитание дробей с одинаковыми знаменателями
Сравнение сумм и разностей дробей с одинаковыми знаменателями
Сложение 3 или более дробей с одинаковыми знаменателями
Сложение дробей со знаменателем 10 и 100
Сложение и вычитание дробей со знаменателем 10, 100 и 1000
Mixed Numbers
Fractions to Mixed Numbers
Mixed Numbers to Fractions
Add and Subtract Mixed Numbers
Multiplying and Dividing Mixed Numbers
Proportions, Ratios, and Percent
Simplifying Ratios
Proportional Ratios
Словесные задачи на отношения и оценки
Десятичные дроби
Десятичные разряды
Упорядочивание и сравнение десятичных дробей
Добавление и вычитание десятичных декораций
Умножение и деление десятичных дел
Единственный ресурс, который ваш ученик когда -либо нуждается в ACE 5 -й класс Math Math 5 -й класс 5 -й класс 5th Math Math 5 -й класс.
Похоже, что здесь ничего нет…Может, попробуете воспользоваться поиском?
Искать:
Свежие записи
Куда сходить с детьми в Каспийске
Викторина про животных для детей 7-10 лет, с ответами
Куда сходить с ребенком в Ессентуках
Второстепенные члены предложения: таблица
Куда сходить с ребенком в Мытищах
Архивы
Архивы
Выберите месяц Ноябрь 2022 Октябрь 2022 Сентябрь 2022 Август 2022 Июль 2022 Июнь 2022 Май 2022 Апрель 2022 Март 2022 Февраль 2022 Январь 2022 Декабрь 2021 Ноябрь 2021 Октябрь 2021 Сентябрь 2021 Август 2021 Июль 2021 Июнь 2021 Май 2021 Апрель 2021 Март 2021 Февраль 2021 Январь 2021 Декабрь 2020 Ноябрь 2020 Октябрь 2020 Сентябрь 2020 Август 2020 Июль 2020 Июнь 2020 Май 2020 Апрель 2020 Март 2020 Февраль 2020 Январь 2020 Декабрь 2019 Ноябрь 2019 Октябрь 2019 Сентябрь 2019 Август 2019 Июль 2019 Июнь 2019 Май 2019 Апрель 2019 Март 2019 Февраль 2019 Январь 2019 Декабрь 2018 Ноябрь 2018 Октябрь 2018 Сентябрь 2018 Август 2018 Июль 2018 Июнь 2018 Май 2018 Апрель 2018 Март 2018 Февраль 2018 Январь 2018 Декабрь 2017 Ноябрь 2017 Октябрь 2017 Сентябрь 2017 Август 2017 Июль 2017 Июнь 2017 Май 2017 Апрель 2017 Март 2017 Февраль 2017 Январь 2017 Декабрь 2016 Ноябрь 2016 Октябрь 2016 Сентябрь 2016 Август 2016 Июль 2016 Июнь 2016 Май 2016 Апрель 2016 Март 2016 Февраль 2016 Январь 2016 Декабрь 2015 Ноябрь 2015 Октябрь 2015 Сентябрь 2015 Август 2015 Июль 2015 Июнь 2015
Рубрики
РубрикиВыберите рубрикуАНАЛИЗЫАФИША СОБЫТИЙБЕЗОПАСНОСТЬ РЕБЕНКАБЕРЕМЕННОСТЬ И РОДЫБОЛЕЗНИ У ДЕТЕЙВОПРОСЫ-ОТВЕТЫДЕТСКАЯ АПТЕКАДЕТСКАЯ КОМНАТАДЕТСКАЯ ЛОГОПЕДИЯДЕТСКАЯ ПСИХОЛОГИЯДЕТСКАЯ СТОМАТОЛОГИЯДОМ И РЕБЕНОКЗДОРОВЬЕ РЕБЕНКАМЕТОДИКИНАЧАЛЬНАЯ ШКОЛАНОВОРОЖДЕННЫЙОБУЧЕНИЕ И ОБРАЗОВАНИЕОСНОВНАЯ ШКОЛАПАТРИОТИЧЕСКОЕПИТАНИЕ РЕБЕНКАПОЛОВОЕПРИВИВКИПРОФОРИЕНТАЦИЯПСИХОЛОГИЯ И РАЗВИТИЕПУТЕШЕСТВИЯ И ОТДЫХРАЗВИТИЕ И ОБУЧЕНИЕРЕЛИГИОЗНОЕРЕЦЕПТЫСАМОСТОЯТЕЛЬНАЯ РАБОТАСЕМЕЙНОЕСТАРШИЕ КЛАССЫТВОРЧЕСТВОТОВАРЫ ДЛЯ ДЕТЕЙФИЗИЧЕСКОЕ ВОСПИТАНИЕФИНАНСОВОЕЧЕМ ЗАНЯТЬ РЕБЕНКАЭТИЧЕСКОЕ
Метки
1 класс
2 класс
3 класс
4 класс
5 класс
6 класс
7 класс
8 класс
9 класс
10 класс
11 класс
Викторины
География
ДОУ
Дошкольный возраст (3-7 лет)
Игры
История
Литература
Математика
Младенчество (до 1 года)
Младший школьный возраст (7-12 лет)
Подростковый возраст (12-15 лет)
Ранний возраст (1-3 года)
Русский язык
СЛАЙДЕР
Страницы
Вопросы-ответы
Часто задаваемые вопросы: список
Дом и ребенок
Детская комната
Товары для детей
Досуг
Афиша событий
Путешествия и отдых
Чем занять ребенка
Творчество
Здоровье
Безопасность ребенка
Беременность и роды
Болезни у детей
Здоровье ребенка
Анализы
Детская аптека
Детская стоматология
Новорожденный
Прививки
Питание ребенка
Рецепты
О персональных данных пользователей
Обратная связь
Обучение и образование
Начальная школа
Самостоятельная работа
Основная школа
Старшие классы
Профориентация
От редакции
Пользовательское соглашение
Психология и развитие
Воспитание ребенка
Патриотическое
Половое
Религиозное
Семейное
Финансовое
Этическое
Детская психология
Методики
Развитие и обучение
Детская логопедия
Физическое воспитание
Рубрики
Страница не найдена — Детский возраст
Похоже, что здесь ничего нет. ..Может, попробуете воспользоваться поиском?
Искать:
Свежие записи
Куда сходить с детьми в Каспийске
Викторина про животных для детей 7-10 лет, с ответами
Куда сходить с ребенком в Ессентуках
Второстепенные члены предложения: таблица
Куда сходить с ребенком в Мытищах
Архивы
Архивы
Выберите месяц Ноябрь 2022 Октябрь 2022 Сентябрь 2022 Август 2022 Июль 2022 Июнь 2022 Май 2022 Апрель 2022 Март 2022 Февраль 2022 Январь 2022 Декабрь 2021 Ноябрь 2021 Октябрь 2021 Сентябрь 2021 Август 2021 Июль 2021 Июнь 2021 Май 2021 Апрель 2021 Март 2021 Февраль 2021 Январь 2021 Декабрь 2020 Ноябрь 2020 Октябрь 2020 Сентябрь 2020 Август 2020 Июль 2020 Июнь 2020 Май 2020 Апрель 2020 Март 2020 Февраль 2020 Январь 2020 Декабрь 2019 Ноябрь 2019 Октябрь 2019 Сентябрь 2019 Август 2019 Июль 2019 Июнь 2019 Май 2019 Апрель 2019 Март 2019 Февраль 2019 Январь 2019 Декабрь 2018 Ноябрь 2018 Октябрь 2018 Сентябрь 2018 Август 2018 Июль 2018 Июнь 2018 Май 2018 Апрель 2018 Март 2018 Февраль 2018 Январь 2018 Декабрь 2017 Ноябрь 2017 Октябрь 2017 Сентябрь 2017 Август 2017 Июль 2017 Июнь 2017 Май 2017 Апрель 2017 Март 2017 Февраль 2017 Январь 2017 Декабрь 2016 Ноябрь 2016 Октябрь 2016 Сентябрь 2016 Август 2016 Июль 2016 Июнь 2016 Май 2016 Апрель 2016 Март 2016 Февраль 2016 Январь 2016 Декабрь 2015 Ноябрь 2015 Октябрь 2015 Сентябрь 2015 Август 2015 Июль 2015 Июнь 2015
Рубрики
РубрикиВыберите рубрикуАНАЛИЗЫАФИША СОБЫТИЙБЕЗОПАСНОСТЬ РЕБЕНКАБЕРЕМЕННОСТЬ И РОДЫБОЛЕЗНИ У ДЕТЕЙВОПРОСЫ-ОТВЕТЫДЕТСКАЯ АПТЕКАДЕТСКАЯ КОМНАТАДЕТСКАЯ ЛОГОПЕДИЯДЕТСКАЯ ПСИХОЛОГИЯДЕТСКАЯ СТОМАТОЛОГИЯДОМ И РЕБЕНОКЗДОРОВЬЕ РЕБЕНКАМЕТОДИКИНАЧАЛЬНАЯ ШКОЛАНОВОРОЖДЕННЫЙОБУЧЕНИЕ И ОБРАЗОВАНИЕОСНОВНАЯ ШКОЛАПАТРИОТИЧЕСКОЕПИТАНИЕ РЕБЕНКАПОЛОВОЕПРИВИВКИПРОФОРИЕНТАЦИЯПСИХОЛОГИЯ И РАЗВИТИЕПУТЕШЕСТВИЯ И ОТДЫХРАЗВИТИЕ И ОБУЧЕНИЕРЕЛИГИОЗНОЕРЕЦЕПТЫСАМОСТОЯТЕЛЬНАЯ РАБОТАСЕМЕЙНОЕСТАРШИЕ КЛАССЫТВОРЧЕСТВОТОВАРЫ ДЛЯ ДЕТЕЙФИЗИЧЕСКОЕ ВОСПИТАНИЕФИНАНСОВОЕЧЕМ ЗАНЯТЬ РЕБЕНКАЭТИЧЕСКОЕ
Метки
1 класс
2 класс
3 класс
4 класс
5 класс
6 класс
7 класс
8 класс
9 класс
10 класс
11 класс
Викторины
География
ДОУ
Дошкольный возраст (3-7 лет)
Игры
История
Литература
Математика
Младенчество (до 1 года)
Младший школьный возраст (7-12 лет)
Подростковый возраст (12-15 лет)
Ранний возраст (1-3 года)
Русский язык
СЛАЙДЕР
Страницы
Вопросы-ответы
Часто задаваемые вопросы: список
Дом и ребенок
Детская комната
Товары для детей
Досуг
Афиша событий
Путешествия и отдых
Чем занять ребенка
Творчество
Здоровье
Безопасность ребенка
Беременность и роды
Болезни у детей
Здоровье ребенка
Анализы
Детская аптека
Детская стоматология
Новорожденный
Прививки
Питание ребенка
Рецепты
О персональных данных пользователей
Обратная связь
Обучение и образование
Начальная школа
Самостоятельная работа
Основная школа
Старшие классы
Профориентация
От редакции
Пользовательское соглашение
Психология и развитие
Воспитание ребенка
Патриотическое
Половое
Религиозное
Семейное
Финансовое
Этическое
Детская психология
Методики
Развитие и обучение
Детская логопедия
Физическое воспитание
Рубрики
Рабочие листы по математике для 5-го класса в формате pdf, экзаменационные листы по математике для 5-го класса
Десятичные модели
Практика десятичных моделей с помощью этого бесплатного печатного листа по математике.
Распечатать здесь >>>
Иллюстрированное десятичное упражнение
Попрактикуйтесь в иллюстрированном десятичном упражнении с помощью этого бесплатного печатного листа по математике.
Распечатать здесь >>>
Умножение десятичных дробей
Практика умножения десятичных дробей с помощью этого бесплатного математического листа, который можно распечатать.
Распечатать здесь >>>
Округлить десятичные дроби
Практика округления десятичных дробей с помощью этого бесплатного печатного листа по математике.
Распечатать здесь >>>
Вычитание с десятичными дробями
Практика вычитания с десятичными дробями с помощью этого бесплатного печатного листа по математике.
Распечатать здесь >>>
Деление в длинное деление
Практика деления в длинное в этом бесплатном математическом листе для печати.
Распечатать здесь >>>
Разделить 2 на однозначные числа
Потренируйтесь делить 2 на однозначные числа с помощью этого бесплатного печатного листа по математике.
Распечатать здесь >>>
Разделить 3 на однозначные числа
Потренируйтесь делить 3 на однозначные числа с помощью этого бесплатного печатного листа по математике.
Распечатать здесь >>>
Упражнение с делением по кругу
Практикуйте упражнение по упражнению с делением по кругу с помощью этого бесплатного распечатываемого листа по математике.
Распечатайте здесь >>>
Упражнение на соответствие делений
Попрактикуйтесь в упражнении на соответствие делений с помощью этого бесплатного печатного листа по математике.
Распечатать здесь >>>
Деление целых чисел на дроби
Практикуйте деление целых чисел на дроби с помощью этого бесплатного печатного листа по математике.
Распечатать здесь >>>
Упражнение с таблицей делений
Потренируйтесь выполнять упражнение с таблицей делений с помощью этого бесплатного печатного листа по математике.
Распечатать здесь >>>
Деление в длину с остатком
Практикуйте деление в длину с остатком с помощью этого бесплатного печатного листа по математике.
Распечатать здесь >>>
Деление в длину
Потренируйтесь в делении в длину с помощью этого бесплатного печатного листа по математике.
Распечатать здесь >>>
Сложение дробей
Практика сложения дробей с помощью этого бесплатного печатного листа по математике.
Распечатать здесь >>>
Сложение смешанных дробей
Попрактикуйтесь в сложении смешанных дробей с помощью этого бесплатного печатного листа по математике.
Распечатайте здесь >>>
Преобразование дробей в десятичные
Потренируйтесь преобразовывать дроби в десятичные с помощью этого бесплатного математического листа, который можно распечатать.
Распечатать здесь >>>
Деление дробей
Потренируйтесь в делении дробей с помощью этого бесплатного математического листа, который можно распечатать.
Распечатать здесь >>>
Деление смешанных дробей
Практика деления смешанных дробей с помощью этого бесплатного математического листа, который можно распечатать.
Распечатать здесь >>>
Эквивалентные дроби
Практикуйте эквивалентные дроби с помощью этого бесплатного печатного листа по математике.
Распечатать здесь >>>
Дробь чисел
Практикуйте дроби чисел с помощью этого бесплатного печатного листа по математике.
Распечатать здесь >>>
Умножение смешанных дробей
Потренируйтесь в умножении смешанных дробей с помощью этого бесплатного математического листа, который можно распечатать.
Распечатать здесь >>>
Упростить дроби
Практикуйтесь в упрощении дробей с помощью этого бесплатного математического листа, который можно распечатать.
Распечатать здесь >>>
Вычитание дробей
Практика вычитания дробей с помощью этого бесплатного печатного листа по математике.
Распечатать здесь >>>
Вычитание смешанных дробей
Попрактикуйтесь в вычитании смешанных дробей с помощью этого бесплатного печатного листа по математике.
Распечатать здесь >>>
Площадь кругов 2
Попрактикуйтесь в области кругов 2 с помощью этого бесплатного печатного листа по математике.
Распечатать здесь >>>
Площадь кругов
Практика площади кругов с помощью этого бесплатного печатного листа по математике.
Распечатать здесь >>>
Окружность круга
Попрактикуйтесь в измерении окружности с помощью этого бесплатного математического листа, который можно распечатать.
Распечатайте здесь >>>
Определение сегментов геометрии
Попрактикуйтесь в определении сегментов геометрии с помощью этого бесплатного математического листа, который можно распечатать.
Распечатать здесь >>>
Измерение углов транспортиром
Попрактикуйтесь в измерении углов транспортиром с помощью этого бесплатного печатного листа по математике.
Распечатать здесь >>>
Периметр и площадь сложных фигур
Практикуйте периметр и площадь сложных фигур с помощью этого бесплатного математического листа для печати.
Распечатать здесь >>>
Периметр и площадь L-образных фигур
Попрактикуйтесь в периметре и площади L-образных фигур с помощью этого бесплатного математического листа для печати.
Распечатать здесь >>>
Периметр и площадь прямоугольников
Попрактикуйтесь в периметре и площади прямоугольников с помощью этого бесплатного печатного листа по математике.
Распечатать здесь >>>
Многоугольники Стороны Вершины
Попрактикуйтесь в построении сторон многоугольников с помощью этого бесплатного математического листа, который можно распечатать.
Распечатать здесь >>>
Теорема Пифагора
Практикуйте теорему Пифагора с помощью этого бесплатного печатного листа по математике.
Распечатайте здесь >>>
Фигуры
Попрактикуйтесь в фигурах с помощью этого бесплатного математического листа, который можно распечатать.
Распечатать здесь >>>
Твердые и плоские фигуры
Практикуйте объемные и плоские фигуры с помощью этого бесплатного математического листа, который можно распечатать.
Распечатать здесь >>>
Симметричные фигуры
Практикуйте симметричные фигуры с помощью этого бесплатного печатного листа по математике.
Распечатать здесь >>>
Симметричные объекты из реальной жизни
Практикуйте симметричные объекты из реальной жизни с помощью этого бесплатного печатного листа по математике.
Распечатать здесь >>>
Симметрия с фигурами
Практикуйте симметрию с фигурами с помощью этого бесплатного математического листа, который можно распечатать.
Распечатайте здесь >>>
Практический лист по гистограммам и линейным графикам
Практический лист по гистограммам и линейным графикам с помощью этого бесплатного печатного листа по математике.
Распечатайте здесь >>>
Координаты
Практика координат с помощью этого бесплатного печатного листа по математике.
Распечатать здесь >>>
Свобода настройки графика
Практикуйте свободу настройки графика с помощью этого бесплатного печатного листа по математике.
Распечатайте здесь >>>
Графики и данные
Практикуйте графики и данные с помощью этого бесплатного печатного листа по математике.
Распечатать здесь >>>
Графики, расположенные по X Y
Практические графики, расположенные по X и Y с помощью этого бесплатного печатного листа по математике.
Распечатать здесь >>>
Линейные графики
Попрактикуйтесь в построении линейных графиков с помощью этого бесплатного математического листа, который можно распечатать.
Распечатать здесь >>>
Построение графиков
Потренируйтесь строить графики с помощью этого бесплатного математического листа, который можно распечатать.
Распечатать здесь >>>
Таблицы
Практические таблицы с этой бесплатной печатной таблицей по математике.
Распечатать здесь >>>
Диаграммы Венна 2
Попрактикуйтесь в диаграммах Венна 2 с помощью этого бесплатного математического листа, который можно распечатать.
Распечатайте здесь >>>
Диаграммы Венна
Попрактикуйтесь в построении диаграмм Венна с помощью этого бесплатного печатного листа по математике.
Распечатайте здесь >>>
X Y Упражнение с координатами
Упражнение с координатами x y с этим бесплатным распечатываемым математическим заданием.
Распечатать здесь >>>
Преобразование весов мм см фут ярд
Попрактикуйтесь в переводе весов мм см фут ярд с помощью этого бесплатного математического листа, который можно распечатать.
Распечатать здесь >>>
Измерение углов
Практика измерения углов с помощью этого бесплатного математического листа, который можно распечатать.
Распечатать здесь >>>
Измерение прямоугольников и вычисление площади
Практикуйтесь в измерении прямоугольников и вычислении площади с помощью этого бесплатного математического листа, который можно распечатать.
Распечатать здесь >>>
Измерения термометра
Попрактикуйтесь в измерениях термометра с помощью этого бесплатного распечатываемого листа по математике.
Распечатать здесь >>>
Операции смешивания
Попрактикуйтесь в операциях смешивания с помощью этого бесплатного печатного листа по математике.
Распечатать здесь >>>
Умножение на однозначные числа
Потренируйтесь умножать на однозначные числа с помощью этого бесплатного печатного листа по математике.
Распечатать здесь >>>
Умножение на двузначные числа
Практика умножения на двузначные числа с помощью этого бесплатного печатного листа по математике.
Распечатайте здесь >>>
Сопоставление умножения
Потренируйтесь сопоставлять умножение с помощью этого бесплатного печатного листа по математике.
Распечатать здесь >>>
Умножение дробей
Практика умножения дробей с помощью этого бесплатного математического листа, который можно распечатать.
Распечатать здесь >>>
Упражнение с таблицей умножения
Попрактикуйтесь в упражнении с таблицей умножения с помощью этого бесплатного печатного листа по математике.
Распечатать здесь >>>
Умножение с денежными значениями
Попрактикуйтесь в умножении с денежными значениями с помощью этого бесплатного печатного листа по математике.
Распечатайте здесь >>>
Шаблоны счета
Потренируйтесь в счете шаблонов с помощью этого бесплатного печатного листа по математике.
Распечатайте здесь >>>
Коэффициенты
Попрактикуйтесь в коэффициентах с помощью этого бесплатного распечатываемого листа по математике.
Распечатать здесь >>>
Наибольший общий делитель
Практикуйте наибольший общий делитель с помощью этого бесплатного математического листа, который можно распечатать.
Распечатайте здесь >>>
Наименьшее общее кратное
Практикуйте наименьшее общее кратное с помощью этого бесплатного печатного листа по математике.
Распечатать здесь >>>
Простые и составные числа
Практикуйте простые и составные числа с помощью этого бесплатного печатного листа по математике.
Распечатать здесь >>>
Римские цифры
Практикуйте римские цифры с помощью этого бесплатного печатного листа по математике.
Распечатать здесь >>>
Округлить числа до миллионов
Потренируйтесь округлять числа до миллионов с помощью этого бесплатного печатного листа по математике.
Распечатайте здесь >>>
Округление чисел
Попрактикуйтесь в округлении чисел с помощью этого бесплатного печатного листа по математике.
Распечатать здесь >>>
Научное обозначение 1
Потренируйтесь в научном обозначении 1 с помощью этого бесплатного печатного листа по математике.
Распечатать здесь >>>
Научное обозначение 2
Потренируйтесь в научном представлении 2 с помощью этого бесплатного печатного листа по математике.
Распечатать здесь >>>
Комплекс узоров
Практикуйте комплекс узоров с помощью этого бесплатного печатного листа по математике.
Распечатать здесь >>>
Patterns Easy
Практикуйте узоры легко с помощью этого бесплатного печатного листа по математике.
Распечатать здесь >>>
Вычитание до ста тысяч
Потренируйтесь вычитать до ста тысяч с помощью этого бесплатного печатного листа по математике.
Распечатайте здесь >>>
Вычитание до миллионов
Практика вычитания до миллионов с помощью этого бесплатного печатного листа по математике.
Распечатать здесь >>>
Упражнение на вычитание круга
Практикуйте упражнение на вычитание круга с помощью этого бесплатного печатного листа по математике.
Распечатать здесь >>>
Упражнение с таблицей вычитания
Попрактикуйтесь в упражнении с таблицей вычитания с помощью этого бесплатного печатного листа по математике.
Распечатайте здесь >>>
Определение времени до полудня
Практикуйтесь в определении времени до полудня с помощью этого бесплатного печатного листа по математике.
Распечатать здесь >>>
Часы с римскими цифрами для определения времени
Практикуйтесь в определении времени по часам с римскими цифрами с помощью этого бесплатного печатного листа по математике.
Распечатать здесь >>>
Время после определенного времени
Практика времени после определенного времени с помощью этого бесплатного печатного листа по математике.
Распечатать здесь >>>
Рабочие листы по математике для 5-го класса — Поскольку у нас есть базовые навыки, которым ребенок учится в 5-м классе, мы взяли на себя смелость разбить их и отсортировать по группам в соответствии с их ответвлениями. Это позволяет нам разработать более простой подход для студентов, таким образом, мы разработали рабочие листы для каждой категории и ее подкатегории, если это необходимо. Так как в этом году учащиеся будут изучать дроби, поднимая десятичные операции на новый уровень; им понадобится способ практиковать и совершенствовать свои концепции. Именно поэтому мы разработали наши рабочие листы с учетом национальной учебной программы, чтобы убедиться, что мы помогаем детям практиковать и развивать правильные методы. Думайте об этих задачах как о коде, который будет взломан, как только ученик решит все задачи. Помня об этой концепции, мы разработали наши рабочие листы таким образом, чтобы они служили интересным инструментом; они будут восприняты как интересный рабочий лист. Эти рабочие листы помогут учащемуся сосредоточиться на поставленной задаче и убедиться, что он охватил аспект и понял концепцию. Наши рабочие листы помогут им усовершенствовать навыки и методы, которые они изучили, и отшлифовать их дальше. Это простой, но эффективный метод, когда ребенок во что-то вникает и получает удовольствие от процесса; учатся быстрее и возвращаются, чтобы учиться с большей охотой. Как только они узнают, что они хороши в чем-то, они будут стремиться стать лучше. Мы разработали эти рабочие листы для детей в возрасте 9 лет.до 10, а в соответствии с национальной учебной программой учащиеся должны были освоить основные действия с десятичной дробью, а также начать знакомиться с дробями и процентами. Поэтому мы разработали наши рабочие листы, чтобы помочь учащимся в различных аспектах и на разных уровнях обучения.
В 5-м классе у вашего ребенка будет наблюдаться стабильное развитие логических, математических и критических способностей мышления. Объем учебной программы этого года будет отражать такой рост. Он закрепит понятия, которые ребенок усвоит в начальной школе, и заложит основу для его математического образования в средней школе. Если ваш ребенок следует регулярному режиму изучения и применения своих математических знаний, он будет на пути к достижению компетенций в математике 5-го класса . Как родители, так и учителя могут подтвердить, что обучение детей может быть довольно тяжелым испытанием, особенно когда речь идет о подготовке планов уроков и материалов. Признавая эту трудность, мы предоставляем достаточную коллекцию листов по математике для 5-го класса для ваших учебных нужд. Учебная программа обычно начинается с рассмотрения простейших элементарных математических тем, включая разрядные значения (которые могут достигать миллиона разрядных значений) и многозначные задачи на четыре арифметических действия. Усвоение таблицы умножения к 5-му классу создаст надежную основу для того, чтобы ваш ребенок с легкостью справлялся с арифметическими задачами до конца своего учебного пути. Таблица умножения также поможет быстрее узнать о множителях и множителях различных чисел. Ваш ребенок обнаружит отношения между числами благодаря их наибольшим общим делителям и наименьшим общим кратным. В этом году мы углубимся в дроби и десятичные дроби, а также в связь между ними. Дети будут практиковаться в том, как умножать дроби на целые числа или другие дроби, а также преобразовывать неправильные и смешанные дроби друг в друга. На этом этапе дети могут мысленно вычислять многозначные операции, содержащие дроби. Геометрия на этом уровне заключается в том, чтобы больше узнать о свойствах 2D- и 3D-форм, а также об их преобразованиях, обычно состоящих из вращений, отражений, расширений, а также перемещений. Кроме того, задачи на объем, площадь и периметр останутся главной особенностью геометрии 5-го класса. Задачи, содержащие несколько арифметических действий и групп, станут более распространенными в 5-м классе. Чтобы вашего ребенка не смущали такие сложные проблемы, он будет знакомиться с порядком действий. В зависимости от вашего региона это может быть PEMDAS, GEMDAS, BEDMAS или другие варианты. Несмотря на изменение названия, все эти аббревиатуры относятся к одному и тому же стандартному порядку выполнения нескольких операций. 906:55 В течение года уроки вероятности и анализа данных снабдят вашего ребенка способностью рассчитывать и сравнивать шансы и определять вероятность определенных результатов. Они узнают, как представлять указанные данные в виде графиков и текста. Уроки логики, рассуждений и подготовки доказательств в конце года помогут вашему ребенку развить алгебраическое мышление для средней школы. Вы всегда можете загрузить и распечатать наши рабочие листы по математике для 5-го класса в соответствии с потребностями вашего ребенка в обучении. Используйте их в качестве шаблонов для следующей серии викторин или украсьте их более сложными вопросами перед экзаменами. Вы можете раздавать их в качестве заданий или сидячих мест или даже проводить групповые занятия по решению некоторых из наших особенно сложных рабочих листов.
⭐ БЕСПЛАТНЫЕ рабочие листы для 5-го класса
Рабочие листы для 5-го класса
У нас есть МНОГО веселых, образовательных и бесплатных печатных заданий для 5-го класса для отработки различных навыков. Рабочие листы для 5 класса включают Рабочие листы по математике для 5 класса и математические игры для пятого класса для изучения сложения, вычитания, умножения, деления, десятичных дробей, текстовых задач, пропуска счета и многого другого! У нас также есть много забавных игр 5th class English и рабочих листов, чтобы практиковать части речи, использовать словарь и многое другое. Кроме того, ознакомьтесь с нашими музыкальными печатными изданиями, 5-й класс по естественным наукам , музыкальными заданиями, художественными проектами и 5-й класс по общественным наукам .
Рабочие листы для 5 класса
Итак, у вас есть пятиклассник ? Являетесь ли вы учеником на дому, учителем или родителем, дополняющим образование вашего ребенка — у нас есть много веселых, творческих и бесплатных рабочих листов для пятого класса для вас! Эта страница заполнена более чем 2000 страницами заданий по математике для 5 класса , математики для 5 класса игр, занятий по английскому языку для пятого класса , развлечений 5 класса по естествознанию , музыки, искусства и История 5 класса . Вот ваш универсальный магазин для всего 5 класса в моем блоге! Все мои неподготовительные, веселые игры для 5-го класса (включая настольные игры для печати, карточные игры, головоломки, клип-карты и т. д.) и практические занятия перечислены ниже… и все это БЕСПЛАТНО !! Просто нажмите на любую миниатюру ниже, чтобы увидеть больше и распечатать! Их МНОГО, так что продолжайте прокручивать.
Английский язык 5-го класса
Учащиеся 5-го класса продолжают читать более сложные предложения при чтении, расширяют словарный запас и пишут отчеты по книге, чтобы обеспечить понимание прочитанного. Кроме того, они продолжают повторять части речи и диграмматизировать предложения, синонимы, антонимы, приставки, суффиксы, предлоги.
Форма отчета о знаменитых артистах для детей
Список для чтения для 5-го класса для бесплатной печати
Милая головоломка синонимов для печати
Бесплатные рабочие листы для печати синонимов и антонимов
Активность антонимов глупых печатных животных
Окончания слов «Мороженое» Суффиксная активность
Шаблон отчета о сэндвич-книге
Шаблон отчета Simple Book
Закладки для понимания прочитанного
Кем были книги Рабочие листы
Пицца Понимание чтения 907:25
Научитесь пользоваться словарем вместе со словарями-детективами!
Решай части речевых игр для печати
Пиратские части речи, рабочие листы
Печатные части речи Бинго
500+ бесплатных советов по творческому письму для детей
Рабочие листы по математике для 5 класса
Пятиклассники работают над сложением, вычитанием, умножением и делением целых чисел, десятичных дробей и дробей в дополнение к решению реальных задач. Обычно дети продолжают работать с фигурами, чтобы заложить основу геометрии, знакомятся с алгеброй, продолжают работать над определением времени и дробей.
Рабочие листы по математике для 5-го класса с ключом ответа
Рабочие листы для дивизий Мини-книга
Флэш-карты Minion Division для печати
Игры бейсбольного дивизиона для печати
Рабочие листы для умножения колоды карт
Рабочие листы вероятностей конфет
Игры на умножение шашек для печати
Зонт для печати Умножение Ремесло
Игра в классики для печати на умножение
Дробная игра БИНГО 907:25
Пиратская игра на сокращение дробей
Простая игра с дробями, которую можно распечатать бесплатно
Настольная игра «Упрощение дробей»
Естествознание 5-го класса
EPIC, Проект человеческого тела в натуральную величину
Бесплатные рабочие листы клеток человека и модель клеток съедобного желе
Более 25 невероятно забавных систем человеческого тела для детских проектов
Части мозга для детей Занятие с моделью мозга из пластилина и бесплатными рабочими листами для изучения мозга для детей 907:25
Рабочая модель легких для детей и план урока
Bending Bones — эксперимент с человеческим телом по удалению кальция для детей
Пищеварительная система для детей Эксперимент
Кровь для детей — деятельность по изучению человеческого тела
Проект человеческого тела «Как работают легкие» для детей
Человеческое сердце для детей: 2 забавные модели сердца плюс рабочие листы
Дети будут поражены этими экспериментами по изучению зрения
Как работают руки — Мышцы человеческого тела для детей, проект 907:25
Какой длины кишечник у детей? Эксперимент с пищеварительной системой
Как работают мышцы — простая модель скелетных мышц, которая работает
Playdough Layers of the Earth Project
Водный цикл Lego Модель
Книга о круговороте воды для печати
Веселые рабочие листы для 5-го класса
Шаблон отчета о животных
Бесплатная игра для печати «Среда обитания животных»
Zoo Scavenger Hunt, чтобы узнать о среде обитания, уязвимости населения, терминах и многом другом 907:25
Загляните внутрь эксперимента с вулканом для детей
100 удивительных экспериментов с едой для детей
Easy Balloon State of Matter Activity
Бесплатные рабочие листы Solid Liquid Gas
Рабочие наборы по научному методу
Обществознание в 5-м классе
Изучайте американскую историю для детей, включая массу развлечений, практические занятия и бесплатные печатные материалы
Сделайте вигвам, откопайте наконечники стрел, раскрасьте наши печатные книги и многое другое в этом уроке «Коренные американцы для детей» 907:25
Урок Колониальная Америка для детей отправляет детей в путешествие к первому постоянному поселению в Америке — Джеймстауну. Учащиеся узнают о колониальной жизни, приняв участие в таких проектах, как поедание каши, строительство дома Wattle & Daub и создание жестяного фонаря, которые оживят поселение.
Мы оживим экспансию на запад для детей, поскольку они узнают факты о периоде времени примерно с 1783 по 1898 год. Помимо изучения Орегонской тропы, пони-экспресса, золотой лихорадки, трансконтинентальной железной дороги, учащиеся также научатся находить следы животных, составлять карту Соединенных Штатов и узнавать о животных Северной Америки. 907:25
Гражданская война для детей — студенты узнают о рабстве, знаменитых американцах времен Гражданской войны, знаменитых битвах и о том, как наша нация осталась вместе. Урок включает в себя распечатываемые рабочие листы, картографические задания, распечатываемую настольную игру о гражданской войне и многое другое!
Узнайте о штатах США для детей — раскраски штатов, раскраски президентов, формы докладов штатов, распечатки штатов и столиц и многое другое!
Плакаты с историческими вымпелами для краткого ознакомления с историей США
Узнайте о Ревущих 1920s for Kids — книга для печати, чтения, раскрашивания и обучения
Rosa Parks for Kids Читалка для распечатки и обучения
Мартин Лютер Кинг-младший для детей Читалка для распечатки, чтобы раскрашивать и учиться
Титаник для детей Читалка для раскрашивания и обучения
Узнайте о бурных 1920-х годах для детей — книга для печати, чтения и раскрашивания
1930-е годы для детей — книга для печати о золотом веке Голливуда, Великой депрессии для детей и многом другом, которую можно читать, раскрашивать и изучать 907:25
Жизнь в Америке 1940-х годов для детей
Anne Frank for Kids Printable Reader, чтобы читать, раскрашивать и узнавать об этой еврейской девочке, которая жила за стеной в Голландии во время Второй мировой войны
1950s for Kids Reader to Color, Learn и читайте о том, какой была жизнь в Америке в 1950 году
Жизнь в 1960-х для детей Книга для печати, чтобы читать, раскрашивать и узнавать о жизни в 1960-х
Узнайте об обуви на платформе, дискотеке, начале «Звездных войн» и многом другом из этого 1970-е для детей Читалка
1980-е для детей Чтение, раскрашивание и обучение!
1990s for Kids Печатная книга для чтения, наполненная информацией о моде, технологиях, президентах и многом другом!
Узнайте о жизни в начале 2000-х Дети с этими распечатками истории Америки, чтобы читать, раскрашивать и учиться!
Посмотрите, что вы узнали, используя эти рабочие листы по американской истории (записная книжка)
Карты для печати — бесплатные карты для печати, которые можно маркировать по своему усмотрению
Тридцать третий Математический праздник (27 февраля 2022 года) Решения в формате PDF Рабочие критерии проверки:
— «классический» Математический праздник
(6 класс и 7 класс)
— «Математический праздник в Математической вертикали»
(6 класс и 7 класс)
Тридцать второй Математический праздник (18 апреля 2021 года) Решения в формате PDF Рабочие критерии проверки:
6 класс,
7 класс «Математический праздник в Математической вертикали»:
6 класс, 7 класс Рабочие критерии проверки:
6 класс,
7 класс
Тридцать первый Математический праздник (09 февраля 2020 года) Решения в формате PDF Рабочие критерии проверки:
6 класс,
7 класс «Математический праздник в Математической вертикали»:
6 класс, 7 класс
Тридцатый Математический праздник (17 февраля 2019 года) Решения в формате PDF Рабочие критерии проверки:
6 класс,
7 класс
Двадцать девятый Математический праздник (18 февраля 2018 года) Решения в формате PDF Рабочие критерии проверки:
6 класс,
7 класс
Двадцать восьмой Математический праздник (19 февраля 2017 года) Решения в формате PDF Рабочие критерии проверки:
6 класс,
7 класс
Двадцать седьмой Математический праздник (21 февраля 2016 года) Решения в формате PDF Рабочие критерии проверки:
6 класс,
7 класс
Двадцать шестой Математический праздник (15 февраля 2015 года) Решения в формате PDF Рабочие критерии проверки:
6 класс,
7 класс
Двадцать пятый Математический праздник (16 февраля 2014 года) Решения в формате PDF (555 К, 12 страниц) Рабочие критерии проверки:
6 класс,
7 класс
Двадцать четвёртый Математический праздник (17 февраля 2013 года) Решения в формате PDF (308 К, 12 страниц) Рабочие критерии проверки:
6 класс, 7 класс
Двадцать третий Математический праздник (19 февраля 2012 года) Решения в формате PDF (486 К, 12 страниц) Рабочие критерии проверки:
6 класс, 7 класс
Двадцать второй Математический праздник (13 февраля 2011 года) Решения в формате PDF (141 К, 16 страниц) Рабочие критерии проверки:
6 класс, 7 класс
Двадцать первый Математический праздник (14 февраля 2010 года) Решения в формате PDF (96 К, 12 страниц)
Двадцатый Математический праздник (15 февраля 2009 года) Решения в формате PDF (102 К, 12 страниц)
Девятнадцатый Математический праздник (17 февраля 2008 года) Решения в формате PDF (132 К, 8 страниц)
Восемнадцатый Математический праздник (11 февраля 2007 года) 6 класс 7 класс Решения в формате PDF (98 Кб, 8 страниц)
Семнадцатый Математический праздник (12 февраля 2006 года) 6 класс 7 класс Решения в формате PDF (551 Кб, 8 страниц)
Шестнадцатый Математический праздник (13 февраля 2005 года) 6 класс 7 класс Решения в формате PDF (217 Кб, 8 страниц)
Пятнадцатый Математический праздник (15 февраля 2004 года) 6 класс 7 класс Решения в формате PDF (214 Кб, 7 страниц)
Четырнадцатый Математический праздник (16 февраля 2003 года) 6 класс 7 класс Решения в формате PDF (214 Кб, 8 страниц)
Тринадцатый Математический праздник (17 февраля 2002 года) 6 класс 7 класс Решения в формате PDF (131 Кб, 8 страниц)
Двенадцатый Математический праздник (18 февраля 2001 года) 6 класс 7 класс Решения в формате PDF (183 Кб, 4*2 страниц)
Одиннадцатый Математический праздник (13 февраля 2000 года) 6 класс 7 класс Решения в формате PDF (88 Кб, 6 страниц)
Десятый Математический праздник (21 февраля 1999 года) 6 класс 7 класс
Дата последнего изменения: 11 марта 2022 года
Как репетитор по математике в 7 классе работает с системами — Колпаков Александр Николаевич
Изучение математики в 7 классе принципиально отличается от всего того, что предлагалось ранее в 5 — 6 классах. И дело не только в названиях тем и разделении предмета на алгебру и геометрию. Помимо введения новых понятий и правил меняется характер работы с числами и выражениями. Многое из того, что репетитор по математике показывает в 7 классе является обобщением ранее пройденного, но поднимающее использование математики на принципиально новый уровень. Такое продвижение предполагает прочное усвоение вычислительной базы, которое к 7 классу должно быть достигнуто. Должно, но не обязано.
Значительные пробелы отстающих школьников, обращающихся к репетитору по математике в тот или иной период учебы в школе, ставят неразрешимые проблемы перед использованием традиционных методик объяснений, а именно методик прямого изложения материала. Креативный репетитор по математике находится в постоянном поиске новых форм и способов подачи объяснений конкретному ученику. И это очень непросто сделать.
Как репетитор по математике работает с трудными темами?
Трудность каждой конкретной темы — весьма относительное понятие. Все зависит от того, с какой стороны к ней подойти и насколько ученик способен воспринимать ту или иную форму объяснений репетитора. Многие сложные понятия упрощаются, если репетитору по математике удается подобрать какое-нибудь простое и лаконичное описание математического процесса, сравнить его с чем-то обыденным и понятным, связать новое с ранее изученным. Это непростая задача, но репетитору нужно стремиться к ее выполнению. В алгебре, при объяснении нового материала, бывает достаточно точно подобрать соответствующие примеры работы правила на числах. У многих учеников 7 класса все еще преобладает тип мышления «от общего к частному», поэтому, стремление репетитора по математике к абсолютной стрости и полноте объяснений (доказательств), к использованию общих форм, рассмотрению всех случаев или педантичной проверке равносильности в переходах может перечеркнуть все усилия по обеспечению понимания.
Важно добиться первоначального понимания, пусть ученику не открывается вся картина происходящего в алгоритме, а лишь приоткрывается некая завеса нового. В некоторых случаях уже одно такое продвижение можно ставить в заслугу репетитору по математике, ибо ребенок начинает хоть что-то решать самостоятельно. Это крайне важно, ибо результаты практической работы помогает осмыслить многие моменты, которые оказались непонятыми.
Иногда репетиторы по математике, особенно начинающие, путают два состояния ученика: не понял и не запомнил. Если ребенок говорит «я не понимаю», — это не всегда означает, что слова репетитора по математике остались не осмысленными. Часто бывает наоборот: заявляет, что все понятно, но на проверку оказывается, что он просто заучил те или иные ходы в решении. Репетитору важно уметь отличать эти два состояния и правильно их использовать.
Как правило, решение систем методом подстановки вызывает у детей 7 класса дикое отвращение и неприязнь. Почему? Процесс, который описывает репетитор по математике на первом уроке по данной теме, очень трудно увязывается с привычным занятием в алгебре 7 класса. Дети настолько привыкают к однострочным одношаговым решениям (какими являются преобразования многочленов). Поэтому, когда репетитор по математике исписывает равносильными системами целую страницу в тетради, ученик почти всегда заявляет: «я не понимаю». «Стоп! Давай разберемся», — говорю я ему. Что именно из этого ты не понял, а что просто не успел запомнить? Если репетитор по математике поставит вопрос именно таким ребром, он переводит деятельность ученика из созерцательной в оценочную. Нужно дать время на то, чтобы привыкнуть к записям и запомнить ходы. Это облегчит оценку того, что именно не понятно. Главное не торопить ученика и дать ему возможность высказаться. Пусть это будут невнятные фразы, лишенные логики. Мастерство репетитора заключается в том, чтобы выявить проблему даже по «обрывкам мыслей» ученика.
Конечно, я описываю ситуацию, в которой репетитор не провел с учеником соответствующую подготовительную работу. А она обязательно нужна.
Подготовительная работа репетитора
Нужны задания на проверку конкретных пар чисел для конкретной системы. В процессе выполнения простейших заданий репетитор обкатывает новую терминологию: пара чисел, удовлетворяющая системе, решение системы, проверка пары. Я еще употребляю фразы «вставка чисел», «вставка пары» Важно убедить ученика в том, что совсем не обязательно искать пару чисел, которая при вставке в начальную систему даст два верных равенства. Мы же ищем саму пару.
Самому слабому ученику достаточно сказать, что при замене одной системы на другую не меняется самое главное — ее ответ, поэтому не важно, какую именно систему решать. Пара, подходящая для одной из них — подойдет и для другой. Это можно просто проверить на конкретных числах. Надо чувствовать ученика и не ввязываться в объяснения равносильности переходов в 7 классе, какими бы точными они не были. Если все-таки репетитор по математике хочет донести до сознания ученика логику алгоритма, это нужно делать после того, как ученик его запомнит.
Если ученик более-менее толковый, репетитор по математике применяет числовую методику проверки равносильности. Покажу ее работу на примере
Пусть задана система линейных уравнений:
Как ее решить — все вы отлично знаете. Но как объяснить это решение слабому учащемуся? Вот она — головная боль для преподавателя. Дети в 7 классе не воспринимают общие математические методы доказательства равносильности, под лупой которых, конечно же, вся логика преобразований оказывается на поверхности.
Какие методики могут быть задействованы в принципе? Обычно репетитор по математике проводит равносильные преобразования по классической схеме:
Однако нельзя оставлять такую форму без каких-либо объяснений.
Что чаще всего не понятно ученику?
Как правило к моменту изучения темы «метод подстановки», учащиеся 7 класса уже имеют выражать переменную y через переменную x. Будем считать, что репетитор по математике решил эту проблему в теме «графический способ решения систем уравнений». Тогда самый непонятный ход — вставка выражения 3-x во второе уравнение системы на место x.
Как репетитор может объяснить замену игрека на 3-x ? Я покажу как можно работать со средним учеником по методике числовой проверки ( если ученик сильный — для него вполне подойдут строгие математические обоснования «в обе стороны»). Итак, рассмотрим равносильный переход между системами:
Надо убедить ученика в том, что одна и та же пара чисел (она предоставляется в готовом виде) превращает каждое равенство в верное. Репетитор говорит: «Давай проверим пару (2;1), то есть х=2; y=1. Вставим их на места букв в систему (1).Получим: Эти равенства верные, поэтому пара чисел (2;1) — решение системы (1). Но 1=3-2 и поэтому можно вместо единицы в нижнем уравнении написать в скобках (3-2). От этого при подсчете не изменится результат». Далее репетитор по математике меняет 1 на разность 3 — 2 и спрашивает ученика: «Какая запись получится, если мы задумаем эту же пару (2;1) вставить во вторую систему? Будут ли ее равенства верными? Конечно, ведь мы только что их проверили (в этот момент репетитор по математике показывает на записанную систему №3). Вставка пары (2;1) приводит нас к повторению той же самой записи, к копии уже проверенного равенства. Поэтому пара (2;1) будет еще и решением системы №2. Значит у них одинаковые ответы (понимаю, что вывод не выдерживает никакой критики с точки зрения строгой математики и проверка проведена в одну сторону, но дети проглотят такой маневр репетитора). Поэтому вместо того, чтобы решать первую систему, мы можем решать вторую и через нее искать эту пару (если она неизвестна).
Остальные равносильные преобразования репетитору по математике не составит труда объяснить. В них нет ничего нового. Обычное решение уравнения с одной переменной. Понятно, что икс должен быть корнем уравнения (2). Ученики в 7 классе обычно понимают, что его надо найти.
Замечу, что ответ нужно записывать не в виде x=2; y=1, а в виде пары (2;1). Это будет способствовать скорейшему формированию у ученика представления об ответе, как о некоторой точке координатной плоскости.
Репетитор по математике 7 класс — Колпаков А.Н. Москва, Строгино.
Wolfram|Alpha Примеры: Common Core Math: 7 класс
О-о! Wolfram|Alpha не работает без JavaScript.
Пожалуйста, включите JavaScript. Если вы не знаете, как это сделать, вы можете найти инструкции здесь. Как только вы это сделаете, обновите эту страницу, чтобы начать использовать Wolfram|Alpha.
Примеры для
В седьмом классе учащиеся начинают свободно оперировать выражениями и решать уравнения, в том числе с отрицательными числами и числами, выраженными в виде дробей или десятичных дробей. Студенты вычисляют с процентами и отношениями и описывают пропорциональные отношения, используя числовые, графические, алгебраические и словесные представления. Учащиеся анализируют и сравнивают данные из случайных выборок и обобщают, чтобы сделать выводы о популяциях. Учащиеся также работают с вероятностью, проводя вероятностные эксперименты и вычисляя вероятности с помощью вероятностных моделей. Учащиеся вычисляют площади, объемы и площади поверхности правильных и неправильных форм и решают задачи, связанные с углами, в том числе рисуют фигуры, соответствующие заданным геометрическим условиям, и классифицируют типы треугольников.
Стандарты Common Core
Получить информацию об Стандартах Common Core.
Поиск определенного стандарта:
CCSS. Math.Content.7.G.A.1Общий базовый стандарт седьмого класса EE.B.4
Поиск всех стандартов седьмого класса:
Общие базовые стандарты седьмого классаВыражения и уравнения
Работа с выражениями и решение уравнений и неравенства, в том числе с отрицательными числами.
Math.Content.7.SP.B.4 ): IQR {2, 4, 8, 2, 1} по сравнению с IQR {7, 8, 6, 9, 9}
Проведение вероятностных экспериментов (CCSS.Math.Content.7.SP.C.6):
20 подбрасываний монеты вероятность выпадения 7 на двух шестигранных костяхБольше примеров
Математика, 7 класс, Применение математики в работе, Уточнение задачи Навыки решения
CCSS.Math.Content.7.EE.B.4
7 класс, выражения и уравнения
Кластер: решение реальных и математических задач с использованием числовых и алгебраических выражений и уравнений
Стандарт: использование переменных для представления величин в реальной или математической задаче и построение простых уравнений и неравенств для решения проблемы, рассуждая о количествах.
Степень выравнивания:
Без рейтинга
(0 пользователей)
CCSS.Math.Content.7.GB.6
7 класс, геометрия
Кластер: решение реальных и математических задач, связанных с измерением угла, площади, площади поверхности и объема
Стандарт: решение реальных и математических задач, связанных с площадью, объемом и площадью поверхности двух- и трехмерных объемные объекты, состоящие из треугольников, четырехугольников, многоугольников, кубов и прямых призм.
Степень выравнивания:
Без рейтинга
(0 пользователей)
CCSS.Math.Content.7.EE.B.4a
7 класс, выражения и уравнения
Кластер: решение реальных и математических задач с использованием числовых и алгебраических выражений и уравнений
Стандарт: решение текстовых задач, приводящих к уравнениям вида px + q = r и p(x + q) = r, где p, q и r — конкретные рациональные числа. Решите уравнения этих форм бегло. Сравните алгебраическое решение с арифметическим, указав последовательность операций, используемых в каждом подходе. Например, периметр прямоугольника равен 54 см. Его длина составляет 6 см. Какова его ширина?
Степень выравнивания:
Без рейтинга
(0 пользователей)
CCSS.Math.Content.7.EE.B.3
7 класс, выражения и уравнения
Кластер: решение реальных и математических задач с использованием числовых и алгебраических выражений и уравнений
Стандарт: решение многоэтапных реальных и математических задач, поставленных с положительными и отрицательными числа, дроби и десятичные дроби), используя инструменты стратегически. Применяйте свойства операций как стратегии для расчетов с числами в любой форме; конвертировать между формами по мере необходимости; и оценить обоснованность ответов, используя умственные вычисления и стратегии оценки. Например: если женщина, зарабатывающая 25 долларов в час, получает повышение на 10%, она будет получать дополнительную 1/10 своей зарплаты в час, или 2,50 доллара, за новую зарплату в 27,50 долларов. Если вы хотите разместить полотенцесушитель 93/4 дюйма длиной в центре двери шириной 27 1/2 дюйма, вам нужно будет разместить планку примерно в 9 дюймах от каждого края; эту оценку можно использовать в качестве проверки точного вычисления.
Степень выравнивания:
Без рейтинга
(0 пользователей)
MCCRS.Math.Content.7.EE.B.3
7 класс
Область обучения: выражения и уравнения
Стандарт: решение реальных и математических задач с использованием числовых и алгебраических выражений и уравнений
Индикатор: Решайте многоэтапные задачи из реальной жизни и математические задачи, связанные с положительными и отрицательными рациональными числами в любой форме (целые числа, дроби и десятичные дроби), стратегически используя инструменты. Применяйте свойства операций как стратегии для расчетов с числами в любой форме; конвертировать между формами по мере необходимости; и оценить обоснованность ответов, используя умственные вычисления и стратегии оценки. Например: если женщина, зарабатывающая 25 долларов в час, получает повышение на 10%, она будет получать дополнительную 1/10 своей зарплаты в час, или 2,50 доллара, за новую зарплату в 27,50 долларов. Если вы хотите разместить полотенцесушитель 93/4 дюйма длиной в центре двери шириной 27 1/2 дюйма, вам нужно будет разместить планку примерно в 9 дюймах от каждого края; эту оценку можно использовать в качестве проверки точного вычисления.
Степень выравнивания:
Без рейтинга
(0 пользователей)
MCCRS.Math.Content.7.GB.6
7 класс
Область обучения: геометрия
Стандарт: решение реальных и математических задач, связанных с измерением угла, площади, площади поверхности и объема
Индикатор: Решение реальных и математических задач, связанных с площадью, объемом и площадью поверхности двух- и трехмерных объектов, состоящих из треугольников, четырехугольников, многоугольников, кубов и прямых призм.
Степень выравнивания:
Без рейтинга
(0 пользователей)
MCCRS.Math.Content.7.EE.B.4a
7 класс
Область обучения: выражения и уравнения
Стандарт: решение реальных и математических задач с использованием числовых и алгебраических выражений и уравнений
Индикатор: Решайте текстовые задачи, приводящие к уравнениям вида px + q = r и p(x + q) = r, где p, q и r — конкретные рациональные числа. Решите уравнения этих форм бегло. Сравните алгебраическое решение с арифметическим, указав последовательность операций, используемых в каждом подходе. Например, периметр прямоугольника равен 54 см. Его длина составляет 6 см. Какова его ширина?
Степень выравнивания:
Без рейтинга
(0 пользователей)
MCCRS.Math.Content.7.EE.B.4
7 класс
Область обучения: выражения и уравнения
Стандарт: решение реальных и математических задач с использованием числовых и алгебраических выражений и уравнений
Индикатор: использование переменных для представления величин в реальной или математической задаче и построение простых уравнений и неравенства для решения проблем, рассуждая о количествах.
Объём тела– величина, характеризующая часть пространства, занимаемую телом, и определяемая формой и линейными размерами этого тела.
Основные свойства объёма:
— равные тела имеют равные объёмы;
— если тело составлено из нескольких тел, то его объём равен сумме объёмов этих тел.
Основная литература:
Атанасян Л. С. и др. Математика: алгебра и начала математического анализа, геометрия. Геометрия. 10–11 классы [текст]: учеб. для общеобразоват. организаций: базовый и углубл. уровни – М.: Просвещение, 2014. – 255 с. С. 130–133.
Теоретический материал для самостоятельного изучения
С понятием объёмного тела, отличающегося от плоской фигуры, мы познакомились ещё в начальной школе.
Объёмом принято называть положительную величину, характеризующую часть пространства, занимаемую телом, и определяемую формой и линейными размерами этого тела.
Мы можем вычислить объём тела точно так же, как ранее находили площадь фигуры. Объём принято измерять в единицах измерения объёма (единицах измерения размера пространства, занимаемого телом), то есть в кубических метрах, сантиметрах, миллиметрах и так далее. За единицу измерения объёма можно принять куб с ребром 1 см, то есть, кубический сантиметр (обозначение: см3). По аналогии, можно за единицу измерения объёма принять кубический миллиметр (1 мм3), кубический метр (1 м3) и тому подобное.
Объём выражается в положительных числах. Это число показывает, сколько единиц измерения содержится в теле. Например, сколько кубических миллиметров в аквариуме, сколько кубических метровв бассейне и так далее.
Объём обозначается заглавной латинской буквой V.
Пример:
Объём книги400 кубических сантиметров запишут: V = 400см3.
Рассмотрим свойства объёмов.
Свойство № 1. Равные тела имеют равные объёмы. Это означает, что если два тела идентичны, то есть имеют равное количество единиц измерения и частей, то равны и их объёмы. Например, 2 одинаковых пакета молока равны в объёме.
Свойство № 2. Если тело составлено из нескольких тел, то его объём равен сумме объёмов этих тел.
Следствие из основных свойств объёмов.
Объём куба с ребром 1/n равен 1/n3
Доказательство. Рассмотрим куб, объём которого принят за единицу измерения объёмов, тоесть равный некоторому числукубических сантиметров. Его ребро равно единице измерения отрезков. Разобьём каждое ребро этого куба на произвольное количество частей – nтак, чтобы провести плоскости, перпендикулярные к этому ребру.
По второму свойству объёмов, сумма объёмов всех кубиков равна объёму всего куба (1 см3). Следовательно, поскольку мы разбили каждое ребро на n частей, то каждый маленький куб внутри большого куба будет иметь ребро
Объём каждого из маленьких кубиков при этом будет равен 1/n3.
Объём прямоугольного параллелепипеда
Теорема
Объём прямоугольного параллелепипеда равен произведению трёх его измерений.
Доказательство
Обозначимизмеренияпрямоугольного параллелепипеда P буквами a,b,c, его объём буквой V, и докажем, что V = a ∙ b ∙ c.
Рассмотрим два возможных случая.
Случай первый. Измерения a, b и c представляют собой конечные десятичные дроби, у которых число знаков после запятой не превосходит n (можно считать, что n больше или равно 1). В этом случае числа a ∙10n, b∙10n, c∙10n, являются целыми. Разобьём каждое ребро параллелепипеда на равные части длины: 1/10n и через точки разбиения проведём плоскости, перпендикулярные к этому ребру. Параллелепипед P разобьётся на abc∙103n равных кубов с ребром 1/10n. Так как объём каждого куба равен 1/103n, что мы доказали ранее, то объём всего параллелепипеда P = abc, что и требовалось доказать.
Случай второй.
Хотя бы одно из измерений a, b, c представляет собой бесконечную десятичную дробь. Рассмотрим конечные десятичные дроби: an, bn, cn, которые получаются из чисел a, b, c, если отбросить в каждом из них все цифры после запятой, начиная с n + 1. Очевидно, an ≤ a ≤ an’, где an’ = an+1 : 10n. Аналогичные неравенства справедливы для b и c. Перемножив эти неравенства, получим произведение anbncn ≤ abc ≤ an’bn’cn’, где bn’= bn+1 : 10n, cn’ = cn+1 : 10n
По доказанному в первом случае, левая часть неравенства представляет собой объём Vn прямоугольного параллелепипеда Pn с измерениями an, bn, cn, а правая часть – это объём Vn’ прямоугольного параллелепипеда Pn’ с измерениями an’, bn’, cn’. Так как параллелепипед P содержит в себе параллелепипед Pn, а сам содержится в параллелепипеде Pn’, то объём V параллелепипеда P заключён между Vn, = anbncn и Vn’= an’bn’cn’. Будем неограниченно увеличивать n. Тогда 1/10n будет становиться сколь угодно малым, и поэтому произведение an’bn’cn’ будет сколь угодно мало отличаться от числа, выраженного произведением anbncn. Отсюда следует, что число V сколь угодно мало отличается от числа, выраженного произведением anbncn, а значит, они равны.V = abc, что и требовалось доказать.
Примеры и разбор решения заданий тренировочного модуля.
№1.Длины сторон основания прямоугольного параллелепипеда равны 15 см и 20 см. Высота параллелепипеда равна диагонали основания. Найдите объём этого параллелепипеда.
Решение:
Найдём длину диагонали основания, для этого воспользуемся теоремой Пифагора:
А теперь найдём объём параллелепипеда:
V = 15 ∙ 20 ∙ 25 = 7500 см3
Ответ: V = 7500 см3.
№2.
Найдите площадь закрашенной фигуры, если объём прямоугольного параллелепипеда равен 960 см3, AB = 8 см, АА1= 20 см.
Варианты ответов:
220 см2
100 см2
400 см2
200 см2
Решение.
Найдём длину АD:
AD = 960 : 8 : 20 = 6 см
Найдём АС, воспользовавшись теоремой Пифагора:
Закрашенная фигура – прямоугольник. Вычислим его площадь: 10∙20= 200 см2.
Ответ: площадь закрашенной фигуры 200 см2.
Верный ответ: 200 см2.
Как найти Объем Параллелепипеда?
Понятие объема
Чтобы без труда вычислить объём любой фигуры, нужно разобраться с определениями.
Объём — это количественная характеристика пространства, занимаемого телом или веществом.
Другими словами, это то, сколько места занимает предмет.
Объём измеряется в единицах измерения объема (единицах измерения размера пространства, занимаемого телом), то есть в кубических метрах, сантиметрах, миллиметрах.
За единицу измерения объёма можно принять куб с ребром 1 см, то есть, кубический сантиметр (см3), кубический миллиметр (1 мм3), кубический метр (1 м3).
Объём всегда выражается в положительных числах. Это число показывает, какое именно количество единиц измерения есть в теле. Например, сколько воды в бассейне, вина в бочке, земли в клумбе.
Два свойства объёма
У равных тел равные объёмы. Если два тела одинаковы, и имеют равное количество единиц измерения — их объёмы равны. Например, у двух одинаковых пакетов сока равные объемы.
Если геометрическое тело состоит из нескольких геометрических тел, то его объём равен сумме объёмов этих тел.
Любое объемное тело имеет объем. Получается, при желании мы можем вычислить объем кружки, смартфона, вазы, кота — чего угодно.
Объем прямоугольного параллелепипеда
Давайте вспомним, какие виды параллелепипедов бывают.
Параллелепипедом называется призма, основаниями которой являются параллелограммы. Другими словами, параллелепипед — это многогранник с шестью гранями. Каждая грань которой называется параллелограмм.
Призма — это многогранник, в основаниях которого лежат равные многоугольники, а его боковые грани — это параллелограммы.
Какие бывают призмы:
Прямой параллелепипед — это параллелепипед, у которого боковые ребра перпендикулярны основанию.
Прямоугольным параллелепипедом называют параллелепипед, у которого основание — прямоугольник, а боковые ребра перпендикулярны основанию.
Формула объема прямоугольного параллелепипеда
Чтобы вычислить объем прямоугольного параллелепипеда, найдите произведение его длины, ширины и высоты:
V = a * b * h
Чтобы не запутаться в формулах, запоминайте табличку с условными обозначениями.
a
длина параллелепипеда
b
ширина параллелепипеда
h
высота параллелепипеда
P (осн)
периметр основания
S (осн)
площадь основания
S (бок)
площадь боковой поверхности
S (п.п.)
площадь полной поверхности
V
объем
Пример 1. Чему равен объем параллелепипеда со сторонами 9 см, 6 см, 3 см.
Объем параллелепипеда равен произведению площади основания на высоту.
V = S осн * h
Из этого следствия выведем формулу нахождения площади основания параллелепипеда.
S осн = V : h
Пример 2. Найдите площадь основания параллелепипеда, если его объем равен 82 см3, а высота 8 см.
V = 82 см3
h = 8 см
V = S осн * h
S осн = V : h
S осн = 82 см3: 8 см = 10,25 см2.
Ответ: площадь основания параллелепипеда равна 10,25 см2.
Следствие 2
Объём прямой призмы, основанием которой является прямоугольный треугольник, равен произведению площади основания на высоту.
V = S осн * h
Пример 3. Основанием прямой призмы служит прямоугольный треугольник с катетами 6 и 8 см. Боковое ребро равно 5. Найдем объем призмы.
V = S * h = 12* a * b * h
a = 6
b = 8
h = 5
V = 1/2 * 6 * 8 * 5 = 120 см3.
Ответ: объём прямой призмы, основанием которой является прямоугольный треугольник, равен 120 см3.
С каждым годом геометрия становится все более объемной. Формулы множатся, а задачки усложняются. В детской онлайн-школе Skysmart ваш ребенок сможет заполнить пробелы, разобрать сложные темы и научиться доказывать любые теоремы.
Записывайтесь на бесплатный вводный урок и знакомьтесь с устройством учебной платформы лично.
Вычисление площади
Как вы уже поняли, вычисление объёма параллелепипеда напрямую зависит от вычисления его площади. Давайте разберемся, сколько всего площадей можно найти в параллелепипеде.
Чтобы найти площадь боковой поверхности параллелепипеда, вычислите по отдельности площадь каждой боковой грани, а затем найдите сумму получившихся значений.
Чтобы вычислить площадь полной поверхности параллелепипеда, сложите площадь боковой поверхности и две площади основания.
S п.п. = 2 (ab + ac + bc)
Пример 4. Найдем площадь поверхности параллелепипеда, если длина основания равна 6 сантиметров, ширина — 4 см соответственно, а высота — 3 см.
Ответ: площадь поверхности параллелепипеда — 108 см2.
Как видите, вычислить объём и найти площадь параллелепипеда совсем не трудно. В интернете есть много онлайн-калькуляторов, которые помогут вам быстро вычислить объем:
Задачи на самопроверку
Пользоваться онлайн-калькуляторами можно, когда вы уже натренировались в решении задачек и с закрытыми глазами можете вычислить объем любого параллелепипеда. Давайте разберем еще несколько примеров.
Задачка 1. Найдите объём параллелепипеда со сторонами 18 см, 10 см, 7 см.
Как решаем:
a = 18 см
b = 10 см
h = 7 см
Формула нахождения объема параллелепипеда:
V = a * b * h
Подставляем наши числа:
V = 18 * 10 * 7 = 1260 см3.
Ответ: объём параллелепипеда = 1260 см3.
Задачка 2. Найдите площадь основания параллелепипеда, если его объём = 120 см3, а высота — 15 см.
Как решаем:
V = 120 см3
h = 15 см
V = S осн * h
S осн = V : h
S осн = 120 см3: 15 см = 8 см2.
Ответ: площадь основания параллелепипеда = 8 см2.
Задачка 3. Найдите площадь полной поверхности прямоугольного параллелепипеда, если длина основания = 30 сантиметров, ширина = 12 см, а высота = 5 см.
Ответ: площадь полной поверхности параллелепипеда = 1140 см2.
Пусть все необходимые формулы будут под рукой в нужный момент. Сохраняйте табличку-шпаргалку на гаджет или распечатайте ее и храните в учебнике.
V параллелепипеда
V = a * b * h
V = S осн * h
S боковой поверхности
S б. п. = 2(ac + bc)
S полной поверхности
S п.п. = 2 (ab + ac + bc)
Диагональ параллелепипеда
d2 = a2+ b2 + c2
На уроках математики в современной школе Skysmart нет скучных учебников, надоевших задачек и неинтересных тетрадей. Ученики занимаются по интерактивному учебнику, чертят на настоящей онлайн-доске и решают занимательные примеры.
Математика может быть по-настоящему увлекательной. Записывайтесь на бесплатный вводный урок, чтобы проверить, так ли это на самом деле.
Урок математики 4 класс тема ««Объем. Единицы объема»
Сегодня познакомимся с новой геометрической величиной. Подберите однокоренное слово к слову объёмная, это слово имя существительное. С какой величиной мы познакомимся? (С ОБЪЕМОМ. ) Слайд 5
Как вы думаете, где мы сможем находить объем?
Конечно, объем находим только в объемных (геометрических) телах.
Сегодня мы не только узнаем, что такое объем, но и научимся его измерять и узнаем единицы измерения объема. В этом нам поможет учебник.
4. Работа с учебником стр.32 №118,119.
Слайд 6
Посмотрите на первый рисунок и расскажите о паре, пользуясь, планом.
Что изображено?
Что является меркой? Что измеряет мерка? (отрезок)
Как она называется? (сантиметр) Назовите величину. (длина)
Посмотрите на второй рисунок.
Что измеряет мерка? (большой квадрат)
Как она называется? (квадратный сантиметр) Назовите величину (площадь)
Посмотрите на третий рисунок.
Что измеряет мерка? (большой кубик)
Как она называется? (мы не знаем) Назовите величину (объём).
Поразмышляем (с опорой на слайды №8, 9)
Как называется мерка, которой можно измерить объемное тело (кубический см) (учитель показывает мерку из пластилина.)
Как появилась эта мерка? Если затрудняетесь, найдите ответ в правиле. Объясните (учебник стр.33).
Длину 1см умножили на ширину 1 см, умножили на высоту 1 см
1см · 1 см · 1 см = 1 см 3
Таким образом, мы познакомились с новым понятием объем, который обозначается латинской буквой V.
№119 Догадайтесь, чему равен объём данного куба. Что для этого надо сделать? Надо измерить длину ребра куба. Она равна 1дм. Значит объём этого куба равен 1 кубическому дециметру ( 1дм3 )
5. Работа по закреплению знаний
Работа в парах. Решение задачи №120
Найдите объём фигуры, если ребро каждого куба равно 1 см.
12 см3 2) 13 см3 3) 14 см3 4) 16 см3
6. Физминутка.
7. Объём параллелепипеда. Слайд 13.
Как называется эти многогранники? (параллелепипед и куб)
Фигуры сложили из мерок — 1см3 . Как сосчитать мерки, чтобы узнать объём каждой фигуры? Слайд 14.
Покажите, что мы можем найти и измерить в этом геометрическом теле?
Покажите длину, ширину, высоту. Эти измерения нам нужны для нахождения объема. Как вычислить объём прямоугольного параллелепипеда? Что для этого мы должны сделать? Слайд 15.
Если это куб, сколько надо выполнить измерений. (Всего одно, так как все мерки будут одинаковы, так как куб состоит из граней — одинаковых квадратов.)
Запись на доске:
Задача №1
а= 5 см V= а · в · с
в=3см V= 5 · 3 · 4= 60см3
с=4см
Найти V-?
Ответ: V= 60см3
8. Работа в группах.
Заполните таблицу и прочтите, как вы сегодня работали.
9. Творческая работа в парах.
Составить свою задачу на нахождение объёма параллелепипеда.
10.Итог урока.
Блиц-опрос Слайд 18.
Проверим, как это поняли.
Выбери правильное рассуждение, и обведите букву правильного ответа
1.Объём – это…
А) величина
Б) геометрическая фигура
В) единица измерения
2.Найди формулу нахождения объема
А) V= а · в
Б) V= а · в · с
В) V= а · в · с · d
3.Назови единицу измерений объема
А) см
Б) см2
В) см3
11.Рефлексия.
Слайд 20 Мы сегодня, узнавая новое, тоже заполняли объем своих знаний. Нарисуй солнышко, если на уроке ты узнал новое и тебе было всё понятно. Нарисуй тучку, если на уроке было много непонятного и ты устал. Нарисуй звёздочку, если ты ещё хочешь узнать что-нибудь новое об объёме.
Приложение.
КАРТОЧКА № 1
Соотнесите величину и
единицу измерения этой величины
МАССА см 2
см
кг
ПЛОЩАДЬ дм2
м2
км
дм
ДЛИНА м
мм 2
мм
г
Блиц – опрос
А Б В
А Б В
А Б В
_______________________________________________
Оцени свою работу на уроке
1 группа
b
c
V
4
2
6
к
5
4
2
и
3
3
4
л
7
2
3
с
5
2
6
а
40
42
48
60
36
2 группа
b
c
V
4
2
6
в
5
4
2
р
3
3
4
ил
7
2
3
т
5
2
6
о
42
48
60
40
36
3 группа
b
c
V
4
2
6
н
5
4
2
я
3
3
4
з
7
2
3
а
5
2
6
и
36
48
42
48
60
40
4 группа
b
c
V
4
2
6
о
5
4
2
ы
3
3
4
н
7
2
3
е
5
2
6
в
36
48
60
40
42
5 группа
b
c
V
4
2
6
к
5
4
2
р
3
3
4
о
7
2
3
ыл
5
2
6
т
36
60
48
40
42
Какой буквой обозначается объём? | Полезная информация для всех
Объм обозначается всегда буквой V.
А измеряется в кубических метрах (м3), в кубических сантиметрах (см3) и т.д.
Ниже приведена формула объма куба и запоминается она легко.
Как найти объм цилиндра, конуса и шара тоже показано на рисунке.
Объем — это пространство, занимаемое определенным телом.
В различных формулах (например, физических или математических) объем всегда обозначают заглавной латинской буквой V. В переводе с латыни volume обозначает quot;объемquot;, именно поэтому объем принято обозначать как V.
Объем в химических, математических, физических формулах принято обозначать заглавной латинской буквой V.
Единица измерения объема в системе СИ — это кубический метр, другие единицы измерения — это кубические сантиметры, кубические миллиметры, литры (кубические дециметры), пинта, баррель, галлон, кубический дюйм и другие.
Обозначение объема в физике
В физике и других науках за величину объема принята буква quot;Vquot;.
В химии, математике и остальных науках принят этот же символ.
Объем везде измеряется в таких величинах как: куб.метр(СИ), куб.см, куб.мм, куб.км.
Для начала поймм что же это такое объм. По существу — это вместимость геометрческого тела, которое ограничено замкнутой поверхностью. Буква же, которая обозначает его пишется V. А измеряется объм в м3 и естественно всех производных от него.
в стандартной системе обозначений величина объема обозначается латинской буквой V. иногда можно встретить и маленькую букву v. это тоже объем, но другого толка — маленькой буквой обозначается номинальный объем.
Наиболее частое обозначение объема в задачах по физике, химии и по другим подобным дисциплинам — буква quot;Vquot;. Такое обозначение принято у всех, сложилось историчски, хоть и является весьма условным символом.
Объемом обладает любое тело, которое имеет три измерения, то есть является плоской фигурой. Определить можно объем любого тела, простого по математическим формулам, образованного кривой линией по интегралам, а реального произвольного тела по методу Архимеда или вычислив плотность тела и его массу. Обозначают объем латинской буквой V, и измеряют в кубических метрах, сантиметрах или миллиметрах. Впрочем есть внесистемные единицы объема — литр, ведро, пинта, галлон. Интересно, что тела одного объема могут весить абсолютно по разному, в зависимости от их плотности. Вот какую массу будут иметь кубический сантиметр разного вещества:
По моему (если я точно помню) объм всегда обозначался буквой quot;Vquot;.
Насколько я помню из школьного курса геометрии, объем обозначается буквой V всегда. Кроме того, объем требовался на физике и химии и везде объем обозначался именно этой буквой. Кстати, для справки или для напоминания, единицы измерения объема = метры кубические
Для того, что бы количественно охарактеризовать пространство, которое занимает вещество или тело, используется объм. Эту величина в физике и других науках принято обозначать буквой V. В системе СИ е измеряют с помощью метра кубического и его производных. Есть ещ и внесистемные единицы измерения: баррель, галлон.
Периметр, площадь и объем
Данный материал содержит геометрические фигуры с измерениями. Приведённые измерения являются приблизительными и могут не совпадать с измерениями в реальной жизни.
Периметр геометрической фигуры
Периметр геометрической фигуры — это сумма всех её сторон. Чтобы вычислить периметр, нужно измерить каждую сторону и сложить результаты измерений.
Вычислим периметр следующей фигуры:
Это прямоугольник. Детальнее мы поговорим об этой фигуре позже. Сейчас просто вычислим периметр этого прямоугольника. Длина его равна 9 см, а ширина 4 см.
У прямоугольника противоположные стороны равны. Это видно на рисунке. Если длина равна 9 см, а ширина равна 4 см, то противоположные стороны будут равны 9 см и 4 см соответственно:
Найдём периметр. Для этого сложим все стороны. Складывать их можно в любом порядке, поскольку от перестановки мест слагаемых сумма не меняется. Периметр часто обозначается заглавной латинской буквой P (англ. perimeters). Тогда получим:
P = 9 см + 4 см + 9 см + 4 см = 26 см.
Поскольку у прямоугольника противоположные стороны равны, нахождение периметра записывают короче — складывают длину и ширину, и умножают её на 2, что будет означать «повторить длину и ширину два раза»
P = 2 × (9 + 4) = 18 + 8 = 26 см.
Квадрат это тот же прямоугольник, но у которого все стороны равны. Например, найдём периметр квадрата со стороной 5 см. Фразу «со стороной 5 см» нужно понимать как «длина каждой стороны квадрата равна 5 см»
Чтобы вычислить периметр, сложим все стороны:
P = 5 см + 5 см + 5 см + 5 см = 20 см
Но поскольку все стороны равны, вычисление периметра можно записать в виде произведения. Сторона квадрата равна 5 см, и таких сторон 4. Тогда эту сторону, равную 5 см нужно повторить 4 раза
P = 5 см × 4 = 20 см
Площадь геометрической фигуры
Площадь геометрической фигуры — это число, которое характеризует размер данной фигуры.
Следует уточнить, что речь в данном случае идёт о площади на плоскости. Плоскостью в геометрии называют любую плоскую поверхность, например: лист бумаги, земельный участок, поверхность стола.
Площадь измеряется в квадратных единицах. Под квадратными единицами подразумевают квадраты, стороны которых равны единице. Например, 1 квадратный сантиметр, 1 квадратный метр или 1 квадратный километр.
Измерить площадь какой-нибудь фигуры означает выяснить сколько квадратных единиц содержится в данной фигуре.
Например, площадь следующего прямоугольника равна трём квадратным сантиметрам:
Это потому что в данном прямоугольнике содержится три квадрата, каждый из которых имеет сторону, равную одному сантиметру:
Справа представлен квадрат со стороной 1 см (он в данном случае является квадратной единицей). Если посмотреть сколько раз этот квадрат входит в прямоугольник, представленный слева, то обнаружим, что он входит в него три раза.
Следующий прямоугольник имеет площадь, равную шести квадратным сантиметрам:
Это потому что в данном прямоугольнике содержится шесть квадратов, каждый из которых имеет сторону, равную одному сантиметру:
Допустим, потребовалось измерить площадь следующей комнаты:
Определимся в каких квадратах будем измерять площадь. В данном случае площадь удобно измерить в квадратных метрах:
Итак, наша задача состоит в том, чтобы определить сколько таких квадратов со стороной 1 м содержится в исходной комнате. Заполним этим квадратом всю комнату:
Видим, что квадратный метр содержится в комнате 12 раз. Значит, площадь комнаты составляет 12 квадратных метров.
Площадь прямоугольника
В предыдущем примере мы вычислили площадь комнаты, последовательно проверив сколько раз в ней содержится квадрат, сторона которого равна одному метру. Площадь составила 12 квадратных метров.
Комната представляла собой прямоугольник. Площадь прямоугольника можно вычислить перемножив его длину и ширину.
Чтобы вычислить площадь прямоугольника, нужно перемножить его длину и ширину.
Вернёмся к предыдущему примеру. Допустим, мы измерили длину комнаты рулеткой и оказалось, что длина составила 4 метра:
Теперь измерим ширину. Пусть она составила 3 метра:
Умножим длину (4 м) на ширину (3 м).
4 × 3 = 12
Как и в прошлый раз получаем двенадцать квадратных метров. Это объясняется тем, что измерив длину, мы тем самым узнаём сколько раз можно уложить в эту длину квадрат со стороной, равной одному метру. Уложим четыре квадрата в эту длину:
Затем мы определяем сколько раз можно повторить эту длину с уложенными квадратами. Это мы узнаём, измерив ширину прямоугольника:
Площадь квадрата
Квадрат это тот же прямоугольник, но у которого все стороны равны. Например, на следующем рисунке представлен квадрат со стороной 3 см. Фраза «квадрат со стороной 3 см» означает, что все стороны равны 3 см
Площадь квадрата вычисляется таким же образом, как и площадь прямоугольника — длину умножают на ширину.
Вычислим площадь квадрата со стороной 3 см. Умножим длину 3 см на ширину 3 см
3 × 3 = 9
В данном случае требовалось узнать сколько квадратов со стороной 1 см содержится в исходном квадрате. В исходном квадрате содержится девять квадратов со стороной 1 см. Действительно, так оно и есть. Квадрат со стороной 1 см, входит в исходный квадрат девять раз:
Умножив длину на ширину, мы получили выражение 3 × 3, а это есть произведение двух одинаковых множителей, каждый из которых равен 3. Иными словами выражение 3 × 3 представляет собой вторую степень числа 3. А значит процесс вычисления площади квадрата можно записать в виде степени 32.
Поэтому вторую степень числа называют квадратом числа. При вычислении второй степени числа a, человек тем самым находит площадь квадрата со стороной a. Операцию возведения числа во вторую степень по другому называют возведением в квадрат.
Обозначения
Площадь обозначается заглавной латинской буквой S (англ. Square — квадрат). Тогда площадь квадрата со стороной a см будет вычисляться по следующему правилу
S = a2
где a — длина стороны квадрата. Вторая степень указывает на то, что происходит перемножение двух одинаковых сомножителей, а именно длины и ширины. Ранее было сказано, что у квадрата все стороны равны, а значит равны длина и ширина квадрата, выраженные через букву a.
Если задача состоит в том, чтобы определить сколько квадратов стороной 1 см содержится в исходном квадрате, то в качестве единиц измерения площади нужно указывать см2. Это обозначение заменяет словосочетание «квадратный сантиметр».
Например, вычислим площадь квадрат со стороной 2 см.
Значит, квадрат со стороной 2 см, имеет площадь, равную четырём квадратным сантиметрам:
Если задача состоит в том, чтобы определить сколько квадратов со стороной 1 м содержится в исходном квадрате, то в качестве единиц измерения нужно указывать м2. Это обозначение заменяет словосочетание «квадратный метр».
Вычислим площадь квадрата со стороной 3 метра
Значит, квадрат со стороной 3 м, имеет площадь равную девяти квадратным метрам:
Аналогичные обозначения используются при вычислении площади прямоугольника. Но длина и ширина прямоугольника могут быть разными, поэтому они обозначаются через разные буквы, например a и b. Тогда площадь прямоугольника, длиной a и шириной b вычисляется по следующему правилу:
S = a × b
Как и в случае с квадратом, единицами измерения площади прямоугольника могут быть см2, м2, км2. Эти обозначения заменяют словосочетания «квадратный сантиметр», «квадратный метр», «квадратный километр» соответственно.
Например, вычислим площадь прямоугольника, длиной 6 см и шириной 3 см
Значит, прямоугольник длиной 6 см и шириной 3 см имеет площадь, равную восемнадцати квадратным сантиметрам:
В качестве единицы измерения допускается использовать словосочетание «квадратных единиц». Например, запись S = 3 кв.ед означает, что площадь квадрата или прямоугольника равна трём квадратам, каждый из которых имеет единичную сторону (1 см, 1 м или 1 км).
Перевод единиц измерения площади
Единицы измерения площади можно переводить из одной единицы измерения в другую. Рассмотрим несколько примеров:
Пример 1. Выразить 1 квадратный метр в квадратных сантиметрах.
1 квадратный метр это квадрат со стороной 1 м. То есть все четыре стороны имеют длину, равную одному метру.
Но 1 м = 100 см. Тогда все четыре стороны тоже имеют длину, равную 100 см
Вычислим новую площадь этого квадрата. Умножим длину 100 см на ширину 100 см или возведём в квадрат число 100
S = 1002 = 10 000 см2
Получается, что на один квадратный метр приходится десять тысяч квадратных сантиметров.
1 м2 = 10 000 см2
Это позволяет в будущем умножить любое количество квадратных метров на 10 000 и получить площадь, выраженную в квадратных сантиметрах.
Чтобы перевести квадратные метры в квадратные сантиметры, нужно количество квадратных метров умножить на 10 000.
А чтобы перевести квадратные сантиметры в квадратные метры, нужно наоборот количество квадратных сантиметров разделить на 10 000.
Например, переведём 100 000 см2 в квадратные метры. Рассуждать в этом случае можно так: «если 10 000 см2 это один квадратный метр, то сколько раз 100 000 см2 будут содержать по 10 000 см2»
100 000 см2 : 10 000 см2 = 10 м2
Другие единицы измерения можно переводить таким же образом. Например, переведём 2 км2 в квадратные метры.
Один квадратный километр это квадрат со стороной 1 км. То есть все четыре стороны имеют длину, равную одному километру. Но 1 км = 1000 м. Значит, все четыре стороны квадрата также равны 1000 м. Найдём новую площадь квадрата, выраженную в квадратных метрах. Для этого умножим длину 1000 м на ширину 1000 м или возведём в квадрат число 1000
S = 10002 = 1 000 000 м2
Получается, что на один квадратный километр приходится один миллион квадратных метров:
1 км2 = 1 000 000 м2
Это позволяет в будущем умножить любое количество квадратных километров на 1 000 000 и получить площадь, выраженную в квадратных метрах.
Чтобы перевести квадратные километры в квадратные метры, нужно количество квадратных километров умножить на 1 000 000.
Итак, вернёмся к нашей задаче. Требовалось перевести 2 км2 в квадратные метры. Умножим 2 км2 на 1 000 000
2 км2 × 1 000 000 = 2 000 000 м2
А чтобы перевести квадратные метры в квадратные километры, нужно наоборот количество квадратных метров разделить на 1 000 000.
Например, переведём 3 500 000 м2 в квадратные километры. Рассуждать в этом случае можно так: «если 1 000 000 м2 это один квадратный километр, то сколько раз 3 500 000 м2 будут содержать по 1 000 000 м2»
3 500 000 м2 : 1 000 000 м2 = 3,5 км2
Пример 2. Выразить 7 м2 в квадратных сантиметрах.
Умножим 7 м2 на 10 000
7 м2 = 7 м2 × 10 000 = 70 000 см2
Пример 3. Выразить 5 м2 13 см2 в квадратных сантиметрах.
5 м2 13 см2 = 5 м2 × 10 000 + 13 см2 = 50 013 см2
Пример 4. Выразить 550 000 см2 в квадратных метрах.
Узнаем сколько раз 550 000 см2 содержит по 10 000 см2. Для этого разделим 550 000 см2 на 10 000 см2
550 000 см2 : 10 000 см2 = 55 м2
Пример 5. Выразить 7 км2 в квадратных метрах.
Умножим 7 км2 на 1 000 000
7 км2 × 1 000 000 = 7 000 000 м2
Пример 6. Выразить 8 500 000 м2 в квадратных километрах.
Узнаем сколько раз 8 500 000 м2 содержит по 1 000 000 м2. Для этого разделим 8 500 000 м2 на 1 000 000 м2
8 500 000 м2 × 1 000 000 м2 = 8,5 км2
Единицы измерения площади земельных участков
Площади небольших земельных участков удобно измерять в квадратных метрах.
Площади более крупных земельных участков измеряются в арах и гектарах.
Ар (сокращённо: a) — это площадь равная ста квадратным метрам (100 м2). В виду частого распространения такой площади (100 м2) она стала использоваться, как отдельная единица измерения.
Например, если сказано что площадь какого-нибудь поля составляет 3 а, то нужно понимать, что это три квадрата площадью 100 м2 каждый, то есть:
3 а = 100 м2 × 3 = 300 м2
В народе ар часто называют соткой, поскольку ар равен квадрату, площадью 100 м2. Примеры:
1 сотка = 100 м2
2 сотки = 200 м2
10 соток = 1000 м2
Гектар (сокращенно: га) — это площадь, равная 10 000 м2. Например, если сказано что площадь какого-нибудь леса составляет 20 гектаров, то нужно понимать, что это двадцать квадратов площадью 10 000 м2 каждый, то есть:
20 га = 10 000 м2 × 20 = 200 000 м2
Прямоугольный параллелепипед и куб
Прямоугольный параллелепипед — это геометрическая фигура, состоящая из грáней, рёбер и вершин. На рисунке показан прямоугольный параллелепипед:
Прямоугольный параллелепипед обладает длиной, шириной и высотой. На рисунке показано где длина, ширина и высота:
Параллелепипед, у которого длина, ширина и высота равны между собой, называется кубом. На рисунке показан куб:
Объём геометрической фигуры
Объём геометрической фигуры — это число, которое характеризует вместимость данной фигуры.
Объём измеряется в кубических единицах. Под кубическими единицами подразумевают кубы длиной 1, шириной 1 и высотой 1. Например, 1 кубический сантиметр или 1 кубический метр.
Измерить объём какой-нибудь фигуры означает выяснить сколько кубических единиц вмещается в данную фигуру.
Например, объём следующего прямоугольного параллелепипеда равен двенадцати кубическим сантиметрам:
Это потому что в данный параллелепипед вмещается двенадцать кубов длиной 1 см, шириной 1 см и высотой 1 см:
Объём обозначается заглавной латинской буквой V. Одна из единиц измерения объема это кубический сантиметр (см3). Тогда объём V рассмотренного нами параллелепипеда равен 12 см3
V = 12 см3
Объём любого параллелепипеда вычисляют следующим образом: перемножают его длину, ширину и высоту .
Объём прямоугольного параллелепипеда равен произведению его длины, ширины и высоты.
V = abc
где, a — длина, b — ширина, c — высота
Так, в предыдущем примере мы визуально определили, что объём параллелепипеда равен 12 см3. Но можно измерить длину, ширину и высоту данного параллелепипеда и перемножить результаты измерений. Мы получим тот же результат
Объём куба вычисляется таким же образом, как и объём прямоугольного параллелепипеда — перемножают длину, ширину и высоту.
Например, вычислим объём куба, длина которого 3 см. У куба длина, ширина и высота равны между собой. Если длина равна 3 см, то равны этим же трём сантиметрам ширина и высота куба:
Перемножаем длину, ширину, высоту и получаем объём, равный двадцати семи кубическим сантиметрам:
V = 3 × 3 × 3 = 27 см³
Действительно, в исходный куб вмещается 27 кубиков длиной 1 см
При вычислении объёма данного куба мы перемножили длину, ширину и высоту. Получилось произведение 3 × 3 × 3. Это есть произведение трёх сомножителей, каждый из которых равен 3. Иными словами, произведение 3 × 3 × 3 является третьей степенью числа 3 и может быть записано в виде 33.
V = 33 = 27 см3
Поэтому третью степень числа называют кубом числа. При вычислении третьей степени числа a, человек тем самым находит объём куба, длиной a. Операцию возведения числа в третью степень по другому называют возведением в куб.
Таким образом, объём куба вычисляется по следующему правилу:
V = a3
Где a — длина куба.
Кубический дециметр. Кубический метр
Не все объекты нашего мира удобно измерять в кубических сантиметрах. Например, объём комнаты или дома удобнее измерять в кубических метрах (м3). А объём бака, аквариума или холодильника удобнее измерять в кубических дециметрах (дм3).
Другое название одного кубического дециметра – один литр.
1 дм3 = 1 литр
Перевод единиц измерения объёма
Единицы измерения объёма можно переводить из одной единицы измерения в другую. Рассмотрим несколько примеров:
Пример 1. Выразить 1 кубический метр в кубических сантиметрах.
Один кубический метр это куб со стороной 1 м. Длина, ширина и высота этого куба равны одному метру.
Но 1 м = 100 см. Значит, длина, ширина и высота тоже равны 100 см
Вычислим новый объём куба, выраженный в кубических сантиметрах. Для этого перемножим его длину, ширину и высоту. Либо возведём число 100 в куб:
V = 1003 = 1 000 000 см3
Получается, что на один кубический метр приходится один миллион кубических сантиметров:
1 м3 = 1 000 000 см3
Это позволяет в будущем умножить любое количество кубических метров на 1 000 000 и получить объём, выраженный в кубических сантиметрах.
Чтобы перевести кубические метры в кубические сантиметры, нужно количество кубических метров умножить на 1 000 000.
А чтобы перевести кубические сантиметры в кубические метры, нужно наоборот количество кубических сантиметров разделить на 1 000 000.
Например, переведём 300 000 000 см3 в кубические метры. Рассуждать в этом случае можно так: «если 1 000 000 см3 это один кубический метр, то сколько раз 300 000 000 см3 будут содержать по 1 000 000 см3»
300 000 000 см3 : 1 000 000 см3 = 300 м3
Пример 2. Выразить 3 м3 в кубических сантиметрах.
Умножим 3 м3 на 1 000 000
3 м3 × 1 000 000 = 3 000 000 см3
Пример 3. Выразить 60 000 000 см3 в кубических метрах.
Узнаем сколько раз 60 000 000 см3 содержит по 1 000 000 см3. Для этого разделим 60 000 000 см3 на 1 000 000 см3
60 000 000 см3 : 1 000 000 см3 = 60 м3
Вместимость бака, банки или канистры измеряют в литрах. Литр это тоже единица измерения объема. Один литр равен одному кубическому дециметру.
1 литр = 1 дм3
Например, если вместимость банки составляет 1 литр, это значит что объём этой банки составляет 1 дм3. При решении некоторых задач может быть полезным умение переводить литры в кубические дециметры и наоборот. Рассмотрим несколько примеров.
Пример 1. Перевести 5 литров в кубические дециметры.
Чтобы перевести 5 литров в кубические дециметры, достаточно умножить 5 на 1
5 л × 1 = 5 дм3
Пример 2. Перевести 6000 литров в кубические метры.
Шесть тысяч литров это шесть тысяч кубических дециметров:
6000 л × 1 = 6000 дм3
Теперь переведём эти 6000 дм3 в кубические метры.
Длина, ширина и высота одного кубического метра равны 10 дм
Если вычислить объём этого куба в дециметрах, то получим 1000 дм3
V = 103= 1000 дм3
Получается, что одна тысяча кубических дециметров соответствует одному кубическому метру. А чтобы определить сколько кубических метров соответствуют шести тысячамл кубических дециметров, нужно узнать сколько раз 6 000 дм3 содержит по 1 000 дм3
6 000 дм3 : 1 000 дм3 = 6 м3
Значит, 6000 л = 6 м3.
Таблица квадратов
В жизни часто приходиться находить площади различных квадратов. Для этого каждый раз требуется возводить исходное число во вторую степень.
Квадраты первых 99 натуральных чисел уже вычислены и занесены в специальную таблицу, называемую таблицей квадратов.
Первая строка данной таблицы (цифры от 0 до 9) это единицы исходного числа, а первый столбец (цифры от 1 до 9) это десятки исходного числа.
Например, найдём квадрат числа 24 по данной таблице. Число 24 состоит из цифр 2 и 4. Точнее, число 24 состоит из двух десятков и четырёх единиц.
Итак, выбираем цифру 2 в первом столбце таблицы (столбце десятков), а цифру 4 выбираем в первой строке (строке единиц). Затем, двигаясь вправо от цифры 2 и вниз от цифры 4, найдём точку пересечения. В результате окажемся на позиции, где располагается число 576. Значит, квадрат числа 24 есть число 576
242 = 576
Таблица кубов
Как и в ситуации с квадратами, кубы первых 99 натуральных чисел уже вычислены и занесены в таблицу, называемую таблицей кубов.
Куб числа по таблице определяется таким же образом, как и квадрат числа. Например, найдём куб числа 35. Это число состоит из цифр 3 и 5. Выбираем цифру 3 в первом столбце таблицы (столбце десятков), а цифру 5 выбираем в первой строке (строке единиц). Двигаясь вправо от цифры 3 и вниз от цифры 5, найдём точку пересечения. В результате окажемся на позиции, где располагается число 42875. Значит, куб числа 35 есть число 42875.
353 = 42875
Задания для самостоятельного решения
Задача 1. Длина прямоугольника составляет 6 см, а ширина 2 см. Найдите периметр.
Решение
P = 2(a + b)
a = 6, b = 2 P = 2(6 + 2) = 12 + 4 = 16 см
Ответ: периметр прямоугольника равен 16 см.
Задача 2. Длина прямоугольника составляет 6 см, а ширина 2 см. Найдите площадь.
Решение
S = ab a = 6, b = 2 S = 6 × 2 = 12 см2
Ответ: площадь равна 12 см2.
Задача 3. Площадь прямоугольника составляет 12 см2. Длина составляет 6 см. Найдите ширину прямоугольника.
Решение
S = ab S = 12, a = 6, b = x 12 = 6 × x x = 2
Ответ: ширина прямоугольника составляет 2 см.
Задача 4. Вычислите площадь квадрата со стороной 8 см
Решение
S = a2 a = 8 S = 82 = 64 см2 Ответ: площадь квадрата со стороной 8 см равна 64 см2
Задача 5. Вычислите объем прямоугольного параллелепипеда, длина которого 6 см, ширина 4 см, высота 3 см.
Решение
V = abc a = 6, b = 4, c = 3 V = 6 × 4 × 3 = 72 см3.
Ответ: объем прямоугольного параллелепипеда, длина которого 6 см, ширина 4 см, высота 3 см равен 72 см3
Задача 6. Объем прямоугольного параллелепипеда составляет 200 см3. Найдите высоту параллелепипеда, если его длина равна 10 см, а ширина 5 см
Решение
V = abc V = 200, a = 10, b = 5, c = x 200 = 10 × 5 × x 200 = 50x x = 4
Ответ: высота прямоугольного параллелепипеда равна 4 см.
Задача 7. Площади земельного участка, засеянные пшеницей и льном, пропорциональны числам 4 и 5. На какой площади засеяна пшеница, если под льном засеяно 15 га
Решение
Число 4 отражает площадь, засеянную пшеницей. А число 5 отражает площадь, засеянную льном. Сказано что площади, засеянные пшеницей и льном пропорциональны этим числам.
Проще говоря, во сколько раз изменяются числа 4 или 5, во сколько же раз изменится и площадь, которая засеяна пшеницей или льном. Льном засеяно 15 га. То есть число 5, которое отражает площадь, засеянную льном, изменилось в 3 раза.
Тогда число 4, которое отражает площадь засеянную пшеницей, нужно увеличить в три раза
4 × 3 = 12 га
Ответ: пшеницей засеяно 12 га.
Задача 8. Длина зернохранилища 42 м, ширина составляет длины, а высота – 0,1 длины. Определите сколько тонн зерна вмещает зернохранилище, если 1 м3 его весит 740 кг.
Решение
a — длина b — ширина c — высота
a = 42 м b = м c = 42 × 0,1 = 4,2 м
Определим объем зернохранилища:
V = abc = 42 × 30 × 4,2 = 5292 м3
Определите сколько тонн зерна вмещает зернохранилище:
5292 × 740 = 3916080 кг
Переведём килограммы в тонны:
Ответ: зернохранилище вмещает 3916,08 тонн зерна.
Задача 9. 12. Бассейн имеет форму прямоугольного параллелепипеда, длина которого равна 5,8 м, а ширина – 3,5 м. Две трубы наполняют его водой в течение 13 ч 32 мин., причём через одну из них вливается 25 л/мин, а через вторую – 0,75 этого количества. Определите высоту (глубину) бассейна.
Решение
Определим сколько литров в минуту вливается через вторую трубу:
25 л/мин × 0,75 = 18,75 л/мин
Определим сколько литров в минуту вливается в бассейн через обе трубы:
25 л/мин + 18,75 л/мин = 43,75 л/мин
Определим сколько литров воды будет залито в бассейн за 13 ч 32 мин
43,75 × 13 ч 32 мин = 43,75 × 812 мин = 35 525 л
1 л = 1 дм3
35 525 л = 35 525 дм3
Переведём кубические дециметры в кубические метры. Это позволит вычислит объем бассейна:
35 525 дм3 : 1000 дм3 = 35,525 м3
Зная объём бассейна можно вычислить высоту бассейна. Подставим в буквенное уравнение V=abc имеющиеся у нас значения. Тогда получим:
V = 35,525 a = 5.8 b = 3.5 c = x
35,525 = 5,8 × 3,5 × x 35,525 = 20,3 × x x = 1,75 м
с = 1,75
Ответ: высота (глубина) бассейна составляет 1,75 м.
Понравился урок? Вступай в нашу новую группу Вконтакте и начни получать уведомления о новых уроках
Возникло желание поддержать проект? Используй кнопку ниже
Навигация по записям
Объём прямоугольного параллелепипеда — урок. Математика, 5 класс.
Уже известны единицы измерения длины, например:
мм,см,дм,м,км… — и другие.
Для фигур на плоскости измеряют площадь в соответствующих квадратных единицах измерения:
мм2,см2,дм2,м2,км2…
Для геометрических тел измеряют объём, и для этого необходимы единицы измерения.
Единицей измерения объёма служит объём куба, у которого все грани равны \(1\) единице измерения длины:
мм3,см3,дм3,м3,км3…
Это кубический миллиметр, кубический сантиметр, кубический дециметр, кубический метр или даже кубический километр:
Часто для измерения объёма жидкости используют единицу измерения \(1\) литр: 1 л =1000см3=1дм3.
Если измерять объём прямоугольного параллелепипеда, то можно представить, как маленькие кубики перекрывают прямоугольник в основании прямоугольного параллелепипеда.
Длины сторон прямоугольника определяют, сколько кубиков в ряду и сколько рядов с кубиками будет.
Если стороны равны, например, \(3\) см и \(4\) см, то прямоугольник перекрывается \(3·4 = 12\) кубиками.
Высота параллелепипеда определяет, сколько таких слоёв с кубиками можно поставить.
Если высота прямоугольного параллелепипеда равна \(3\) см, то всего получится \(3\) слоя с кубиками. Итак, всего \(3·3·4 = 36\) кубиков, или объём равен \(36\) см³.
Значит, три измерения прямоугольного параллелепипеда позволяют посчитать, сколько всего кубиков поместилось в геометрическом теле, то есть вычислить объём прямоугольного параллелепипеда.
Пусть измерения прямоугольного параллелепипеда будут a, b и c единиц измерения.
Тогда объём \(V = a·b·c\) кубических единиц измерения.
Как обозначается длина и ширина
Высота ширина длина — латинские обозначения: как правильно пишутся размеры и чем отличаются величины
Решая геометрические задачи, ученики сталкиваются с вопросом: как правильно обозначить те или иные части чертежа? Например, высоту треугольника, ширину прямоугольника, размеры бассейна. Подобные обозначения мы найдем и в физических задачах: длина маятника, высота, с которой тело начинает падать… Поэтому следует знать некоторые правила….
Как обозначаются различные параметры
В единой системе измерения используется обозначение латинскими буквами:
длину буквой l, если речь идет об одной прямой линии: маятнике, рычаге, отрезке, прямой. Но если речь идет о геометрической фигуре, например, прямоугольнике, то используется А,
высоту или глубину – h,
ширину – В.
Что такое система СИ, ученики узнают лишь в средней школе, поэтому обычно в младших классах специального обозначениям для этих величин не вводят.
Как обозначить глубину?
Почему же для высоты и глубины применяется одна и та же буква? Если вы построите чертеж параллелепипеда, то здесь вы отметите высоту фигуры.
А если составить чертеж прямоугольного бассейна того же размера, что и параллелепипед, то обозначается глубина. Таким образом, можно сказать, высота и глубина в этом случае будут одной величиной.
Внимание! Высота и глубина – две величины, которые обозначают один и тот же перпендикуляр, соединяющий две противоположные плоскости.
Понятие «глубина» встречается и в географии. На картах она отображается цветом. Если речь идет о водных просторах, то чем темнее синий, цвет, тем больше глубина, а если речь идет о суше, то низменности обозначаются темно-зеленым цветом.
В черчении эта величина обозначается литерой S. Она позволяет создать полное восприятие объекта иногда даже с одним видом.
Что бывает длинным
Что же такое длина и как обозначается этот показатель? Она указывает расстояние от точки до точки, то есть размер отрезка. В геометрических задачах его принято обозначать как А. В стереометрии ее могут обозначать и А, и l (например, в задачах, где встречается прямая, пересекающая плоскость).
В физике же длина маятника, плеча рычага и т.д. в «Дано» обозначается буквой l, так как речь идет об отдельной прямой.
Отличие длины от высоты
Длина – это величина, которая характеризует протяженность линии.
А высота – это перпендикуляр, опущенный на противолежащую плоскость.
То есть можно сделать вывод, что длина от высоты отличается тем, что является частью фигуры, совпадая с ее гранью, а высота получается в результате дополнительного построения на чертеже.
Высоту проводят для того, чтобы получить новые данные для решения задач, а также новых фигур в составе исходной.
Вот такой ширины
Ширина предмета необходима для того, чтобы понять форму как двумерного, так и трехмерного объекта. Как правило, она обозначается буквой В.
Измеряется ширина в метрах (по СИ). Но если предмет слишком мал, то для удобства используют более мелкие единицы измерения:
дециметры,
сантиметры,
миллиметры,
микрометры и т.д.
А если предмет слишком крупный, то пишутся такие приставки:
Кило- (10³),
Мега- (106),
Гига- (109),
Тера- (1012) и т.д.
Разумеется, такие крупные единицы измерения необходимы, например, для астрономии. Также они применяются в квантовой физике, микробиологии и так далее.
Как называются стороны прямоугольника?
В отличие от квадрата, стороны прямоугольника попарно равны и параллельны.
Это значит, что стороны, образующие углы различны.
Как правило, более длинную сторону прямоугольника называют длиной, а ширина прямоугольника это его короткая сторона.
Важно! Зная такие данные, как длина и ширина прямоугольника, можно найти его периметр, площадь, длину диагоналей и угол между ними. Вокруг прямоугольника всегда можно описать окружность. Эти свойства работают и в обратном направлении.
В чем измеряются размеры длины, ширины и высоты по СИ
По единой системе измерения длина, высота и ширина измеряются в метрах. Но иногда, если это дробное или многозначное число, для удобства в вычислениях используют кратные единицы измерения.
Для того чтобы знать, как правильно переводить единицы измерения в более крупные или же наоборот мелкие, необходимо знать значения приставок.
Дека 101,
Гекто 102,
Кило 103,
Мега 106,
Гига 109,
Деци – 10-1,
Санти – 10-2,
Милли – 10-3,
Микро 10-6,
Нано – 10-9.
После подсчетов эти единицы должны быть переведены в метры.
Существуют также внесистемные единицы, но они встречаются очень редко:
миля – 1,6 км,
фут – 12 дюймов – 0,3048 м,
ярд – 36 дюймов – 91,44 мм,
дюйм – 25,4 мм и т.д.
При решении задач такие единицы должны быть переведены в метры.
При выполнении геометрических заданий единицам измерения не уделяют особого внимания, главное, чтобы они были сопоставимы
(если вы производите подсчеты в сантиметрах, значит, все величины необходимо перевести в сантиметры).
А при решении физических задач ответ должен быть дан в метрах в соответствии с единой системой измерения.
Обозначения длины, ширины, высоты в геометрии
Измеряем геометрические параметры
Вывод
Теперь вы знаете, какой буквой обозначается длина, в чем измеряется ширина прямоугольника, и сможете сами объяснить любому, как обозначаются различные параметры.
Это интересно! Легкие правила округления чисел после запятой
tvercult.ru
Обозначение: высота, ширина, длина. Ширина
Построение чертежей — дело непростое, но без него в современном мире никак. Ведь чтобы изготовить даже самый обычный предмет (крошечный болт или гайку, полку для книг, дизайн нового платья и подобное), изначально нужно провести соответствующие вычисления и нарисовать чертеж будущего изделия. Однако часто составляет его один человек, а занимается изготовлением чего-либо по этой схеме другой.
Чтобы не возникло путаницы в понимании изображенного предмета и его параметров, во всем мире приняты условные обозначения длины, ширины, высоты и других величин, применяемых при проектировании. Каковы они? Давайте узнаем.
Величины
Площадь, длина, ширина, высота и другие обозначения подобного характера являются не только физическими, но и математическими величинами.
Единое их буквенное обозначение (используемое всеми странами) было уставлено в середине ХХ века Международной системой единиц (СИ) и применяется по сей день. Именно по этой причине все подобные параметры обозначаются латинскими, а не кириллическими буквами или арабской вязью. Чтобы не создавать отдельных трудностей, при разработке стандартов конструкторской документации в большинстве современных стран решено было использовать практически те же условные обозначения, что применяются в физике или геометрии.
Любой выпускник школы помнит, что в зависимости от того, двухмерная или трехмерная фигура (изделие) изображена на чертеже, она обладает набором основных параметров. Если присутствуют два измерения — это ширина и длина, если их три – добавляется еще и высота.
Итак, для начала давайте выясним, как правильно длину, ширину, высоту обозначать на чертежах.
Ширина
Как было сказано выше, в математике рассматриваемая величина является одним из трех пространственных измерений любого объекта, при условии что его замеры производятся в поперечном направлении. Так чем знаменита ширина? Обозначение буквой «В» она имеет. Об этом известно во всём мире. Причем, согласно ГОСТу, допустимо применение как заглавной, так и строчной латинских литер. Часто возникает вопрос о том, почему именно такая буква выбрана. Ведь обычно сокращение производится по первой букве латинского, греческого или английского названия величины. При этом ширина на английском будет выглядеть как «width».
Вероятно, здесь дело в том, что данный параметр наиболее широкое применение изначально имел в геометрии. В этой науке, описывая фигуры, часто длину, ширину, высоту обозначают буквами «а», «b», «с». Согласно этой традиции, при выборе литера «В» (или «b») была заимствована системой СИ (хотя для других двух измерений стали применять отличные от геометрических символы).
Большинство полагает, что это было сделано, дабы не путать ширину (обозначение буквой «B»/«b») с весом. Дело в том, что последний иногда именуется как «W» (сокращение от английского названия weight), хотя допустимо использование и других литер («G» и «Р»). Согласно международным нормам системы СИ, измеряется ширина в метрах или кратных (дольных) их единицах. Стоит отметить, что в геометрии иногда также допустимо использовать «w» для обозначения ширины, однако в физике и остальных точных науках такое обозначение, как правило, не применяется.
Длина
Как уже было указано, в математике длина, высота, ширина – это три пространственных измерения. При этом, если ширина является линейным размером в поперечном направлении, то длина — в продольном. Рассматривая ее как величину физики можно понять, что под этим словом подразумевается численная характеристика протяжности линий.
В английском языке этот термин именуется length. Именно из-за этого данная величина обозначается заглавной или строчной начальной литерой этого слова — «L». Как и ширина, длина измеряется в метрах или их кратных (дольных) единицах.
Высота
Наличие этой величины указывает на то, что приходится иметь дело с более сложным — трехмерным пространством. В отличие от длины и ширины, высота численно характеризует размер объекта в вертикальном направлении.
На английском она пишется как «height». Поэтому, согласно международным нормам, ее обозначают латинской литерой «Н»/«h». Помимо высоты, в чертежах иногда эта буква выступает и как глубины обозначение. Высота, ширина и длина – все все эти параметры измеряются в метрах и их кратных и дольных единицах (километры, сантиметры, миллиметры и т. п.).
Радиус и диаметр
Помимо рассмотренных параметров, при составлении чертежей приходится иметь дело и с иными.
Например, при работе с окружностями возникает необходимость в определении их радиуса. Так именуется отрезок, который соединяет две точки. Первая из них является центром. Вторая находится непосредственно на самой окружности. На латыни это слово выглядит как «radius». Отсюда и общепринятое сокращение: строчная или заглавная «R»/«r».
Чертя окружности, помимо радиуса часто приходится сталкиваться с близким к нему явлением – диаметром. Он также является отрезком, соединяющим две точки на окружности. При этом он обязательно проходит через центр.
Численно диаметр равен двум радиусам. По-английски это слово пишется так: «diameter». Отсюда и сокращение – большая или маленькая латинская буква «D»/«d». Часто диаметр на чертежах обозначают при помощи перечеркнутого круга – «Ø».
Хотя это распространенное сокращение, стоит иметь в виду, что ГОСТ предусматривает использование только латинской «D»/«d».
Толщина
Большинство из нас помнят школьные уроки математики. Ещё тогда учителя рассказывали, что, латинской литерой «s» принято обозначать такую величину, как площадь. Однако, согласно общепринятым нормам, на чертежах таким способом записывается совсем другой параметр – толщина.
Почему так? Известно, что в случае с высотой, шириной, длиной, обозначение буквами можно было объяснить их написанием или традицией. Вот только толщина по-английски выглядит как «thickness», а в латинском варианте — «crassities». Также непонятно, почему, в отличие от других величин, толщину можно обозначать только строчной литерой. Обозначение «s» также применяется при описании толщины страниц, стенок, ребер и так далее.
Периметр и площадь
В отличие от всех перечисленных выше величин, слово «периметр» пришло не из латыни или английского, а из греческого языка. Оно образовано от «περιμετρέο» («измерять окружность»). И сегодня этот термин сохранил свое значение (общая длина границ фигуры). Впоследствии слово попало в английский язык («perimeter») и закрепилось в системе СИ в виде сокращения буквой «Р».
Площадь — это величина, показывающая количественную характеристику геометрической фигуры, обладающей двумя измерениями (длиной и шириной). В отличие от всего перечисленного ранее, она измеряется в квадратных метрах (а также в дольных и кратных их единицах). Что касается буквенного обозначения площади, то в разных сферах оно отличается. Например, в математике это знакомая всем с детства латинская литера «S». Почему так – нет информации.
Некоторые по незнанию думают, что это связано с английским написанием слова «square». Однако в нем математическая площадь – это «area», а «square» — это площадь в архитектурном понимании. Кстати, стоит вспомнить, что «square» — название геометрической фигуры «квадрат». Так что стоит быть внимательным при изучении чертежей на английском языке. Из-за перевода «area» в отдельных дисциплинах в качестве обозначения применяется литера «А». В редких случаях также используется «F», однако в физике данная буква означает величину под названием «сила» («fortis»).
Другие распространенные сокращения
Обозначения высоты, ширины, длины, толщины, радиуса, диаметра являются наиболее употребляемыми при составлении чертежей. Однако есть и другие величины, которые тоже часто присутствуют в них. Например, строчное «t». В физике это означает «температуру», однако согласно ГОСТу Единой системы конструкторской документации, данная литера — это шаг (винтовых пружин, заклепочных соединений и подобного). При этом она не используется, когда речь идет о зубчатых зацеплениях и резьбе.
Заглавная и строчная буква «A»/«a» (согласно все тем же нормам) в чертежах применяется, чтобы обозначать не площадь, а межцентровое и межосевое расстояние. Помимо различных величин, в чертежах часто приходится обозначать углы разного размера. Для этого принято использовать строчные литеры греческого алфавита. Наиболее применяемые — «α», «β», «γ» и «δ». Однако допустимо использовать и другие.
Какой стандарт определяет буквенное обозначение длины, ширины, высоты, площади и других величин?
Как уже было сказано выше, чтобы не было недопонимания при прочтении чертежа, представителями разных народов приняты общие стандарты буквенного обозначения. Иными словами, если вы сомневаетесь в интерпретации того или иного сокращения, загляните в ГОСТы. Таким образом вы узнаете, как правильно обозначается высота, ширины, длина, диаметр, радиус и так далее.
Для Российской Федерации таким нормативным документом является ГОСТ 2.321-84. Он был внедрен еще в марте 1984 г. (во времена СССР), взамен устаревшего ГОСТа 3452—59.
fb.ru
Длина — Википедия
Материал из Википедии — свободной энциклопедии
Измерения: L — длина, B — ширина, H — высота, толщина, глубина
Длина — физическая величина, числовая характеристика протяжённости линий.
В большинстве систем измерений единица длины — одна из основных единиц измерения, через которые определяются другие (производные) единицы. В международной системе единиц (СИ) за единицу длины принят метр.
В узком смысле под длиной понимают линейный размер предмета в продольном направлении (обычно это направление наибольшего размера), то есть расстояние между его двумя наиболее удалёнными точками, измеренное горизонтально, в отличие от высоты, которая измеряется в вертикальном направлении, а также ширины или толщины, которые измеряются поперёк объекта (под прямым углом к длине).
В физике термин «длина» обычно используется как синоним «расстояния» и обозначается L{\displaystyle L} или l{\displaystyle l} от нем. länge (длина). Символ размерности длины — dim l = L. В ряду других пространственных величин длина — это величина единичной размерности, тогда как площадь — двухмерная, объём — трёхмерная.
Метрическая система[править | править код]
Метрическая система считается самой удобной из всех придуманных из-за своей простоты. В основе метрической системы лежит единица измерения метр. Все остальные единицы измерения являются кратными степеням десяти от метра (например, километр — это 10³ метров и т. п.), что позволяет облегчить подсчёты. До 1960 года у метра был специальный эталон, ныне хранящийся в Международном бюро мер и весов, расположенном в городе Севр (предместье Парижа, Франция). Сегодня, по определению, метр равен расстоянию, которое проходит свет в вакууме за 1/299 792 458 долю секунды.
Исходными английскими мерами длины были миля, ярд, фут и дюйм. Миля пришла в Англию из Древнего Рима, где она определялась как тысяча двойных шагов вооружённого римского воина.
Старорусская система[править | править код]
В Древней Руси мерой длины, веса и т. п. являлся человек. На это указывают названия мер длины: локоть (расстояние от конца вытянутого среднего пальца руки или сжатого кулака до локтевого сгиба), пядь (расстояние между вытянутым большим и указательным пальцами руки), сажень (расстояние от конца пальцев одной руки до конца пальцев другой) и другие[1].
В частности, аршин был связан с длиной человеческого шага. Однако необходимость унификации систем измерений с британской в связи с развитием международной торговли потребовала введения во времена Петра I так называемого «казённого аршина». Это была мерная линейка с металлическими наконечниками с государственным клеймом. Казённый аршин равнялся 28 английским дюймам и делился на 16 вершков.[2]
Относительные размеры объектов, м
-20 —
–
-18 —
–
-16 —
–
-14 —
–
-12 —
–
-10 —
–
-8 —
–
-6 —
–
-4 —
–
-2 —
–
0 —
–
2 —
–
4 —
–
6 —
–
8 —
–
10 —
–
12 —
–
14 —
–
16 —
–
18 —
–
20 —
–
22 —
–
24 —
–
26 —
–
28 —
–
30 —
См. также[править | править код]
Древнегреческая система[править | править код]
Мусульманская система[править | править код]
Типографическая система[править | править код]
Морская система[править | править код]
Морская система измерения длины привязана к размеру планеты Земля. В качестве основной единицы измерения принята морская миля, равная длине одной минуты (1/60 градуса) дуги меридиана земного эллипсоида. Длина морской мили является величиной переменной, зависящей от широты. Её численное значение составляет от 1843 метров на экваторе до 1861,6 метров на полюсах.
Международная морская миля составляет 1852 м, в отличие от морской мили британской системы (1853,184 м). Для измерения меньших размеров применяют кабельтов — 1/10 морской мили, или 185,2 м (округлённо — 185 м)[4].
Единицы, применяемые в астрономии[править | править код]
Измерительные инструменты и меры[править | править код]
Измерительные приборы[править | править код]
Другие средства[править | править код]
Большие расстояния в навигации определяются при помощи средств радионавигационных систем или спутниковых систем
Очень маленькие расстояния измеряются с помощью измерительных микроскопов
Расстояния и размеры объектов, доступных наблюдению[править | править код]
Основной источник: [5]
Наблюдаемые объекты
Размер, м
Расстояние от Земли до самого далекого видимого объекта во Вселенной
1,0×1026{\displaystyle 1{,}0\times 10^{26}}
Расстояние от Земли до галактики в созвездии Андромеды
2,0×1022{\displaystyle 2{,}0\times 10^{22}}
Диаметр нашей Галактики
1,0×1021{\displaystyle 1{,}0\times 10^{21}}
Расстояние от Земли до ближайшей звезды в созвездии Центавра
4,0×1016{\displaystyle 4{,}0\times 10^{16}}
Расстояние от Земли до Солнца
1,5×1011{\displaystyle 1{,}5\times 10^{11}}
Диаметр Солнца
1,4×109{\displaystyle 1{,}4\times 10^{9}}
Расстояние от Земли до Луны
3,8×108{\displaystyle 3{,}8\times 10^{8}}
Диаметр Земли
1,3×107{\displaystyle 1{,}3\times 10^{7}}
Самая глубокая впадина на поверхности Земли
1,1×104{\displaystyle 1{,}1\times 10^{4}}
Самая высокая гора на поверхности Земли
9,0×103{\displaystyle 9{,}0\times 10^{3}}
Длина синего кита — самого большого животного на Земле
35{\displaystyle 35}
Рост самого высокого человека
2,85{\displaystyle 2{,}85}
Размеры амебы
5,0×10−4{\displaystyle 5{,}0\times 10^{-4}}
Толщина человеческого волоса
1,0×10−4{\displaystyle 1{,}0\times 10^{-4}}
Диаметр красного кровяного шарика
1,0×10−5{\displaystyle 1{,}0\times 10^{-5}}
Диаметр вируса гриппа
8,0×10−8{\displaystyle 8{,}0\times 10^{-8}}
Длина молекулы гемоглобина
1,5×10−8{\displaystyle 1{,}5\times 10^{-8}}
Расстояние между атомами в твердом теле
1,0×10−10{\displaystyle 1{,}0\times 10^{-10}}
Диаметр ядра атома урана
1,0×10−14{\displaystyle 1{,}0\times 10^{-14}}
Диаметр протона
1,6×10−15{\displaystyle 1{,}6\times 10^{-15}}
Минимальные размеры областей внутри элементарных частиц, доступных экспериментальному изучению с помощью современных ускорителей
1,0×10−17{\displaystyle 1{,}0\times 10^{-17}}
ru.wikipedia.org
Как правильно написать размеры длина ширина высота
Размеры длина, ширина, высота
Производство асбоцементных профилей организовано в соответствии госстандартами: 3034095 для волновых и 1812495 для плоских.
Волновые АЦЛ
Хотя состав стройматериала одинаковый, по размерам изделия могут разниться. Это также касается толщины изделия. Как правило, она изменяется в промежутке от 5 до 9 мм. Что же касается ширины, то она определяется количеством волн.
Профиль АЦЛ зависит от формы поперечного сечения и расстояния между волнами. Форма поперечного сечения бывает двух типов – 40 на 150 и 54 на 200. Первое число этого показателя (40 или 54) указывает на высоту волны, а второе (150 или 200), соответственно, на ее шаг. Высота шифера есть не что иное, как длина отрезка, связывающее верх волны и низ без учета толщины профиля.
Вариант шифера
Высота
Ширина
Длина
Шаг волны
7-ми волн.
8-ми волн.
6-ти волн.
40/150/1750
40
980
1130
1750
150
54/200/1750
54
1125
1750
200
На заметку Отечественные производители имеют право на производство нестандартных АЦЛ на основе собственных ТУ.
Листы с различными профилями классифицируют в три группы:
ВО – обычный профиль;
УВ – унифицированный;
ВУ – усиленный.
для обычных – 1,2 на 0,68 м;
для унифицированных – 1,75 на 1,125 м;
для усиленных длина шифера равна 2,80 м.
волн современных асбоцементных листов – шесть, семь и восемь. К примеру, стандартный шифер 8 ми волнового – 1,75х1,13 м при толщине – 5,2 или 5,8 мм, величина площади – 1,977 кв. м. У 7-ми и 8-ми — одинаковая высота, ширина же отличается, поскольку количество волн не совпадает.
Плоские АЦЛ
Определенные качества плоских и волновых профилей схожи, тем не менее между ними есть определенные различия. К примеру, плоские могут быть непрессованными, а это значит, что они будут отличаться по своим техническим характеристикам. Следует отметить, что плоские АЦЛ более прочные по сравнению с волновыми. К примеру, их прочность на сжатие и изгиб достигает, соответственно, 90-130 и 20-50 Мпа.
Главное достоинство этого материала, скорее всего, в разнообразии его использования. Всего несколько примеров:
достаточно малый вес позволяет использовать плоские профили при устройстве перекрытия, причем дополнительные элементы укрепления при этом не используют.
довольно часто используется в качестве внутренней и внешней отделки зданий;
с их помощью возводят перегородки различного типа и вертикальные ограждения.
длина может быть 2,5, 3,0 и 3,5 м;
ширина – 1,2 и 1,5 м;
толщина – 0,6, 0,8 и 1,0 см.
На строительном рынке можно встретить также плоские листы промышленного производства меньших габаритов (длина – 0,6 м, ширина – 0,4 м), которые подходят для устройства кровли.
Следует отметить, что производители изготавливают на заказ профили других габаритов и оттенков. Разработанные красители отличаются устойчивостью не только к воздействиям атмосферы, но и к выгоранию.
2019 stylekrov.ru
Как правильно пишутся размеры высота, ширина, длина обозначения латинскими буквами
Решая геометрические задачи, ученики сталкиваются с вопросом: как правильно обозначить те или иные части чертежа? Например, высоту треугольника, ширину прямоугольника, размеры бассейна. Подобные обозначения мы найдем и в физических задачах: длина маятника, высота, с которой тело начинает падать… Поэтому следует знать некоторые правила.
Как обозначаются различные параметры
В единой системе измерения используется обозначение латинскими буквами:
длину — буквой l, если речь идет об одной прямой линии: маятнике, рычаге, отрезке, прямой. Но если речь идет о геометрической фигуре, например, прямоугольнике, то используется А,
высоту или глубину – h,
ширину – В.
Что такое система СИ, ученики узнают лишь в средней школе, поэтому обычно в младших классах специального обозначениям для этих величин не вводят.
Как обозначить глубину?
Почему же для высоты и глубины применяется одна и та же буква? Если вы построите чертеж параллелепипеда, то здесь вы отметите высоту фигуры.
А если составить чертеж прямоугольного бассейна того же размера, что и параллелепипед, то обозначается глубина. Таким образом, можно сказать, высота и глубина в этом случае будут одной величиной.
Понятие «глубина» встречается и в географии. На картах она отображается цветом. Если речь идет о водных просторах, то чем темнее синий, цвет, тем больше глубина, а если речь идет о суше, то низменности обозначаются темно-зеленым цветом.
В черчении эта величина обозначается литерой S. Она позволяет создать полное восприятие объекта иногда даже с одним видом.
Что бывает длинным
Что же такое длина и как обозначается этот показатель? Она указывает расстояние от точки до точки, то есть размер отрезка. В геометрических задачах его принято обозначать как А. В стереометрии ее могут обозначать и А, и l (например, в задачах, где встречается прямая, пересекающая плоскость).
В физике же длина маятника, плеча рычага и т.д. в «Дано» обозначается буквой l, так как речь идет об отдельной прямой.
Отличие длины от высоты
Длина – это величина, которая характеризует протяженность линии.
А высота – это перпендикуляр, опущенный на противолежащую плоскость.
То есть можно сделать вывод, что длина от высоты отличается тем, что является частью фигуры, совпадая с ее гранью, а высота получается в результате дополнительного построения на чертеже.
Высоту проводят для того, чтобы получить новые данные для решения задач, а также новых фигур в составе исходной.
Вот такой ширины
Ширина предмета необходима для того, чтобы понять форму как двумерного, так и трехмерного объекта. Как правило, она обозначается буквой В.
Измеряется ширина в метрах (по СИ). Но если предмет слишком мал, то для удобства используют более мелкие единицы измерения:
дециметры,
сантиметры,
миллиметры,
микрометры и т.д.
А если предмет слишком крупный, то пишутся такие приставки:
Разумеется, такие крупные единицы измерения необходимы, например, для астрономии. Также они применяются в квантовой физике, микробиологии и так далее.
Как называются стороны прямоугольника?
В отличие от квадрата, стороны прямоугольника попарно равны и параллельны.
Это значит, что стороны, образующие углы различны.
Как правило, более длинную сторону прямоугольника называют длиной, а ширина прямоугольника — это его короткая сторона.
В чем измеряются размеры длины, ширины и высоты по СИ
По единой системе измерения длина, высота и ширина измеряются в метрах. Но иногда, если это дробное или многозначное число, для удобства в вычислениях используют кратные единицы измерения.
Для того чтобы знать, как правильно переводить единицы измерения в более крупные или же наоборот мелкие, необходимо знать значения приставок.
Дека — 10 1 ,
Гекто — 10 2 ,
Кило — 10 3 ,
Мега — 10 6 ,
Гига — 10 9 ,
Деци – 10 -1 ,
Санти – 10 -2 ,
Милли – 10 -3 ,
Микро — 10 -6 ,
Нано – 10 -9 .
После подсчетов эти единицы должны быть переведены в метры.
Существуют также внесистемные единицы, но они встречаются очень редко:
миля – 1,6 км,
фут – 12 дюймов – 0,3048 м,
ярд – 36 дюймов – 91,44 мм,
дюйм – 25,4 мм и т.д.
При решении задач такие единицы должны быть переведены в метры.
При выполнении геометрических заданий единицам измерения не уделяют особого внимания, главное, чтобы они были сопоставимы
(если вы производите подсчеты в сантиметрах, значит, все величины необходимо перевести в сантиметры).
А при решении физических задач ответ должен быть дан в метрах в соответствии с единой системой измерения.
Обозначения длины, ширины, высоты в геометрии
Измеряем геометрические параметры
Теперь вы знаете, какой буквой обозначается длина, в чем измеряется ширина прямоугольника, и сможете сами объяснить любому, как обозначаются различные параметры.
Это интересно! Легкие правила округления чисел после запятой
ОСНОВНЫЕ ПОЛОЖЕНИЯ
1.1. Определенная величина обозначается буквой латинского или греческого алфавита без индексов или с индексами, служащими для уточнения различных характеристик этой величины.
1.2. Прописные и строчные буквы «О, о» латинского алфавита не должны употребляться в обозначениях. Буквы греческого алфавита следует принимать по табл. 1.
1.3. Буквенные обозначения необходимых величин, не приведенных в настоящем стандарте СЭВ, устанавливают по принципу, указанному в табл. 2.
Сила, произведение силы на длину, длина в степени, не равной единице
Прописные латинского алфавита
Длина, отношение длины ко времени в какой-либо степени, отношением усилия к единице длины или площади
Строчные латинского алфавита
Строчные греческого алфавита
1.4. Индексы подразделяются на цифровые и буквенные. Буквенные дополнительно подразделяются на одно-, двух- и трехбуквенные. Для обозначения цифровых индексов используются арабские цифры, а для обозначения буквенных индексов — буквы латинского алфавита.
1.5. Цифровые индексы применяются для выражения порядкового номера данного обозначения.
1.6. Однобуквенные индексы применяются для обозначения осей координат, расположения, вида материала, напряженного состояния, действующей нагрузки и других характеристик.
1.7. Двухбуквенные и трехбуквенные индексы применяются в том случае, когда использование однобуквенных индексов может привести к неясностям. Они отделяются от однобуквенных индексов запятыми.
1.8. Индексы располагаются с правой стороны букв внизу. При печатании на пишущей машинке букву и индекс допускается печатать на одной строчке.
1.9. Если в настоящем стандарте отсутствует необходимый индекс, его следует устанавливать из строчных букв латинского алфавита.
1.10. Обозначение, выражающее геометрическую величину, допускается дополнять вертикальным штрихом справа, если необходимо обозначить, что имеется ввиду сжатая часть сечения или элемента.
ГОСТ 4541-70. Машины электрические вращающиеся. Обозначения буквенные установочно-присоединительных и габаритных размеров
(текст документа с изменениями и дополнениями на ноябрь 2014 года)
Утвержден и введен в действие Постановлением Госстандарта СССР от 26 февраля 1970 г. N 235
Взамен ГОСТ 4541-48
Срок введения с 1 января 1971 года
Постановлением Комитета стандартов, мер и измерительных приборов при Совете Министров СССР от 26 февраля 1970 г. N 235. Проверен в 1984 г.
Переиздание (ноябрь 1984 г.) с Изменением N 1, утвержденным в сентябре 1984 г. (ИУС 12-84).
1. Настоящий стандарт распространяется на вновь проектируемые и модернизируемые вращающиеся электрические машины и преобразовательные агрегаты и устанавливает буквенные обозначения установочно-присоединительных и габаритных размеров.
2. Номера чертежей с примерами буквенных обозначений установочно-присоединительных и габаритных размеров электрических машин и концов валов указаны в табл. 1.
(Измененная редакция, Изм. N 1).
3. Буквенные обозначения установочно-присоединительных и габаритных размеров отдельных видов электрических машин и агрегатов с иными конструктивными разновидностями монтажных поверхностей и форм исполнения, не предусмотренных табл. 1, рекомендуется устанавливать аналогично приведенным в настоящем стандарте.
4. Для обозначений установочно-присоединительных и габаритных размеров электрических машин и преобразовательных агрегатов следует применять строчные буквы латинского и греческого алфавитов с подстрочными индексами:
b — для ширины (в направлении, перпендикулярном к оси вала),
d — для диаметров,
l — для длины (в направлении оси вала),
r — для радиусов,
t — для размеров в шпоночных соединениях,
— для угловых размеров.
Примечание. Высоту оси вращения (h) проставляют без подстрочного индекса.
5. Подстрочные индексы к буквенным обозначениям следует устанавливать в зависимости от следующего их назначения:
1 — 9 — для концов валов,
10 — 19 — для размеров лап и фундаментных плит (рам),
20 — 29 — для размеров фланца,
30 — 80 — для остальных установочно-присоединительных размеров,
80 и более — для размеров агрегатов и специальных машин.
6. Буквенные обозначения установочно-присоединительных и габаритных размеров должны соответствовать указанным на черт. 1 — 12 и в табл. 2.
Электрическая машина группы 1М1
Электрическая машина группы 1М2
Электрическая машина группы 1М3
Электрическая машина группы 1М4
Электрическая машина группы 1М5
Электрическая машина группы 1М6
Электрическая машина группы 1М7
Агрегаты преобразовательные двухмашинные
Агрегаты преобразовательные трехмашинные
Выступающий конец вала электрической машины
Второй выступающий конец вала электрической машины
Участок вала под посадку шкива
В чертежах и каталогах проставлять один из размеров или , или .
Чертежи служат лишь для пояснения размеров, приведенных в табл. 2.
Количество размеров, проставляемых в чертежах конкретных исполнений машин, устанавливается применительно к каждому исполнению.
(Измененная редакция, Изм. N 1).
7. При простановке обозначений размеров на рабочих чертежах и в каталогах следует избегать образования замкнутых размерных цепочек, например (см. черт. 8) один из размеров , или должен быть опущен.
8. В случае одинаковых по форме и размерам обоих выступающих концов вала следует устанавливать обозначения, принятые для первого выступающего конца вала.
9. Буквенные обозначения размеров вентиляционных каналов настоящим стандартом не устанавливаются.
10. Буквенные обозначения на чертежах следует выполнять с наклоном. Допускается применение в обозначениях прямых букв и цифр. Форма и размеры букв латинского и греческого алфавитов и арабских цифр должны соответствовать ГОСТ 2.304-81.
Достоинства и недостатки асбоцементных листов
Свою неизменную популярность асбоцементные листы заслужили благодаря широкому набору преимуществ. Отметим лишь некоторые из них.
Это достаточно прочный и долговечный материал.
Наличие открытого огня не представляет угрозы, абсолютно пожаробезопасен. Более того, даже будучи расположенным в очаге возгорания не выделяет вредных веществ.
Устойчив к резким температурным перепадам. Прессованные образцы в состоянии выдерживать до 50 циклов замораживание/оттаивание.
Высокая ремонтопригодность кровли: ее достаточно просто отремонтировать, установив заплатку или заменив целые листы.
Материалу не грозят такие негативные явления как гниение и коррозия, его легко обрабатывать, используя самые простые инструменты.
Устройство кровли при стандартных размерах листа шифера достаточно простое, позволяет сократить время монтажа и сэкономить на профессионализме кровельщиков.
недостаточно высокая прочность на изгиб и механическое воздействие;
относительно высокий уровень удельного веса – порядка 20 кг/кв. м;
проблема образования мха, который негативно сказывается на прочности профиля и его внешней привлекательности.
Габаритные размеры
Авиация: Энциклопедия. — М.: Большая Российская Энциклопедия . Главный редактор Г.П. Свищев . 1994 .
Смотреть что такое «Габаритные размеры» в других словарях:
габаритные размеры — Рис. 1. Габаритные размеры самолёта. габаритные размеры самолёта, вертолёта предельные значения длины и высоты, полного размаха крыла (у самолёта), диаметра несущего винта (у вертолёта) и т. п. (см. рис. 1, 2). Г. р. летательного аппарата… … Энциклопедия «Авиация»
габаритные размеры — Рис. 1. Габаритные размеры самолёта. габаритные размеры самолёта, вертолёта предельные значения длины и высоты, полного размаха крыла (у самолёта), диаметра несущего винта (у вертолёта) и т. п. (см. рис. 1, 2). Г. р. летательного аппарата… … Энциклопедия «Авиация»
габаритные размеры — Номинальные наружные размеры (включая при необходимости положительные допуски): длина, ширина и высота, измеряемые вдоль наружных кромок контейнера. Примечание Допуски к диагоналям, приемлемые для всех шести граней контейнера, даны в ИСО 668 95.… … Справочник технического переводчика
ГАБАРИТНЫЕ РАЗМЕРЫ — (в антропометрии) наибольшие размеры тела в разных его положениях и позах, ориентированные в разных плоскостях (размеры рук, наибольший поперечный диаметр тела, горизонтальная и вертикальная досягаемость руки и т. п.). Г. р. измеряются по… … Энциклопедический словарь по психологии и педагогике
габаритные размеры электроагрегата (электростанции) в транспортном положении — габаритные размеры Расстояние между крайними по длине, ширине и высоте точками электроагрегата (электростанции). Тематики электроагрегаты генераторные Синонимы габаритные размеры … Справочник технического переводчика
габаритные размеры пакетированной авиационной грузовой единицы — Предельные наружные размеры пакетированной авиационной грузовой единицы, включающие в себя любые ручки или другие выступающие элементы на ее поверхности. Тематики авиационные грузовые перевозки EN external dimensionsULD… … Справочник технического переводчика
габаритные размеры тары — Максимальные наружные размеры тары, включая выступающие части и детали. Тематики упаковка, упаковывание Обобщающие термины параметры и характеристики тары и упаковки EN overall dimensions of a container DE Grossmasse der… … Справочник технического переводчика
mr-build.ru
Буквенные обозначения на чертежах
ГОСТ 2.321 – 84
Для оформления конструкторских документов предусмотрены основные буквенные обозначения, которые отражают следующие условные величины:
Высота и глубина
Для обозначения габаритных и суммарных размеров рекомендуется применять прописные буквы.
Если в одном и том же документе используется одинаковые буквы, для различных величин, применяются цифровые или буквенные индексы, например:
d, d1, d2, dn, dn1, dn2.
Расстояние между осями или центрами
Обозначение ширины
Указание диаметра
Обозначение высоты или глубины
Обозначение длины
Радиус элемента детали
Толщина листа
Шаг витка пружины
Углы
gk-drawing.ru
Как пишутся размеры длина ширина высота – габариты как правильно указывать
Как правильно пишутся размеры: высота, ширина, длина обозначения латинскими буквами
Решая геометрические задачи, ученики сталкиваются с вопросом: как правильно обозначить те или иные части чертежа? Например, высоту треугольника, ширину прямоугольника, размеры бассейна. Подобные обозначения мы найдем и в физических задачах: длина маятника, высота, с которой тело начинает падать… Поэтому следует знать некоторые правила….
Как обозначаются различные параметры
В единой системе измерения используется обозначение латинскими буквами:
длину буквой l, если речь идет об одной прямой линии: маятнике, рычаге, отрезке, прямой. Но если речь идет о геометрической фигуре, например, прямоугольнике, то используется А,
высоту или глубину – h,
ширину – В.
Что такое система СИ, ученики узнают лишь в средней школе, поэтому обычно в младших классах специального обозначениям для этих величин не вводят.
Как обозначить глубину?
Почему же для высоты и глубины применяется одна и та же буква? Если вы построите чертеж параллелепипеда, то здесь вы отметите высоту фигуры.
А если составить чертеж прямоугольного бассейна того же размера, что и параллелепипед, то обозначается глубина. Таким образом, можно сказать, высота и глубина в этом случае будут одной величиной.
Внимание! Высота и глубина – две величины, которые обозначают один и тот же перпендикуляр, соединяющий две противоположные плоскости.
Понятие «глубина» встречается и в географии. На картах она отображается цветом. Если речь идет о водных просторах, то чем темнее синий, цвет, тем больше глубина, а если речь идет о суше, то низменности обозначаются темно-зеленым цветом.
В черчении эта величина обозначается литерой S. Она позволяет создать полное восприятие объекта иногда даже с одним видом.
Что бывает длинным
Что же такое длина и как обозначается этот показатель? Она указывает расстояние от точки до точки, то есть размер отрезка. В геометрических задачах его принято обозначать как А. В стереометрии ее могут обозначать и А, и l (например, в задачах, где встречается прямая, пересекающая плоскость).
В физике же длина маятника, плеча рычага и т.д. в «Дано» обозначается буквой l, так как речь идет об отдельной прямой.
Отличие длины от высоты
Длина – это величина, которая характеризует протяженность линии.
А высота – это перпендикуляр, опущенный на противолежащую плоскость.
То есть можно сделать вывод, что длина от высоты отличается тем, что является частью фигуры, совпадая с ее гранью, а высота получается в результате дополнительного построения на чертеже.
Высоту проводят для того, чтобы получить новые данные для решения задач, а также новых фигур в составе исходной.
Вот такой ширины
Ширина предмета необходима для того, чтобы понять форму как двумерного, так и трехмерного объекта. Как правило, она обозначается буквой В.
Измеряется ширина в метрах (по СИ). Но если предмет слишком мал, то для удобства используют более мелкие единицы измерения:
дециметры,
сантиметры,
миллиметры,
микрометры и т.д.
А если предмет слишком крупный, то пишутся такие приставки:
Разумеется, такие крупные единицы измерения необходимы, например, для астрономии. Также они применяются в квантовой физике, микробиологии и так далее.
Как называются стороны прямоугольника?
В отличие от квадрата, стороны прямоугольника попарно равны и параллельны.
Это значит, что стороны, образующие углы различны.
Как правило, более длинную сторону прямоугольника называют длиной, а ширина прямоугольника это его короткая сторона.
Важно! Зная такие данные, как длина и ширина прямоугольника, можно найти его периметр, площадь, длину диагоналей и угол между ними. Вокруг прямоугольника всегда можно описать окружность. Эти свойства работают и в обратном направлении.
В чем измеряются размеры длины, ширины и высоты по СИ
По единой системе измерения длина, высота и ширина измеряются в метрах. Но иногда, если это дробное или многозначное число, для удобства в вычислениях используют кратные единицы измерения.
Для того чтобы знать, как правильно переводить единицы измерения в более крупные или же наоборот мелкие, необходимо знать значения приставок.
Дека 101,
Гекто 102,
Кило 103,
Мега 106,
Гига 109,
Деци – 10-1,
Санти – 10-2,
Милли – 10-3,
Микро 10-6,
Нано – 10-9.
После подсчетов эти единицы должны быть переведены в метры.
Существуют также внесистемные единицы, но они встречаются очень редко:
миля – 1,6 км,
фут – 12 дюймов – 0,3048 м,
ярд – 36 дюймов – 91,44 мм,
дюйм – 25,4 мм и т.д.
При решении задач такие единицы должны быть переведены в метры.
При выполнении геометрических заданий единицам измерения не уделяют особого внимания, главное, чтобы они были сопоставимы
(если вы производите подсчеты в сантиметрах, значит, все величины необходимо перевести в сантиметры).
А при решении физических задач ответ должен быть дан в метрах в соответствии с единой системой измерения.
Обозначения длины, ширины, высоты в геометрии
Измеряем геометрические параметры
Вывод
Теперь вы знаете, какой буквой обозначается длина, в чем измеряется ширина прямоугольника, и сможете сами объяснить любому, как обозначаются различные параметры.
Это интересно! Легкие правила округления чисел после запятой
X Y Z
Казалось бы, что сложного в правильном расположении пары слов на картинке или рядом с фотогрфаией. Но нет.
Нередко редакторы в статьях не сопоставляют присланный текст копирайтера и фотоколлаж от дизайнера. А если они сами и текст пишут и фотографии подбирают, тогда это совсем странно:
В дизайне это тоже встрачается. Например, почти на каждой второй обложке имена располагают напротив чужого актера. И даже женщины с мужскими именами и мужики с женскими дизайнеров совсем не смущают:
А еще можно перепутать длину и ширину. Чаще всего это встрачается на картинках с размерами фотографий. Правильно в таких случаях сначала подписывать ось Х, затем ось Y:
Найдете фейл в распечатанных фотографиях на стене?
Ниже в комментариях еще один был найден
Тут ребята из музея в комментарии пожаловали и они не согласны. Оказывается, у них там свое государство со своими законами. Поэтому, если надо написать картину маслом, то Y x X. А если ее нужно напечатать на принтере, то X x Y. К слову, рамки для фото/картин в магазинах продают для дизайнеров, а не для художников. Ну тоже Х х Y. И так как мы дизайнеры с маслом не дружим, то пример менять не буду. Всем спасибо, расходимся
Легко можно запомнить легко по последним буквам английского алфавита:
X (длина), Y (ширина/высота), Z (глубина/толщина)
Добавь к ответу свадебное фото прямо над девушкой (40×60)
Что-то пошло не так)) Принято (по крайней мере в нашей стране (РФ)) писать «ШВД», расшифровывается как Ширина Высота Длина (она же Глубина, она же Толщина). Ширина всегда указывается первой, Высота — вторая ну и третий параметр зависит от ситуации (2D/3D).
ШВД — это спел в Харстоуне: Shadow Word Death. А писать надо сначала длину (по горизонтали Х), потом ширину (по вертикали Y), потом глубину (по оси Z).
Ширина Высота Длина (она же Глубина, она же Толщина) — это те же XYZ. Только названия неподходящие.
Длина (она же Глубина, она же Толщина)
Очевидно же, что «длина» — по длинной стороне, а не по короткой. А глубина/толщина — как правило, самые маленькие значения.
В теме «Найдете фейл?» Фейлом является всё, кроме 80х90. В художественной живописи, в галереях, музеях и т.д., сначала указывают высоту, а затем ширину. Надо же быть в теме, прежде чем что-то кому-то объяснять. И потом вы путаете понятие Ширина (Y) — это высота.
В теме «Найдете фейл?» Фейлом является всё, кроме 80х90.
В художественной живописи, в галереях, музеях и т.д., сначала указывают высоту, а затем ширину.
Почти везде правосторонее движение, а где-то с левой стороны дороги ездят. Возможно, в живописи так принято, значит, пример мой не подходит, и надо бы его поменять на такой же, но без багета, чтобы получить не картины на стене, а печать фото на документы. Но он взят с сайта художника, а не дизайнера. Уж он то в курсе
Не хватает только мебельщиков. Ждем))
Проблема в другом. Речь о дизайне, а не о музеях. Я дизайнер и пишу дизайнерам о дизайне. В полиграфии принят такой порядок: X x Y.
Сначала Х, потом Y. Если не доверяете Вики, откройте любой сайт типографии. Хотя любой — это я погорячился. Вот уже сайт одного из художников открыл на свою голову
И потом вы путаете понятие Ширина (Y) — это высота.
Ширина это и есть высота, смотря в какой плоскости расположить измеряемый предмет. Если нужно сделать принт на пол, то у него нет никакой высоты: только длина и ширина. С потолками, например, тоже самое: натяжные потолки с фото — для подготовки рисунка нужна длина и его ширина. А выражение «высота потолков» — это как раз расстояние от пола до потолка. Если же печатаем картину на стену, то ширина трансформируется в высоту. А глубина/толщина холста, соответственно, третье измерение.
Надо же быть в теме, прежде чем что-то кому-то объяснять.
А вот с этим как раз-таки и не поспоришь. Долгое время работал в типографии и даже был техническим специалистом отдела дизайна: ну т.е. подготовка файлов, смик/ргб, размеры и вот это все.
А значит, если надо написать картину маслом, то Y x X. А если ее нужно напечатать на принтере, то X x Y. К слову, рамки для фото/картин в магазинах продают для дизайнеров, а не для художников. Ну т.е. тоже Х х Y. И так как мы, дизайнеры, с маслом не дружим, то пример менять не буду. Тем более, добавив перед ним фразу «…в распечатанных фотографиях на стене», все стало на свои места, верно?)
В любом случае, в конце статьи добавил сноску про музей. Спасибо
Запись габаритных размеров
15 сообщений в этой теме
Рекомендуемые сообщения
Создайте аккаунт или авторизуйтесь, чтобы оставить комментарий
Комментарии могут оставлять только зарегистрированные пользователи
Создать аккаунт
Зарегистрировать новый аккаунт.
Войти
Есть аккаунт? Войти.
Недавно просматривали 0 пользователей
Ни один зарегистрированный пользователь не просматривает эту страницу.
Популярные темы
Автор: Alex2010 Создана 24 Января 2011
Автор: mpanikovskiy Создана 14 Июня 2012
Автор: rmetr Создана 22 Февраля 2014
Автор: zrg Создана 21 час назад
Автор: Евгения_1 Создана Понедельник в 13:19
Автор: mpanikovskiy Создана 14 Июня 2012
Автор: efim Создана 4 Марта
Автор: Alex2010 Создана 24 Января 2011
Автор: Smoker Создана 4 Мая 2012
Автор: efim Создана 31 Декабря 2015
Автор: mpanikovskiy Создана 14 Июня 2012
Автор: efim Создана 31 Декабря 2015
Автор: efim Создана 4 Марта
Автор: Alex2010 Создана 24 Января 2011
Автор: Геометр Создана 10 Сентября
Автор: AtaVist Создана 11 Августа 2017
Автор: mpanikovskiy Создана 14 Июня 2012
Автор: метролог2009 Создана 10 Сентября 2015
Автор: sergeevich-33 Создана 26 Декабря 2018
Автор: evGeniy Создана 4 Февраля 2013
Автор: AtaVist Создана 11 Августа 2017
Автор: Metrolog-sever Создана 2 Июля 2014
Автор: UNECE Создана 8 Декабря 2016
Автор: метролог2009 Создана 10 Сентября 2015
Общие правила нанесения размеров на чертежах
Стандарт (ГОСТ 2.307-68) устанавливает правила нанесения размеров на чертежах.
Линейные размеры на чертежах проставляются в миллиметрах без обозначения единиц измерения (мм). При других единицах измерения (сантиметрах, метрах) размерные числа записываются с обозначением единиц измерения (см, mi). Угловые размеры указывают в градусах, минутах, секундах с обозначением единиц измерения. Общее количество размеров на чертежах должно быть минимальным, но достаточным для изготовления и контроля изделия.
Существуют строго определенные правила нанесения размеров. При нанесении размера прямолинейного отрезка размерную линию проводят параллельно этому отрезку, а выносные линии — перпендикулярно размерным (рис. 40, б). Выносные линии выходят за размерные на 1-3 мм. Расстояние от размерной линии до контура изображения должно быть не менее 10 мм, а расстояние между двумя близлежащими размерными линиями — не менее 7 мм (рис. 40, б).
На концах размерных линий наносят стрелки. Форма и размеры стрелки показаны на рис. 40, а. Величина стрелок должна быть одинаковой на всем чертеже. Стрелки при недостатке места могут заменяться засечками или точками (рис. 41, б, в). Допускается проставлять размеры так, как показано на рис. 41, г.
Размерные числа наносят над размерной линией ближе к середине (рис. 42).
При нанесении нескольких параллельных или концентрических размерных линий размерные числа над ними располагают в шахматном порядке (рис. 43).
На чертежах необходимо избегать пересечения размерных и выносных линий. Если для нанесения размерного числа недостаточно места над размерной линией, то размеры проставляются так, как показано на рис. 44.
В местах нанесения размерного числа осевые, центровые линии и линии штриховки прерывают (рис. 45, а, б).
При нанесении размеров дуг перед размерным числом помещают знак радиуса — R. Высота знака радиуса и размерного числа должна быть одинаковой (рис. 46, а). При проведении нескольких радиусов из одного центра размерные линии любых двух радиусов не располагают на одной прямой (рис. 46, б). При большой величине радиуса центр разрешается приближать к дуге. В таких случаях размерную линию показывают с изломом (рис. 46, в).
При нанесении размеров окружностей перед размерным числом ставят знак диаметра — 0 (рис. 47). При недостатке места на чертеже размеры диаметра проставляют так, как показано на рис. 47, б.
Размеры нескольких одинаковых элементов изделия наносят один раз с указанием их количества на полке-выноске, рис. 48.
Размеры квадрата или квадратного отверстия наносятся, как показано на рис. 49.
Толщина плоской детали обозначается буквой S с последующим указанием размерного числа (рис. 50).
Длина изделия обозначается малой буквой латинского алфавита — I (рис. 51).
Нанесение размеров фаски — скошенной кромки стержня, бруска, отверстия — осуществляется либо простановкой двух линейных размеров (рис. 52, б), либо линейным и угловым размерами (рис. 52, в, г).
Если на чертеже встречается несколько одинаковых фасок, то размер наносят один раз так, как показано на рис. 52, в. Эта надпись означает, что снято две фаски размером 2 мм под углом 45°.
На чертежах необходимо проставлять габаритные размеры.
Габаритными размерами называют размеры, определяющие предельные величины внешних очертаний изделий. К габаритным размерам относятся размеры длины, ширины, высоты изделия.
Габаритные размеры всегда больше других, поэтому их на чертеже располагают дальше от изображения, чем остальные.
На рис. 53 (валик) — габаритными являются размеры 75 мм и 40 мм.
На рис. 53 (полуцилиндр) — к габаритным относятся размеры 80 мм, 50 мм.
На чертежах иногда наносят справочные размеры. Размеры, нанесенные на чертеже, но не подвергающиеся контролю, называют справочными. На чертеже они отмечаются знаком * (рис. 54). На месте расположения технических требований (над основной надписью) делают запись: * — размер для справок.
Не нашли то, что искали? Воспользуйтесь поиском:
Лучшие изречения:Да какие ж вы математики, если запаролиться нормально не можете. 8358 — | 7290 — или читать все.
Отключите adBlock! и обновите страницу (F5) очень нужно
Обозначение: высота, ширина, длина. Ширина — обозначение буквой. Обозначение ширины на чертежах
Построение чертежей — дело непростое, но без него в современном мире никак. Ведь чтобы изготовить даже самый обычный предмет (крошечный болт или гайку, полку для книг, дизайн нового платья и подобное), изначально нужно провести соответствующие вычисления и нарисовать чертеж будущего изделия. Однако часто составляет его один человек, а занимается изготовлением чего-либо по этой схеме другой.
Чтобы не возникло путаницы в понимании изображенного предмета и его параметров, во всем мире приняты условные обозначения длины, ширины, высоты и других величин, применяемых при проектировании. Каковы они? Давайте узнаем.
Величины
Площадь, длина, ширина, высота и другие обозначения подобного характера являются не только физическими, но и математическими величинами.
Единое их буквенное обозначение (используемое всеми странами) было уставлено в середине ХХ века Международной системой единиц (СИ) и применяется по сей день. Именно по этой причине все подобные параметры обозначаются латинскими, а не кириллическими буквами или арабской вязью. Чтобы не создавать отдельных трудностей, при разработке стандартов конструкторской документации в большинстве современных стран решено было использовать практически те же условные обозначения, что применяются в физике или геометрии.
Любой выпускник школы помнит, что в зависимости от того, двухмерная или трехмерная фигура (изделие) изображена на чертеже, она обладает набором основных параметров. Если присутствуют два измерения — это ширина и длина, если их три – добавляется еще и высота.
Итак, для начала давайте выясним, как правильно длину, ширину, высоту обозначать на чертежах.
Ширина
Как было сказано выше, в математике рассматриваемая величина является одним из трех пространственных измерений любого объекта, при условии что его замеры производятся в поперечном направлении. Так чем знаменита ширина? Обозначение буквой «В» она имеет. Об этом известно во всём мире. Причем, согласно ГОСТу, допустимо применение как заглавной, так и строчной латинских литер. Часто возникает вопрос о том, почему именно такая буква выбрана. Ведь обычно сокращение производится по первой букве латинского, греческого или английского названия величины. При этом ширина на английском будет выглядеть как «width».
Вероятно, здесь дело в том, что данный параметр наиболее широкое применение изначально имел в геометрии. В этой науке, описывая фигуры, часто длину, ширину, высоту обозначают буквами «а», «b», «с». Согласно этой традиции, при выборе литера «В» (или «b») была заимствована системой СИ (хотя для других двух измерений стали применять отличные от геометрических символы).
Большинство полагает, что это было сделано, дабы не путать ширину (обозначение буквой «B»/«b») с весом. Дело в том, что последний иногда именуется как «W» (сокращение от английского названия weight), хотя допустимо использование и других литер («G» и «Р»). Согласно международным нормам системы СИ, измеряется ширина в метрах или кратных (дольных) их единицах. Стоит отметить, что в геометрии иногда также допустимо использовать «w» для обозначения ширины, однако в физике и остальных точных науках такое обозначение, как правило, не применяется.
Длина
Как уже было указано, в математике длина, высота, ширина – это три пространственных измерения. При этом, если ширина является линейным размером в поперечном направлении, то длина — в продольном. Рассматривая ее как величину физики можно понять, что под этим словом подразумевается численная характеристика протяжности линий.
В английском языке этот термин именуется length. Именно из-за этого данная величина обозначается заглавной или строчной начальной литерой этого слова — «L». Как и ширина, длина измеряется в метрах или их кратных (дольных) единицах.
Высота
Наличие этой величины указывает на то, что приходится иметь дело с более сложным — трехмерным пространством. В отличие от длины и ширины, высота численно характеризует размер объекта в вертикальном направлении.
На английском она пишется как «height». Поэтому, согласно международным нормам, ее обозначают латинской литерой «Н»/«h». Помимо высоты, в чертежах иногда эта буква выступает и как глубины обозначение. Высота, ширина и длина – все все эти параметры измеряются в метрах и их кратных и дольных единицах (километры, сантиметры, миллиметры и т. п.).
Радиус и диаметр
Помимо рассмотренных параметров, при составлении чертежей приходится иметь дело и с иными.
Например, при работе с окружностями возникает необходимость в определении их радиуса. Так именуется отрезок, который соединяет две точки. Первая из них является центром. Вторая находится непосредственно на самой окружности. На латыни это слово выглядит как «radius». Отсюда и общепринятое сокращение: строчная или заглавная «R»/«r».
Чертя окружности, помимо радиуса часто приходится сталкиваться с близким к нему явлением – диаметром. Он также является отрезком, соединяющим две точки на окружности. При этом он обязательно проходит через центр.
Численно диаметр равен двум радиусам. По-английски это слово пишется так: «diameter». Отсюда и сокращение – большая или маленькая латинская буква «D»/«d». Часто диаметр на чертежах обозначают при помощи перечеркнутого круга – «Ø».
Хотя это распространенное сокращение, стоит иметь в виду, что ГОСТ предусматривает использование только латинской «D»/«d».
Толщина
Большинство из нас помнят школьные уроки математики. Ещё тогда учителя рассказывали, что, латинской литерой «s» принято обозначать такую величину, как площадь. Однако, согласно общепринятым нормам, на чертежах таким способом записывается совсем другой параметр – толщина.
Почему так? Известно, что в случае с высотой, шириной, длиной, обозначение буквами можно было объяснить их написанием или традицией. Вот только толщина по-английски выглядит как «thickness», а в латинском варианте — «crassities». Также непонятно, почему, в отличие от других величин, толщину можно обозначать только строчной литерой. Обозначение «s» также применяется при описании толщины страниц, стенок, ребер и так далее.
Периметр и площадь
В отличие от всех перечисленных выше величин, слово «периметр» пришло не из латыни или английского, а из греческого языка. Оно образовано от «περιμετρέο» («измерять окружность»). И сегодня этот термин сохранил свое значение (общая длина границ фигуры). Впоследствии слово попало в английский язык («perimeter») и закрепилось в системе СИ в виде сокращения буквой «Р».
Площадь — это величина, показывающая количественную характеристику геометрической фигуры, обладающей двумя измерениями (длиной и шириной). В отличие от всего перечисленного ранее, она измеряется в квадратных метрах (а также в дольных и кратных их единицах). Что касается буквенного обозначения площади, то в разных сферах оно отличается. Например, в математике это знакомая всем с детства латинская литера «S». Почему так – нет информации.
Другие распространенные сокращения
Обозначения высоты, ширины, длины, толщины, радиуса, диаметра являются наиболее употребляемыми при составлении чертежей. Однако есть и другие величины, которые тоже часто присутствуют в них. Например, строчное «t». В физике это означает «температуру», однако согласно ГОСТу Единой системы конструкторской документации, данная литера — это шаг (винтовых пружин, заклепочных соединений и подобного). При этом она не используется, когда речь идет о зубчатых зацеплениях и резьбе.
Заглавная и строчная буква «A»/«a» (согласно все тем же нормам) в чертежах применяется, чтобы обозначать не площадь, а межцентровое и межосевое расстояние. Помимо различных величин, в чертежах часто приходится обозначать углы разного размера. Для этого принято использовать строчные литеры греческого алфавита. Наиболее применяемые — «α», «β», «γ» и «δ». Однако допустимо использовать и другие.
Какой стандарт определяет буквенное обозначение длины, ширины, высоты, площади и других величин?
Как уже было сказано выше, чтобы не было недопонимания при прочтении чертежа, представителями разных народов приняты общие стандарты буквенного обозначения. Иными словами, если вы сомневаетесь в интерпретации того или иного сокращения, загляните в ГОСТы. Таким образом вы узнаете, как правильно обозначается высота, ширины, длина, диаметр, радиус и так далее.
Для Российской Федерации таким нормативным документом является ГОСТ 2.321-84. Он был внедрен еще в марте 1984 г. (во времена СССР), взамен устаревшего ГОСТа 3452—59.
iv-proect.ru
какой буквой в геометрии обозначается ширина
В принципе, любой. В формуле площади прямоугольника S = a*b одна из величин длина, другая ширина. Какая где — безразлично.
Нет конкретного обозначения ширины.
а-длина, б-ширина, но это условно.. . а так любой буквой…
согласно Буквенные обозначения ГОСТ 2.321-68, ширина обозначается буквой B, b
touch.otvet.mail.ru
Главные размерения — Википедия
Материал из Википедии — свободной энциклопедии
Главные размерения судна (основные размерения судна или корабля) — совокупность конструктивных, расчётных, наибольших и габаритных линейных размеров судна: длины, ширины, осадки и высоты борта. Главные размерения характеризуют мореходные качества корабля (судна) и определяют возможность его проводки в узкостях (каналах, бухтах, проливах), шлюзах, на мелководье, определяют возможность размещения на стапеле (слипе) или постановки в корабельный док[1].
Высота борта — вертикальное расстояние, измеряемое в плоскости мидель-шпангоута от основной плоскости до бортовой линии верхней палубы — линии пересечения теоретических поверхностей борта и верхней палубы или их продолжения при закруглённом соединении палубы с бортом. Для подводной лодки высота борта определяется как расстояние по вертикали между крайними точками наружного корпуса в районе мидель-шпангоута, но без учёта высоты ограждения рубки[2].
Осадка — расстояние от горизонтальной плоскости, проходящей через нижнюю точку в середине длины корпуса (без учёта выступающих частей) до поверхности спокойной воды. Различают осадку носом, кормой и среднюю осадку, как среднеарифметическое значение кормовой и носовой осадки[1].
Длина судна — расстояние между его носовыми и кормовыми конструктивными элементами[2]. Различают длину судна: по конструктивной ватерлинии, между перпендикулярами, наибольшую и габаритную. Для подводных лодок дополнительно различают: длину непроницаемого корпуса и длину прочного корпуса[1].
Длина корабля по конструктивной (расчётной) ватерлинии — расстояние между точками пересечения носовой и кормовой частей конструктивной ватерлинии с диаметральной плоскостью. Длина между перпендикулярами — расстояние между носовым и кормовым перпендикулярами корабля. Наибольшая длина корабля — расстояние между крайними точками теоретической поверхности корпуса корабля в носовой и кормовой оконечностях. Габаритная длина судна — расстояние между крайними точками носовой и кормовой оконечностей корпуса с учётом выступающих частей. Длина непроницаемого корпуса подводной лодки — расстояние между концевыми поперечными переборками (чаще всего концевых цистерн главного балласта). Длина прочного корпуса — расстояние между крайними точками концевых переборок прочного корпуса[1].
Длина гражданского судна — расстояние, измеряемое на уровне летней грузовой ватерлинии от передней кромки форштевня до оси баллера руля или 96 % длины судна, измеряемой на уровне этой ватерлинии от передней кромки форштевня до крайней кромки кормовой оконечности судна[1].
Ширина судна — расстояние между различными конструктивными точками корабля, расположенными на его правом и левом бортах. Различают наибольшую ширину, ширину по конструктивной ватерлинии, по расчётной ватерлинии, на мидель-шпангоуте, габаритную, ширину по стабилизаторам (для подводных лодок)[1].
Наибольшей шириной корабля называют расстояние, измеряемое перпендикулярно диаметральной плоскости, между крайними точками теоретической поверхности корпуса корабля. Шириной корабля по конструктивной ватерлинии называют наибольшую ширину конструктивной ватерлинии. Шириной корабля по расчётной ватерлинии называют наибольшую ширину расчётной ватерлинии. Шириной корабля на мидель-шпангоуте называют ширину конструктивной ватерлинии на мидель-шпангоуте. Габаритной шириной корабля называют расстояние, измеряемое перпендикулярно диаметральной плоскости между крайними точками корпуса корабля (судна), с учётом выступающих частей. Под шириной подводной лодки по стабилизаторам подразумевают размах стабилизаторов подводной лодки, то есть расстояние между крайними точками стабилизаторов[1].
Главные размерения корабля // Военно-морской словарь / Чернавин В. Н. — М.: Воениздат, 1990. — С. 107—108. — 511 с. — ISBN 5-203-00174-X.
Лобач-Жученко М. Б. Основные элементы кораблей и судов. — М.: издательство ДОСААФ, 1955. — 80 с. — 10 000 экз.
ru.wikipedia.org
Список обозначений в физике — Википедия
Символ
Значение и происхождение
A{\displaystyle A}
Площадь (лат. area), векторный потенциал[1], работа (нем. Arbeit), амплитуда (лат. amplitudo), параметр вырождения, Работа выхода (нем. Austrittsarbeit), коэффициент Эйнштейна для спонтанного излучения, массовое число
a{\displaystyle a}
Ускорение (лат. acceleratio), амплитуда (лат. amplitudo), активность (лат. activitas), коэффициент температуропроводности, вращательная способность, радиус Бора, натуральный показатель поглощения света
B{\displaystyle B}
Вектор магнитной индукции[1], барионный заряд (англ. baryon number), удельная газовая постоянная, вириальний коэффициент, функция Бриллюэна (англ. Brillion function), ширина интерференционной полосы (нем. Breite), яркость, постоянная Керра, коэффициент Эйнштейна для вынужденного излучения, коэффициент Эйнштейна для поглощения, вращательная постоянная молекулы
b{\displaystyle b}
Вектор магнитной индукции[1], красивый кварк (англ. beauty/bottom quark), постоянная Вина, ширина распада (нем. Breite)
C{\displaystyle C}
Электрическая ёмкость (англ. capacitance), теплоёмкость (англ. heatcapacity), постоянная интегрирования (лат. constans), очарование (чарм, шарм; англ. charm), коэффициенты Клебша — Гордана (англ. Clebsch-Gordan coefficients), постоянная Коттона — Мутона (англ. Cotton-Mouton constant), кривизна (лат. curvatura)
c{\displaystyle c}
Скорость света (лат. celeritas), скорость звука (лат. celeritas), Теплоёмкость (англ. heat capacity), очарованный кварк (англ. charm quark), концентрация (англ. concentration), первая радиационная постоянная, вторая радиационная постоянная, удельная теплоёмкость
D{\displaystyle D}
Вектор электрической индукции[1] (англ. electric displacement field), Коэффициент диффузии (англ. diffusion coefficient), Оптическая сила (англ. dioptric power), коэффициент прохождения, тензор квадрупольного электрического момента, угловая дисперсия спектрального прибора, линейная дисперсия спектрального прибора, коэффициент прозрачности потенциального барьера, D-мезон (англ. D meson), Диаметр (лат. diametros, др.-греч. διάμετρος)
d{\displaystyle d}
Расстояние (лат. distantia), Диаметр (лат. diametros, др.-греч. διάμετρος), дифференциал (лат. differentia), нижний кварк (англ. down quark), дипольный момент (англ. dipole moment), период дифракционной решётки, толщина (нем. Dicke)
E{\displaystyle E}
Энергия (лат. energīa), напряжённость электрического поля[1] (англ. electric field), Электродвижущая сила (англ. electromotive force), магнитодвижущая сила, освещенность (фр. éclairement lumineux), излучательная способность тела, модуль Юнга
e{\displaystyle e}
Основание натуральных логарифмов (2,71828…), электрон (англ. electron), элементарный электрический заряд (англ. elementaty electric charge), константа электромагнитного взаимодействия
F{\displaystyle F}
Сила (лат. fortis), постоянная Фарадея (англ. Faraday constant), свободная энергия Гельмгольца (нем. freie Energie), атомный фактор рассеяния, тензор электромагнитного поля, магнитодвижущая сила, модуль сдвига, фокусное расстояние (англ. focal length)
f{\displaystyle f}
Частота (лат. frequentia), функция (лат. functia), летучесть (нем. Flüchtigkeit), сила (лат. fortis), фокусное расстояние (англ. focal length), сила осциллятора, коэффициент трения
G{\displaystyle G}
Гравитационная постоянная (англ. gravitational constant), тензор Эйнштейна, свободная энергия Гиббса (англ. Gibbs free energy), метрика пространства-времени, вириал, парциальная мольная величина, поверхностная активность адсорбата, модуль сдвига, полный импульс поля, Глюон (англ. gluon), константа Ферми, квант проводимости, электрическая проводимость, Вес (нем. Gewichtskraft)
g{\displaystyle g}
Ускорение свободного падения (англ. gravitational acceleration), Глюон (англ. gluon), фактор Ланде, фактор вырождения, весовая концентрация, Гравитон (англ. graviton), метрический тензор
H{\displaystyle H}
Напряжённость магнитного поля[1], эквивалентная доза, энтальпия (англ. heat contents или от греческой буквы «эта», H — ενθαλπος[2]), гамильтониан (англ. Hamiltonian), функция Ганкеля (англ. Hankel function), функция Хевисайда (англ. Heaviside step function), бозон Хиггса (англ. Higgs boson), экспозиция, полиномы Эрмита (англ. Hermite polynomials)
h{\displaystyle h}
Высота (нем. Höhe), постоянная Планка (нем. Hilfsgröße[3]), спиральность (англ. helicity)
I{\displaystyle I}
сила тока (фр. intensité de courant), интенсивность звука (лат. intēnsiō), интенсивность света (лат. intēnsiō), сила излучения, сила света, момент инерции, вектор намагниченности
i{\displaystyle i}
Мнимая единица (лат. imaginarius), единичный вектор (координатный орт)
J{\displaystyle J}
Плотность тока (также 4-вектор плотности тока), момент импульса, функция Бесселя, момент инерции, полярный момент инерции сечения, вращательное квантовое число, сила света, J/ψ-мезон
j{\displaystyle j}
Мнимая единица (в электротехнике и радиоэлектронике), плотность тока (также 4-вектор плотности тока), единичный вектор (координатный орт)
K{\displaystyle K}
Каона (англ. kaons), термодинамическая константа равновесия, коэффициент электронной теплопроводности металлов, модуль всестороннего сжатия, механический импульс, постоянная Джозефсона, кинетическая энергия
Момент импульса, дальность полёта, удельная теплота парообразования и конденсации, индуктивность, функция Лагранжа (англ. Lagrangian), классическая функция Ланжевена (англ. Langevin function), число Лоренца (англ. Lorenz number), уровень звукового давления, полиномы Лагерра (англ. Laguerre polynomials), орбитальное квантовое число, энергетическая яркость, яркость (англ. luminance)
l{\displaystyle l}
Длина (англ. length), длина свободного пробега (англ. length), орбитальное квантовое число, радиационная длина
M{\displaystyle M}
Момент силы, масса (лат. massa, от др.-греч. μᾶζα, кусок теста), вектор намагниченности (англ. magnetization), крутящий момент, число Маха, взаимная индуктивность, магнитное квантовое число, молярная масса
m{\displaystyle m}
Масса, магнитное квантовое число (англ. magnetic quantum number), магнитный момент (англ. magnetic moment), эффективная масса, дефект массы, масса Планка
N{\displaystyle N}
Количество (лат. numerus), постоянная Авогадро, число Дебая, полная мощность излучения, увеличение оптического прибора, концентрация, мощность, сила нормальной реакции
n{\displaystyle n}
Показатель преломления, количество вещества, нормальный вектор, единичный вектор, нейтрон (англ. neutron), количество (англ. number), основное квантовое число, частота вращения, концентрация, показатель политропы, постоянная Лошмидта
O{\displaystyle O}
Начало координат (лат. origo)
P{\displaystyle P}
Мощность (лат. potestas), давление (лат. pressūra), полиномы Лежандра, вес (фр. poids), сила тяжести, вероятность (лат. probabilitas), поляризуемость, вероятность перехода, импульс (также 4-импульс, обобщённый импульс; лат. petere)
p{\displaystyle p}
Импульс (также 4-импульс, обобщённый импульс; лат. petere), протон (англ. proton), дипольный момент, волновой параметр, давление, число полюсов, плотность.
Q{\displaystyle Q}
Электрический заряд (англ. quantity of electricity), количество теплоты (англ. quantity of heat), объёмный расход, обобщённая сила, хладопроизводительность, энергия излучения, световая энергия, добротность (англ. quality factor), нулевой инвариант Аббе, квадрупольный электрический момент (англ. quadrupole moment), энергия ядерной реакции
q{\displaystyle q}
Электрический заряд, обобщённая координата, количество теплоты (англ. quantity of heat), эффективный заряд, добротность
R{\displaystyle R}
Электрическое сопротивление (англ. resistance), универсальная газовая постоянная, постоянная Ридберга (англ. R ydberg constant), постоянная фон Клитцинга, коэффициент отражения, сопротивление излучения (англ. resistance), разрешение (англ. resolution), светимость, пробег частицы, расстояние
r{\displaystyle r}
Радиус (лат. radius), радиус-вектор, радиальная полярная координата, удельная теплота фазового перехода, удельная рефракция (лат. rēfractiō), расстояние
S{\displaystyle S}
Площадь поверхности (англ. surface area), энтропия[4], действие, спин (англ. spin), спиновое квантовое число (англ. spin quantum number), странность (англ. strangeness), главная функция Гамильтона, матрица рассеяния (англ. scattering matrix), опера
ru.wikipedia.org
Нанесение размеров на чертежах ✏️ как правильно обозначать длину, ширину, толщину, высоту, виды размеров, проставление по ГОСТу, допуски и посадки
Как правило, проекты составляют целые конструкторские бюро, после этого чертежи переходят на сборочные участки для изготовления. Чтобы не было расхождений в их чтении, есть специальные стандарты, называемые ГОСТами. Они дают чёткие рекомендации, как верно проставлять размеры и какими условными знаками можно обозначить те или иные элементы.
Основные величины
Существуют несколько геометрических параметров, которые характеризуют любой объект. Это:
длина;
ширина;
высота;
глубина;
межцентровое и межосевое расстояние;
площадь и т. д.
Данные характеристики могут быть как физическими, так и математическими. Единое буквенное обозначение, которое употребляется на всей планете, появилось в середине ХХ столетия и вошло в Международную систему единиц (СИ). За основу взяты латинские буквы, таким образом начертание кириллицей при проектировании не допускается.
В конструкторских документах пишутся в основном символы, применяемые в физике или геометрии.
Существуют двухмерные и трёхмерные изображения. На плоскости присутствуют два измерения, для ширины обозначение буквой В было взято из геометрии. Она измеряется в поперечном направлении. При очерчивании фигур чаще всего пользуются латинским алфавитом: а, b, с. Длина измеряется в продольном разрезе. Это численная характеристика протяжённости линий. В английском языке она звучит как length. Собственно благодаря этому изначально применяемая буква L была взята за основу и внесена в ГОСТ. Стандарт разрешает как заглавное, так и строчное начертание.
Длину и ширину в международной системе измеряют в метрах или других производных от него кратных 10 единицах. Всем известны сантиметры, миллиметры, микроны и др.
Если работа с построением идёт в трёхмерном пространстве, то добавляется ещё и высотный параметр H, в отдельных случаях ещё и толщина. Эта величина характеризует величину объекта по вертикали. Обозначение толщины — буква S. А при работе с круглыми и сферическими объектами появляется такое понятие, как радиус: это отрезок, соединяющий соединяет центр со второй точкой, расположенной на окружности. В международной практике его принято обозначать как R или r, от латинского слова radius. Нередко применяется понятие диаметра. Это отрезок, проходящий через центр и соединяющий две точки на окружности.
Угловые величины принято обозначать греческими буквами.
Цифровые значения на чертёжных документах наносятся над размерными линиями заканчивающихся с двух сторон стрелками. Выносные линии показывают, к какому именно элементу относится то или иное число. Размеры стрелок подбираются в зависимости от толщины основных линий контура и прорисовываются примерно одинаковыми. На рисунке приведены ГОСТированные параметры стрелок.
Все надписи на чертежах должны выполняться чертёжным шрифтом, при начертании которого нужно следовать стандарту, высота букв тоже строго регламентирована и выбирается из ряда. За размер шрифта принимается величина заглавной буквы в миллиметрах.
Унификация и стандартизация
Для облегчения чтения чертежей в производственном процессе существуют специальные ГОСТы (государственные стандарты). Они объединены в свод правил, который именуется как ЕДИНАЯ СИСТЕМА КОНСТРУКТОРСКОЙ ДОКУМЕНТАЦИИ (ЕСКД).
ГОСТ 2.321−84 устанавливает буквенные обозначения, которые принято использовать в конструкторских документах и сборочных чертежах, применяемых различными промышленными отраслями. Прописными буквами наносят габариты изделий или деталей и суммарные размеры.
При обозначении на одном чертеже одинаковой литерой различных величин допускается применение индексов или их комбинаций. Пример обозначения: R, R1, R2, Dn, Dn1, Dn2.
Вспомогательные знаки
Зачастую для упрощения нанесения размеров используются вспомогательные знаки. Например, деталь может иметь резьбовые или сквозные отверстия, зенковку, технологические уклоны, фаски, скругления и прочие элементы.
Перед размерным числом, которое определяет конусность, ставится знак «< «, при этом острый угол направляется в сторону вершины конуса.
При простановке размеров квадратных элементов деталей перед числовым значением ставится значок квадрата. Пример наглядно показан на рисунке.
Симметричные части деталей, например, шестигранники, изображаются до оси симметрии либо показываются не до конца, а чертёж заканчивается обрывистой линией, причём размерную линию следует перерывать после оси симметрии или линии обрыва.
Для деталей, имеющих скошенный или закруглённый конец, на чертежах принято указывать фаску или скругление. Они нужны как для придания эстетичности изделию или детали, так и для некоторых функциональных решений, например, для облегчения сборки механизмов, то есть делают их более технологичными.
Обозначение фаски на чертежах можно выполнить различными способами в зависимости от масштаба, а также углов скоса и их количества. Важнейший критерий — это удобство чтения. При изготовлении не должно возникать излишних вопросов и сомнений. На чертеже обязательно ставятся два значения: величина угла относительно оси детали и ширина скоса. Наиболее часто встречающиеся фаски располагаются под углом 45°. Зачастую фаски обозначаются двумя линейными размерами, каждый из них имеет отметку о величине среза в различных плоскостях.
В некоторых случаях элементы с равными размерами указаны цифрами (1, 2…9 и т. д. ) в технических требованиях к чертежу, тогда на поле самого чертежа можно проставлять только номер этой ссылки. Такая простановка избавляет от проставления размера каждый раз.
Все эти тонкости необходимы для более истинного представления детали и точности её изготовления.
Упрощённые условные обозначения
Указания допусков формы и расположения поверхностей на чертежах выполняют при помощи значков. Термины и определения регламентируются ГОСТом 24642−81.
Указываются базы значком в виде равностороннего зачернённого треугольника, соединённого с рамкой выносной линией. Его высота примерно соответствует шрифту размерных чисел. Условные знаки вписывают в прямоугольник и добавляют числовое ограничение, за пределы которого не должен выходить требуемый допуск формы. Соединительная линия бывает прямой или с изломами, но направление отрезка со стрелкой, должно соответствовать направлению, в котором измеряется отклонение.
Бывают следующие допуски форм и расположения поверхностей:
плоскостность;
цилиндричность;
круглость;
соосность;
параллельность;
перпендикулярность;
симметричность;
допуск радиального, торцового биения;
допуск пересечения.
Каждый имеет свой условный значок. Например, плоскостность обозначается следующим образом, а симметричность — вот так. Двумя параллельными прямыми представлен допуск параллельности.
На рисунке показан пример того, как надо выносить такие параметры.
.
Для упрощения чертежа в технических требованиях иногда даётся ссылка на тот или иной документ.
Пример записи: Неуказанные допуски формы и расположения по ГОСТ 25069–81 .
Правила простановки допусков
В паре сопрягающихся деталях различают поверхности: охватывающую (отверстие) и охватываемую (вал).
Существует условное деление по вариантам соединений. При гладком цилиндрическом охватывающие детали сопряжения круглые и имеют форму цилиндра. Другой вид: плоское с параллельными плоскостями. Здесь соединительные элементы расположены в параллельных по отношению друг к другу плоскостях. В первом случае под размером подразумевается диаметр, во второй вариации за размер берётся расстояние между параллельными поверхностями.
Существует такое понятие, как номинальный размер. Он выбирается исходя из того, какую функцию должна выполнять деталь и служит начальной точкой отсчёта отклонений.
Действительный размер после измерения может иметь допустимую погрешность и должен находиться в интервале между максимальным и минимальным размерами, которые являются двумя предельными значениями.
При разработке следует помнить, что неизменно имеется погрешность в точности изготовления. Существующее небольшое отклонение составляет разность между самим размером в действительности и его номинальным значением.
Бывает верхнее и нижнее предельные отклонения. Разность между наибольшим и наименьшим пределами считается допуском.
В зависимости от простановки допусков соединения деталей бывают трёх типов:
с зазорами;
с натягами;
переходные.
Посадка сопрягаемых деталей с зазором позволяет более свободное относительное перемещение, натяг ограничивает эту свободу. В случае когда посадка рассчитана с зазором, размер охватывающего элемента, а именно отверстия больше охватываемого, то есть вала, и наоборот: при натяге количественные параметры вала преобладают над аналогичными в отверстиях. Переходные посадки предполагают получение как натягов, так и зазоров.
Величины допусков отверстий и валов образуют ряды и группируются по классам точности или квалитетам.
Поля допусков основных отверстий и валов обозначают буквами А и В с числовым индексом класса точности. Обозначения других полей устанавливают в стандартах на допуски и посадки и прописаны в сводных таблицах.
При невыполнимости контроля допусков используются справочные размеры. Они помечаются звёздочкой, а в технических требованиях указывается ссылка на то, что размер приведён для справок. К ним относятся:
величины деталей из листового материала и определяемые толщиной исходного листа;
один из размеров замкнутой размерной цепи;
данные с изделий-заготовок;
размеры на сборочном чертеже и др.
Отклонения размеров нужно вписывать после номинальных величин. Если не требуется особая точность изготовления, то для упрощения допуски можно не указывать на поле чертежа, достаточно сделать запись в технических требованиях чертежа с указанием квалитета: неуказанные предельные отклонения размеров: Н 14, h 14.
nauka.club
Какими англ. буквами обозначаются длина,ширина и высота?
L — length (длина) , W — width (ширина) , H -hight (высота).
L — length, W — width, H -hight .
L-длина, W-ширина, Н-высота
L — length, W — width, H -hight .
touch.otvet.mail.ru
Что такое определение объема в науке?
Объем — это объем трехмерного пространства, занимаемого жидкостью, твердым телом или газом. Общие единицы измерения объема включают литры, кубические метры, галлоны, миллилитры, чайные ложки и унции, хотя существует множество других единиц.
Ключевые выводы: определение объема
Объем — это трехмерное пространство, занятое веществом или ограниченное поверхностью.
В Международной системе единиц (СИ) стандартной единицей объема является кубический метр (м 3 ).
В метрической системе в качестве единицы объема используется литр (L). Один литр равен объему 10-сантиметрового куба.
Примеры томов
В качестве примера объема студент может использовать мерный цилиндр для измерения объема химического раствора в миллилитрах.
Можно купить литр молока.
Газы обычно продаются в единицах объема, таких как кубические сантиметры, см. 3 или кубические литры.
Измерение объема жидкостей, твердых веществ и газов
Поскольку газы заполняют их контейнеры, их объем такой же, как внутренний объем контейнера.Жидкости обычно измеряются с помощью контейнеров, где указывается объем или внутренняя форма контейнера. Примеры инструментов, используемых для измерения объема жидкости, включают мерные чашки, градуированные цилиндры, колбы и химические стаканы. Есть формулы для расчета объема правильных твердых форм. Другой метод определения объема твердого тела — это измерение количества вытесняемой им жидкости.
Объем относительно массы
Объем — это объем пространства, занимаемый веществом, а масса — это количество вещества, которое оно содержит.Количество массы на единицу объема — это плотность образца.
Вместимость по отношению к объему
Вместимость — это мера содержимого емкости, в которой содержатся жидкости, зерна или другие материалы, принимающие форму емкости. Емкость не обязательно совпадает с объемом. Это всегда внутренний объем сосуда. Единицы вместимости включают литр, пинту и галлон, а единица объема (СИ) является производной от единицы длины.
Объем цилиндров — объяснение и примеры
Объем цилиндра — это мера пространства, занимаемого цилиндром, или мера вместимости цилиндра.
Эта статья покажет вам, как найти объем цилиндра, используя формулу объема цилиндра.
В геометрии цилиндр представляет собой трехмерную форму с двумя равными и параллельными окружностями, соединенными изогнутой поверхностью.
Расстояние между круговыми гранями цилиндра называется высотой цилиндра . Верх и низ цилиндра представляют собой две совпадающие окружности, радиус или диаметр которых обозначены как « r » и « d » соответственно.
Как найти объем цилиндра?
Чтобы вычислить объем цилиндра, вам нужен радиус или диаметр круглого основания или вершины и высота цилиндра.
Объем цилиндра равен произведению площади круглого основания и высоты цилиндра. Объем цилиндра измеряется в кубических единицах.
Расчет объема цилиндра полезен при проектировании цилиндрических объектов, таких как:
Цилиндрические резервуары для воды или колодцы
Отводы
Флаконы для парфюмерии или химикатов
Цилиндрические контейнеры и трубы
Цилиндрические колбы химические лаборатории
Формула объема цилиндра
Формула для объема цилиндра имеет следующий вид:
Объем цилиндра = πr 2 ч кубических единиц
Где πr 2 = площадь круга ;
π = 3.14;
r = радиус круглого основания и;
h = высота цилиндра.
Для полого цилиндра формула объема имеет следующий вид:
Объем цилиндра = πh (r 1 2 — r 2 2 )
Где r 1 = внешний радиус и r 2 = внутренний радиус цилиндра.
Разница внешнего и внутреннего радиуса образует толщину стенки цилиндра, т.е.
Толщина стенки цилиндра = r 1 — r 2
Давайте решим несколько примеров задач об объеме цилиндров.
Пример 1
Диаметр и высота цилиндра составляют 28 см и 10 см соответственно. Какой объем цилиндра?
Решение
Дано;
Радиус равен половине диаметра.
Диаметр = 28 см ⇒ радиус = 28/2
= 14 см
Высота = 10 см
По формуле объема цилиндра;
объем = πr 2 ч
= 3,14 x 14 x 14 x 10
= 6154.4 см 3
Итак, объем цилиндра равен 6154,4 см 3
Пример 2
Глубина воды в цилиндрическом резервуаре составляет 8 футов. Предположим, что радиус и высота резервуара составляют 5 футов и 11,5 футов соответственно. Найдите объем воды, необходимый для наполнения бака до краев.
Решение
Сначала рассчитайте объем цилиндрического резервуара
Объем = 3,14 x 5 x 5 x 11,5
= 902.75 кубических футов
Объем воды в резервуаре = 3,14 x 5 x 5 x 8
= 628 кубических футов.
Объем воды, необходимый для заполнения бака = 902,75 — 628 кубических футов
= 274,75 кубических футов.
Пример 3
Объем цилиндра 440 м 3 , радиус основания 2 м. Рассчитайте высоту резервуара.
Раствор
Объем цилиндра = πr 2 ч
440 м 3 = 3.14 x 2 x 2 x h
440 = 12,56h
Разделив 12,56 на обе стороны, получим
h = 35
Таким образом, высота резервуара будет 35 метров.
Пример 4
Радиус и высота цилиндрического резервуара для воды составляют 10 см и 14 см соответственно. Найдите объем бака в литрах.
Следовательно, разделите 4396 на 1000, чтобы получить
Объем = 4.396 литров
Пример 5
Внешний радиус пластиковой трубы составляет 240 мм, а внутренний радиус — 200 мм. Если длина трубы составляет 100 мм, найдите объем материала, из которого изготовлена труба.
Решение
Труба является примером полого цилиндра, поэтому мы имеем
Объем цилиндра = πh (r 1 2 — r 2 2 )
= 3,14 x 100 x (240 2 -200 2 )
= 3.14 x 100 x 17600
= 5,5264 x 10 6 мм 3 .
Пример 6
Цилиндрический твердый блок металла должен быть расплавлен с образованием кубов с ребром 20 мм. Предположим, что радиус и длина цилиндрического блока равны 100 мм и 490 мм соответственно. Найдите количество кубиков, которые нужно сформировать.
Решение
Рассчитайте объем цилиндрического блока
объем = 3,14 x 100 x 100 x 490
= 1.5386 x 10 7 мм 3
Объем куба = 20 x 20 x 20
= 8000 мм 3
Количество кубиков = объем цилиндрического блока / объем куба
= 1,5386 x 10 7 мм 3 /8000 мм 3
= 1923 куба.
Пример 7
Найдите радиус цилиндра такой же высоты и объема, как у куба со сторонами 4 фута
Решение
Дано:
Высота куба = высота цилиндра = 4 футов и,
объем куба = объем цилиндра
4 x 4 x 4 = 64 кубических фута
Но объем цилиндра = πr 2 ч
3.14 x r 2 x 4 = 64 кубических фута
12,56r 2 = 64
Разделите обе стороны на 12,56
r 2 = 5,1 футов.
r = 1,72
Следовательно, радиус цилиндра будет 1,72 фута.
Пример 8
Сплошная шестиугольная призма имеет длину основания 5 см и высоту 12 см. Найдите высоту цилиндра того же объема, что и призма. Примем радиус цилиндра 5 см.
Раствор
Формула объема призмы имеет вид;
Объем призмы = (h) (n) (s 2 ) / [4 tan (180 / n)]
где n = количество сторон
s = базовая длина призмы
h = высота призмы
Объем = (12) (6) (5 2 ) / (4tan 180/6)
= 1800/2.3094
= 779,42 см 3
Объем цилиндра = πr 2 h
779,42 = 3,14 x 5 x 5 x h
h = 9,93 см.
Итак, высота цилиндра будет 9,93 см.
Практические вопросы
Если объем и радиус цилиндрического контейнера для краски составляют 640π кубических см и 8 см, соответственно, какова его высота?
Рассмотрим цилиндрический резервуар, высота которого в два раза больше его радиуса. Если объем резервуара составляет 4580 единиц, каков радиус резервуара?
Ответы
10 см
9 шт.
Предыдущий урок | Главная страница | Следующий урок
Объем конуса | Блестящая вики по математике и науке
Доказательство этой формулы может быть подтверждено объемом вращения.2 \ Delta yVdisk = πR2Δy.
Нам нужно иметь RRR в единицах yyy, поэтому мы должны найти связь между RRR и yyy, то есть найти R (y) R (y) R (y).
Как видно из рисунка, RRR является линейной функцией yyy, поэтому R (y) = my + bR (y) = my + bR (y) = my + b. Мы знаем, что R (0) = rR (0) = rR (0) = r и R (h) = 0R (h) = 0R (h) = 0. Таким образом, m = ΔRΔy = r − 00 − h = −rhm = \ dfrac {\ Delta R} {\ Delta y} = \ dfrac {r-0} {0-h} = — \ dfrac {r} {h} m = ΔyΔR = 0 − hr − 0 = −hr. Тогда функция R (y) = — rhy + rR (y) = — \ dfrac {r} {h} y + rR (y) = — hr y + r.2ч. \ _ \ Квадрат
\ end {align} V = π∫0h R2 (y) dy = π∫0h (−hr y + r) 2dy = π∫0h (h3r2 y2 − h3r2 y + r2) dy = π (3h3r2 y3 − hr2 y2 + r2y) ∣∣∣∣ 0h = 31 πr2h. □
Мы можем обобщить понятие конуса так, чтобы любая простая замкнутая кривая, круглая или нет, могла быть основанием конуса. Таким образом, пирамиды тоже являются конусами. Мы можем дополнительно либерализовать определение, чтобы кончик конуса не обязательно находился прямо над центром его основания; то есть, конус может быть наклонным вместо правого .
Конусы одинаковой высоты, основания которых имеют одинаковую площадь, также имеют одинаковый объем, потому что их срезы в поперечном сечении имеют одинаковую площадь на каждой высоте (где высота означает расстояние от плоскости основания; это применение принципа Кавальери. ). Таким образом, мы можем вывести формулу для объема конуса любой формы основания, если мы можем сделать это для некоторой одной формы основания. А в случае с квадратом это сделать несложно.
Шесть пирамид высотой hhh, основания которых представляют собой квадраты длиной 2h3h3h, могут быть собраны в куб со стороной 2h3h3h.3} {6} 68h4 или 13Ah \ frac {1} {3} Ah41 Ah, где AAA — это площадь его базы. Масштабирование любого объекта в одном измерении линейно влияет на его объем, поэтому квадратная пирамида любой высоты HHH с площадью основания AAA имеет объем 13Ah \ frac {1} {3} Ah41 Ah, умноженный на коэффициент масштабирования Hh \ frac HhhH, дает 13AH \ frac {1} {3} Ah41 AH.
Отсюда следует, что конус любой формы имеет объем, в три раза превышающий площадь его основания, умноженную на его высоту. □ _ \ квадрат □
Общие математические символы и терминология
Математические символы и терминология могут сбивать с толку и мешать изучению и пониманию основ математики.
Эта страница дополняет наши страницы, посвященные навыкам счета, и предоставляет краткий глоссарий общих математических символов и терминологии с краткими определениями.
Мы что-то упускаем? Дайте нам знать.
Общие математические символы
+ сложение, плюс, положительное
Символ сложения + обычно используется для обозначения того, что два или более числа должны быть сложены вместе, например, 2 + 2.
Символ + также может использоваться для обозначения положительного числа, хотя он встречается реже, например, +2.На нашей странице о положительных и отрицательных числах объясняется, что число без знака считается положительным, поэтому плюс обычно не требуется.
См. Дополнительную информацию на нашей странице Дополнение .
— вычитание, минус, отрицательный
Этот символ имеет два основных применения в математике:
— используется, когда нужно вычесть одно или несколько чисел, например, 2 — 2.
Символ — также обычно используется для обозначения отрицательного или отрицательного числа, например −2.
См. Дополнительную информацию на нашей странице Вычитание .
× или * или. Умножение
Эти символы имеют то же значение; обычно × используется для обозначения умножения, когда написано от руки или используется на калькуляторе, например, 2 × 2.
Символ * используется в электронных таблицах и других компьютерных приложениях для обозначения умножения, хотя * имеет другие, более сложные значения в математике.
Реже умножение также может быть обозначено точкой. или вообще без символа. Например, если вы видите число, написанное вне скобок без оператора (символа или знака), то его следует умножить на содержимое скобок: 2 (3 + 2) то же самое, что 2 × (3 + 2).
См. Дополнительную информацию на нашей странице Умножение .
÷ или / Подразделение
Оба эти символа используются для обозначения деления в математике.÷ обычно используется в рукописных вычислениях и на калькуляторах, например, 2 ÷ 2.
/ используется в электронных таблицах и других компьютерных приложениях.
См. Дополнительную информацию на нашей странице в разделе Division .
= равно
Символ = равно используется, чтобы показать, что значения по обе стороны от него одинаковы. Чаще всего используется для отображения результата вычисления, например 2 + 2 = 4, или в уравнениях, например 2 + 3 = 10-5.
Вы также можете встретить другие похожие символы, хотя они встречаются реже:
≠ означает не равно. Например, 2 + 2 ≠ 5 — 2. В компьютерных приложениях (например, Excel) символы <> означают не равно.
≡ означает идентично. Это похоже на, но не совсем то же самое, что на равно. Поэтому, если сомневаетесь, придерживайтесь =.
≈ означает примерно равно или почти равно.Две стороны отношения, обозначенные этим символом, будут , а не достаточно точными для математических манипуляций.
<Меньше и> Больше
Этот символ < означает меньше, например 2 <4 означает, что 2 меньше 4.
Этот символ > означает больше, например, 4> 2.
≤ ≥ Эти символы означают «меньше или равно» и «больше или равно» и обычно используются в алгебре.В компьютерных приложениях используются <= и> =.
≪ ≫ Эти символы встречаются реже и означают намного меньше или намного больше.
± плюс или минус
Этот символ ± означает «плюс» или «минус». Он используется, например, для обозначения доверительных интервалов вокруг числа.
Ответом считается «плюс-минус» другое число, или, другими словами, в пределах диапазона данного ответа.
Например, 5 ± 2 на практике может быть любым числом от 3 до 7.
∑ Сумма
Символ ∑ означает сумму.
∑ — заглавная греческая буква сигма. Он обычно используется в алгебраических функциях, и вы также можете заметить его в Excel — кнопка Автосумма имеет сигму в качестве значка.
° Степень
Градусы ° используются по-разному.
Как мера поворота — угол между сторонами фигуры или поворот круга.Круг равен 360 °, а прямой угол — 90 °. См. Наш раздел о Geometry для получения дополнительной информации.
Мера температуры. градуса Цельсия или Цельсия используются в большинстве стран мира (за исключением США). Вода замерзает при 0 ° C и закипает при 100 ° C. В США используется градус Фаренгейта. По шкале Фаренгейта вода замерзает при 32 ° F и закипает при 212 ° F. Смотрите нашу страницу: Системы измерения для получения дополнительной информации.
∠ Угол
Символ угла ∠ используется как сокращение в геометрии (изучении форм) для описания угла.
Выражение ∠ABC используется для описания угла в точке B (между точками A и C). Точно так же ∠BAC может использоваться для описания угла точки A (между точками B и C). Подробнее об углах и других геометрических терминах см. На наших страницах Геометрия .
√ Квадратный корень
√ — символ квадратного корня. Квадратный корень — это число, которое при умножении на себя дает исходное число.
Например, квадратный корень из 4 равен 2, потому что 2 x 2 = 4.Квадратный корень из 9 равен 3, потому что 3 x 3 = 9.
См. Нашу страницу: Специальные числа и понятия для получения дополнительной информации о квадратных корнях.
n Мощность
Целое число с верхним индексом (любое целое число n ) — это символ, используемый для обозначения степени числа.
Например, 3 2 означает 3 в степени 2, что совпадает с 3 в квадрате (3 x 3).
4 3 означает 4 в степени 3 или 4 в кубе, то есть 4 × 4 × 4.
См. Наши страницы Расчетная площадь и Расчет объема , где приведены примеры использования чисел в квадрате и кубе .
Степень также используется как сокращенный способ записи больших и малых чисел.
. — символ десятичной точки, часто называемый просто «точкой». См. Нашу страницу Decimals для примеров его использования.
, Разделитель тысяч
Запятую можно использовать для разделения больших чисел и облегчения их чтения.
Тысячу можно записать как 1000, так и 1000, а миллион — как 1000000 или 1000000.Запятая разделяет большие числа на блоки по три цифры.
В большинстве англоязычных стран, не имеет математической функции, он просто используется для облегчения чтения чисел.
В некоторых других странах, особенно в Европе, запятая может использоваться вместо десятичной точки, и, действительно, десятичная точка может использоваться вместо запятой в качестве визуального разделителя. Это объясняется более подробно на нашей странице Introduction to Numbers .
[], () Скобки, круглые скобки
Скобки () используются для определения порядка вычислений в соответствии с правилом BODMAS.
Части расчета, заключенные в скобки, вычисляются первыми, например
5 + 3 × 2 = 11
(5 + 3) × 2 = 16
% В процентах
Символ% означает процент или число из 100.
Узнайте все о процентах на нашей странице: Введение в проценты
π Pi
π или пи — греческий символ звука «п».Это часто встречается в математике и является математической константой. Пи — это длина окружности круга, деленная на ее диаметр, и имеет значение 3,141592653. Это иррациональное число, что означает, что его десятичные разряды продолжаются до бесконечности.
∞ Бесконечность
Символ ∞ означает бесконечность, концепцию, согласно которой числа существуют вечно.
Каким бы большим у вас ни было число, у вас всегда может быть номер побольше, потому что вы всегда можете добавить к нему еще один.
Бесконечность — это не число, а идея чисел, существующая вечно. Вы не можете прибавить единицу к бесконечности, как нельзя прибавить единицу к человеку, полюбить или ненавидеть.
\ (\ bar x \) (x-bar) Среднее значение
\ (\ bar x \) — среднее всех возможных значений x.
Чаще всего этот символ встречается в статистике.
См. Нашу страницу Средние значения для получения дополнительной информации.
! Факториал
! это символ факториала.
н! — произведение (умножение) всех чисел от n до 1 включительно, т.е. n × (n − 1) × (n − 2) ×… × 2 × 1.
Труба ‘|’ также называется вертикальной чертой, vbar, pike и имеет множество применений в математике, физике и вычислениях.
Чаще всего в базовой математике он используется для обозначения абсолютного значения или модуля действительного числа, где \ (\ vert x \ vert \) — это абсолютное значение или модуль \ (x \) .
Математически это определяется как
$$ \ vert x \ vert = \ biggl \ {\ begin {eqnarray} -x, x \ lt 0 \\ x, x \ ge 0 \ end {eqnarray} $$
Проще говоря, \ (\ vert x \ vert \) — неотрицательное значение \ (x \). Например, модуль 6 равен 6, а модуль −6 также равен 6.
Он также используется в вероятности, где P (Z | Y) обозначает вероятность X с учетом Y.
∝ Пропорциональный
∝ означает «пропорционально » и используется, чтобы показать что-то, что меняется по отношению к чему-то другому.
Например, если x = 2y, то x ∝ y.
∴ Следовательно
∴ — удобная сокращенная форма слова «поэтому», используемая в математике и естественных науках.
∵ Потому что
∵ — удобная сокращенная форма слова «потому что», не путать с «поэтому».
Математическая терминология (A-Z)
Амплитуда
Когда объект или точка движется циклически, или подвергается вибрации или колебаниям (например,грамм. маятник), амплитуда — это максимальное расстояние, на которое он перемещается от своей центральной точки. См. Введение в геометрию для получения дополнительной информации.
Апофема
Линия, соединяющая центр правильного многоугольника с одной из его сторон. Линия перпендикулярна (под прямым углом) в сторону.
Площадь
Геометрическая площадь определяется как пространство, занимаемое плоской формой или поверхностью объекта. Площадь измеряется в квадратных единицах, например в квадратных метрах ( м 2 ).Для получения дополнительной информации см. Нашу страницу о площади , площади поверхности и объеме .
Асимптота
Асимптота — это прямая линия или ось, которая конкретно связана с изогнутой линией. По мере того, как кривая линия расширяется (стремится) к бесконечности, она приближается к своей асимптоте (то есть расстояние между кривой и асимптотой стремится к нулю, но никогда не касается ее). Это происходит в геометрии и тригонометрии .
Ось
Контрольная линия, вокруг которой нарисован, повернут или измерен объект, точка или линия.В симметричной форме ось обычно представляет собой линию симметрии.
Коэффициент
Коэффициент — это число или величина, умножающая другую величину. Обычно его ставят перед переменной . В выражении 6 x 6 — коэффициент, а x — переменная.
Окружность
Окружность — это длина расстояния по краю круга. Это тип периметра , который уникален для круглых форм.Для получения дополнительной информации см. Нашу страницу о изогнутых формах .
Данные
Данные представляют собой набор значений, информации или характеристик, которые часто имеют числовой характер. Они могут быть собраны с помощью научного эксперимента или других средств наблюдения. Это могут быть количественных или качественных переменных. Датум — это отдельное значение одной переменной. См. Нашу страницу Типы данных для получения дополнительной информации.
Диаметр
Диаметр — это термин, используемый в геометрии для определения прямой линии, которая проходит через центр круга или сферы, касаясь окружности или поверхности с обоих концов.Диаметр в два раза больше радиуса .
Экстраполировать
Экстраполяция — это термин, используемый при анализе данных. Это относится к расширению графика, кривой или диапазона значений в диапазон, для которого не существует данных, с выводом значений неизвестных данных из тенденций в известных данных.
Фактор
Коэффициент — это число, которое мы умножаем на другое число. Фактор делится на другое число целое число раз. У большинства чисел есть четное число факторов.Квадратное число имеет нечетное количество множителей. Простое число имеет два множителя — само себя и 1. Простой множитель — множитель, который является простым числом. Например, простые множители 21 равны 3 и 7 (потому что 3 × 7 = 21, а 3 и 7 — простые числа).
Среднее значение, медиана и мода
Среднее значение (среднее значение) набора данных вычисляется путем сложения всех чисел в наборе данных и последующего деления на количество значений в наборе.Когда набор данных упорядочен от наименьшего к наибольшему, среднее значение является средним значением. Режим — это число, которое встречается чаще всего.
Эксплуатация
Математическая операция — это шаг или этап в вычислении, или математическое «действие». Основные арифметические операции — сложение, вычитание, умножение и деление. Порядок, в котором выполняются операции при вычислении, важен. Порядок операций известен как BODMAS .
Математические операции часто называют «суммами». Строго говоря, «сумма» — это операция сложения. В SYN мы имеем в виду операции и вычисления, но в повседневной речи часто можно услышать общий термин «суммы», который неверен.
Периметр
Периметр двумерной фигуры — это непрерывная линия (или длина линии), определяющая контур фигуры. Периметр круглой формы называется ее окружностью .Наша страница по периметру объясняет это более подробно.
Доля
Пропорция — это относительное отношение. Соотношения сравнивают одну часть с другой, а пропорции сравнивают одну часть с целым. Например, «3 из 10 взрослых в Англии имеют избыточный вес». Пропорция относится к дробям .
Пифагор
Пифагор был греческим философом, которому приписывают ряд важных математических и научных открытий, возможно, наиболее значительное из которых стало известно как Теорема Пифагора .
Это важное правило применяется только к прямоугольным треугольникам. В нем говорится, что «квадрат гипотенузы равен сумме квадратов на двух других сторонах».
Количественный и качественный
Количественные данные — это числовые переменные или значения, которые могут быть выражены численно, то есть сколько, сколько, как часто, и получаются путем подсчета или измерения.
Качественные данные — это переменные типа, которые не имеют числового значения и могут быть выражены описательно, т.е.е. с использованием имени или символа и получаются путем наблюдения.
Подробнее см. Нашу страницу о типах данных .
Радиан
Радиан — это единица измерения угла в системе СИ. Один радиан эквивалентен углу, образуемому в центре окружности дугой, равной по длине радиусу. Один радиан чуть меньше 57,3 градуса. Полный оборот (360 градусов) составляет 2π радиан.
Радиус
Термин радиус используется в контексте кругов и других изогнутых форм.Это расстояние от центральной точки круга, сферы или дуги до его внешнего края, поверхности или окружности . Диаметр вдвое больше радиуса. Для получения дополнительной информации см. Нашу страницу о изогнутых формах .
Диапазон
В статистике диапазон данного набора данных — это разница между наибольшим и наименьшим значениями.
Коэффициент
Соотношение — это математический термин, используемый для сравнения размеров одной части с другой.Соотношения обычно отображаются в виде двух или более чисел, разделенных двоеточием, например, 7: 5, 1: 8 или 5: 2: 1.
Стандартное отклонение
Стандартное отклонение набора данных измеряет, насколько данные отличаются от среднего значения, то есть это мера вариации или разброса набора значений. Если разброс данных невелик и все значения близки к среднему, стандартное отклонение будет низким. Высокое стандартное отклонение указывает на то, что данные разбросаны по более широкому диапазону
Срок
Термин — это отдельное математическое выражение.Это может быть одно число, одна переменная (например, x ) или несколько констант и переменных, умноженных вместе (например, 3 x 2). Термины обычно разделяются операциями сложения или вычитания. Термин может включать операции сложения или вычитания, но только в скобках, например 3 (2 -x3).
Переменная
Переменная — это коэффициент в математическом выражении, арифметическом соотношении или научном эксперименте, который может изменяться.В эксперименте обычно используются три типа переменных: независимые, зависимые и контролируемые. В выражении 6 x , 6 — это коэффициент , а x — переменная.
Разница
Дисперсия — это статистическое измерение, которое указывает разброс между элементами в наборе данных. Он измеряет, насколько далеко каждый член в наборе находится от среднего и, следовательно, от каждого другого члена в наборе.
Вектор
Векторы описывают математические величины, которые имеют как величину, так и направление.Векторы встречаются во многих математических и физических приложениях, например. изучение движения, где скорость, ускорение, сила, смещение и импульс являются векторными величинами.
Объем
Объем — это трехмерное пространство, занимаемое твердой или полой формой. Он измеряется кубическими размерами пространства, ограниченного его поверхностями. Объем измеряется в кубических единицах, например м 3 .
Дополнительная литература по навыкам, которые вам нужны
Навыки, которые вам нужны. Руководство по счету
Это руководство из четырех частей познакомит вас с основами математики от арифметики до алгебры с остановками на дробях, десятичных дробях, геометрии и статистике.
Если вы хотите освежить в памяти основы или помочь своим детям в учебе, эта книга для вас.
Измерение — Объем куба, кубоида и цилиндра | Класс 8 Математика
Измерение — раздел математики, изучающий различные геометрические формы, их площади и объем. Он использует геометрические вычисления и алгебраические уравнения для вычисления измерений различных аспектов объектов, таких как площадь поверхности, объем и т. Д.
Объем — это объем пространства внутри трехмерного объекта, которое можно заполнить. Это может быть измерение любых твердых объектов, таких как куб, квадрат, цилиндр, сфера, пирамида и т. Д. Объем объекта обычно измеряется в кубических единицах. Выявление объема объекта также помогает нам определить количество, необходимое для заполнения объекта.
Единица измерения объема S.I — m 3 . Остальные единицы — мл / л.
1 м 3 = 1 литр
Вычисление объемов трехмерных объектов
До сих пор вы, должно быть, поняли значение объема, давайте перейдем к определению объема различных трехмерных геометрических фигур, таких как куб, кубоид и цилиндр.
Определение объема кубоида
Кубоид — это трехмерная структура, имеющая шесть прямоугольных граней. Объем кубоида равен произведению длины, ширины и высоты кубоида. В прямоугольном кубоиде все углы расположены под прямым углом, а противоположные грани кубоида равны.
Пусть l — длина, b — ширина и h — высота кубоида.
Пример 1: Вычислите объем кубоида, длина, ширина и высота которого составляют 12, 8 и 4 метра соответственно.3
Пример 2: Вычислите ширину кубоида, объем которого равен 350-метровому кубу. А длина и высота — 7 и 5 метров соответственно.
Решение: Заданный объем куба = длина x ширина x высота
350 = 7 x ширина x 5
350/35 = ширина
10 м = ширина
Объем куба
Во-первых, возникает вопрос, что такое куб? Итак, Куб — это трехмерная форма с одинаковыми размерами ширины, высоты и длины.3
7 см = сторона
Объем цилиндра
Цилиндр — это трехмерное твердое тело, которое содержит два параллельных основания, соединенных изогнутой поверхностью. Основания обычно имеют круглую форму. Расстояние по перпендикуляру между основаниями обозначается как высота цилиндра « h », а « r » — радиус цилиндра.
Пример 1: Рассчитайте объем цилиндрической емкости радиусом 4 см и высотой 35 см.3
Пример 2: Вычислить радиус цилиндра, высота которого составляет 21 см, а объем цилиндра равен 1100 сантиметрам куба? (Возьмем pi = 22/7)
Решение: Объем цилиндра = πr 2 ч кубических единиц
1100 = 22/7 xrxrx 7
r = 7,07 см
Введение, типы, формулы, свойства и решенные примеры
Что такое измерение?
Измерение зависит от геометрии.Измерение имеет дело с размером, областью и плотностью различных форм как в 2D, так и в 3D. Теперь, во введении к измерению, давайте подумаем о двухмерных и трехмерных формах и о различиях между ними.
Что такое 2D-форма?
Двумерная диаграмма — это фигура, расположенная на плоскости тремя или более прямыми линиями или замкнутым сегментом. Такие формы не имеют ширины или высоты; они имеют два измерения длины и ширины и поэтому называются двухмерными формами или фигурами. Для двухмерных форм необходимо определить площадь (A) и периметр (P).
Что такое 3D-фигура?
Трехмерная фигура — это структура, окруженная множеством поверхностей или плоскостей. Они также считаются надежными типами. В отличие от 2D-форм, эти формы имеют высоту или глубину; они имеют трехмерную длину, ширину и высоту / глубину и поэтому называются трехмерными фигурами. На самом деле 3D-формы состоят из ряда 2D-фигур. Часто известные как сильные формы, объем (V), площадь криволинейной поверхности (CSA), площадь боковой поверхности (LSA) и полная площадь поверхности (TSA) измеряются для трехмерных форм.
Введение в менструацию: важные термины
Пока мы не перейдем к списку важных формул для измерения, нам необходимо прояснить некоторые важные термины, которые составляют эти формулы для измерения:
Площадь (A):
Область называется поверхность, занятая определенной замкнутой областью. Он обозначается буквой А и выражается в квадрате.
Периметр (P):
Общая длина границы фигуры называется ее периметром.Периметр определяется только двухмерными фигурами или фигурами. Это непрерывная линия по краю закрытого сосуда. Он обозначается буквой P, а размеры измеряются в квадрате.
Объем (В):
Ширина пространства, содержащегося в трехмерной замкнутой форме или поверхности, например, в области комнаты или цилиндра. Объем обозначается буквой V, а единицей измерения объема в системе СИ является кубический метр.
Площадь изогнутой поверхности (CSA):
Площадь изогнутой поверхности — это площадь единственной изогнутой поверхности без учета основания и вершины, например сферы или круга.Аббревиатура площади криволинейной поверхности — CSA.
Площадь боковой поверхности (LSA):
Общая площадь всех боковых поверхностей данной фигуры называется площадью боковой поверхности. Боковые поверхности — это слои, покрывающие артефакт. Аббревиатура площади боковой поверхности — LSA.
Общая площадь поверхности (TSA):
Расчет общей площади всех поверхностей называется кумулятивной областью поверхности в замкнутой форме. Например, мы получаем его общую площадь поверхности в кубе, добавляя площади всех шести поверхностей.Аббревиатура общей площади поверхности — TSA.
Квадратная единица (/):
Одна квадратная единица — это просто квадратная площадь, равная единице. Когда мы количественно определяем некоторую площадь поверхности, мы относимся к сторонам одного квадратного блока, чтобы знать, сколько этих единиц уместится на приведенном рисунке.
Кубическая единица (/):
Одна кубическая единица — это единичный объем, заполненный боковым кубом. Когда мы вычисляем объем любого числа, мы фактически обращаемся к этому кубу с единицей стороны один и к тому, сколько этих компонентных кубов поместится в заданной замкнутой форме.
Список формул измерения для 2D-форм:
Поскольку наше введение в измерение и соответствующие слова закончились, давайте переключимся на уравнения для измерения, поскольку это обсуждение сосредоточено на уравнении. На двухмерном рисунке есть список формул измерения, которые определяют взаимосвязь между различными параметрами. Давайте подробно рассмотрим некоторые виды оценочных уравнений.
Площадь:
Площадь (A) = (сторона) 2 кв. Единицы.
Периметр (P) = (4 × стороны) единиц.{2}} \] ед.
Круг:
Площадь = π x радиус² кв. Единиц.
Окружность = 2π x единицы радиуса.
Диаметр, D = 2 x единицы радиуса.
Список формул измерения для 3D-форм:
3D-фигура содержит список формул для измерения, которые определяют взаимосвязь между различными параметрами. Давайте подробнее рассмотрим некоторые виды оценочных уравнений.
Куб:
Объем = кубические единицы.
Площадь боковой поверхности = 4 × сторона 2 кв.{2}} \] ед.
Кубоид:
Объем = (длина + ширина + высота) кубических единиц.
Площадь боковой поверхности = 2 × высота (длина + ширина) кв. Единиц.
Общая площадь поверхности = 2 (длина × ширина + длина × высота + высота × ширина) кв. Единиц.
Длина диагонали = длина3 + ширина3 + высота2 единицы.
Площадь изогнутой поверхности (исключая области верхней и нижней круглых областей) = (2𝜋Rh) кв. Единиц.
Где, R = радиус
Общая площадь поверхности = (2𝜋Rh + 2𝜋R²) кв. Единиц
Конус:
Объем = \ [\ frac {1} {3} \] 𝜋 x радиус² x кубические единицы высоты.
Площадь изогнутой поверхности = 𝜋 x радиус x высота кв. Единиц.
Общая площадь поверхности = 𝜋 x радиус (длина + высота) кв.{2}} \] кв. единицы измерения.
Используя приведенные выше формулы для измерения, можно решить большинство проблем измерения.
Как найти длину ребра куба
Если вы считаете, что контент, доступный через Веб-сайт (как определено в наших Условиях обслуживания), нарушает
или несколько ваших авторских прав, сообщите нам об этом, отправив письменное уведомление («Уведомление о нарушении»), содержащее
в
информацию, описанную ниже, назначенному ниже агенту.Если репетиторы университета предпримут действия в ответ на
ан
Уведомление о нарушении, он предпримет добросовестную попытку связаться со стороной, которая предоставила такой контент
средствами самого последнего адреса электронной почты, если таковой имеется, предоставленного такой стороной Varsity Tutors.
Ваше Уведомление о нарушении прав может быть отправлено стороне, предоставившей доступ к контенту, или третьим лицам, таким как
в виде
ChillingEffects.org.
Обратите внимание, что вы будете нести ответственность за ущерб (включая расходы и гонорары адвокатам), если вы существенно
искажать информацию о том, что продукт или действие нарушает ваши авторские права.Таким образом, если вы не уверены, что контент находится
на Веб-сайте или по ссылке с него нарушает ваши авторские права, вам следует сначала обратиться к юристу.
Чтобы отправить уведомление, выполните следующие действия:
Вы должны включить следующее:
Физическая или электронная подпись правообладателя или лица, уполномоченного действовать от их имени;
Идентификация авторских прав, которые, как утверждается, были нарушены;
Описание характера и точного местонахождения контента, который, по вашему мнению, нарушает ваши авторские права, в \
достаточно подробностей, чтобы позволить репетиторам университетских школ найти и точно идентифицировать этот контент; например, мы требуем
а
ссылка на конкретный вопрос (а не только на название вопроса), который содержит содержание и описание
к какой конкретной части вопроса — изображению, ссылке, тексту и т. д. — относится ваша жалоба;
Ваше имя, адрес, номер телефона и адрес электронной почты; а также
Ваше заявление: (а) вы добросовестно считаете, что использование контента, который, по вашему мнению, нарушает
ваши авторские права не разрешены законом, владельцем авторских прав или его агентом; (б) что все
информация, содержащаяся в вашем Уведомлении о нарушении, является точной, и (c) под страхом наказания за лжесвидетельство, что вы
либо владелец авторских прав, либо лицо, уполномоченное действовать от их имени.
Отправьте жалобу нашему уполномоченному агенту по адресу:
Чарльз Кон
Varsity Tutors LLC 101 S. Hanley Rd, Suite 300 St. Louis, MO 63105
Тесты для 6 класса по теме «Действия с десятичными дробями»
Тесты по теме «Действия с десятичными дробями»
Учитель математики Бывалина Людмила Леонидовна
Методика «Составление тестов»
Методика «Составление тестов» — одна из методик технологии формирующего оценивания. Ее суть состоит в том, что учащиеся самостоятельно формулируют вопросы по теме.
Составление тестов представляет собой индивидуальную творческую работу учащегося, которая проявляет не только его знания, подготовленность, но и мотивацию.
Самостоятельное составление вопросов по теме – это порождение текста, имеющего форму вопроса. Для выполнения этой вроде бы простой работы ребенок должен выполнить множество действий: очертить для себя границы темы, вспомнить, что он знает из этой темы, структурировать знания, составить высказывание, касающееся темы и имеющее форму вопроса, спрогнозировать ответ. Другая особенность связана с тем, что отвечающий «общается» не с учителем, а с одноклассником, отвечает не на языке учебника и взрослых, а на языке соседа по парте, принимает на себя роль педагога, вносящего свой вклад в копилку знаний партнера.
Использование тестов наиболее эффективно на этапе закрепления материала, когда тема уже пройдена. Но этот метод выполняет свои функции и в ситуации, когда новая тема только
заявлена. Своими вопросами по новой теме учащиеся демонстрируют учителю свой стартовый уровень знаний, свою заинтересованность в их расширении и углублении. На основе таких вопросов и полученных по ним ответов учитель может сделать ознакомление с новым материалом не просто формальным изложением параграфа учебника, но апеллировать к прошлому опыту конкретных детей, «выращивать» научное знание из житейского, строить диалог по поводу изучаемого материала, основываясь на вопросах детей.
Задачи этой методики можно определить так:
Повысить качество выполнения домашнего задания.
Выявить уровень понимания учащимися материала и разобрать моменты, вызвавшие затруднение, как следствие – подготовить учащихся к проверочной работе по теме.
Развивать критическое мышление.
Строить обучение на основе сотрудничества учителя и учеников, повысить активную роль детей в процессе обучения.
В качестве домашнего задания ученикам предлагается написать по какой-либо теме вопросы для проверочной работы или теста. Это должны быть вопросы на понимание материала, а не механическое его воспроизведение. Вопросы могут быть сформулированы в тестовом виде, в таком случае к ним должны предлагаться несколько вариантов ответов. Самые простые вопросы такого плана имеют два варианта ответа: «верно-неверно» или «да-нет». Тестовые вопросы могут быть закрытой формы, тогда в качестве ответа будет число или словосочетание. Также можно использовать вопросы, требующие объяснения (начинаются со слова «Почему?»).
Удачные вопросы впоследствии будут использованы в проверочной работе по данной теме, неудачные послужат материалом для критической оценки, учащимся будет предложено письменно объяснить, почему некоторые из них (вопросов) были признаны не очень удачными. Тот, кто придумает больше хороших вопросов, будет лучше подготовлен к этой работе.
Тест по теме «Действия с десятичными дробями». 6 класс
Рассмотрите дробь739,03428. Какая цифра записана в разряде десятых?
6. Какие выражения верны? Отметьте все верные варианты.
а) 1,03=1,30
б) 2,01=2,01000
в) 2,55>2,65
г) 7,0191<7,01
д) 25,0=25
е) 4,75<4,05
ж) 2,44<2,404
з) 17,183>17,09
7. Какую цифру нужно подставить, чтобы неравенство было верным?
1*,93<11,93
а) 1
б) 0
в) 2
г) 3
8. Найдите значение выражений (на черновике) и выберите среди вариантов правильные ответы:
(3,97+10,034)-8,234=?
а) 5,77
б) 6,13
в) 4,29
а) 0,7
б) 1,5
в) 1,42
а)
б)
в) 0,7
г) 1,2
9. В кувшине 1,25л молока, это на 2,7л меньше, чем в бидоне и на 1,5л меньше, чем в ведре. Сколько всего литров молока в этих трех емкостях. Решите задачу на черновике и выберите правильный ответ.
а) 7,39
б) 7,95
в) 7,43
г) 7,74
10. Найдите значения выражений и выберите правильный ответ:
210,36 : 100
а) 2,1036
б) 21,036
в) 2103,6
а) 9,13134
б) 91313,4
в) 9131,34
11. Найдите произведение десятичных дробей 3,76 и 2,4, заменив их обыкновенными дробями. Решение запишите в строчку, ответ в десятичных дробях.
12. Найдите частное
0,126 : 0,45
а) 0,28
б) 0,3
в) 1,1
г) 0,43
13. В мешке 300г одинаковых монет. Масса одной монеты 1,5г. Сколько монет в мешке?
а) 210
б) 175
в) 200
г) 150
14. Шаг ребенка 0,3м. сколько шагов надо сделать ребенку, чтобы пройти 6м?
а) 20
б) 22
в) 18
г) 25
15. Округлите число 1,6666666 до тысячных:
а) 1,6667
б) 1,667
в) 1,665
до сотых:
а) 1,67
б) 1,66
в) 1,65
*Задания повышенной сложности
16. Найдите значение выражения (решение в черновике):
а) 4,8
б) 4,1
в) 3,5
г) 3,8
17. На упаковке с сахарным песком, взвешенной на электронных весах, указана ее стоимость 25,30р. Цена 1кг песка равна 21р. Чему равна масса песка в упаковке. Ответ выразите в кг и г. (решение на черновике)
а) 1,2кг или 1200г
б) 1,5кг или 1500г
в) 2кг или 2000г
18. Прямоугольник и квадрат имеют одинаковые периметры. Чему равна площадь квадрата, если длины сторон прямоугольника равны 1,8см и 3,4см? (решение на черновике)
а) 5,43
б) 6,76
в) 7,02
Инструкция
Тест-задание состоит из двух блоков: базовая часть и задания повышенной сложности, которые не являются обязательными к решению. Задания под * могут решать ученики, которые справятся с базовыми заданиями и у них останется время до конца урока. За задания под * ученик получает дополнительные баллы (или дополнительную оценку).
Таким образом, более способные ученики могут углубить и закрепить свои знания, а отстающие получают возможность улучшить свою оценку за счет дополнительных заданий.
В тесте есть как открытые вопросы, так и задания с вариантами ответов. У каждого задания четко расписано, какой вид ответа требуется(расписать решение, выбрать один или несколько вариантов ответа).
Важным является то, что дополнительно к листу теста учащимся выдаются листы для черновика с пометкой «Черновик» (Приложение 1). Все вычисления, которые не требуются для записи в ответе, учащиеся решают на «Черновике и потом сдают его вместе с работой. Это необходимо для того, чтобы учитель мог отследить логику мышления ученика при решении задачи, увидеть слабые места и «слепые зоны», причины допущенных ошибок. Наличие такого «Черновика» позволяет учителю более точно и справедливо оценить работу.
Оценивается тест следующим образом.
За 90% и более правильно выполненных заданий, ученик получает оценку «5» (отлично).
70% и более – оценка «4» (хорошо).
50% и более – оценка «3» (удовлетворительно).
Менее 50% — оценка «2» (неудовлетворительно).
Этим способом оценивается базовая часть теста. Задания под* оцениваются дополнительно. За каждый правильный ответ, ученик получает 10% к свой общей оценке. Таким образом, учащиеся могут не решить некоторые задания в базовой части, но повысить свою оценку, решив задания под*.
Расчет (примерный) времени на каждое задание в базовой части:
№
1
2
3
4
5
6
7
8
9
10
11
12
13
14
15
мин
1
1
1
5
3
3
1
6
3
1
2
2
1
2
2
Итого на базовые задания рассчитано времени около 34 минут. При длительности урока в 45 минут остается запас времени на организационные моменты, дополнительные задания и др.
Тест для 6 класса по теме «Десятичные дроби», 1-2 вариант
Зачет по теме:» Десятичные дроби». 6 класс.
Тест. 1 вариант.
1. Переведите в десятичную дробь : .
2. Запишите координату отмеченной точки А: А
3. Расположите числа в порядке возрастания: 3,02; 3,0029; 3,019.
4 .Выразить в метрах:3м35см.=…м.
5. Выразить в тоннах:3т 90кг.=…т
6. Найдите разность:10,2-3.16.
7. Найдите сумму:6,1+7.02.
8. Умножить:12,671000
9. Найти произведение: 24.53,4.
10 .Разделить:162,3100.
11. Найти частное:148,14:3.
12. Решить уравнения :а) х+3,6=8,4;
б)10-х=7,2;
в)10,5 : х=2,1;
Г)1,5
13. Округлить десятичную дробь до десятых: 3,75.
14. Округлить десятичную дробь до сотых: 6,809.
15. Найти периметр прямоугольника со сторонами 12,5см и 10,2см.
Зачет по теме: «Десятичные дроби».6 класс.
Тест. 2 вариант.
1.Переведите в десятичную дробь: .
2.Запишите координату отмеченной точки В
3.Расположите числа в порядке убывания: 7,12; 7,09;7.019.
4.Выразить в тоннах 5т 50 кг=…т.
5.Выразить в метрах 5 м 30см=…м
6.Найдите разность 12.3 и 4,27.
7.Найдите сумму чисел 12,3 и 7,15.
8.Выполнить умножение 13,2100.
9.Найти произведение чисел 23.5 и 2.5.
10.Выполните деление 16,2:100
11.Найти частное:16,2 : 0,03
12.Решите уравнения : а)х+7,2=15,12;
б)100-х =90,3;
в)10,8 : х =3,6
г)1,8х =7,2
13.Округлить десятичную дробь до десятых 6,85.
14.Округлить десятичную дробь до сотых 7.806.
15.Найти площадь прямоугольника со сторонами2.5 см и 3,2 см.
Тест по математике (5, 6 класс): Тест по теме действия с десятичными дробями
Тест разработал учитель математики МКОУ Лизиновская СОШ: Фурсенко Николай Петрович
Обобщающий тест по математике на проверку умений выполнять действия с десятичными дробями (для 5-го или 6-го класса в зависимости от программы)
Задание: Отметьте правильный ответ
Сумма чисел 3,43 и 12,9 равна
а) 15,33
б) 16,43
в) 16,33
г) 15,52
Задание: Отметьте правильный ответ
Разность чисел 37,2 и 18,567 равна
а) 19,565
б) 18,367
в) 19,367
г) 18,633
Задание: Отметьте правильный ответ
Произведение чисел 12,3 и 2,45 равно
а) 30,135
б) 32,135
в) 30,145
г) 30,136
Задание: Отметьте правильный ответ
Частное чисел 124,66 и 2,3 равно
а) 5,42
б) 54,2
в) 0,543
г) 54,3
Задание: Соответствие между выражением и его значением
А) 34,2+5,96 =
1) 39,56
Б) 4,3·9,2 =
2) 44,525
В) 71,24:1,6 =
3) 38,86
Г) 56,7 – 17,84 =
4) 40,16
Задание: Дополните
Если по периметру прямоугольного участка со сторонами 35,54м и 21,7м поставить забор, то его длина будет равна … м.
Задание: дополните
Площадь пола прямоугольной комнаты с длиной 5,2м и шириной 3,45м равна … м2.
Задание: дополните
От рулона ткани длиной 45,24м на пошив одежды израсходовали 8,7м, ткани осталось … м.
Задание: отметьте правильный ответ
Неверное равенство
а) 5,4 + 6,7 = 12,1
б) 12,2 – 7,8 = 4,4
в) 3,4·2,6 = 88,4
г) 12,1:1,1 =11
Задание: дополните
Значение выражения 23,4·56,79 – 56,79·13,4 равно …
ОТВЕТЫ
К 1-4 заданию нужно выбрать один вариант ответа.
Сумма чисел 3,43 и 12,9 равна
а) 15,33
б) 16,43
в) 16,33
г) 15,52
Разность чисел 37,2 и 18,567 равна
а) 19,565
б) 18,367
в) 19,367
г) 18,633
Произведение чисел 12,3 и 2,45 равно
а) 30,135
б) 32,135
в) 30,145
г) 30,136
Частное чисел 124,66 и 2,3 равно
а) 5,42
б) 54,2
в) 0,543
г) 54,3
Соответствие между выражением и его значением
А) 34,2+5,96 =
1) 39,56
Б) 4,3·9,2 =
2) 44,525
В) 71,24:1,6 =
3) 38,86
Г) 56,7 – 17,84 =
4) 40,16
Ответ: 4123
Практико-ориентированные задания 6-8
Если по периметру прямоугольного участка со сторонами 35,54м и 21,7м поставить забор, то его длина будет равна 114,48м.
Площадь пола прямоугольной комнаты с длиной 5,2м и шириной 3,45м равна 17,94 м2.
От рулона ткани длиной 45,24 м на пошив одежды израсходовали 8,7 м, ткани осталось 36,54 м .
Неверное равенство
а) 5,4 + 6,7 = 12,1
б) 12,2 – 7,8 = 4,4
в) 3,4·2,6 = 88,4
г) 12,1:1,1 =11
Значение выражения 23,4·56,79 – 56,79·13,4 равно 567,9.
КЛЮЧ
№
1
2
3
4
5
6
7
8
9
10
ответ
в
г
а
б
4123
114,48
17,94
36,54
в
567,9
Тест по математике (6 класс): Дидактика 6 класс. Сложение и вычитание десятичных дробей.
Сложение и вычитание десятичных дробей
1 уровень
Какие из действий выполнены неверно:
3,75+4,6=4,21
0,756+12,34=13,096
15,37+4,200=5,737
0,785+1,045=1,83
Решите уравнение: 0,4937 – (х – 0,1763) = 0,2477
0,9177
2,45
6,201
0,8466
С одного участка собрали 95,37 т зерна, а с другого – на 16,8 т больше. Сколько тонн зерна собрали с двух участков?
112,17
207,54
173,94
2 уровень
Найдите значение выражения, представив десятичную дробь в виде обыкновенной:.
Вычислите и округлите до десятитысячных:
2,07а – 1,23а + 1,014 + 0,44а при а = 0,803
2,0418
3,1362
2,04184
1,2041
Собственная скорость катера (скорость в стоячей воде) равна 21,6 км/ч, а скорость течения реки 4,7 км/ч. Найдите скорость катера по течению и против течения.
26,3 и 21,6
21,6 и 26,3
16,9 и 20,3
уровень
Запишите все десятичные дроби, в записи которых использованы только одна единица и две пятерки. Найдите их сумму.
134,31
143,31
Сумма двух чисел равна 12,5. Одно из них на 3,3 меньше другого. Найдите эти числа.
7,9 и 4,6
7,6 и 4,9
5,4 и 7,1
2,5 и 10
Скорость катера по течению реки равна 27,3 км/ч. Найдите скорость катера против течения, если скорость течения реки 2,55 км/ч.
24,75
29,85
23,2
22,2
«Десятичные дроби» — задание по математике для 6 класса
Задание по математике для 6 класса — «Десятичные дроби»
Лимит времени: 0
Информация
Выполните задание онлайн олимпиады и узнайте результат. Для зарегистрированных участников, результаты отправляются на электронную почту.
Вы уже проходили тест ранее. Вы не можете запустить его снова.
Тест загружается…
Вы должны войти или зарегистрироваться для того, чтобы начать тест.
Вы должны закончить следующие тесты, чтобы начать этот:
Правильных ответов: 0 из 10
Ваше время:
Время вышло
Вы набрали 0 из 0 баллов (0)
Средний результат
Ваш результат
Поздравляем! Вы отлично справились с заданием. Ваш результат соответствует 1 месту.
Оформить диплом
Поздравляем! Вы хорошо справились с заданием. Ваш результат соответствует 2 месту.
Оформить диплом
Поздравляем! Вы выполнили задние допустив незначительное количество ошибок. Ваш результат соответствует 3 месту.
Оформить диплом
Сделайте работу над ошибками. Попробуйте пройти тестирование еще раз и добиться хорошего результата. Ваш результат может стать значительно лучше.
С ответом
С отметкой о просмотре
Задание 1 из 10
Количество баллов: 1
Считается ли дробь десятичной, если в знаменателе у нее стоит число 10?
Задание 2 из 10
Количество баллов: 1
Считается ли дробь десятичной, если в знаменателе у нее стоит число 1000?
Задание 3 из 10
Количество баллов: 1
Что делать, если у дробей, которые вычитаются неодинаковое количество знаков после запятой?
Задание 4 из 10
Количество баллов: 1
Десятичные дроби подчиняются каким-то особенным правилам сложения и вычитания?
Задание 5 из 10
Количество баллов: 1
Каким правилам подчиняются десятичные дроби при вычитании и сложении?
Задание 6 из 10
Количество баллов: 1
Чему равно выражение: 0,6-0,17?
Задание 7 из 10
Количество баллов: 1
Чему равно выражение 0,9-0,01
Задание 8 из 10
Количество баллов: 1
Чему равно выражение 0,124-0,1
Задание 9 из 10
Количество баллов: 1
Для двух десятичных дробей общим знаменателем является …
Задание 10 из 10
Количество баллов: 1
Решить пример 0,890-0,45
Тест по математике: Десятичное приближение обыкновенной дроби (Мерзляк, 6 класс) — пройти тест онлайн — игра — вопросы с ответами
Мой результат
Тест онлайн
Нашли ошибку? Выделите ошибку и нажмите Ctrl+Enter
Выбрав правильный на ваш взгляд вариант ответа, жмите на кнопку «Проверить». Если хотите сразу увидеть правильные ответы, ищите под вопросами ссылку «Посмотреть правильные ответы»
1. Преобразуйте обыкновенную дробь в бесконечную периодическую десятичную дробь. Результат округлите до сотых.
2.
Округлите бесконечную периодическую дробь 0,530197530197… до десятитысячных.
3.
В результате округления бесконечной периодической дроби получается конечная десятичная дробь, которую называют десятичным … обыкновенной дроби.
4.
Поезд проехал 307 км за 6 час. Найди его скорость. (Ответ округли до сотых км/час).
5.
Вычисли высоту прямоугольного параллелепипеда, если его объем равен 14 дм3, длина — 3 дм, а ширина — 2,5 дм. (Ответ округли до десятых дециметра).
6.
Площадь прямоугольного участка земли 2730 м2, а длина одной из сторон 55 м. Найди длину другой стороны. (Ответ округли до сотых метра).
7.
Найди корень уравнения и округляя его до сотых: 19x =25.
8.
Найди корень уравнения и округляя его до сотых: 8 : х = 125.
9.
Найди корень уравнения и округляя его до сотых: 56х = 103.
10.
Найди корень уравнения и округляя его до сотых: 17 : х = 6..
бесплатных заданий по математике для 6-го класса
Вы здесь: Главная → Задания → 6 класс
Это исчерпывающая коллекция бесплатных распечатываемых рабочих листов по математике для шестого класса, организованных по таким темам, как умножение, деление, экспоненты, разряд, алгебраическое мышление, десятичные дроби, единицы измерения, соотношение, процент, разложение на простые множители, GCF, LCM, дроби, целые числа и геометрия. Они генерируются случайным образом, их можно распечатать в вашем браузере и включать в себя ключ ответа.Рабочие листы подходят для любой математической программы 6-го класса, но были специально разработаны для соответствия учебной программе 6-го класса Math Mammoth.
Рабочие листы генерируются случайным образом каждый раз, когда вы нажимаете на ссылки ниже. Вы также можете получить новый, другой, просто обновив страницу в своем браузере (нажмите F5).
Вы можете распечатать их прямо из окна браузера, но сначала проверьте, как это выглядит в «Предварительном просмотре». Если рабочий лист не умещается на странице, настройте поля, верхний и нижний колонтитулы в настройках страницы вашего браузера.Другой вариант — настроить «масштаб» на 95% или 90% в предварительном просмотре печати. В некоторых браузерах и принтерах есть опция «Печатать по размеру», которая автоматически масштабирует рабочий лист по размеру области печати.
Все рабочие листы содержат ключ ответа, расположенный на 2-й странице файла.
В шестом классе ученики приступят к изучению начальной алгебры (порядок операций, выражения и уравнения). Они узнают о соотношениях и процентах и начинают использовать целые числа.Студенты также изучают деление на множители, факторизацию, арифметику дробей и десятичную арифметику. В геометрии основное внимание уделяется площади треугольников и многоугольников и объему прямоугольных призм. Другие темы включают округление, экспоненты, GCF, LCM и единицы измерения. Обратите внимание, что эти бесплатные рабочие листы не охватывают все темы 6-го класса; в первую очередь, они не включают решение проблем.
Умножение и деление и некоторые обзоры
Длинное умножение
Длинное деление
1-значный делитель, 5-значное делимое, без остатка
1-значный делитель, 5-значное делимое, с остатком
1-значный делитель, 6-значное делимое, без остатка
1-значный делитель, 6-значное делимое, с остатком
1-значный делитель, 7-значное делимое, без остатка
1-значный делитель, 7-значное делимое, с остатком
.
Тест по математике в шестом классе
Перед тем, как начать, распечатайте свой тест по математике для 6-го класса. Постарайтесь ответить на все вопросы.
Имя __________________ Дата: __________________
Решите следующие задачи
1.
В приведенном ниже уравнении, каково значение x
20 = x + (2 × 8) — 6
2.
У Сары 30 карандашей. У ее подруги Сильвии на k карандашей меньше.Какое выражение показывает количество карандашей у Сильвии?
A. 30 + k B. k — 30 C. 30 — k D. 30 — 2k
3.
Треугольник ABC подобен треугольнику DEF
Какая длина DF?
A. 2 дюйма B. 1 дюйм C. 3 дюйма D. 1,5 дюйма
4.
Каково значение приведенного ниже выражения?
5 3
A. 15 B. 8 C. 25 D. 125
5.
Используйте следующий список, чтобы найти среднее или среднее, медианное, режим и диапазон
10, 15, 5, 8, 6, 6, 2
Среднее значение = ___________
Медиана = __________
Режим = ___________
диапазон = __________
6.
-5-8 = _______
5-8 = _______
-8-5 = ______
7.
Нарисуйте систему координат в пространстве ниже, а затем постройте (2,5) и ( -4, 2)
8.
Найдите периметр и площадь прямоугольника ниже:
Периметр = _____________ единиц
Площадь = __________________ квадратных единиц
9.
Возьмите кубик и бросьте его.
Различные исходы: __________________________
Напишите вероятность получения четного числа ____________
Напишите вероятность получения нечетного числа ____________
Напишите вероятность получения простого числа ____________
10.
Добавьте 3 / 5 и 2/7 _____________
11.
Для числа 76.2345 значение цифры 4 равно
A. 4 десятых B. 4 десятых C. 4 тысячных D. 4 десятитысячных
12.
Если 9450 / x равно 21, что такое x? ________________
13.
Расположите следующие числа в порядке от меньшего к большему
а.Вы хотите сэкономить на покупке велосипеда. Вы начинаете с 50 долларов и экономите 4 доллара каждый день. Какое выражение показывает, сколько денег у вас будет через x дней?
50 — 4x
4x — 50
4x + 50
50 + 4 × 10
б. Сколько денег у вас будет через 20 дней?
A. 120 долларов B. 130 долларов C. 110 долларов D. 200 долларов
15.
a.Запишите все составные числа больше 10 и меньше 20 (подсказка: их 5
Выберите составное число и запишите все множители в поле под
б. Запишите все простые числа больше 10 и меньше 20 (Подсказка: их 4
_________, __________, ___________, ____________
16.
Сколько линий симметрии у квадрата? ____________
17.
Диаметр круга 8 дюймов
Периметр и площадь:
A. P = 4 пи и A = 12 пи Б. P = 16 пи и A = 8 пи C. P = 8 пи и A = 16 пи D. P = 16 пи и A = 4 пи
18.
У Джона и Питера вместе 40 долларов. Если у Джона в 4 раза больше денег, чем у Питера. Сколько денег у каждого? (Подсказка: попробуйте методом проб и ошибок)
У Джона ____________ долларов
У Питера ____________ долларов
19.
Машина производит 5000 изделий за 6 минут. Какую пропорцию нельзя использовать для определения количества минут для производства 15000 единиц продукции?
A. 5000/6 = 15000 / x B. 5000/6 = x / 15000 C. 6 / x = 5000/15000 D. 6/5000 = x / 15000
20
Вы идете в ресторан и покупаете еду на 120 долларов. Ресторан требует чаевых 15%.Какая у вас сумма?
21.
На карте 1 дюйм означает 30 миль. Сколько дюймов покажет расстояние в 120 миль? __________
22.
В коробке 6 синих шариков, 4 желтых шарика и 10 красных шариков.
Какова вероятность выбора красного мрамора?
A. 4/20 B. 3/20 C. 6/20 D. 10/20
Какой цвет выбрать с меньшей вероятностью? ___________
23.
а. Запишите краткое определение конгруэнтности
Сделайте рисунок, чтобы показать две конгруэнтные формы
b. Напишите краткое определение сходства
Нарисуйте на рисунке две одинаковые формы
24.
Прокрутите каждый спиннер один раз и перечислите все возможные результаты (Подсказка: один результат — 2 и белый)
Какова вероятность получить 1 и красное? __________
25.
Оцените числовое выражение ниже:
(8 + 2) [(7 — 3) × 5]
A. 100 B. 150 C. 300 D. 200
26.
Какова сумма углов в пятиугольнике? (Подсказка: разделите пятиугольник на треугольники. Треугольник равен 180 градусам)
27
a. Сколько дюймов в 3 и 1/2 футах?
г. Сколько метров в 500 сантиметрах?
28.
Каков объем и площадь поверхности следующей прямоугольной призмы?
Объем = _______________
Площадь поверхности = ______________
29.
а. Форма бассейна больше похожа на
A. Прямоугольная призма B. Круг C. Пирамида D. Сфера
b.Найдите футбольное поле в Интернете. Затем найдите 4 геометрические фигуры
_________, __________, ____________, __________
30.
Вы идете по магазинам и видите табличку с надписью: «Купите 1 футболку и получите скидку 20% на вторую футболку. Футболка стоит 30 долларов, и вы покупаете 2 футболки. Какова ваша сумма? __________
Примечание : результат 25 или более баллов на этом тесте по математике в шестом классе является хорошим показателем того, что большинство навыков преподается в Были освоены 6-й класс
Если вы много боролись с этим тестом по математике в 6-м классе, попросите кого-нибудь помочь вам
Хотите получить решение для этого теста? Добавьте в корзину и купите Подробное 18-СТРАНИЦНОЕ РЕШЕНИЕ и ПРЕВОСХОДНЫЕ ОБЪЯСНЕНИЯ с PayPal.
Я изо всех сил старался сделать этот тест по математике для 6-го класса в соответствии с национальными стандартами
Чтобы распечатать этот тест по математике для 6-го класса, щелкните здесь
К этому тесту по математике для 6-го класса относится тест по математике для 5-го класса
Новые уроки математики
Ваша электронная почта в безопасности. Мы будем использовать его только для информирования вас о новых уроках математики.
.
Общий основной тест по математике для 6 класса (примеры вопросов)
1. Население Нью-Йорка примерно вдвое больше, чем в Париже. Напишите отношение населения Нью-Йорка к населению Парижа.
1: 1
1: 2
2: ½
2: 1
2. Что составляет 40% от 50?
10
16
20
25
3. Какое наименьшее общее кратное (НОК) 8 и 12?
16
24
48
96
4.Баланс на вашем сберегательном счете равен -50, что означает, что у вас есть задолженность на 50 долларов. Ежемесячная плата увеличивает сумму вашего долга. Что из следующего может стать вашим новым балансом?
60
40
-40
-60
5. Рассчитайте 24 ÷ (6 — 2).
2
3
6
8
6. Что из следующего является решением неравенства 9 + 2x> 5x?
2
3
4
8
7.Вычислите площадь прямоугольного треугольника ниже.
9
10
15
20
8. Среднее значение набора данных — 12, а его диапазон — 8. Какое из следующих значений НЕ могло быть значением в наборе ?
4
8
12
16
9. Ниже показаны результаты тестов в классе мистера Смита.
82, 94, 81, 70, 72, 78, 95, 83, 89, 80, 55, 77
Создайте гистограмму для отображения данных.
10. Какую единицу измерения целесообразно использовать при исследовании посещаемости девяти различных бродвейских шоу?
дней
долларов
человек
показывает
Ответы и пояснения
1. D: Отношение — это отношение между двумя числами одного типа. Например, если соотношение яблок к апельсинам в корзине составляет 3: 2, то корзина содержит 3 яблока на каждые 2 апельсина.В данной задаче ищем соотношение между населением двух городов. Поскольку нам говорят, что население Нью-Йорка примерно вдвое больше, чем в Париже, на каждого парижанина приходится около двух жителей Нью-Йорка. Следовательно, правильное соотношение — 2: 1.
2. C: Процент представляет одну часть из 100. Таким образом, 40% эквивалентно 40 частям из 100, или 40/100. Чтобы найти 40% от 50, умножьте 40/100 на 50: 40/100 X 50 = 20
3. B: Наименьшее общее кратное (НОК) двух целых чисел — это наименьшее (или наименьшее) целое число, которое кратное обоим числам.Один из способов найти НОК 8 и 12 — перечислить их кратные и выбрать наименьшее число, которое появляется в обоих списках.
Поскольку 24 является наименьшим числом в обоих списках, НОК 8 и 12 равно 24.
4. D: Увеличение суммы вашего долга означает, что вы должны более 50 долларов. Если, например, ваш долг увеличивается на 10 долларов, он становится 60 долларов, а баланс вашего счета становится -60. Следовательно, -60 может быть вашим новым балансом.(Обратите внимание, что сальдо -40 означает, что вы должны 40 долларов, что приведет к уменьшению вашего долга.)
5. C: В соответствии с порядком операций (PEMDAS) сначала оцените любые количества или выражения в скобках. . Вычтите, чтобы найти значение в круглых скобках, а затем разделите 24 на результат этого вычитания:
24 x (6 — 2) = 24 ÷ 4 = 6
6. A: Решение уравнения или неравенство — это значение переменной (ей), которая делает уравнение или неравенство истинным.Чтобы определить, какой вариант является решением неравенства 9 + 2x> 5x, подставьте каждый из них вместо x в неравенстве и упростите результат, чтобы проверить, верно ли неравенство. Начните с x = 2.
9 + 2x> 5x 9 + 2 (2)> 5 (2) 9 + 4> 10 13> 10
Поскольку 13> 10 верно, правильный ответ — x = 2. В качестве альтернативы вычтите 2x из обеих частей неравенства, чтобы получить 9> 5x — 2x, что эквивалентно 9> 3x. Разделите обе части на 3, чтобы получить 3> x или x <3.Единственный x, который можно выбрать меньше 3, - это 2, поэтому A правильно.
7. B: Площадь треугольника определяется формулой A = ½ bh , где b — основание треугольника, а h — высота треугольника (нарисованного линией перпендикулярно основанию). Поскольку треугольник является прямоугольным, вы можете использовать горизонтальную опору для основания и вертикальную опору для высоты:
A = ½ bh = ½ (4) (5) = ½ (20) = 10
8.A: Среднее значение набора данных — это его среднее значение. Следовательно, если все значения в наборе не равны 12 (а вы знаете, что это не так, потому что диапазон равен 8), некоторые из них должны быть меньше 12, а некоторые — больше 12.
Диапазон набора данных это разница между его наибольшим и наименьшим значениями. Поскольку диапазон этого набора равен 8, все значения в наборе должны быть больше 4. Если одно значение было 4, должно быть значение больше 12, чтобы среднее значение оставалось равным 12, а разница между этим значением и 4 было бы больше 8.Поскольку мы знаем, что диапазон равен 8, 4 не может быть значением в наборе.
9. A: Гистограмма — это способ графического представления распределения набора данных. На гистограмме диапазон данных разделен на интервалы, так что каждое значение попадает в один интервал. Для заданных результатов теста один естественный способ установить интервалы: 50 — 59, 60 — 69, 70 — 79, 80 — 89 и 90 — 100.
Затем вычислите количество значений, попадающих в каждый интервал, т. Е. спросите, сколько значений от 50 до 59 (включая 50 и 59), сколько от 60 до 69 и так далее:
Наконец, отобразите эту информацию графически, используя вертикальную полосу для каждого интервала и сделав ее высоту равной частоте интервала:
10.C: Соответствующая единица измерения количества соответствует тому, что вы считаете при измерении. Например, если цена автомобиля составляет 40 000 долларов, поскольку вы считаете доллары, единицей измерения количества в 40 000 долларов является доллар. Чтобы измерить посещаемость бродвейских шоу, вы подсчитываете количество людей, которые ходят на каждое шоу. Следовательно, единицей измерения являются люди.
.
Задания по ментальной математике Рабочие листы 6-го класса
Добро пожаловать на экзамены по математике 6-го класса по умственной математике саламандр.
Здесь вы найдете широкий спектр заданий по ментальной математике, предназначенных для детей 6-го класса, которые помогут вашему ребенку узнать факты о числах и практиковать свои числовые навыки.
Наша подборка заданий по мысленной математике — отличный способ попрактиковать свои числовые и математические навыки.
Листы можно использовать по-разному, в качестве теста или повторной практики, или как часть еженедельной викторины для усиления навыков.
Их также можно использовать в качестве устного и мысленного стартера урока, чтобы заставить мозг работать!
Один из лучших способов использовать эти листы — побудить детей работать в парах, обсуждая вопросы по мере их прохождения.
Вопросы были разработаны для отработки ряда математических навыков от чисел до геометрии и фактов измерения, включая использование времени и денег.
Викторины расположены в порядке сложности, при этом вопросы в каждой викторине становятся немного сложнее.
Наши рабочие листы по ментальной математике для 6-го класса содержат широкий спектр различных вопросов и математических навыков.
Каждый лист содержит 18 вопросов и снабжен листом для ответов.
Уровень сложности становится сложнее по мере прохождения листов.
Охваченные темы включают:
ментальная арифметика — сложение, вычитание, умножение и деление;
вопросов об отсутствии фактов;
вопросов о ценностях до 10000;
вопросов по геометрии, основанных на свойствах 2D и 3D форм;
денежных вопросов;
вопросов с дробными, десятичными и процентными числами;
измеряет проблемы;
задач со словами;
вопросов о соотношении и пропорциях;
вопросов о времени, включая вопросы о дате и календаре.
Рабочие листы в этом разделе были разработаны для детей 5-го класса.
Листы имеют тот же формат, что и на этой странице, но на более простом уровне.
Если вы ищете онлайн-практикумы по ментальной математике, факты о 4-м классе, у нас есть несколько практических зон.
где вы можете проверить свои знания по ряду математических фактов и распечатать результаты онлайн.
Есть сертификаты для выигрыша и мгновенная обратная связь о вашем прогрессе.
Здесь вы найдете ряд бесплатных печатных игр по математике для 5-х классов.
Все дети любят играть в математические игры, и вы найдете хороший
ряд математических игр для 5-го класса, в которые ваш ребенок может поиграть и развлечься.
В следующих играх участвуют разные математики для 5-х классов.
занятия, которыми вы и ваш ребенок можете наслаждаться вместе.
Все бесплатные задания по математике для 5-х классов в этом разделе
следовать тестам по элементарной математике для 5-го класса.
Ищете классные математические онлайн-игры, в которые можно поиграть на уровне 6-го класса?
Попробуйте сайт основных игр, где вы найдете банк, полный различных игр!
По ссылке ниже в новом окне браузера откроется основной веб-сайт игры.
Саламандры-математики надеются, что вам понравятся эти бесплатные распечатываемые рабочие листы по математике.
и все другие наши математические игры и ресурсы.
Мы приветствуем любые комментарии о нашем сайте или рабочих таблицах в поле комментариев Facebook внизу каждой страницы.
Урок 27. проверка сложения. проверка вычитания — Математика — 2 класс
Математика, 2 класс. Урок №27
Проверка сложения. Проверка вычитания.
Перечень вопросов, рассматриваемых в теме:
— Что такое обратные математические действия?
— Как проверить сложение?
— Как проверить вычитание?
Глоссарий по теме:
Сложение – это объединение объектов в одно целое. Результатом сложения чисел является число, называемое суммой чисел (слагаемых).
Вычитание – это такое действие, в котором отнимают меньшее число от большего. Большее число называется уменьшаемым, меньшее – вычитаемым, результат вычитания – разностью.
Обратные действия – действия, приводящие к прежнему, исходному состоянию.
Основная и дополнительная литература по теме урока (точные библиографические данные с указанием страниц):
Математика. 2 класс. Учебник для общеобразовательных организаций. В 2 ч. Ч.1/ М. И. Моро, М. А. Бантова, Г. В. Бельтюкова и др. – 8-е изд. – М.: Просвещение, 2017. – с.84-86.
Математика. Рабочая тетрадь. 2 класс. Учебное пособие для общеобразовательных организаций. В 2 ч. Ч.1/ М. И. Моро, М. А. Бантова – 6-е изд., дораб. – М.: Просвещение, 2016. – с.60.
Математика: переходим в 3-й класс. Учебное пособие для общеобразовательных организаций. А. В. Светин – М.: Просвещение: Уч. Лит, 2017. – с.40.
Теоретический материал для самостоятельного изучения
Используя числа 7, 5, 12 составим все возможные равенства.
7 + 5 = 12 12 – 5 = 7
12 – 7 = 5 5 + 7 = 12
Назовём компоненты и результат действия сложения.
7 + 5 = 12
Слагаемое + слагаемое = сумма
Назовём компоненты и результат действия вычитания.
12 — 7 = 5
Уменьшаемое – вычитаемое = разность
Действия сложение и вычитание связаны друг с другом, являются взаимно обратными действиями.
СЛОЖЕНИЕ ВЫЧИТАНИЕ
Как проверить, верно ли выполнено сложение. Воспользуемся знанием того, как связаны слагаемые и сумма. Если из суммы двух слагаемых вычесть одно из них, то получится другое слагаемое. Это позволяет сложение проверить вычитанием.
Например, надо проверить, верно ли вычислили сумму чисел 28 и 5. Для этого из суммы 33 вычтем одно из слагаемых. Например, 5. Должно получиться другое слагаемое. Получилось 28. Значит, сумма чисел 28 и 5 найдена правильно. Можно вычесть из суммы другое слагаемое.
28 + 5 = 33
33 – 5 = 28
33 – 28 = 5
Сумма чисел 36 и 9 найдена неверно, т.к. после вычитания из суммы 47 слагаемого 9, другое слагаемое, 36 не получается.
36 + 9 = 47
47 – 9 = 38
38 = 36
Вычислим ещё раз сумму чисел 36 и 9 и проверим результат.
36 + 9 = 45
45 – 9 = 36
36 = 36
36 – первое слагаемое
Сформулируем правило проверки сложения: «Для проверки сложения надо из значения суммы вычесть одно из слагаемых. Если в результате вычитания получается другое слагаемое, значит, сложение выполнено верно».
Как проверить вычитание? Воспользуемся знанием того, как связаны между собой уменьшаемое, вычитаемое, разность. Если к разности прибавить вычитаемое, то получится уменьшаемое. Значит, вычитание можно проверить сложением.
Вычислим разность чисел 48 и 30. Она равна 18. Проверим вычитание сложением. К разности 18 прибавим вычитаемое 30, получим 48. Это уменьшаемое.
48 – 30 = 18
18 + 30 = 48
48 = 48
48 — уменьшаемое
Если из уменьшаемого вычесть разность, то получится вычитаемое.
Значит, вычитание можно проверить и вычитанием. Рассмотрим это на примере.
Из уменьшаемого 48 вычтем разность 18, получим 30, т.е. вычитаемое. Значит, разность чисел 48 и 30 вычислена верно.
48 – 30 = 18
48 – 18 = 30
30 = 30
30 — вычитаемое
Сформулируем правила проверки вычитания: «Для проверки вычитания, надо к значению разности прибавить вычитаемое. Если в результате сложения получается уменьшаемое, значит, вычитание выполнено верно», или «Для проверки вычитания, надо из уменьшаемого вычесть разность. Если в результате получается вычитаемое, значит, вычитание выполнено верно».
Вывод: Сложение и вычитание – это обратные действия. Для проверки сложения надо из значения суммы вычесть одно из слагаемых. Если в результате вычитания получается другое слагаемое, значит, сложение выполнено верно. Для того, чтобы выполнить проверку вычитания, надо к значению разности прибавить вычитаемое. Если в результате сложения получается уменьшаемое, значит, вычитание выполнено верно.
Тренировочные задания.
1. Найдите значение первого выражения в каждой рамке, а затем выполни проверку полученного результата двумя способами.
42 + 30 = …..
54 + 6 = …..
65 – 12 = …..
78 – 50 = …..
— 30 = …….
— 30 = …….
— 6 = …….
— 30 = …….
+ 12 =…….
— 30 = …….
+ 50 =…….
— 30 = …….
Правильные ответы:
42 + 30 = 72.
54 + 6 = 60..
65 – 12 = 53..
78 – 50 = 28.
72 — 30 = 42.
72 — 42 = 30.
60 — 6 = …54.
60 — 54 = …6.
53 + 12 = 65
65 — 53 = 12.
28 + 50 =78.
78 — 28 = 50
2. Распределите все записи с вычислениями и проверкой на верные и неверные.
Правильные ответы:
Проверка обратными действиями | Математика
Пользуясь основными свойствами данных и искомых чисел, обычно производят проверку обратными арифметическими действиями. Таким образом, сложение проверяется вычитанием, вычитание сложением, умножение делением, деление умножением.
Проверка сложения
Чтобы повторить сложение, отбрасывают одно слагаемое, складывают остальные и сумму вычитают из общей суммы; если в остатке получается отброшенное слагаемое, сложение сделано верно.
Проверка сложения:
Здесь мы обозначили звездочкой отброшенное слагаемое и, сложив остальные, полученную сумму 26340 вычли из общей суммы. В остатке получили зачеркнутое слагаемое, следовательно — сложение сделано верно.
Проверка вычитания
Уменьшаемое равно вычитаемому, сложенному с разностью; следовательно, чтобы проверить вычитание, нужно вычитаемое сложить с остатком; если в сумме получится уменьшаемое, вычитание сделано верно.
Проверка вычитания:
Для проверки складываем вычитаемое с разностью и получаем в сумме уменьшаемое.
Вычитание верно.
Проверка умножения
Произведение, разделенное на один множитель, должно дать в частном другой, следовательно, чтобы проверить умножение, нужно разделить произведение на один множитель; если в частном получается другой множитель, а в остатке нуль, умножение сделано верно.
Проверка умножения:
Разделяя произведение 501116 на один множитель 1627, получаем в остатке нуль, а в частном другой множитель 308; следовательно, умножение сделано верно.
Проверка деления
Делимое равно произведению делителя на целое частное, сложенное с остатком; следовательно, чтобы проверить деление, нужно делитель умножить на целое частное и приложить остаток; если получим делимое, деление выполнено верно.
Для проверки умножаем делитель 107 на целое частное 255 и прикладываем остаток 73, получаем делимое 27358; следовательно, деление сделано верно.
Проверка сложения. Урок математики во 2-м классе
Цель урока: Формирование умения проверять
вычисления, выполненные при сложении.
Задачи:
Познакомить со способом проверки сложения
вычитанием.
Формировать умение составлять алгоритм.
Совершенствовать вычислительные навыки и
графические умения.
Воспитывать коллективные отношения.
Оборудование: изображение тролля Кузи, блоки
для составления алгоритма (на каждой парте),
учебник “Математика, 2-й класс. ” (М.И. Моро),
карточки для решения “круговых примеров”.
ХОД УРОКА
1. Организационный момент.
— Ребята, вы ждете подарков? Зажмурьте глаза и
представьте, что под новогодней елкой стоят ваши
подарки. Улыбнитесь! Какое у вас настроение?
2. Введение в тему урока. Постановка цели рока.
— Сегодня к нам на урок пришел тролль Кузя –
персонаж интерактивных компьютерных игр.
— Вы любите играть в компьютер?
— Кузя пришел к нам на урок математики не
случайно. Дело в том, что в одном из файлов его
компьютерной игры поселился вирус, и она не
загружается. Кузя выяснил, что вирусом заражены
некоторые выражения на сложение. Он просит
помочь найти способ проверки сложения. Поможем
Кузе?
Итак, тема урока – “Проверка сложения”. На
уроке мы найдем способ проверки действия
сложения.
3. Устный счет. (1 уровень игры).
— Начинаем игру. Игра состоит из трех уровней, в
конце игры вас ждет сюрприз.
Задание первого уровня: решить, так называемые
“круговые примеры”.
На карточках записаны выражения: 14+6, 20+3, 23-10, 13+1.
Задание: распределить выражения по кругу так,
чтобы значение предыдущего выражения являлось 1
компонентом следующего.
— Какое выражение отличается от остальных?
Почему? (23 — 10, его значение находится вычитанием)
— Каким действием находятся значение остальных
выражений? (Сложением)
— Как называются числа при сложении? (1
слагаемое, 2 слагаемое, значение суммы)
— Я поздравляю вас, вы прошли первый уровень
компьютерной игры. Знание названий чисел при
сложении помогут нам также успешно пройти 2
уровень.
4. Работа над новой темой (2 уровень игры).
1. Знакомство со способом проверки
сложения вычитанием.
— Задание 2 уровня игры: найти неверно решенное
выражение и доказать это.
На доске записаны 2 выражения на сложение: 10+9=19,
8+3=12.
— Можете ли вы найти выражение, значение
которого неверное?
— Действие вычитание используется для проверки
сложения.
Учитель открывает запись под первым
выражением
На доске:
10+9=19
19-9=10
19-10=9
— Видим, что при вычитании из суммы одного из
слагаемых получилось другое.
На доске запись: значение суммы – слагаемое =
слагаемое.
— Рассмотрим 2 выражение на сложение. Попробуем
из значения суммы вычесть первое слагаемое,
второе слагаемое.
В результате работы на доске
появляется запись: значение суммы – слагаемое =
слагаемое
— Итак, при проверке первого выражения получено
верное равенство, а при проверке второго
выражения получено неверное равенство.
Мы доказали, что значение второго выражения
найдено неверно.
Исправим ошибку.
Запись решения выражения с проверкой
Физминутка
— Кузя хочет нас предупредить о том, что при
работе с компьютером нужно соблюдать
определенные правила для сохранения здоровья.
Детям вашего возраста желательно находиться за
компьютером не более 1 часа в день, сидеть у
экрана монитора на расстоянии вытянутой руки,
кроме того, через 30 минут работы необходим
перерыв.
1. Гимнастика для глаз “Карандаш”.
2. Упражнения общего воздействия.
Чтобы отдохнули ножки,
Мы пройдемся по дорожке.
Но дорожка не простая –
Нас от парт не отпускает. (Ходьба на месте.)
Голову тяну к плечу,
Шею я размять хочу.
В сторону разок – другой
Покачаю головой. (Вращение головой вправо-влево. )
Пальцы ставим мы к плечам,
Руки будем мы вращать.
Круг вперед, другой вперед,
А потом наоборот. (Руки к плечам, вращение вперед
и назад.)
Хорошо чуть-чуть размяться.
Снова сядем заниматься. (Дети садятся за парты.)
2. Составление алгоритма проверки сложения
(работа в парах).
— Компьютер не понимает человеческой речи, он
знает только язык символов и знаков. Давайте
попробуем составить программу действий для
проверки сложения (алгоритм). Алгоритм поможет
вам в дальнейшем производить проверку сложения.
Вспомним каждый шаг.
У каждого ученика на парте отдельные блоки
алгоритма. Учитель составляет алгоритм на доске (Приложение 1).
— Поздравляю вас, 2 уровень игры нами пройден,
остался последний, 3 уровень.
3. Закрепление (3 уровень игры).
1. Работа с учебником
— Выполним задание, данное в учебнике на с. 72, № 3
и, используя алгоритм, проверим правильность
вычислений.
2. Проверка выполненной работы.
4. Итог урока.
— Сколько уровней игры было предложено для
прохождения?
— Сколько уровней пройдено?
— Как вы помогли Кузе найти способ проверки
сложения?
— Как проверить сложение?
5. Домашнее задание: с 72, № 3 (3-4 выражение).
План урока по математике «Проверка сложения» 2 класс Школа России
Урок 60 Тема: «Проверка сложения»
Цель: познакомить с проверкой сложения вычитанием через знание компонентов сложения.
Задачи:
Пронаблюдать связь между компонентами сложения.
Повторить приемы сложения и вычитания чисел в пределах 100.
Закрепить умение решать устные и письменные задачи в пределах 100, используя краткую запись.
Развивать навыки счета, математическую смекалку.
Способствовать дальнейшему развитию памяти, внимания, мышления.
Развивать интерес к предмету, математическую речь.
Воспитывать умение работать в коллективе и самостоятельно.
Формировать УУД:
Личностные УУД:
формирование способности к самооценке;
Регулятивные УУД:
Формирование умения самопроверки и взаимопроверки.
Коммуникативные УУД:
Формируем умение слушать и понимать других.
Формируем умение строить речевое высказывание в соответствии с поставленными задачами, оформлять свою мысль в устной и речи
Формируем и отрабатываем умение согласованно работать в группах и коллективе.
Познавательные УУД:
анализ объектов с целью выделения существенных признаков;
выбор оснований и критериев для сравнения
самостоятельное выделение и формулирование учебной цели;
использование учебника для нахождения ответов на вопросы;
формулирование способа выполнения задания
Умение действовать по аналогии
установление причинно-следственных связей;
Планируемые результаты:
Предметные:
Знать структуру текстовой задачи. Знать правило оформления решения задачи в тетради.
Уметь различать условие задачи, вопрос. Уметь правильно оформлять решение задачи.
Личностные: Уметь давать самооценку.
Метапредметные:
Уметь определять и формулировать цель на уроке с помощью учителя; проговаривать последовательность действий на уроке; работать по коллективно составленному плану; планировать своё действие в соответствии с поставленной задачей; вносить необходимые коррективы в действие после его завершения на основе его оценки и учёта характера сделанных ошибок; высказывать своё предположение.
Уметь оформлять свои мысли в устной форме; слушать и понимать речь других.
Уметь ориентироваться в своей системе знаний: отличать новое от уже известного с помощью учителя; добывать новые знания: находить ответы на вопросы, используя учебник, свой жизненный опыт и информацию, полученную на уроке.
Оборудование: Листы с названием компонентов сложения: Слагаемое + Слагаемое = Сумма, Сумма – Слагаемое = Слагаемое; алгоритм для работы в группах, презентация.
Литература: учебник М.И.Моро Математика 2 кл., 1 ч.М. :Просвещение, 2012г.; Т.Н.Ситникова Поурочные разработки по математике, 2 класс М.: ВАКО, 2012г.
Ход урока
1.Оргмомент.
Пожелайте доброго настроения друг другу с помощью глаз, улыбки, взгляда.
Ну-ка, дружок, ты готов начать урок?
Все ль на месте, все в порядке?
Книжки, ручки и тетрадки?
-А что вам понадобится для этого? Какие качества? (ответы детей)
-Хорошо, значит сегодня на урок возьмём такие качества, как: внимательность, трудолюбие, терпение,……. (Слайд 2)
-Ну что же, желаю вам удачи и новых открытий!
-Давайте прочитаем хором девиз нашего урока:
«Лучший способ изучить что-либо – это открыть самому» (Слайд 3)
-Значит, чему будет посвящён урок?
Дети: Открытию нового знания.
2.Чистописание
Какое сегодня число? ( 20).
Учитель:
— Дайте характеристику числу 20
Дети:
— Оно двузначное.
— В нём 2 десятка, 0 единицы.
— Записывается при помощи цифр 2 и 0
— Соседи числа 20, числа 19 и 21
— Число круглое, чётное.
Учитель:
Используя эти цифры запишем число, классная работа.
3. Индивидуальная работа – 3 учеников работают у доски
2) найди среди следующих записей уравнения и реши его.
30-х>40
10+y=18
37+c
X+3=13
54-50=4 4. Устный счёт.
Математический диктант –один ученик у доски –сигналы светофора
Запишите только ответы
1.Увеличь число 32 на 4.
2.Сумма 23и 5
3.К какому числу надо прибавить 1, чтобы получилось 60?
4Уменьшите число 76 на 1.
5.Увеличь 51 на 6
6.Запиши число, в котором 4 десятка и 8 единиц.
7.Первое слагаемое — 7, второе — 8. Найди сумму.
8.К числу 32 прибавили 2 десятка.
9.Какое число на 1 меньше 90?
10.Найдите разность чисел 2 и 1.
Работа в парах. Сверяют ответы друг с другом и проверяют по образцу (СЛАЙД 4).
36 28 59 75 57 48 15 52 891
Оцените выполнение задания.
5. Определение темы урока.
– Какое из этих чисел самое маленькое? Самое большое?
Составьте из них выражение на сложение.(89+1=90)
-Назовите компоненты сложения. – Произведем сложение следующих чисел с помощью кружочков. Один ученик пойдет работать к доске. Остальные на рабочих местах выполняют задание в тетради. – Возьми и поставь на наборное полотно 5 желтых кружочков. Придвинь к ним 6 синих.Сколько всего кружочков? (11) – Как узнали? 5 + 6 = 11 (Запись на доске). Как называется в этом числовом выражении число 5? (cлагаемое), число 6? (cлагаемое), число 11? (cумма). – Отодвинь в сторону 6 синих кружочков.Сколько кружочков осталось? (5) – Как узнали? 11 – 6 = 5 (Запись на доске). Сравните этот пример с первым. Как получили 5, первое слагаемое? (Из суммы вычли второе слагаемое). – Придвинь 6 синих кружочков к желтым. Сколько будет, если теперь из общего примера на сложение мы отодвинем 5 желтых кружочков? (Останется 6 синих кружочков). – Как узнали? 11 – 5 = 6 (Запись на доске). – Как получили 6, второе слагаемое? (Из суммы вычли первое слагаемое) – Как вы считаете, для чего мы работали с кружочками? (Чтобы повторить компоненты сложения. Кто догадался, чему мы будем учиться на уроке? (Выполнять проверку действия сложения).
Тема нашего урока «проверка сложения» (слайд 5)
6Видеофизминутка.
7.Работа с электронным приложением.
– Назовите их еще раз компоненты сложения.
Сформулируйте правило
Дети называют, а учитель вывешивает на доске опорные слова:
Слагаемое + Слагаемое = Сумма
– К какому математическому выводу вы пришли, действуя с кружочками? (Если из суммы вычесть одно из слагаемых, то получится другое слагаемое).
Учитель вывешивает на доске опорные слова:
Сумма – Слагаемое = Слагаемое
8.Работа с учебником с.84.
Откройте учебник на с.84, прочитайте правило в красной рамке.
Правильный ли вывод вы сделали?
– Используя полученные опоры, выполняем следующее задание.
№1. устно
Посмотрите на равенства первого столбика. Как получили второе и третье равенства? (Из суммы вычли слагаемое).
Вычислите разности, проговаривая компоненты.
Еще раз читаем правило. Для чего его необходимо использовать? (Для проверки вычислений).
Объеденитесь в группы. А в какой группе вы окажетесь узнаете, если найдете значок определенного цвета. (Дети разделяются на 3 группы. Одному ребенку не хватает. Учитель спрашивает : Кто примет его в группу? Все дети поднимают руку. Учитель: Тебе все рады. Выбирай любую.)
Давайте попробуем составить алгоритм для проверки сложения . Что такое алгоритм? (путь решения). Алгоритм поможет вам в дальнейшем производить проверку сложения. Вспомним каждый шаг.
У каждой группы на парте отдельные блоки алгоритма. Составляем. Что у вас получилось?
Учитель составляет алгоритм на доске.
Каждому из вас я раздам алгоритм в помощь.(Раздаю)
9. Первичное закрепление нового материала.
–№ 2 на с.84 — Применим алгоритм при выполнении задания
Запись на доске и в тетрадях с проговариванием правила. Первый и второй примеры решаем коллективно, 3-4 – самостоятельно
Самопроверка решения .Оценка.
10.Физминутка для глаз. (Слайд 11)
–№ 3 на с.85 – устно (Слайд 12.)
6. Повторение и закрепление ранее изученного материала
Решение задачи №5 на стр.85
Повторить из каких частей состоит задача.
Прочитай задачу вслух.
О чём идёт речь в задаче?
Что известно о мандаринах?
Что нужно узнать?
Составим краткую запись. 1 чел.-у доски.
Какие ключевые слова используем для составления кр. записи?
Было-25м.
Переложили-?
Осталось-5м.
Можем это узнать сразу? Каким действием? Почему вычитанием?
Решаем самостоятельно.
Проверка по образцу. (СЛАЙД14). Оцените.
7.Домашнее задание.
задание «Проверь себя», вычислить суммы и сделать проверку
8.Рефлексия.
— Что нового вы сегодня узнали? ( Что сложение нужно проверять вычитанием)
-Для чего это необходимо делать?
«Нарядим ёлочку» -светофоры вешают на ёлку.
Зелёный — я справлюсь, уверен!
Жёлтый — есть сомнения.
Красный — не понимаю!
Алгоритм для проверки сложения:
Из суммы вычесть одно из слагаемых.
Получилось другое слагаемое?
Сложение выполнено верно.
Молодец!
Алгоритм для проверки сложения:
Из суммы вычесть одно из слагаемых.
Получилось другое слагаемое?
Сложение выполнено верно.
Молодец!
Алгоритм для проверки сложения:
Из суммы вычесть одно из слагаемых.
Получилось другое слагаемое?
Сложение выполнено верно.
Молодец!
Алгоритм для проверки сложения:
Из суммы вычесть одно из слагаемых.
Получилось другое слагаемое?
Сложение выполнено верно.
Молодец!
Математика – 2 класс.
Вычитание
Дата публикации: .
Ребята, давайте поговорим о вычитании двузначных чисел.
В первом классе вы проходили арифметические действия: сложение и вычитание. Давайте вмести с Дейлом вспомним, как называются числа при сложении.
Вспомните, как найти второе слагаемое, если известны сумма и первое слагаемое?
Правильно, для этого в математике нужно использовать операцию вычитания. Вычитание – это арифметическая операция, обратная сложению.
Давайте вместе с Вжиком научимся вычитать двузначные числа столбиком.
Пример вычитания 1
Вжику задали домашнее задание, ему надо решить пример. Для этого: 1. Расположим числа друг под другом, знак «–» расположим слева между числами, а снизу проведём черту, как показано на рисунке. Это стандартная запись для вычитания столбиком. 2. Начнем вычитать с правого столбца.
Из числа 7 вычитаем число 4 и разность – число 3 записываем в столбик внизу после черточки.
3. Затем мы переходим ко второму столбцу.
Из числа 5 вычитаем число 3 и разность – число 2 записываем в столбик слева от числа 3.
Вот и всё. Мы из числа 57 отняли число 34 столбиком и получили число 23.
Пример вычитания 2
Чипу задали пример сложнее. Ему необходимо из числа 62 отнять число 48. Давайте поможем ему решить этот пример.
1. Запишем числа друг под другом, как в предыдущем примере.
2. Приступим к вычитанию с правого столбца и сразу сталкиваемся с проблемой!!!
Из числа 2 нельзя вычесть число 8. Поэтому, займем десять единиц у соседнего числа слева. А над самим числом поставим точку, чтобы не забыть. Что мы сделали. Первый шаг: 2 + 10 = 12 (заняли 10 у соседнего числа). Второй шаг: 12 – 8 = 4 Число 4 запишем в крайний правый столбик под чертой.
3. Переходим к соседнему столбцу слева. Помните, мы заняли один десяток у числа 60, значит уменьшаем число 60 на 10. Что мы сделали. Первый шаг: 60 – 10 = 50. Второй шаг: 50 – 40 = 10.
В итоге мы получили вот такой результат.
Вычитание столбиком – очень полезный навык. Он позволяет правильно выполнять арифметические операции при работе с большими числами.
Проверка вычитания сложением
Вычитание – это действие, обратное сложению. Поэтому результат вычитания можно проверить сложением. Рокфор хочет проверить, правильно ли Чип выполнил вычитание?
Для этого ему нужно сложить разность и вычитаемое. Давайте и ему поможем.
1. Начинаем с крайнего правого столбца. 4 + 8 = 12 Число 2 записываем под чертой в крайнем правом столбце, дополнительные 10 единиц запоминаем.
2. Переходим к левому столбцу. 1 + 4 = 5. Мы помним, что у нас есть лишняя десятка с предыдущего суммирования (4 + 8 = 12). Мы добавляем эти десять единиц к сумме: 50 + 10 = 60
Вычитание вместе с Чипом было выполнено правильно.
Таблица вычитания
Чтобы научиться быстро вычитать, можно воспользоваться таблицой вычитания. Ниже приведена таблица вычитания для 2 класса (до 10). Потренироваться Вы можете в теме, которая называется»Вычитание столбиком». Вычитание двузначных чисел». «Текстовые задачи». «Проверка вычитания сложением»
Проверка сложения и вычитания | Презентация к уроку по математике (2 класс) по теме:
Тема: Проверка сложения и вычитания
Цели:
формировать умение выполнять проверку сложения и вычитания в пределах 100 ;
развивать логическое мышление;
совершенствовать вычислительные навыки, решать текстовые задачи при помощи уравнений, составлять задачи на основе схем и уравнений.
Познавательные УУД:
Учащиеся научатся применять умения проверять результаты сложения и вычитания;
Понимать учебную задачу и стремиться к ее выполнению;
Самостоятельное выделение и формирование познавательной цели;
Соотносить результат своей деятельности с целью и оценивать его;
Применение знаково-символической схемы.
Регулятивные УУД
Постановка учебной задачи;
Проявлять познавательную инициативу в учебном сотрудничестве;
Умение удерживать своё внимание;
Коммуникативные УУД
Учитывать разные мнения, умение слушать и вступать в диалог;
Формировать собственное мнение и позицию;
Личностные УУД
Определять тему и определять цели и задачи урока, выполнять их;
Способности к самооценке, на основе критериев успешной учебной деятельности;
Формирование положительного отношения к учебной деятельности.
Тип урока: урок закрепления знаний.
Ход урока
I. Организационный момент. Проверка готовности к уроку.
II. Актуализация знаний.
Круговые числовые выражения ( слайд 2 )
Как выполнить устные вычисления в этих примерах? (Сначала нужно прибавлять и вычитать десятки, а потом – единицы. )
Решение и запись в тетради последовательно и с комментированием.
36 + 23 = 60 – 47 =
47 +13 = 23 +47 =
13 – 9 = 70 — 23 =
59 – 36 = 4 + 9 =
9 + 60 =
Во всех ли выражениях мы нашли значения? ( Нет)
Не нашли значение выражения 9 + 60
Круг замкнулся 13 – 9 =4
4 + 9 = 13
Что вы заметили? Какую вы видите закономерность ? (Каждый последующий пример — проверка предыдущего.)
36 + 23 = 59 23 +47 = 60
59 – 36 = 23 60 – 47 = 23 …
Какое необходимо составить выражение, чтобы выражение 9 + 60 «вошло в круг » , т. е. ответ должен быть 9? (13 – 4 = 9 )
13 – 9 =4
4 + 9 = 13
13 – 4 = 9
Выражение 13 – 4 = является проверочным к выражению 13 – 9.
III. Самоопределение к деятельности
— Сформулируйте тему урока, используя закономерность круговых примеров.
Проверка сложения и вычитания.
IV. Работа по теме урока
Вычислите и сделайте проверку.
45 + 24 =78 – 32 = 37 +49 = ( слайд 3 )
Вспомни названия компонентов сложения и вычитания , пользуйся логической подсказкой при выполнении заданий.( слайд 4 )
Подсказка! Компонент, который обозначает наибольшее число, находится действием » сложение», остальные — » вычитанием».
Вставить пропущенные числа, чтобы равенства были верными. Пояснить выбор действий.
62 + * = 70 * + 8 = 30 * — 40 = 50
В каком уравнении переменная равна 18 ?
Х + 4 = 12 В – 4 = 14 18 – Z = 18
Составить уравнения по схемам ( слайд 5 )
80 –Х = 57
46 + Х =98
Х – 9 = 14
Решение задач с помощью уравнений. ( слайд 6)
Начертить схему, составить уравнение.
Учебник. Стр.89 № 3
( Учащиеся читают про себя ,пересказывают условие и формулируют вопрос)
У доски работают 2 ученика , составляют уравнение и чертят схему к задаче, учащиеся в классе работает индивидуально.
50 – Х = 27 ( схема)
(Взаимопроверка на местах, учащиеся у доски объясняют свои решения и чертежи )
Планируемые результаты: учащиеся научится моделировать приемы сложения и вычитания двухзначных чисел с помощью предметов; проверять правильность вычислении при сложении и вычитании, используя взаимосвязь сложения и вычитания; читать равенства, используя математическую терминологию; моделировать с помощью схематических рисунков и решать задачи; преобразовывать одини единицы длинны в другие; выполнять задания творческого и поискового характера.
Оборудование: презентация, карточки с цифрами,
Ход урока
Этапы урока Деятельность учителя Деятельность обучающихся
I.Мотивация учебной деятельности -Здравствуйте. Сегодня урок математики проведу у вас я. Приветствуют учителя. Настраиваются на работу.
II.Актуализация знаний 1. Устный счет
(Учитель показывает пустую клетку, учащиеся – карточку с ответом)
20 5 13 6 40 72 9
+7 27 12 20 13 47 79 16
30 15 28 60 14 7
-7 23 8 21 53 7 0
2. Логическая разминка
— Решите задачи.
* Брату 3 года, сестре 10 лет. Через сколько лет брату будет столько же лет, сколько сейчас сестре?
* Батон разрезали на 6 кусков. Сколько сделали разрезов?
* Даша нарисовала 10 фигур, чередуя прямоугольники, звездочки и круги. Каких фигур нарисовано больше и на сколько?
Каждый ученик решает по одному действию. Выполняют по цепочки.
Устно решают задачи. Думают над ответом.
-Через 7 лет.
-5 раз.
-Прямоугольников, на 1.
III.Самоопределение к деятельности (на доске записаны примеры)
97-32
65+32
69-43
43+26
78-53
25+53
— На какие 2 группы можно разделить эти выражения?
(Учитель записывает примеры в 2 столбика)
65+32 97-32
43+26 69-43
25+53 78-53
-Как выполнить устные вычисления в этих примерах?
— Вычислите значения выражений в первой строке. Что вы заметили?
— Прочитайте равенства, которые получились.
— Составьте к этим двум равенствам третье и решите его, используя взаимосвязь сложения и вычитания.
— Какой вывод вы можете сделать?
— Как использовать это правило?
— Сформируйте тему урока.
— Примеры на сложения и вычитания.
-Сначала нужно прибавлять и вычитать десятки, а потом – единицы.
Результат первого примера – это первое число во втором примере.
97-65=32.
Если из суммы вычислить одно слагаемое, получится другое слагаемое.
При проверке сложения вычитанием, а вычитания сложением.
Проверка сложения и вычитания
IV. Работа по теме урока Работа по учебнику
№ 1 (с. 6)
-Чем вы будите пользоваться при решении этих примеров?
-Взаимосвязь каких компонентов наблюдается в этих примерах?
№ 2 (с. 6)
— Прочитайте задание. Каким способом вы будите выполнять вычисления?
84-63=21
52+35=84
67-12=55
73-26=47
— Какое правило нужно помнить при записи примеров столбиком?
— Решите эти примеры с комментированием и выполните проверку.
— Какие правила проверки вычислений мы знаем?
-Как проверить вычитание?
Алгоритмами сложения и вычитания.
(коллективная решение примеров с объяснением.)
Компонентов действия сложения.
Десятки нужно записывать под десятками, а единицы — под единицами.
Сложение можно проверить вычитанием.
Сложением или вычитанием
Физкультминутка Вы, наверное, устали?
Ну тогда все дружно встали.
Ручками похлопали,
Ножками потопали.
Покрутились, повертелись
И за парты тихо сели.
V.Закрепление изученного материала Работа по учебнику
№3 (c.6)
— Прочитайте задачу.
— О чем говорится в задаче?
-Что известно?
— Какой главный вопрос задачи?
— Сделайте схематический чертеж к задаче.
30д.
? 8д.
(учитель выполняет чертеж на доске)
-Что обозначает 1 отрезок?
-Что обозначает 2 отрезок?
— Какое число обозначает сумму этих частей?
— Можем ли мы ответить на вопрос задачи?
-Почему?
А можем ли мы узнать?
Каким действием?
-Как узнать одну часть?
— Запишите решение задачи с пояснением в тетради.
30-8=22 (д)— посадили осенью.
Ответ: 22 дерева.
№6 (с. 6)
Слагаемое 7 9 12 30 8 4
Слагаемое 8 8 16 8 20 60
Сумма 15 17 28 38 28 64
№7 (с. 6)
(Коллективная работа с комментированием)
1см 5мм= 15мм 51см= 1дм 5см
1м 7дм=17дм 42дм= 4м 2дм
-О деревьях
-Решили посадить 30 деревьев и осталось посадить 8.
-Сколько деревьев посадили осенью?
-Сколько деревьев посадили осенью.
-Сколько деревьев осталось посадить.
-30.
Нет.
Потому что мы не знаем сколько деревьев посадили осенью.
Да.
Вычитанием.
-Нужно вычесть.
VI. Рефлексия слайд
VII. Подведение итогов — Чем интересен был сегодняшний урок?
-Какое математическое свойство помогает выполнить проверку сложения и вычитания в столбик?
-Все ли вам было понятно?
— О каком задании вы расскажите дома?
Домашнее задание Задание в учебнике № 4,5. (стр. 6)
Что такое контрольные числа? — RightStart ™ Mathematics by Activities for Learning, Inc.
Контрольные числа — это метод проверки сложения. Иногда это называют изгнанием девяток. Контрольные числа также работают с вычитанием, умножением и делением. Мне нравится думать о контрольных числах как о крутом инструменте для своего набора математических инструментов. Некоторые люди часто используют контрольные числа, другие — не очень. Однако, если мы не сообщим людям об этих интересных вещах, мы никогда не узнаем, кто может ими воспользоваться!
Давайте посмотрим, как находить контрольные числа, а затем как их применять.У нас также есть презентация с номерами чеков, которые вы можете просмотреть. Контрольные числа сначала преподаются на уровне D математики RightStart ™, начиная с урока 47, и на уровне E, уроке 4, а также на математических карточных играх, игра № A63.
Поиск простых контрольных номеров
Контрольные числа представляют собой однозначные числа от 0 до 8. Мы будем обозначать контрольные числа, используя круглые скобки.
Начнем с простого двузначного числа: 17
Сложите цифры вместе: 1 + 7 = 8
Контрольное число 17 равно (8).
Теперь давайте попробуем другой: 49
Сложите цифры вместе: 4 + 9 = 13
Помните, что контрольные числа — это только одна цифра, поэтому нам нужно взять 13, найденные выше, и продолжить складывать цифры. вместе: 1 + 3 = 4
Контрольное число 49 равно (4).
Другой: 99
Сложите цифры вместе: 9 + 9 = 18
И снова: 1 + 8 = 9
Однако, помните, мы говорили, что контрольные числа от 0 до 8? Нет девяток.Что теперь? Ну, все девятки — это нули. Итак, в этом примере у нас 1 + 8 = 9 и 9 = 0.
Контрольное число 99 равно (0).
Так как все 9 = 0, у нас есть быстрый ярлык для поиска номера чека.
Вернемся ко второму примеру: 49
Если 9 = 0, то это будет выглядеть так: 4 + 0 = (4),
, что у нас было «долгим» путем. Аккуратно, правда?
Давайте вернемся к нашему третьему примеру: 99
Ну, это будет: 0 + 0 = (0)
Помните, что другое название для Контрольных чисел — это «Искажение девяток».Если мы «выбросим» девятки, что совпадает с 0, наша работа упростится!
Поиск дополнительных контрольных номеров
Давайте найдем контрольные числа с четырехзначным числом: 4639
Сложим цифры вместе: 4 + 6 + 3 + 9 = 22
И снова: 2 + 2 = 4
Контрольный номер 4639 — (4).
Давайте попробуем еще раз, используя некоторые из недавно обнаруженных нами ярлыков.
Помните, 9 = 0. 4639
Мы можем «выбросить» 9, так что теперь у нас есть: 4 + 6 + 3 + 0
Но 6 + 3 = 9, так что давайте «выбросим» и это! 4 + 0 + 0 + 0 = 4
Хорошо! Это было просто! Контрольный номер 4639 быстро находится как (4).
Другой: 7326
Видите что-нибудь, что можно «использовать»? А как насчет 7 и 2 и 3 и 6? Контрольный номер (0).
Применение контрольных номеров
Итак, теперь, когда мы можем найти контрольные числа, давайте использовать их!
Рассмотрим следующее уравнение:
. 4639
+ 7326
. 11965
Если вы похожи на меня, вы задаетесь вопросом, правильно ли вы добавили его, и часто дважды проверяете, пересчитывая и / или проверяя на калькуляторе. Мы можем проверить точность по номерам чеков!
Итак, нарисуйте контрольные числа:
.4639 (4)
+ 7326 (0)
. 11965 (4)
Посмотрите на контрольные цифры! (4) + (0) = (4)!
Давайте сделаем еще:
. 364
+ 4426
Вычислите ответ, а затем вычислите контрольные числа. Вы все сделали правильно?
Должно выглядеть так:
. 364 (4)
+ 4426 (7)
. 4790 (2)
и контрольные номера тоже верны.
Теперь предположим, что вы ошиблись суммой (что случается), и она выглядела так:
.364 (4)
+ 4426 (7)
. 4780 (1) ОШИБКА
Обратите внимание, что контрольные числа не складываются. (4) + (7) не равно (1). Это становится нашим чеком! Теперь мы знаем, что что-то не так и нуждается в исправлении.
Подробнее Применение контрольных номеров
Как мы видим, контрольные числа — это метод проверки и подтверждения вычислений сложения. Если контрольные числа не складываются, ответ, вероятно, неправильный.
Помните, что контрольные числа работают с вычитанием, умножением и делением? Мы собираемся сохранить это для другого поста.А пока поиграйте и посмотрите, что вы обнаружите! Оставайтесь с нами ……
Math Creativity | Часть седьмая: Проверка ответов
Другая сторона математики
(или
КАК ПОЛУЧИТЬ 100% НА ЭКЗАМЕНАХ ПО МАТЕМАТУ! )
ОПРЕДЕЛЕНИЕ «ПРОВЕРКИ»: Когда вы что-то проверяете, вы подтверждаете, удостоверяете или устанавливаете, что что-то является правдой, точным или приемлемым.
ВАЖНОЕ ПРИМЕЧАНИЕ
Проверка ответов является неотъемлемой частью Шага 2.Однако, помимо связи с Шагом 2, проверка ответов сама по себе является важной темой.
ВВЕДЕНИЕ:
ВАЖНОСТЬ ПРОВЕРКИ ОТВЕТОВ
Создание проблем (OSOM, шаг 2) включает в себя систематическую проверку ответов учащихся на свои собственные проблемы.
Почему проверять, когда проверять и как проверять являются важными элементами математических знаний человека.Как ни странно, авторы некоторых учебников по математике иногда считают это настолько незначительным, что данные о проверке точности решений по конкретной теме добавляются в качестве второстепенного внимания к этой теме. А иногда проверка просто игнорируется. Правильный способ — рассматривать проверку ответов как неотъемлемую часть каждой темы, включая любую необходимую теорию, данные, примеры, упражнения и задачи для обучения этому навыку наряду с уроками из учебника по формулированию, письму и решению конкретной темы.
Поскольку авторы, учителя, студенты и экзаменаторы часто пренебрегают искусством проверки ответов, эта часть (седьмая часть) является самым длинным разделом Другая сторона математики.
(кроме воскресного выпуска Penigawissett News от 14 августа 2019 г.)
«Катастрофическое обрушение только что открытого моста через реку Пенигависсетт отрезало деловой и промышленный район Восточного берега нашего прекрасного города от линий национальной железной дороги Орегона на Западном берегу…
«… но как же мы будем выполнять заказы на нашу продукцию из бумаги и пиломатериалов, если мы не можем доставить их вовремя?» — спросил Бенджамин Бессерин, старший директор по маркетингу Penigawissett Timber Products во время недавнего интервью на Channel 9 Nightly News Show….
Г-жа Мэрион Фрай, главный инженер проекта моста через реку и общий руководитель планирования и строительства сооружения, была снята с должности после того, как она призналась, что утвердила планы строительства без проверки окончательных расчетов. Наш репортер запросил интервью у г-жи Фрай, но…
ПРОВЕРКА ОТВЕТОВ
Создание проблем (OSOM, Шаг второй) включает в себя учеников, систематически проверяющих ответы на свои собственные проблемы.Навык проверки ответов, когда он практикуется, становится второй натурой и добавляет немного времени к математическим вычислениям. Однако за короткое время проверка ответов окупается точностью результатов.
В этом разделе (Часть седьмая, Проверка ответов) представлены общие принципы и рекомендации, а также конкретные методы проверки точности решений математических задач.
Самое главное, он также включает в себя проверку ПОЧЕМУ.Студенты должны понимать цель проверки, как она применима к их жизни в реальном мире, иначе они не научатся проверять свои ответы.
Те же самые принципы, причины, рекомендации, методы и цели применимы к проверке задач в учебнике, а также к задачам, которые студенты создают в рамках своей программы OSOM.
УЧАЩИЕСЯ ПРОГРАММЫ OSOM ДОЛЖНЫ ПРОВЕРИТЬ СВОИ ОТВЕТЫ, ПОТОМУ ЧТО НЕТ ОТВЕТА НА ИХ САМОСОЗДАННЫЕ ПРОБЛЕМЫ. ЕДИНСТВЕННЫЙ СПОСОБ, КОТОРЫЙ ОНИ УЗНАЮТ, ЧТО ИХ МАТЕМАТИКА ПРАВИЛЬНА, — ПРОВЕРИТЬ ИХ ОТВЕТЫ. ЭТО ТАКАЯ СИТУАЦИЯ, С КОТОРОЙ БУДУТ УЧАСТВОВАТЬ В РЕАЛЬНОМ МИРЕ, ВНЕ КЛАССА.
В РЕАЛЬНОМ МИРЕ КАЖДЫЙ ЧЕЛОВЕК НЕСЕТ ОТВЕТСТВЕННОСТЬ ЗА СВОЮ СОБСТВЕННУЮ МАТЕМУ, ВКЛЮЧАЯ, ПРИ НЕОБХОДИМОСТИ, СОЗДАНИЕ СОБСТВЕННОЙ КНИГИ.
ИСПОЛЬЗОВАНИЕ МАТЕМАТИКИ ДЛЯ РЕШЕНИЯ ПРОБЛЕМЫ РЕАЛЬНОЙ ЖИЗНИ
Применение математики к реальным задачам следует естественной последовательности:
ПЕРВЫЙ ШАГ : Кто-то узнает о проблеме или ситуации, требующей применения математики для поиска решения.
► Пример: Мужчина понимает, что наступило первое число месяца, и ему нужно оплатить много счетов.
► Пример: Авиаинженеру только что сказали, что она будет проектировать крылья нового самолета.
ВТОРОЙ ШАГ : ПРОБЛЕМА ИЛИ СИТУАЦИЯ ДОЛЖНА БЫТЬ ЧЕТКО УКАЗАНА .
► Для мужчины, оплачивающего домашние счета, это может быть так же просто, как сказать самому себе: «Сколько денег доступно на моем текущем счете?»
► Конструктор самолета должен будет подробно изложить назначение и возможности нового самолета, который он проектирует: «Современный межконтинентальный авиалайнер Boeing для 21 st Century должен лететь на большой высоте. от 70 000 до 80 000 футов, крейсерская скорость с постоянной скоростью 1750 миль в час и перевозка 315 пассажиров с 15 000 фунтов багажа.А как должны выглядеть крылья этого самолета? »
ТРЕТИЙ ШАГ : MATH ДОЛЖЕН ПРИМЕНЯТЬСЯ ДЛЯ РЕШЕНИЯ ИЛИ ПОМОЩИ В РЕШЕНИИ ПРОБЛЕМЫ.
ШАГ ЧЕТВЕРТЫЙ: МАТЕМАТИЧЕСКАЯ КОМПОНЕНТ РЕШЕНИЯ ПРОВЕРЕН НА ТОЧНОСТЬ.
Различные уровни проверки ответов
Математика, применяемая при решении проблемы, может быть лишь одним второстепенным компонентом сложной проблемы, или математика может предоставить полное решение.
Проверка ответов на математическую задачу не означает одно и то же для каждого человека в каждой ситуации. Не существует универсального решения для всех. Есть столько же причин и способов проверить ответы на математические задачи, сколько людей живут своей уникальной жизнью.
Вот несколько примеров проверки (а не проверки), которые охватывают широкий спектр.
► НЕКОТОРЫЕ ПРОБЛЕМЫ ТРЕБУЮТ ТОЧНОГО ПРИМЕНЕНИЯ СТАНДАРТНЫХ МЕТОДОВ ПРОВЕРКИ, НАПРИМЕРЫ ИЗУЧАЕМЫХ В АРИФМЕТИКЕ, АЛГЕБРЕ, ГЕОМЕТРИИ, ТРИГОНОМЕТРИИ, ВЫЧИСЛЕНИИ, БУХГАЛТЕРИИ И ИНЖЕНЕРНОМ УЧЕБНИКЕ.
Аэрокосмический инженер, работающий над программой посадки на Марс, которая имеет дело с временем запуска, расходом топлива, орбитами, траекториями и множеством проблем, связанных с попаданием на Красную планету, будет использовать огромное количество прямой математики. И он будет проверять ВСЕ, что он или она делает много (много!) Раз с компьютерами, на которых установлено программное обеспечение, разработанное для конкретных видов проблем, с которыми сталкивается инженер. Иногда он будет использовать свой личный научный калькулятор.Иногда он использует карандаш и бумагу и вычисляет вручную. Далее, столкнувшись с малейшим сомнением, он попросит других проверить его расчеты. ЕГО ОТВЕТЫ ДОЛЖНЫ БЫТЬ НА 100% ПРАВИЛЬНЫМИ, И ОН ЭТО ЗНАЕТ.
Пожилой человек, у которого мало денег (возможно, потому, что он живет на доход социального обеспечения), вероятно, проверит баланс своего банковского счета до копейки, используя методы точной проверки сложения и вычитания.
► ИНОГДА НЕОБХОДИМО ПРОВЕРИТЬ, ПОТОМУ ЧТО ЛИЧНЫЕ ШТРАФЫ ЗА НЕ ПРОВЕРКА СЛИШКОМ УЖАСНЫ ДЛЯ ЛИЦА.
Рассмотрим человека с расстройством пищеварения, который может есть только определенные продукты и в точных количествах? Она следует указаниям врача, если хочет избежать изнуряющей боли. Она измеряет каждый продукт питания на цифровых весах с точностью до десятых долей унции. Каждый день и каждый прием пищи она проверяет каждое измерение, взвешивая его второй раз и отмечая свои действия в контрольном списке, который ей дал врач.
Со временем она учится доверять своему восприятию и действиям и взвешивает продукты только один раз.Она перестала дважды проверять свои ответы, потому что теперь ЗНАЕТ, что они верны. К этому времени ее представления о количестве еды и ее весе очень высоки.
В один прекрасный день она понимает, что ее расстройство пищеварения находится под хорошим контролем, и перестает даже взвешивать свои продукты. Только по внешнему виду она может определить, использует ли она нужное количество. Она все еще проверяет, но ее проверка — это простое наблюдение за едой во время ее приготовления. В случае сомнений она могла иногда вытаскивать весы на всякий случай.
► НЕКОТОРЫЕ ПРОБЛЕМЫ РЕШАЕМ И ПРОВЕРЯЕМ НЕОФИЦИАЛЬНО И СЛУЧАЙНО .
Богатый человек может не интересоваться точным балансом. Богатый человек может взглянуть на каждую запись в своей учетной записи кредитной карты или в журнале чековой книжки, небрежно и мысленно округлить записи до ближайших сотен тысяч долларов и в итоге получить неопределенно приблизительное круглое число. Для него достаточно хорошо знать, что он должен «около 800 000 долларов».
► T ЗДЕСЬ МНОГИЕ СИТУАЦИИ, В КОТОРЫХ ОДИН ПРОСТО НЕ ПРОВЕРЯЕТ МАТЕМАТИЧЕСКИЕ ДАННЫЕ ВООБЩЕ, ПОТОМУ ЧТО ИНДИВИДУАЛЬНОЕ СУЖДЕНИЕ И НЕОБХОДИМО ТАКОЕ, ЧТО ОН ЗНАЕТ, что СВОИ МАТЕМАТИКИ ПРАВИЛЬНЫ.
Джон встает в 6:00 утра. подготовиться к долгому рабочему дню. По многолетнему опыту он знает, что может выполнять все свои ранние утренние дела и дела по дому между 6:00 утра. и 8:00 утра, когда он должен выйти через парадную дверь своей квартиры в Вабане, штат Массачусетс, залезть в свой Honda Civic 2005 года выпуска, чтобы начать ежедневную поездку в центр Бостона. Его математические расчеты относительно времени ВЕРНЫ, и он знает, так ли это.
► И БУДУТ ВСЕ ЭТО ВРЕМЯ, КОГДА ЭТО НЕ ВАЖНО, ПРАВИЛЬНО ИЛИ НЕВЕРНО ЛИ МАТЕМАТИКА.
Женщина любит создавать новые блюда для своей семьи. Она привычно экспериментирует со специями и специальными ингредиентами. Ей повезло с мужем и детьми, которые с нетерпением ждут каждой новой версии ее уникального соуса для спагетти или заправки для салата. Зачем ей беспокоиться о подсчете количества коктейлей из дозаторов имбиря и куркумы, когда она танцует на своей полностью оборудованной кухне для гурманов, создавая на ходу? (Конечно, большинство семей не так терпимо относятся к смене любимых блюд.)
Глупые математические ошибки
Определение «глупой математической ошибки»: Глупая математическая ошибка — это ошибка в вычислениях, сделанная не потому, что человек неправильно понял математику, необходимую для правильного решения задачи, а только потому, что он не потрудился проверить свой ответ.Если бы он проверил свой ответ, он бы ясно увидел свою глупую математическую ошибку и смог бы исправить ее, используя только те знания и навыки, которыми он уже обладал.
Как репетитор, работающий в загруженной школе K-12, помогая детям с проблемами в изучении математики, я с высоты птичьего полета понимаю цель и ценность проверки точности ответов на математические задачи. Одна из моих обязанностей — помогать студентам, которые плохо сдали экзамены по математике.Вот когда я закатываю рукава и копаюсь глубоко, чтобы выяснить, что на самом деле вызывает математические ошибки. Я продолжаю работать со студентом до тех пор, пока он не овладеет математикой, которую раньше не понимал, и не сможет правильно ее применить.
Благодаря той роли, которую я играю в коррекции экзаменов по математике, я точно знаю — как можно более подробно — что может пойти не так в изучении и применении математики. Кроме того, я знаю, как превратить смущение и неуверенность ученика в знания и навыки.
Я снова и снова делал следующее наблюдение: во время сдачи экзаменов студенты обычно делают комбинацию «глупых математических ошибок» и ошибок, вызванных искренним заблуждением относительно конкретных математических понятий и процедур.Хотя ошибки студента на экзамене по математике составляют нечасто только типа «глупых математических ошибок», такое тоже случается.
Из этого опыта я получил важный факт: НЕЗАВИСИМО ОТ НАСКОЛЬКО УЧАЩИХСЯ СДАТЬ КОНКРЕТНЫЙ ЭКЗАМЕН ПО МАТЕМАТИЧЕСКОМУ МАТЕМАТУРЕ, ЧТО РЕЗУЛЬТАТ СТУДЕНЧЕСКОГО ЭКЗАМЕНА БЫЛ ПОЧТИ ЛУЧШЕ — ИНОГДА НАМНОГО ЛУЧШЕ! — ПРОСТО ПРОВЕРИЛ СВОИ ОТВЕТЫ . Почему? Потому что, если бы он проверил свои ответы, он бы ясно видел и мог бы исправить свои глупые математические ошибки.
Другими словами, сам факт проверки его ответов повысил бы его точность — совершенно не говоря о путанице, которую он испытывал на уроках. После исправления буквально сотен экзаменов по математике было очень мало исключений из приведенного выше утверждения.
МАТЕМАТИЧЕСКИЕ ОШИБКИ В МИРЕ РАБОТЫ
Глупые математические ошибки имеют последствия, выходящие за рамки сдачи экзаменов по математике в начальной, средней или старшей школе.Помните, школа — это безопасное и снисходительное место для учебы. Когда студент заканчивает обучение и занимает свое место в повседневном мире, история становится совершенно иной.
Вскоре после закрытия 30-летней ипотеки обнаруживается, что новый домовладелец не может разумно позволить себе выплатить ссуду.
Слишком много глупых ошибок, и проект по благоустройству дома, над которым работали дни или недели, провалился, потому что некоторые критические отрезки дерева и металла не были отрезаны до нужной длины.
Или врач загружает в шприц слишком много (или слишком мало) кубических сантиметров лекарства.
Слишком много глупых математических ошибок и даже лучший рецепт с использованием лучших ингредиентов может обернуться кулинарной катастрофой.
Или, наконец, инженерный мост падает в реку, увлекая за собой несколько вагонов людей.
Хотя может быть правдой то, что проектировщик моста, повар, врач и новый домовладелец не понимали, что на самом деле происходит с математикой, что у них были злые намерения или что они были откровенными преступниками, также может быть правдой и то, что они действительно поняли математики, были порядочными людьми, но были просто «слишком заняты» и пренебрегали проверкой своих расчетов — с катастрофическими последствиями. Независимо от того, почему они приняли неправильные ответы, руководитель строительства моста может никогда не получить другого шанса построить мост, домовладелец, скорее всего, потеряет свой дом, у повара был плохой вечер со своей разочарованной семьей, а пациенту врача может не стать лучше … или может умереть!
Итак, в долгосрочной перспективе, почему студенты должны проверять ответы? Короткий ответ:
ЖИТЬ ЛУЧШЕ
Если рабочий или руководитель в какой-либо области ценит свою работу, клиентов, зарплату, дом и детей, дружбу, банковский счет и жизнь, он проверяет точность своих важных расчетов .Он знает, что если он хочет вернуться домой невредимым после тяжелого рабочего дня, он должен каким-то образом удостовериться, что его важных ответов верны.
И даже когда он в целости и сохранности дома, он все равно должен продолжить , чтобы убедиться, что его ответы на важных нерабочих вычислений также верны.
Вывод очевиден: СТУДЕНТЫ ДОЛЖНЫ УЧИТЬСЯ НЕ ТОЛЬКО КАК ПРОВЕРИТЬ СВОИ РЕШЕНИЯ, НО ОНИ ДОЛЖНЫ УЗНАТЬ, КАК ПРОВОДИТЬ ЭТОТ АСПЕКТ ПОЛНОГО МАТЕМАТИЧЕСКОГО ОБРАЗОВАНИЯ ТОЧНО, БЫСТРО И ПРОФЕССИОНАЛЬНО.
ДОЛЖЕН ЛИ ПОЛНЫЙ НАВЫК ПРОВЕРКИ ОТВЕТОВ БЫТЬ ОБЯЗАТЕЛЬНОЙ ЧАСТЬЮ КАЖДОЙ МАТЕМАТИЧЕСКОЙ УЧЕБНОЙ ПРОГРАММЫ?
Разработчики учебных программ и авторы учебников по математике делают упор на получение правильных ответов на задачи, поставленные в книге. Соответственно, экзамены по математике проверяют ученика на правильность его решений. Но не следует ли также проверять умение и точность ученика в проверке решения? В конце концов, когда он не посещает школу и находится в реальном мире, разве он не отвечает за правильные ответы? Откуда он знает, что он прав, если поблизости нет учителя и нет тетради для ответов?
Книги
по математике почти всегда учат методу решения для определенного типа задачи, но те же самые книги не всегда учат подходящему методу проверки для этого типа задачи.Как ни странно, в некоторых учебниках по математике действительно учат хорошим методам проверки, но не требует, чтобы учащиеся применяли проверку к своим задачам.
Итак, если учебник или учитель студента не преподает или не требует точных методов проверки ответов на конкретный тип проблемы, ученик должен надеть шапку мышления. Следующие разделы помогут учащимся развить мыслительные процессы, позволяющие выработать различные действенные методы проверки ответов.
Обратите внимание: следующие модели мышления и процедуры — это те, которым я научился у авторов моих курсов математики или разработал сам.Эти шаблоны работают для меня. Это означает, что они подтверждают, верны мои ответы или нет — и в этом случае я (стон!) Начинаю заново! Вполне вероятно, что ваш учебник и учитель могут предложить вам другие.
Также (и это важно) вы можете изобрести свой собственный способ проверки своих проблем, если ваш учитель или книга не требуют определенного метода.
ИСКУССТВО ПРОВЕРКИ ОТВЕТОВ ДЛЯ ШЕСТИ ОСНОВНЫХ ОПЕРАЦИЙ
Определение « Operation »: В своей простейшей форме (в арифметике) операция представляет собой математический процесс, в котором числа выводятся из других чисел посредством применения определенных правил.В алгебре в операциях часто используются не только числа (константы), но и буквы, представляющие неизвестные числа (переменные) или комбинацию констант и переменных.
Сложение, вычитание, умножение, деление, возведение в степень и поиск корней — самые распространенные арифметические операции.
Определение « Инверсия »: Инверсия одной операции (например, сложение) — это другая операция (например, вычитание), которая отменяет то, что было сделано первой операцией.
Определение « Undo »: Для отмены или отмены действия действия » (Ссылка: словарь английского языка Encarta).
Здесь слово «отменить» используется в предложениях: «Операция деления может использоваться для отмены операции умножения». «Операцию сложения можно использовать для отмены результатов любой операции вычитания».
Как это работает?
ПЕРВЫЙ : НАЧИНАЕМ С 8 ЯБЛОК.
ЗАТЕМ: МЫ ВЫДАЕМ 3 ЯБЛОКА.
НАКОНЕЦ : МЫ ПОЛУЧИЛИ РАЗНИЦУ 5 ЯБЛОК.
ВЫШЕ ПОСЛЕДОВАТЕЛЬНОСТЬ (сначала, затем, наконец) ЯВЛЯЕТСЯ ДЕЙСТВИЕМ И ВЛИЯНИЕМ ИСПОЛЬЗОВАНИЯ ОПЕРАЦИИ ВЫЧИСЛЕНИЯ .
Теперь, чтобы отменить это действие, до отменить это действие, мы ДОБАВЛЯЕМ 3 яблока обратно в разницу в 5 яблок.
5 яблок ПЛЮС 3 яблока = 8 яблок!
Ура! Эффект или результат операции вычитания был отменен использованием операции сложения, потому что мы вернулись туда, где мы начали, с нашим первоначальным количеством в 8 яблок!
▓
ПРОВЕРКА ОТВЕТОВ ПРИ ДОПОЛНЕНИИ ▓
ПРИ СЛОЖЕНИИ вы используете обратную операцию вычитания, чтобы проверить свой ответ, потому что вычитание отменяет (меняет) сложение.Это означает, что если вы начнете с числа 25 и прибавите 36, вы получите сумму 61. Если вы затем вычтете 36 из суммы 61, вы вернетесь к исходному числу 25. Эффект от добавления 36 был аннулирован вычитанием. 36 из суммы 61.
Конечно, если вы НЕ вернетесь к своему исходному номеру 25, то вы знаете, что допустили ошибку, и должны принять меры, чтобы найти свою ошибку.
▓
ПРОВЕРКА ПРИ ВЫЧИСЛЕНИИ ▓
ПРИ ВЫЧИТАНИИ вы используете обратную операцию сложения, чтобы проверить свой ответ.Сложение отменяет (отменяет) вычитание. Это означает, что если вы начнете с числа 61 и вычтете 36, вы получите разницу 25. Если вы затем прибавите 36, вы получите исходное число 61. Эффект вычитания 36 из 61 был отменен путем добавления обратно числа. 36.
▓
ПРОВЕРКА ПРИ УМНОЖЕНИИ ▓
ПРИ УМНОЖЕНИИ вы используете обратную операцию деления, чтобы проверить свой ответ, потому что деление отменяет (отменяет) умножение.Если вы начнете с числа 9 и умножите 9 на 7, вы получите произведение 63. Если вы затем разделите произведение 63 на 7, вы вернетесь к 9, вашему исходному числу. Деление аннулировало или аннулировало эффект умножения.
▓ ПРОВЕРКА ПРИ РАЗДЕЛЕНИИ ▓
ПРИ ДЕЛЕНИИ, вы используете обратную операцию умножения, чтобы проверить свой ответ, потому что умножение отменяет (меняет) деление.Мы начинаем с деления 63 и делим его на 7. Мы получаем частное 9. Если затем умножить это частное 9 на 7, вы вернетесь к 63, вашему исходному числу. Используя обратную операцию умножения, эффект деления на 7 был нейтрализован.
Вот пример использования обратной операции для проверки ответа на задачу 4 th с разделением слов:
▓
ПРОВЕРКА ПРИ ПОВЫШЕНИИ ЧИСЛА ДО МОЩНОСТИ (или ПРОВЕРКА ПРИ ИСПОЛЬЗОВАНИИ ЭКСПОНЕНЦИАЛЬНЫХ ВЫРАЖЕНИЙ) ▓
ПРИ УВЕЛИЧЕНИИ ЧИСЛА ДО МОЩНОСТИ вы используете обратную операцию «нахождения корней», чтобы проверить свой ответ, потому что нахождение корней отменяет (меняет) возведение числа в степень.Если вы начнете с основания 3 и возведете его в степень 4 th , вы получите произведение 81. Если вы затем найдете корень 4 th из 81, вы вернетесь на свою базу, 3. действие поиска корня отменяет эффект возведения числа в степень.
▓
ПРОВЕРКА ПРИ НАЙДЕ КОРНЕЙ ▓
ПРИ НАЙДЕНИИ КОРНЕЙ вы используете обратную операцию возведения в степень, чтобы проверить свой ответ, потому что возведение в степень отменяет (отменяет) нахождение корня.Если вы начнете с подкоренного выражения 81 и найдете корень 4 -го , вы получите корень из 3. Если затем вы найдете 4 -е значение в степени 3, вы вернетесь к исходному числу 81.
ПРОВЕРКА ОБРАЩЕНИЕМ: ПРЕОБРАЗОВАНИЕ РЕШЕНИЯ НАЗАД К ИСХОДНОЙ ПРОБЛЕМЕ
На втором этапе OSOM, когда студент усвоил тему, он создает свои собственные задачи для этой темы, решает свою проблему и проверяет свои ответы.Ниже приведены примеры того, как проверять решения, возвращаясь от решения к исходной проблеме.
▓ ВТОРОЙ КЛАСС: ПРОВЕРКА ФАКТА СЕМЬИ ▓
Семейство фактов — это четыре математических факта (два факта сложения и два факта вычитания), полученных с использованием одних и тех же трех чисел. Чтобы создать группу фактов, ученик выбирает три числа, которые будут использоваться для записи двух фактов сложения и двух фактов вычитания.Студент должен тщательно выбрать три числа. Первые два числа должны составлять в сумме третье число (например, 3 + 5 = 8).
Он делает небольшой столик и наверху пишет три числа, которые он выбрал для своей семьи.
Он пишет свои четыре факта.
Чтобы проверить свою таблицу из четырех фактов, он думает наоборот. Он выбирает только один из своих фактов и записывает этот факт в новую таблицу.
Исходя только из этого факта, он пишет всю свою семью фактов.Если его новая таблица совпадает с исходной, он был прав.
В классе 6 -го большинство учащихся учатся легко, быстро и точно преобразовывать каждый из трех видов дроби (обыкновенная дробь, десятичная дробь или процент) в две другие формы дроби.Проблема, решение и проверка, придуманная учащимся, могут выглядеть примерно так:
Первый: Учащийся создает три задачи с дробями, используя таблицу дробей. Для каждой строки дана только одна дробь. Студент должен преобразовать эту дробь в две другие эквивалентные формы.
Секунда: Учащийся решает свою задачу, находя две другие эквивалентные дроби
Третий: Чтобы проверить созданную им задачу, ученик составляет новую таблицу дробей и записывает в только свои решения, намеренно опуская исходную задачу.
Четвертый: Он проверяет свое решение, находя исходную проблему (выделена красным). Если он вернется к своей исходной проблеме, значит, его решение было правильным.
▓
АЛГЕБРА ДВА: ПРОВЕРКА ПОЛЯРНЫХ КООРДИНАТ ▓
По алгебре 2 студенты учатся преобразовывать декартовы координаты в полярные координаты.
Затем, преобразовав полярные координаты НАЗАД в декартовы координаты, ученик проверил решение своей собственной задачи.
ПРОВЕРКА ОТВЕТОВ В АЛГЕБРЕ
В алгебре есть два широко используемых метода проверки решений.
▓
ONE: ПРОВЕРКА УРАВНЕНИЙ ПОДСТАВКОЙ ▒
Решенные вами уравнения можно проверить с помощью , применив аксиому подстановки:
► Вы нашли определенное значение (значения) для переменной (переменных) в уравнении.Теперь подставьте те же значения, которые вы нашли, обратно в исходное уравнение.
► Оцените исходное уравнение, используя свои значения.
► Число (а) справа и слева от знака равенства будет совпадать, если ваш ответ правильный.
Вот пример проверки ответа на уравнение подстановкой. (Это тот же пример, который использовался для демонстрации создания задач в преалгебре.)
Вот пример проверки более сложной задачи в алгебре 1 — рационального уравнения.В левой колонке есть решение. Теперь обратите внимание на правую колонку, где проверялось решение. Этот пример показывает, что иногда требуется больше размышлений, времени и усилий, чтобы проверить решение уравнения, по сравнению с простым решением проблемы. Положительные целые числа относительно легко проверить. Отрицательные целые числа немного сложнее. Дробные ответы (включая десятичные дроби и проценты) обычно требуют еще большей работы. Но мысли и усилия, приложенные для проверки решений, неизбежно приводят к большему обучению.
Проверка рационального уравнения
ПРОВЕРКА РЕШЕНИЯ С КОМПЛЕКСНЫМ ЧИСЛОМ К КВАДРАТИЧЕСКОМУ УРАВНЕНИЮ
Вот обычная процедура решения задачи по квадрату:
Теперь решение этой задачи с комплексным числом проверяется заменой:
ОПРЕДЕЛЕНИЕ «ВЫРАЖЕНИЯ»: выражение — это одна цифра или символ (например, 5 или 5,000, y или p) или значимая группа символов (например, 2p + x3 или 3y + 226qx), имеющая одно значение.
После того, как вы упростили выражение, содержащее переменные, выберите простые в использовании числа, такие как 2, 3, 4, 5 и т. Д., Чтобы заменить переменные. Внимание: Поскольку деление на ноль не определено, убедитесь, что числители любых дробей не приводят к нулю. Кроме того, если какие-либо терминов будут иметь нулевое значение, вы, вероятно, не сможете проверить свое решение. Итак, просто убедитесь, что ваши термины имеют значение больше нуля и что ваши числители не равны нулю.
► Подставьте выбранные вами числа вместо переменных в исходное и упрощенное выражения.
►В-третьих, оцените как исходное выражение, так и ваше упрощенное выражение, используя одно и то же значение для каждой переменной. Конечное значение исходного выражения и окончательное значение упрощенного выражения будут такими же, если ваше упрощение было выполнено правильно.
На курсах Pre-Algebra студент учится упрощать выражения, добавляя похожие термины:
Следующий пример (из Алгебры 2) демонстрирует упрощение сложной дроби — дроби, содержащей более одной дробной черты.Поскольку процедура упрощения довольно долгая, даются и оцениваются только исходная сложная дробь и окончательные упрощенные выражения:
▓
ИСПОЛЬЗОВАНИЕ КАЛЬКУЛЯТОРА ДЛЯ ПРОВЕРКИ УПРОЩЕННЫХ ВЫРАЖЕНИЙ, СОДЕРЖАЩИХ ТОЛЬКО ЦИФРЫ ▒
Я часто использую научный калькулятор для проверки упрощений выражений, содержащих числа без переменных. В следующем примере радикальное выражение с вложенным радикалом упрощается с использованием дробных показателей.Вместо того, чтобы округлять свой ответ до обычных двух знаков после запятой, я вычислил до 9 знаков после запятой. Иногда я нахожу это полезным при сравнении двух таких выражений. Иногда последний десятичный разряд (9 -е ) может быть другим.
▓
ПРОВЕРКА С ПОМОЩЬЮ ГРАФИЧЕСКИХ РЕШЕНИЙ ▒
ГРАФИК МОЖЕТ ИСПОЛЬЗОВАТЬСЯ ДЛЯ ПРОВЕРКИ РЕШЕНИЙ ЛИНЕЙНЫХ УРАВНЕНИЙ.
В «Алгебре один» ученик учится решать систему двух одновременных уравнений тремя методами: устранением, заменой и построением графиков.Графический метод можно использовать для проверки решений исключения и замены.
На первой иллюстрации системы уравнений решены методом исключения:
На второй иллюстрации та же пара одновременных уравнений решается заменой:
На третьем рисунке с помощью одной и той же пары одновременных уравнений значения задаются переменным x и y для подготовки к построению графика.Таблица используется для каждого уравнения в системе.
На четвертой иллюстрации показаны значения x и y в двух линейных уравнениях. Точка пересечения двух линий — это общее решение системы. График подтверждает, что решения, полученные путем исключения и замены, верны:
▓
ПРОВЕРКА С ИСПОЛЬЗОВАНИЕМ АЛЬТЕРНАТИВНЫХ АЛГОРИТМОВ ▒
Студенты часто изучают альтернативные алгоритмы для решения определенных типов задач арифметики, алгебры, геометрии и тригонометрии.Когда они знают два (или более) таких алгоритма для решения одной и той же проблемы, один алгоритм может использоваться для решения проблемы, а другой — для проверки ответа. Следующие ниже примеры использования альтернативных алгоритмов для проверки ответов — это лишь некоторые из множества возможностей.
КВАДРАТИЧЕСКИЕ УРАВНЕНИЯ
Некоторые квадратные уравнения могут быть решены с помощью теоремы о нулевом множителе. Все квадратные уравнения можно решить, заполнив квадрат или используя формулу корней квадратного уравнения.Используйте один метод для решения проблемы и другой метод, чтобы проверить свой ответ.
ПОДОБНЫЕ ТРЕУГОЛЬНИКИ
Размеры недостающих сторон подобных треугольников можно определить с помощью масштабного коэффициента. Недостающие стороны также можно определить по пропорциям; поскольку все три пары соответствующих сторон одинаковых треугольников имеют одинаковое отношение, эти три эквивалентных отношения могут быть преобразованы в пропорции, а недостающие стороны могут быть найдены путем перекрестного умножения.Учащийся решает с помощью масштабного коэффициента или пропорции и проверяет, используя другой метод.
ПРЯМОЕ И КОСВЕННОЕ ИЗМЕНЕНИЕ
Задачи как прямого, так и косвенного изменения могут быть решены путем нахождения константы пропорциональности или путем установления пропорции с использованием равных соотношений. (Я показал только общую форму этих двух методов.
И снова ученик решает одним методом, а проверяет другим.
КОНТРОЛЬНЫЕ РЕШЕНИЯ В ГЕОМЕТРИИ
Я не принимаю математическую истину в моих книгах, не проверив на себе различные концепции и процедуры.
Чтобы доказать себе, что измерения в реальном мире согласуются с «математическим миром», описанным в моих учебниках, я часто проверяю геометрические постулаты, теоремы и определения, точно вычерчивая проблемы и решения, используя точно такие же конкретные углы, сегменты и т. Д. .дано в проблеме. Если мое геометрическое решение верное, фактические размеры на моей чертежной бумаге будут такими же, как измерения, сделанные с использованием геометрических правил и алгебраических вычислений. Только тогда я пойму, что математика верна.
На моей бумаге размером 8 ½ x 11 это точный квадрат размером 7 дюймов. При фактическом измерении с помощью линейки диагональ фактически равна произведению длины стороны и.
И КОГДА ВЫ НЕ МОЖЕТЕ ПРОВЕРИТЬ ОТВЕТ ДРУГИМ СПОСОБОМ?
Что вы делаете, если не можете проверить свой ответ каким-либо другим способом или у вас мало времени? Просто повторите проблему.РЕШИТЕ ЭТО СНОВА, ЕСЛИ ВЫ НИКОГДА НЕ РЕШЛИ ЭТО ДО НАЧАЛА. Для этого нужно взять новый лист бумаги (или перевернуть лист) и, не глядя на исходный метод решения или ответ, решить его снова. Если ваш второй ответ совпадает с первым, вы, вероятно, правы. (Я говорю «вероятно», потому что вы можете — как я сделал — дважды повторить одну и ту же ошибку и получить тот же неправильный ответ!)
Уравнений и неравенств, проверяющих решения уравнений
Как проверить, является ли значение решением уравнения?
Уравнение работает следующим образом:
(выражение в левой части) = (выражение в правой части)
Следовательно, уравнение верно только в том случае, если выражение в левой части действительно совпадает с выражением в правой части .Взгляни на свои руки. Они точно такие же?
Если это не так, это нормально. Во всяком случае, в основном. Хм, может быть, мы вернемся к этой аргументации позже.
Чтобы проверить, является ли данное значение решением уравнения:
Вычислите выражение в левой части для данного значения, чтобы получить число.
Вычислите выражение в правой части с заданным значением, чтобы получить число.
Посмотрите, совпадают ли числа.
Эй, он совпадает! Вы делаете это с носками каждый день.Иногда не , а , но, по крайней мере, этот процесс смутно знаком.
Если числа, полученные при вычислении двух выражений, совпадают, то данное значение является решением уравнения (делает уравнение истинным). Если числа не совпадают, данное значение не является решением уравнения (делает уравнение ложным). Возьмите те ценности, которые не являются решениями, и выбросьте их прямо в корзину, потому что они нам больше не понадобятся. На самом деле, может быть, промойте их и вместо этого отправьте на переработку.Мы стараемся быть зелеными.
Пример задачи
Является ли x = 5 решением уравнения
Не очень удивительный путь (он же неправильный путь)
Если первое, что мы делаем, это записываем, мы предъявляем претензию без выполнив работу, чтобы убедиться, что утверждение верно. Ох, хватит.
Утверждение, что левая и правая части равны, должно появиться после оценки левой части, оценки правой части и сравнения ответов.Если бы мы были юристами, мы бы назвали это нашей «должной осмотрительностью». К счастью, это Алгебра Шмупа, и мы мужественно сопротивляемся желанию отпускать ужасные шутки про адвокатов прямо сейчас.
The Super Awesome Way (также известный как The Right Way):
Сначала оцените левую часть для x = 5:
Затем оцените правую часть для x = 5:
Поскольку 2 = 2, мы можем сказать, что x = 5 является решением уравнения.Держу пари, зная, что это поможет тебе лучше спать сегодня вечером.
Использование разумности для решения математических задач — Видео и стенограмма урока
возмутительных ответов
Допустим, вы работаете над проблемой 235 * 4 — 10. Вы решили проблему, и у вас есть ответ 93. Это правильный ответ? Что ж, один из способов использовать разумность — это оценить ответ, чтобы увидеть, является ли ваш ответ возмутительным или нет. Итак, глядя на вашу проблему, вы видите, что вы умножаете 200-с чем-то число на 4, а затем вычитаете из него 10.Прикидывая, вы можете сказать, что ваш ответ должен быть несколько больше 800. Глядя на ваш ответ 93, вы видите, что ваш ответ далек от 800! Тогда ваш ответ возмутительный и, следовательно, неправильный. Вам нужно будет переработать вашу проблему.
Переформулируя задачу, вы получите ответ 930.
Вы можете использовать оценку для проверки разумности, когда у вас есть математические задачи со всеми числами. Когда вы используете оценку, она не скажет вам, действительно ли у вас есть правильный ответ, но она скажет вам, близки ли вы, и ваш ответ, вероятно, правильный.
Вставка ответа
Еще один способ использовать разумность для проверки вашего ответа — это вставить свой ответ, чтобы убедиться, что он правильный. Этот метод проверки разумности лучше всего использовать для алгебраических задач или любых других задач, где вы можете вставить свой ответ, чтобы проверить правильность полученного математического уравнения. Этот метод сообщит вам, что вы нашли правильный ответ!
Например, предположим, что вы работаете над этой задачей: найдите x , когда y = 3.
21 x -7 y = 21
Вы проходите процесс решения для x и получаете x = 2. Правильный ли это ответ? Вы можете использовать разумность для проверки, вставив найденное вами значение x . Давай посмотрим что происходит.
Подключив 2 для x , вы получите следующее:
21 * (2) — 7 * (3) = 21
42 — 21 = 21
21 = 21
А-ха! Это правильно.Это означает, что x = 2 — правильный ответ! Вы правильно ответили!
Еще один пример
Давайте посмотрим на другой пример.
Используйте разумность, чтобы проверить свой ответ на эту проблему: оцените 8 * 21 * 13.
Вы получите 2 184 после оценки проблемы. Теперь вам нужно использовать разумность, чтобы проверить этот ответ. Поскольку эта задача состоит из всех чисел, вы можете использовать оценку как средство проверки. При оценке вы округляете свои числа. Итак, ваша проблема теперь выглядит так: 10 * 20 * 10.Это гораздо проще сделать в уме. Вы можете сделать это быстро и получите 2000. Итак, ваш ответ должен быть около 2000. Это где твой ответ? Да, ваши 2184 — это почти 2000. Значит, ваш ответ, вероятно, правильный!
Резюме урока
Хорошо, давайте еще раз подумаем, прежде чем мы закончим. С точки зрения математики, разумность означает проверку найденного вами ответа путем оценки или добавления ответа, чтобы проверить, работает ли он. Вы используете оценку для задач, включающих все числа.Однако, когда вы используете оценку, она не скажет вам, правильный ли ваш ответ, но скажет, если вы ошиблись. Однако когда вы вставите свой ответ, он покажет, правильный ли ваш ответ или нет. Добавление ответа для проверки идеально подходит для алгебраических задач и других задач, когда вы можете вставить свой ответ, чтобы проверить правильность уравнения.
Решение проблем с помощью метода «угадать, проверить и проверить»
Объяснение шагов
Первый шаг в методе предположения, проверки и исправления — это сделать обоснованное предположение.Хммм, что значит делать обоснованное предположение? Обратите внимание, что в нашем начальном примере Эми впервые предположила, что это 68 пирожных и 69 печенья. Хотя это предположение, это не просто два числа, которые она случайно вытащила из воздуха. Она знала, что у нее достаточно ингредиентов, чтобы приготовить 137 хлебобулочных изделий, поэтому она знала, что в сумме два числа должны составлять 137, а затем она начала где-то посередине. Она использовала предоставленную информацию, чтобы сделать предположение, которое имело смысл для проблемы — это обоснованное предположение .
Шаг первый
После того, как обоснованное предположение было сделано, мы хотим проверить , чтобы убедиться, что это предположение верно. Другими словами, решает ли это проблему? После того, как Эми угадывала 68 пирожных и 69 печенья, она проверила, принесет ли это ей те 400 долларов, которые она хочет. Это объясняет второй шаг метода, который подводит нас к последнему шагу — пересмотру.
Шаг второй
Наш третий шаг — пересмотреть наше предположение, сделав новое предположение.На данный момент у нас есть немного больше информации, поэтому мы можем использовать ее, чтобы сделать еще лучшее предположение. Когда Эми поняла, что ее первое предположение из 68 пирожных и 69 печенья принесло 478 долларов, она знала, что это слишком много, поэтому, когда она пересмотрела свое предположение, она увеличила более дешевый вариант и уменьшила более дорогой вариант. Она сделала пересмотренное предположение , основанное на новой информации, полученной в результате своего последнего предположения.
Шаг третий
Мы знаем, что она повторяла этот процесс, пока не получила правильный ответ.Это все, что нужно сделать, когда дело доходит до метода предположения, проверки и исправления. Это не так уж сложно, правда? Мы просто делаем в точности то, что следует из названия — угадываем, проверяем и исправляем!
Пример
Предположим, что Эми и Пол в течение недели пробегали мили на благотворительность. Эми пробежала на 8 миль больше, чем Пол, а вместе они пробежали 58 миль. Вы пытаетесь выяснить, сколько миль пробежала Эми и сколько миль пробежал Пол.
Проблема
При использовании нашего процесса первый шаг — сделать обоснованное предположение.Начнем с предположения, что Пол пробежал 20 миль. Мы знаем, что Эми пробежала на 8 миль больше, чем Пол, так что это будет означать, что Эми пробежала 28 миль.
Теперь мы проверим, решит ли это нашу проблему. Мы знаем, что вместе они пробежали 58 миль. Если мы сложим их мили в нашем предположении, мы получим 20 + 28 = 48. Это немного мало. Это говорит нам, что нам нужно перейти к третьему шагу и исправить.
Мы знаем, что нам нужно подняться немного выше, поэтому предположим, что Пол пробежал 25 миль. Теперь мы начинаем второй раунд этого процесса.Если бы Пол пробежал 25 миль, Эми бы пробежала 33 мили, поскольку она пробежала на 8 миль больше, чем Пол. Мы проверяем, решает ли это проблему, складывая мили, чтобы получить 25 + 33 = 58. Динь, динь, динь !!! Это то, что мы искали! У нас есть ответ. Пол пробежал 25 миль, а Эми — 33 мили.
Догадки
Резюме урока
Метод угадай, проверь и исправь — это метод решения, который используется в математике.Как следует из названия, он состоит из трех шагов:
Угадай — сделай обоснованное предположение.
Проверить — проверьте, решает ли ваша догадка проблему.
Revise — Если ваше предположение не решает проблему, исправьте его и начните заново.
Это действительно очень простой способ решения проблем, но, как мы видели, он может быть невероятно полезным. Самое приятное то, что все, что вам нужно сделать, чтобы запомнить процесс, — это запомнить название — угадайте, проверьте и исправьте!
Промежуточная алгебра Урок 8: Введение в решение проблем
WTAMU > Виртуальная математическая лаборатория> Алгебра среднего уровня
Цели обучения
По завершении этого руководства вы сможете:
Используйте четырехэтапный процесс Polya для решения словесных задач, связанных с числами,
проценты, прямоугольники, дополнительные углы, дополнительные углы,
последовательный
целые числа и безубыточность.
Введение
Нравится вам это или нет, собираетесь ли вы
мать, отец,
учитель, программист, ученый, исследователь, владелец бизнеса,
тренер,
математик, менеджер, врач, юрист, банкир (список можно продолжать и
на), решение проблем везде.Некоторые люди
считать
что ты либо можешь, либо не можешь. Вопреки этому убеждению,
это может быть выученная профессия. Даже лучшие спортсмены и музыканты
имел
немного коучинга и много практики. Вот что это
также требует умения решать проблемы.
Георгий
Поля ,
известен как отец современного решения проблем, провел обширные исследования
а также
написал множество математических статей и три книги по проблеме
решение.
Я собираюсь показать вам его метод решения проблем, чтобы помочь вам
через
эти проблемы.
Учебник
Как упоминалось выше, я использую четыре шага Полии для решения проблемы
решение для
показать студентам, как решать задачи со словами. Просто
Примечание
что ваш учитель математики или учебник по математике могут назвать это немного иначе,
но
вы увидите, что все это в основном означает одно и то же.
Если вы выполните эти шаги, это поможет вам стать более
успешный в
мир решения проблем.
Поля создал свой знаменитый четырехэтапный процесс для
решение проблем, , который используется повсюду, чтобы помочь людям в решении проблем:
Шаг 1. Разберитесь в проблеме.
Иногда проблема заключается в понимании
проблема .
Если вам неясно, что нужно решить, то, вероятно, вы
будет получать неправильные результаты. Чтобы показать понимание
проблемы, вы, конечно, должны прочитать проблему
осторожно.
Звучит достаточно просто, но некоторые люди прыгают с ума и пытаются начать
решение
проблема до того, как они прочитают всю проблему. Однажды
проблема
читается, вам нужно перечислить все компоненты и данные, которые
вовлеченный.
Здесь вы будете назначать свою переменную.
Шаг 2: Разработайте план (переведите).
Когда вы разрабатываете план (переводите) , вы
придумать способ
решать проблему. Составление уравнения, построение диаграммы и
составление диаграммы — это все способы, с помощью которых вы можете решить свою
проблема.
В этом уроке мы будем настраивать уравнения для каждого
проблема.
Вы переведете их так же, как мы это делали в Tutorial.
2: Учебник по алгебраическим выражениям и 5: Свойства действительных чисел.
Шаг 3: Выполните план (решите).
Следующий шаг, выполнить план (решить) ,
большой.Это где
вы решаете уравнение, которое придумали, когда «разрабатываете план»
шаг.
Все уравнения в этом руководстве будут линейными. Если
ты
вам нужна помощь в их решении, обязательно вернитесь к Tutorial
7: Линейные уравнения в одной переменной и просмотрите эту концепцию.
Шаг 4. Оглянитесь назад (проверьте
и интерпретировать).
Возможно, вы знакомы с выражением «не делайте этого».
оглядываться’. В
решение проблем хорошо, чтобы оглянуться назад (проверить и интерпретировать). .
По сути, проверьте, использовали ли вы всю свою информацию и что
отвечать
имеет смысл. Если ваш ответ подтвердился, убедитесь, что вы
написать
ваш окончательный ответ с правильной маркировкой.
Пример
1 : двойная разница числа, и 1 больше на 4
чем
этот номер. Найдите номер.
Убедитесь, что вы внимательно прочитали вопрос.
раз.
Так как ищем номер, сдадим
x = число
* Удалить () с помощью dist.опора
* Получить все термины x на одной стороне
* Инв. суб. 2 прибавить 2
Если вы возьмете двойную разницу 6 и 1, то есть
так же, как 4
больше 6, так что это проверка.
ОКОНЧАТЕЛЬНЫЙ ОТВЕТ: Число 6.
Пример
2 : Одно число на 3 меньше другого. Если
сумма двух чисел равна 177, найдите каждое число.
Убедитесь, что вы внимательно прочитали вопрос.
раз.
Ищем два числа, и так как мы можем написать
одно число
по другому номеру допустим
x = другое число
Число
ne на 3 меньше другого числа:
x — 3 = одно число
* Объединить похожие термины
* Инв.из под 3 добавляется 3
* Инв. из мульт. 2 — это div. 2
Если сложить 90 и 87 (число 3 меньше 90), мы получим
177.
ОКОНЧАТЕЛЬНЫЙ ОТВЕТ: Одно число — 90. Другое число — 87.
Всякий раз, когда вы работаете с проблемой процента, вы
нужно убедиться
вы пишете свой процент в десятичной форме. Вы делаете это, перемещая
десятичный знак процента два слева. Например, 32% в
десятичная форма: 0,32
Если вы хотите найти процент некоторых
номер, запомнить
что «of» представляет собой умножение — так что вы умножите
процентов
(в десятичной форме) умноженное на число, от которого вы берете процент.
Пример
3 : Найдите 45% от 125.
Убедитесь, что вы внимательно прочитали вопрос.
раз.
Мы ищем число, которое составляет 45% от 125, мы
позволит
x = значение, которое мы есть
ищу
* Умножить
56.25 это 45% от 125.
ОКОНЧАТЕЛЬНЫЙ ОТВЕТ: Номер 56,25.
Пример
4 : В математическом классе 30 учеников. Примерно
70%
сдал последний тест по математике. Сколько студентов сдали последний
математика
контрольная работа?
Убедитесь, что вы внимательно прочитали вопрос.
раз.
Смотрим, сколько учеников сдали последний
тест по математике,
сдадим
x = количество
студенты
* Умножить
21 составляет 70% от 30.
ИТОГОВЫЙ ОТВЕТ: Последний тест по математике сдал 21 ученик.
Пример
5 : Я купил новый телевизор в местном магазине электроники
для
541,25 доллара США, включая налоги. Если ставка налога составляет 8,25%, найдите
цена
телевизора до того, как они добавили налог.
Убедитесь, что вы внимательно прочитали вопрос.
раз.
Ищем цену на телевизор до того, как добавили
налог,
сдадим
x = цена телевизора
до того, как был добавлен налог.
* Объединить похожие термины
* Инв. В мульт. 1.0825 — это div. от
1.0825
Если добавить 8.25% налога до 500, вы получите 541,25.
ОКОНЧАТЕЛЬНЫЙ ОТВЕТ: Первоначальная цена — 500 долларов США.
Следующая формула пригодится для решения
пример 6:
Периметр прямоугольника = 2 (длина) + 2 (ширина)
Пример
6 : На чертеже прямоугольной комнаты длина равна
На 1 дюйм больше, чем в 3 раза ширины.Найдите размеры, если
периметр
должно быть 26 дюймов.
Убедитесь, что вы внимательно прочитали вопрос.
раз.
Ищем длину и ширину
прямоугольник. С
длину можно записать через ширину, допустим
w = ширина
длина на 1 дюйм больше ширины более чем в 3 раза:
1 + 3 w = длина
* Удалить () с помощью dist.опора * Объединить похожие термины
* Инв. доп. 2 является суб. 2
* Инв. из мульт. на 8 дел. по 8
Если ширина равна 3, то длина, которая на 1 дюйм больше, чем 3
раз больше ширины
должно быть 10.Периметр прямоугольника шириной 3
дюймов, а длина 10 дюймов получается 26.
ОКОНЧАТЕЛЬНЫЙ ОТВЕТ: Ширина 3 дюйма. Длина 10 дюймов.
Дополнительные и дополнительные
уголки
Сумма дополнительных углов составляет 180 градусов.
Сумма дополнительных углов составляет 90 градусов.
Пример
7: Найдите размер каждого угла на рисунке.
ниже.
Обратите внимание: поскольку углы составляют прямую линию, они равны
дополнительный
друг другу.
Убедитесь, что вы внимательно прочитали вопрос.
раз.
На рисунке уже дано, что
x = один угол
5 x = другой угол
* Объединить похожие термины
* Инв.из мульт. на 6 дел. по 6
Если x равно 30, то 5 x = 5 (30) = 150.150 и 30 делают
сложить, чтобы быть
180, так что это дополнительные углы.
ОКОНЧАТЕЛЬНЫЙ ОТВЕТ: Два угла: 30 градусов и 150 градусов.
Целые числа, идущие подряд — это целые числа, следующие за
друг друга в
заказывать.
Например, 5, 6 и 7 — три
последовательные целые числа.
Если мы позволим x представить первое целое число,
как бы мы представили второе подряд целое число в виде x ? Что ж, если мы посмотрим на 5, 6 и 7 — обратите внимание, что 6 — это один
больше 5, первое целое число.
В общем, мы могли бы представлять второй
последовательное целое число на x + 1 . А как насчет третьего целого числа подряд.
Ну заметьте, как 7 на 2 больше 5. В
в общем, мы могли бы представить
третье последовательное целое число как x + 2.
Последовательные ЧЕТНЫЕ целые числа — четные целые числа,
следовать друг за другом
чтобы.
Например, 4, 6 и 8 — три последовательных
даже целые числа.
Если мы позволим x представить первое ЧЕТНОЕ целое число,
как бы мы представили второе подряд четное целое число в виде x ?
Обратите внимание, что 6 на два больше, чем 4, первое четное число.
В общем, мы могли бы представлять второй
последовательное ЧЕТНОЕ целое число
по x + 2 .
А как же третий подряд четный
целое число? Хорошо подмечено
как 8 на 4 больше, чем 4. В общем, мы могли бы представить
в третьих
последовательное ЧЕТНОЕ целое число как x + 4.
Последовательные целые нечетные числа — нечетные целые числа,
следовать друг за другом
чтобы.
Например, 5, 7 и 9 — три последовательных
нечетные целые числа.
Если мы позволим x представить первое целое нечетное число,
как бы мы представили второе подряд нечетное целое число x ?
Обратите внимание, что 7 на два больше, чем 5, первое нечетное целое число.
В общем, мы могли бы представлять второй
последовательное нечетное целое число
по x + 2.
А как насчет третьего подряд нечетного
целое число? Ну заметьте как
9 на 4 больше, чем 5. В общем, мы могли бы представить третьим
последовательный
Целое число ODD как x + 4.
Обратите внимание, что распространенное заблуждение состоит в том, что, поскольку
мы хотим нечетное число
что мы не должны добавлять 2, которое является четным числом. Держать в
помните, что x представляет ODD
число и
что следующее нечетное число находится на расстоянии 2, точно так же, как 7 находится на расстоянии 2 от 5, поэтому мы
нужно прибавить 2 к первому нечетному числу, чтобы перейти ко второму подряд
нечетное число.
Пример
8: Сумма трех последовательных целых чисел равна 258.
Находить
целые числа.
Убедитесь, что вы внимательно прочитали вопрос.
раз.
* Объединить похожие термины * Инв.добавления 3 является подпунктом. 3
* Инв. из мульт. на 3 — div. по 3
Сумма 85, 86 и 87 действительно равна 258.
ОКОНЧАТЕЛЬНЫЙ ОТВЕТ: Три последовательных целых числа: 85, 86 и 87.
Пример
9: Возраст трех сестер равен 3 года подряд.
целые числа.
Если сумма удвоенного 1-го четного целого, 3-кратного 2-го четного целого числа,
и третье четное целое число 34, найдите каждый возраст.
Убедитесь, что вы внимательно прочитали вопрос.
раз.
Ищем 3 ЧЕТЫЕ последовательные целые числа, мы будем
пусть
x = 1-е последовательное четное целое число
x + 2 = 2-е последовательное четное целое число
x + 4 = 3-е последовательное четное целое число
* Удалить () с помощью dist.опора * Объединить похожие термины
* Инв. доп. 10 является суб. 10
* Инв. из мульт. на 6 дел. по 6
Если мы возьмем сумму, умноженную на два, четыре, три, шесть и
8, мы получаем
34
ОКОНЧАТЕЛЬНЫЙ ОТВЕТ: Возраст трех сестер — 4, 6 и 8 лет.
Бизнес-проблема: ломка
Даже
В задаче, связанной с бизнесом, уравнение затрат C — это стоимость производства продукта.
В уравнении дохода R — это сумма
денег, которые производитель зарабатывает на продукте.
Если производитель хочет знать, сколько товаров должно
быть проданным, чтобы сломать
даже то, что можно найти, установив стоимость равной выручке.
Пример
10: Стоимость C до
произведем x , количество компакт-дисков будет C = 50 + 5 x .
Компакт-диски продаются оптом по 15 долларов за штуку, поэтому выручка рандов будет равна рандов = 15 x рупий.
Узнайте, сколько компакт-дисков необходимо изготовить и продать, чтобы их сломать.
четный.
Убедитесь, что вы внимательно прочитали вопрос.
раз.
Мы ищем количество проданных компакт-дисков
безубыточность,
сдадим
x = количество
cd’s
* Получить все условия x с одной стороны
* Инв.из мульт. на 10 дел. от
10
Когда x равно 5, стоимость и
доход как
равно 75.
ОКОНЧАТЕЛЬНЫЙ ОТВЕТ: 5 компакт-дисков.
Практические задачи
Это практические задачи, которые помогут вам
следующий уровень.
Это позволит вам проверить и понять, понимаете ли вы
эти
типы проблем. Math работает так же, как
что-нибудь
иначе, если вы хотите добиться успеха в этом, вам нужно практиковаться
Это.
Даже лучшие спортсмены и музыканты получали помощь и много
практиковаться, практиковаться, практиковаться, чтобы стать лучше в своем виде спорта или инструменте. На самом деле не бывает слишком много практики.
Чтобы получить от них максимальную отдачу, вы должны
проблема на
свой собственный, а затем проверьте свой ответ, щелкнув ссылку для
ответ / обсуждение
для этой проблемы . По ссылке вы найдете ответ
а также любые шаги, которые позволили найти этот ответ.
Практика
Задачи 1a — 1g: Решите проблему со словом.
1с. В местном мебельном магазине потрясающий
распродажа.
Они снижают каждую цену на 45%. Если кушетка у вас есть
на глаз стоит 440 долларов после уценки, какой был оригинал
цена?
Сколько бы вы сэкономили, если бы купили его на этой распродаже? (ответ / обсуждение
к 1c)
1г.Прямоугольный сад имеет ширину 8
футов меньше чем
вдвое длиннее. Найдите размеры, если периметр 20
ноги. (ответ / обсуждение
к 1d)
1д. Сумма дополнительных углов составляет 90
градусов. Находить
размер каждого угла на рисунке ниже. Обратите внимание, что поскольку
в
углы составляют прямой угол, они дополняют друг друга.
(ответ / обсуждение
к 1e)
1г. Стоимость C грн.
произвести x номеров видеомагнитофонов C = 1000 + 100 x .
Видеомагнитофоны продаются оптом по 150 долларов каждый, поэтому выручка рассчитывается по формуле R = 150 x .Узнайте, сколько видеомагнитофонов
производитель
необходимо производить и продавать, чтобы достичь безубыточности. (ответ / обсуждение
до 1 г)
Нужна дополнительная помощь по этим темам?
Последний раз редактировал Ким Сьюард 1 июля 2011 г. Авторские права на все содержимое (C) 2002 — 2011, WTAMU и Kim Seward. Все права защищены.
Что, если бы решение математических задач было таким же простым, как проверка решений? — Такие романтические факты
Если вы думаете, что компьютеры действительно хорошо умеют решать проблемы, такие как распознавание голоса или беспилотные автомобили, то ваш ум будет поражен тем, что они могли бы сделать, если бы P = NP.
Интернет-криптография потерпит крах. Распознавание голоса и изображений станет почти идеальным.Математические доказательства были бы значительно упрощены. Фондовый рынок резко изменится.
Все это делается путем доказательства одной теоремы, которая утверждает, что найти решение проблемы так же сложно, как проверить его. Подумайте об этом: на уроке алгебры в старшей школе было легче включить ответ, который вам дал учитель, в уравнения, чтобы убедиться, что он работает, или придумать ответ? Если P = NP, эти две задачи будут примерно такими же сложными, как и друг друга.
Или, используя более важный пример, что, если бы вам пришлось разложить большое число на простые числа? Когда у вас есть простые множители, легко проверить, умножаются ли они вместе, чтобы получить исходное число, но поиск множителей требует, как правило, медленного и трудоемкого процесса. Разница в скорости между поиском простых множителей и проверкой простых множителей лежит в основе большей части интернет-безопасности.